Academia.eduAcademia.edu
Radmila Bulajich Manfrino José Antonio Gómez Ortega Rogelio Valdez Delgado Topics in Algebra and Analysis Preparing for the Mathematical Olympiad Radmila Bulajich Manfrino José Antonio Gómez Ortega Rogelio Valdez Delgado Topics in Algebra and Analysis Preparing for the Mathematical Olympiad Radmila Bulajich Manfrino Facultad de Ciencias Universidad Autónoma del Estado de Morelos Cuernavaca, Morelos, México José Antonio Gómez Ortega Facultad de Ciencias Universidad Nacional Autónoma de México Distrito Federal, México Rogelio Valdez Delgado Facultad de Ciencias Universidad Autónoma del Estado de Morelos Cuernavaca, Morelos, México ISBN 978-3-319-11945-8 ISBN 978-3-319-11946-5 DOI 10.1007/978-3-319-11946-5 (eBook) Library of Congress Control Number: 2015930195 Mathematics Subject Classification (2010): 00A07, 11B25, 11B65, 11C08, 39B22, 40A05, 97Fxx Springer Cham Heidelberg New York Dordrecht London © Springer International Publishing Switzerland 2015 This work is subject to copyright. All rights are reserved by the Publisher, whether the whole or part of the material is concerned, specifically the rights of translation, reprinting, reuse of illustrations, recitation, broadcasting, reproduction on microfilms or in any other physical way, and transmission or information storage and retrieval, electronic adaptation, computer software, or by similar or dissimilar methodology now known or hereafter developed. The use of general descriptive names, registered names, trademarks, service marks, etc. in this publication does not imply, even in the absence of a specific statement, that such names are exempt from the relevant protective laws and regulations and therefore free for general use. The publisher, the authors and the editors are safe to assume that the advice and information in this book are believed to be true and accurate at the date of publication. Neither the publisher nor the authors or the editors give a warranty, express or implied, with respect to the material contained herein or for any errors or omissions that may have been made. Printed on acid-free paper Springer International Publishing AG Switzerland is part of Springer Science+Business Media (www.springer.com) Introduction Topics in algebra and analysis have become fundamental for the mathematical olympiad. Today, the problems in these topics that appear in the contests are frequent, and the problems from other areas that use algebra and analysis in their solutions are also frequent. In this book, we want to point out the principal algebra and analysis tools that a student must assimilate and learn to use gradually in training for mathematical contests and olympiads. Some of the topics that we study in the book are also part of the mathematical syllabus in high school courses, but there are other topics that are presented at the college level. That is, the book can be used as a reference text for undergraduates in the first year of college who will be facing algebra and analysis problems and will be interested in learning techniques to solve them. The book is divided in ten chapters. The first four correspond to topics from high school and they are basic for the students that are training for the mathematical olympiad contest, at a local and national level. The next four chapters are usually studied in the first year of college, but they have become fundamental tools, for the students competing in an international level. The last two chapters contain the problems and solutions of the theory studied in the book. The first chapter covers the basic algebra, as are the numerical systems, absolute value, notable products, and factorization, among others. We expect that the reader gain some skills for the manipulation of equations and algebraic formulae to carry them in equivalent forms, which are easier to understand and work with them. In Chapter 2 the study of the finite sums of numbers is presented, for instance, the sum of the squares of the first n natural numbers. The telescopic sums, arithmetic and geometric progressions are analyzed, as well as some of its properties. Chapter 3 talks about the mathematical technique to prove mathematical statements that involve natural numbers, known as the principle of mathematical induction. Its use is exemplified with several problems. Many equivalent statements of the principle of mathematical induction are presented. To complete the first part of the book, in Chapter 4 the quadratic and cubic polynomials are studied, with emphasis in the study of the discriminant of a quadratic polynomial and Vieta’s formulas for these two classes of polynomials. v vi Introduction The second part of the text begins with Chapter 5, where the complex numbers are studied, as well as its properties and some applications are given. All these with examples related to mathematical olympiad problems. In addition, a proof of the fundamental theorem of algebra is included. In Chapter 6, the principal properties of functions are studied. Also, there is an introduction to the functional equations theory, its properties and a series of recommendations are given to solve the problems where appear functional equations. Chapter 7 talks about the notion of sequence and series. Special sequences are studied as bounded, periodic, monotone, recursive, among others. In addition, the concept of convergence for sequences and series is introduced. In Chapter 8, the study of polynomials from the first part of the book is generalized. The theory of polynomials of arbitrary degree is presented, as well as several techniques to analyze properties of the polynomials. At the end of the chapter, the polynomials of several variables are studied. Most of the sections of these first eight chapters have at the end a list of exercises for the reader, selected and suitable to practice the topics in the corresponding sections. The difficulty of the exercises vary from being a direct application of a result seen in the section to being a contest problem that with the technique studied is possible to solve. Chapter 9 is a collection of problems, each one of them close to one or more of the topics seen in the book. These problems have a degree of difficulty greater than the exercises. Most of the problems have appeared in some mathematical contests around the world or olympiads. In the solution of each problem is implicit the knowledge and skills that are need to manipulate algebraic expressions. Finally, Chapter 10 contains the solutions to all exercises and problems presented in the book. The reader can notice that at the end of some sections there is a ⋆ symbol, this means that the level of the section is harder than others sections. In a first lecture, the reader can skip these sections; however, it is recommended that the reader have them in mind for the techniques used in them. We thank Leonardo Ignacio Martı́nez Sandoval and Rafael Martı́nez Enrı́quez for his always-helpful comments and suggestions, which contribute to the improvement of the material presented in this book. Contents Introduction . . . . . . . . . . . . . . . . . . . . . . . . . . . . . . . . . . . v 1 Preliminaries 1.1 Numbers . . . . . . . . . . . . 1.2 Absolute value . . . . . . . . . 1.3 Integer part and fractional part 1.4 Notable products . . . . . . . . 1.5 Matrices and determinants . . 1.6 Inequalities . . . . . . . . . . . 1.7 Factorization . . . . . . . . . . . . . . . . . . . . . . . . . . of a number . . . . . . . . . . . . . . . . . . . . . . . . . . . . . . . . . . . . . . . . . . . . . . . . . . . . . . . . . . . . . . . . . . . . . . . . . . . . . . . . . . . . . . . . . . . . . . . . . . . . . . . . . . . . . . . . . . . . 1 10 12 15 17 21 25 2 Progressions and Finite Sums 2.1 Arithmetic progressions 2.2 Geometric progressions 2.3 Other sums . . . . . . . 2.4 Telescopic sums . . . . . . . . . . . . . . . . . . . . . . . . . . . . . . . . . . . . . . . . . . . . . . . . . . . . . . . . . . . . . . . . . . . . . . . . . . . . . . . . . . . . . . . . . . . . . . . . . . . . 31 36 38 40 3 Induction Principle 3.1 The principle of mathematical induction 3.2 Binomial coefficients . . . . . . . . . . . 3.3 Infinite descent . . . . . . . . . . . . . . 3.4 Erroneous induction proofs . . . . . . . . . . . . . . . . . . . . . . . . . . . . . . . . . . . . . . . . . . . . . . . . . . . . . . . . . . . . . . . . . . . 43 53 57 60 4 Quadratic and Cubic Polynomials 4.1 Definition and properties . . . . . . . . . . . . . . . . . . . . . . . 4.1.1 Vieta’s formulas . . . . . . . . . . . . . . . . . . . . . . . . 4.2 Roots . . . . . . . . . . . . . . . . . . . . . . . . . . . . . . . . . . 63 65 66 5 Complex Numbers 5.1 Complex numbers and their properties . . . . . . 5.2 Quadratic polynomials with complex coefficients 5.3 The fundamental theorem of algebra . . . . . . . 5.4 Roots of unity . . . . . . . . . . . . . . . . . . . 5.5 Proof of the fundamental theorem of algebra ⋆ . 75 79 81 82 85 . . . . . . . . . . . . . . . . . . . . . . . . . . . . . . . . . . . . . . . . . . . . . . . . . . vii viii Contents 6 Functions and Functional Equations 6.1 Functions . . . . . . . . . . . . . . . . . . . . . . . . 6.2 Properties of functions . . . . . . . . . . . . . . . . . 6.2.1 Injective, surjective and bijective functions . 6.2.2 Even and odd functions . . . . . . . . . . . . 6.2.3 Periodic functions . . . . . . . . . . . . . . . 6.2.4 Increasing and decreasing functions . . . . . . 6.2.5 Bounded functions . . . . . . . . . . . . . . . 6.2.6 Continuity . . . . . . . . . . . . . . . . . . . 6.3 Functional equations of Cauchy type . . . . . . . . . 6.3.1 The Cauchy equation f (x + y) = f (x) + f (y) 6.3.2 The other Cauchy functional equations ⋆ . . 6.4 Recommendations to solve functional equations . . . 6.5 Difference equations. Iterations . . . . . . . . . . . . . . . . . . . . . . . . . . . . . . . . . . . . . . . . . . . . . . . . . . . . . . . . . . . . . . . . . . . . . . . . . . . . . . . . . . . . . . . . . . . . . . . . . . . . . . . . . . . . . . . . . . . . 89 94 94 96 97 98 99 99 102 102 105 108 111 7 Sequences and Series 7.1 Definition of sequence . . . . . . . . . . 7.2 Properties of sequences . . . . . . . . . 7.2.1 Bounded sequences . . . . . . . . 7.2.2 Periodic sequences . . . . . . . . 7.2.3 Recursive or recurrent sequences 7.2.4 Monotone sequences . . . . . . . 7.2.5 Totally complete sequences . . . 7.2.6 Convergent sequences . . . . . . 7.2.7 Subsequences . . . . . . . . . . . 7.3 Series . . . . . . . . . . . . . . . . . . . 7.3.1 Power series . . . . . . . . . . . . 7.3.2 Abel’s summation formula . . . . 7.4 Convergence of sequences and series ⋆ . . . . . . . . . . . . . . . . . . . . . . . . . . . . . . . . . . . . . . . . . . . . . . . . . . . . . . . . . . . . . . . . . . . . . . . . . . . . . . . . . . . . . . . . . . . . . . . . . . . . . . . . . . . . . . . . . . . . . . . . . . . . . . . . . . . . . . . . . . . . . . . . . . . . . . . . . . . . . . . . . . . . . . . . . . . . . . . . . . . . . . . . . . . . . . . . . . . . 115 118 118 119 120 124 125 126 127 129 131 133 135 8 Polynomials 8.1 Polynomials in one variable . . . . . . . . . 8.2 The division algorithm . . . . . . . . . . . . 8.3 Roots of a polynomial . . . . . . . . . . . . 8.3.1 Vieta’s formulas . . . . . . . . . . . 8.3.2 Polynomials with integer coefficients 8.3.3 Irreducible polynomials . . . . . . . 8.4 The derivative and multiple roots ⋆ . . . . 8.5 The interpolation formula . . . . . . . . . . 8.6 Other tools to find roots . . . . . . . . . . . 8.6.1 Parameters . . . . . . . . . . . . . . . . . . . . . . . . . . . . . . . . . . . . . . . . . . . . . . . . . . . . . . . . . . . . . . . . . . . . . . . . . . . . . . . . . . . . . . . . . . . . . . . . . . . . . . . . . . . . . . . . . . . . . . . . . . . . . . . . . . . . . . . . . . . . . . . . 139 140 145 145 145 146 150 151 153 153 ix Contents 8.7 8.8 8.6.2 Conjugate . . . . . . . . . 8.6.3 Descartes’ rule of signs ⋆ . Polynomials that commute . . . Polynomials of several variables . . . . . . . . . . . . . . . . . . . . . . . . . . . . . . . . . . . . . . . . . . . . . . . . . . . . . . . . . . . . . . . . . . . . . . . . . . . . . 154 155 157 161 9 Problems . . . . . . . . . . . . . . . . . . . . . . . . . . . . . . . . . . . 165 10 Solutions to Exercises and Problems 10.1 Solutions to exercises of Chapter 1 10.2 Solutions to exercises of Chapter 2 10.3 Solutions to exercises of Chapter 3 10.4 Solutions to exercises of Chapter 4 10.5 Solutions to exercises of Chapter 5 10.6 Solutions to exercises of Chapter 6 10.7 Solutions to exercises of Chapter 7 10.8 Solutions to exercises of Chapter 8 10.9 Solutions to problems of Chapter 9 . . . . . . . . . . . . . . . . . . . . . . . . . . . . . . . . . . . . . . . . . . . . . . . . . . . . . . . . . . . . . . . . . . . . . . . . . . . . . . . . . . . . . . . . . . . . . . . . . . . . . . . . . . . . . . . . . . . . . . . . . . . . . . . . . . . . . . . . . . . . . . . . . . . . . . . . . . . . . . . . . . 179 190 199 214 220 229 239 248 261 Notation . . . . . . . . . . . . . . . . . . . . . . . . . . . . . . . . . . . . . . 305 Bibliography . . . . . . . . . . . . . . . . . . . . . . . . . . . . . . . . . . . 307 Index . . . . . . . . . . . . . . . . . . . . . . . . . . . . . . . . . . . . . . . 309 Chapter 1 Preliminaries 1.1 Numbers We will assume that the reader is familiar with the notion of the set of numbers that we usually use to count. This set is called the set of natural numbers and it is usually denoted by N, that is, N = {1, 2, 3, . . . }. In this set we have two operations, the sum and the product, that is, if we add or multiply two numbers in the set we obtain a natural number. In some books 0 is considered a natural number, however, in this book it is not, but we will suppose that 0 is such that n + 0 = n, for every natural number n. Now, suppose that we want to solve the equation x + a = 0, with a ∈ N, that is, we want to find an x such that the equality is true. This equation does not have a solution in the set of natural numbers N, therefore we need to define another set which includes the set of numbers N but also the negative numbers. In other words, we need to extend the set of numbers N in order that such equation can be solved in the new set. This new set is called the set of integers and is denoted by Z, that is, Z = {. . . , −3, −2, −1, 0, 1, 2, 3, . . .}. In this set we also have two operations, the sum and the product, which satisfy the following properties. Properties 1.1.1. (a) The sum and the product of integers are commutative. That is, if a, b ∈ Z, then a + b = b + a and ab = ba. (b) The sum and the product of integers are associative. That is, if a, b and c ∈ Z, then (a + b) + c = a + (b + c) and (ab)c = a(bc). © Springer Internationl Publishing Switzerland 2015 R.B. Manfrino et al, Topics in Algebra and Analysis, DOI 10.1007/978-3-319-11946-5_1 1 2 Chapter 1. Preliminaries (c) There exists in Z a neutral element for the sum, the number 0. That is, if a ∈ Z, then a + 0 = 0 + a = a. (d) There exists in Z a neutral element for the multiplication, the number 1. That is, if a ∈ Z, then a1 = 1a = a. (e) For each a ∈ Z there exists an inverse element under the sum which is denoted by −a. That is, a + (−a) = (−a) + a = 0. (f) In Z a distributive law holds, in which addition and multiplication are involved. That is, if a, b and c ∈ Z, then a(b + c) = ab + ac. Note that the existence of the additive inverse allows us to solve equations of the type mentioned above, that is, x + a = b, where a and b are integers. However, there is not necessarily an integer number x that solves the equation qx = p, with p and q integers. Therefore the necessity arises to extend the set of integers. Consider the set of rational numbers, which is denoted by Q, defined as  p | p ∈ Z and q ∈ Z\{0} . Q= q In general, when working with the rational number pq we ask that p and q do not have common prime factors, that is, the numbers are relative primes, which is denoted by (p, q) = 1. In the set of rational numbers we also have two operations, the sum and the product, which satisfy all the properties valid in the set of integers, but in the case of the product we have an extra property, the multiplicative inverse element. Property 1.1.2. If pq ∈ Q, with p = 0 and (p, q) = 1, then there exists a unique number, pq ∈ Q, which is called the multiplicative inverse of pq , such that p q · = 1. q p Using this property we can solve equations of the form qx = p, however, there are numbers that we cannot write as the quotient of two integers. For example, if we want to solve the equation x2 − 2 = 0, this equation does not √ have a solution 2 and proceed to in the set Q. We write the solutions of the equation as x = ± √ prove that 2 is not in Q. √ Proposition 1.1.3. The number 2 is not a rational number. √ Proof. Suppose√the contrary, that is, 2 is a rational number. Therefore, it can be written as 2 = pq , where p and q do not have common factors. Squaring both sides of the equation we get 2 = p2 q2 , that is, 2q 2 = p2 . This means that 3 1.1 Numbers p2 is an even number, but then p is also even. But if p is even, p is of the form p = 2m, then 2q 2 = (2m)2 = 4m2 . Dividing by 2 both sides of the equation, give us q 2 = 2m2 , that is, q 2 is even and therefore q is also even. Hence, p and q are even numbers, which contradicts the fact that p and q were assumed as not having √  common factors. Thus, 2 is not a rational number. We can give a geometric representation of the rational numbers as points on a straight line, which in this case is called the number line. A straight line can be travelled in two directions, one which we call the positive direction and the other the negative direction. Once we have agreed about which of the directions is to be taken as positive we talk of an oriented straight line. For example, we can decide that the positive direction goes from left to right. If we consider two points O and U on the straight line, we will give the same orientation to a line segment contained on the straight line. That is, if we let the point O be 0 and U be on the right of it, we will say that the line segment OU is traversed in the positive direction. If U represents the point 1, we will call OU an unitary oriented straight line segment. In this way, we place, to the right of 0, what we take to be all the positive integers equally spaced along the line, that is, two consecutive integers are spaced a distance equal to the length of the segment OU . To represent the negative numbers it is sufficient to do the same, starting at O and traversing the straight line in the opposite direction. The rational number pq is defined as the oriented segment pq OU . This segment is obtained when we sum p times the qth part of the segment OU . More precisely, we do the following: (a) Divide the segment OU into q equal parts. To do this, we make use of an extra straight line passing through O and not perpendicular to OU , and in this line we take q points W1 , . . . , Wq , where two consecutive points are separated a distance OW1 . Now, we draw the line segment from Wq to U and for each Wj we draw a parallel straight line to U Wq , the intersection points of the parallels with OU will divide OU in q equal parts. If V is the intersection point of the parallel to U Wq through W1 , we have that V is the point that represents the number q1 (note that OV has the same orientation as OU ). We also consider V ′ the symmetric point to V , with respect to O. In the following figure, we took q = 4. Wq W1 V′ 0 O V 1 U 2 4 Chapter 1. Preliminaries (b) If p is a non-negative integer, take OP = OV + OV + · · · + OV = p · OV.    p times p q OU . The segment OP is, by definition, point P , with p = 6 and q = 4. U′ In the next figure we have marked the 0 1 O U 2 P (c) If p is negative, let p′ be the positive integer such that p = −p′ . Then OP = OV ′ + OV ′ + · · · + OV ′ = p′ OV ′ = (−p′ )OV = p · OV.    p′ times p q OU . The segment OP is, by definition, point is simply denoted by pq . Since OU is the unitary segment, this With this representation of the rational numbers, we have that every rational number defines a point on the number line, but there are points on the number line that are not represented by a rational number. For example, √ we want to determine, on the number line, which point represents the number 2, which we proved above is not a rational number. To do this, take the right triangle whose legs are each equal to 1. √ Then, by the Pythagorean theorem, the hypotenuse of √ the triangle is equal to 2. If we take a compass and draw a circle of radius 2 and center at 0, the point √where the circle intersects the positive part of the number line corresponds to 2. √ 2 1 0 O 1 U √ 2 A point on the number line that does not correspond to a rational number represents an irrational number, and the set of irrational numbers is denoted by I. The union of these two sets is called the set of real numbers and is denoted by R, that is, R = Q ∪ I. The set of real numbers R contains the set of natural numbers, the set of integers and the set of rational numbers. In fact, we have the following chain of inclusions N ⊂ Z ⊂ Q ⊂ R. 5 1.1 Numbers Given two points on the number line that we know represent two real numbers, we can find the point which represents the sum of these two numbers in the following way: if P and Q are two points on the straight line and O is the origin, the sum will be the addition of the oriented segments OP and OQ, as we can see in the following figure. OP + OQ OP OQ OP 0 O P +Q Q P We can also find the point that represents the product of two points P and Q which are on the number line. In order to do that we consider an extra straight line passing through the origin O and not perpendicular to the line OP . We mark on the extra straight line the unity U and the point Q. Through Q we draw a parallel line to U P which will intersect the real line at a point R. OR Since the triangles ORQ and OP U are similar it follows that OP = OQ OU , therefore OR · OU = OP · OQ, hence OR represents the product of P and Q. Q U O P R With this geometric representation of the numbers it is easy to find, in the real line, the sum and the product of any two real numbers, no matter whether they are rational or irrational numbers. Similarly, as it happens in the set of integers, the operations in the set of real numbers have the same properties. Properties 1.1.4. (a) (b) (c) (d) The sum of two real numbers is a real number. The sum of two real numbers is commutative. The sum is associative. The number 0 is called the additive neutral element. That is, x + 0 = x for all x ∈ R. (e) Every real number x has an additive inverse. That is, there is a real number, which we denote by −x, such that x + (−x) = 0. 6 Chapter 1. Preliminaries (f) (g) (h) (i) The product of two real numbers is a real number. The product of two real numbers is commutative. The product is associative. The number 1 is the multiplicative neutral element. That is, x · 1 = x for all x ∈ R. (j) Every real number x different from 0, has a multiplicative inverse. That is, there exists a real number, which we denote by x−1 , such that x · x−1 = 1. (k) For any three real numbers x, y, z it follows that x (y + z) = x · y + x · z. This property is called the distributive law. In the set of integers there is an order. With this we want to point out that given two integers a and b we can say which one is greater. We say that a is greater than b if a − b is a natural number. In symbols we have that a > b if and only if a − b ∈ N. This is equivalent to saying that a − b > 0. In general, the notation a > b is equivalent to b < a. The expression a ≥ b means that a > b or a = b. Similarly, a ≤ b means that a < b or a = b. Properties 1.1.5. If a is an integer number, one and only one of the following relations holds: (a) a > 0, (b) a = 0, (c) a < 0. In the set of rational numbers and in the set of real numbers, we also have the order properties. The order of the real numbers enables us to compare two numbers and to decide which one of them is greater or whether they are equal. Let us assume that the real number system contains a set P , which we will call the set of positive numbers, and we will express in symbols a > 0 if a belongs to P . In the geometric representation of the real numbers, the set P in the number line is, of the two pieces in which O has divided the straight line, the piece which contains U (the number 1). The following properties are satisfied. Properties 1.1.6. Every real number x has one and only one of the following properties: (a) x = 0. (b) x ∈ P , that is, x > 0. Properties 1.1.7. (c) −x ∈ P , that is, −x > 0. (a) If x, y ∈ P , then x + y ∈ P (in symbols x > 0, y > 0, then x + y > 0). (b) If x, y ∈ P , then xy ∈ P (in symbols x > 0, y > 0, then xy > 0). We will denote by R+ the set P of positive real numbers. Now we can define the relation x is greater than y, by saying that it holds x − y ∈ P (in symbols x > y). Similarly, x is smaller than y, if y − x ∈ P (in 7 1.1 Numbers symbols x < y). Observe that x < y is equivalent to y > x. We can also define that x is smaller than or equal to y, if x < y or x = y, (using symbols x ≤ y). Example 1.1.8. (a) If x < y and z is any number, then x + z < y + z. (b) If x < y and z > 0, then xz < yz. In fact, to prove (a) we see that x + z < y + z if and only if (y + z) − (x + z) > 0 if and only if y − x > 0 if and only if x < y. To prove (b), we proceed as follows: x < y implies that y − x > 0, and since z > 0, then (y − x)z > 0, therefore yz − xz > 0, hence xz < yz. Exercise 1.1. Prove the following statements: (i) (ii) (iii) (iv) (v) (vi) If If If If If If a < 0, b < 0, then ab > 0. a < 0, b > 0, then ab < 0. a < b, b < c, then a < c. a < b, c < d, then a + c < b + d. a > 0, then a−1 > 0. a < 0, then a−1 < 0. Exercise 1.2. Let a, b be real numbers. Prove that, if a + b, a2 + b and a + b2 are rational numbers, and a + b = 1, then a and b are rational numbers. Exercise 1.3. Let a, b be real numbers such that a2 + b2 , a3 + b3 and a4 + b4 are rational numbers. Prove that a + b, ab are also rational numbers. Exercise 1.4. √ (i) Prove that if p is a prime number, then p is an irrational number. √ (ii) Prove that if m is a positive integer which is not a perfect square, then m is an irrational number. Exercise 1.5. Prove that there are an infinite number of pairs of irrational numbers a, b such that a + b = ab is an integer number. Exercise 1.6. If the coefficients of ax2 + bx + c = 0 are odd integers, then the roots of the equation cannot be rational numbers. Exercise 1.7. Prove that for real numbers a and b, it follows that  √ a+ b= a+ √ a2 − b + 2 a− √ a2 − b . 2 8 Chapter 1. Preliminaries Exercise 1.8. For positive numbers a and b, find the value of:   √ √ a a a a . . .. (ii) a b a b . . .. (i) Exercise 1.9 (Romania, 2001). Let x, y and z be non-zero real numbers such that xy, yz and zx are rational numbers. Prove that: (i) x2 + y 2 + z 2 is a rational number. (ii) If x3 + y 3 + z 3 is a rational number different from zero, then x, y and z are rational numbers. Exercise√1.10 (Romania, 2011). Let a, b be different real positive numbers, such √ that a− ab and b− ab are both rational numbers. Prove that a and b are rational numbers. The decimal system is a positional system in which every digit takes a value based on its position with respect to the decimal point. That is, the digit is multiplied by a power of 10 according to the position it occupies. For the units digit, that is, the digit which is just to the left-hand side of the decimal point, it is multiplied by 10n , with n = 0. Accordingly, the digit in the tens position must be multiplied by 101 = 10. The exponent increases one by one when we move from right to left and decreases one by one when we move in the other direction. For example, 87325.31 = 8 · 104 + 7 · 103 + 3 · 102 + 2 · 101 + 5 · 100 + 3 · 10−1 + 1 · 10−2 . In general, every real number can be written as an infinite decimal expansion in the following way bm . . . b1 b0 .a1 a2 a3 . . . , where bi and ai belong to {0, 1, . . . , 9}, for every i. The symbol . . . means that on the right-hand side of the number we can have an infinite number of digits, in this way the number bm . . . b1 b0 .a1 a2 a3 . . . , represents the real number bm · 10m + · · · + b1 · 10 + b0 · 100 + a1 · 10−1 + a2 · 10−2 + · · · . For example, 1 3 = 0.3333 . . . , 1 2 = 0.50000 . . . , 3 7 √ = 0.428571428571 . . ., 2 = 1.4142135 . . . . With this notation we can distinguish between rational and irrational numbers. The rational numbers are those in which the decimal expansion is finite or infinite, but there is, always, a certain number of digits which, after a certain point, repeat 34 = 0.123636 . . . is periodic of period 2 periodically, for example the number 275 after the third digit. Meanwhile, for the irrational numbers, the decimal expansion is infinite and never becomes periodic. 9 1.1 Numbers This manner of representing numbers is essential to solve some of the problems that appear in the mathematical olympiad. In the same way, as in the decimal representation of the numbers in base 10, we can represent the integers in any base. If m is a positive integer, we can find its representation in base b if we write the number as a sum of powers of b, that is, m = ar br + · · · + a1 b + a0 . The integers which appear as coefficients of the powers of b in the representation must be smaller than b. Observation 1.1.9. To identify a number which is not written in base 10, we will use a subindex indicating the base, for example, 12047 means that the number 1204 is a number expressed using base 7. Let us analyze the following example. Example 1.1.10. In which base is the number 221 a factor of 1215? The number 1215 in base a is written as a3 + 2a2 + a + 5 and the number 221 in base a is 2a2 + 2a + 1. Therefore, if we divide a3 + 2a2 + a + 5 by 2a2 + 2a + 1 we obtain that 1 1 a+ 2 2 9 1 + − a+ . 2 2   Therefore, since 1215a has to be a multiple of 221a, the remainder − 12 a + 29 has to be 0 and 12 a + 21 has to be an integer number. Both conditions are satisfied if a = 9. a3 + 2a2 + a + 5 = (2a2 + 2a + 1) Exercise 1.11. Write the following numbers as integers: (i) 0.11111 . . . (ii) 1.14141414 . . . . m n, where m and n are positive Exercise 1.12. (i) Prove that 121b is a perfect square in any base b ≥ 2. (ii) Determine the smallest value of b such that 232b is a perfect square. Exercise 1.13 (IMO, 1970). Let a, b and n be positive integers greater than 1. Let An−1 and An be numbers expressed in the numerical system in base a and Bn−1 and Bn be two numbers in the numerical system in base b. These numbers are related in the following way, An = xn xn−1 . . . x0 , An−1 = xn−1 xn−2 . . . x0 , Bn = xn xn−1 . . . x0 , Bn−1 = xn−1 xn−2 . . . x0 , with xn = 0 and xn−1 = 0. Prove that a > b if and only if Bn−1 An−1 < . An Bn 10 Chapter 1. Preliminaries 1.2 Absolute value We define the absolute value of a real number x as |x| =  x, if −x, if x≥0 x < 0. (1.1) For a real non-negative number k, the identity |x| = k is satisfied only by x = k and x = −k. The inequality |x| ≤ k is equivalent to −k ≤ x ≤ k. This can be seen as follows: If x ≥ 0, then 0 ≤ x = |x| ≤ k. On the other hand, if x ≤ 0, then −x = |x| ≤ k, therefore, x ≥ −k. As a consequence of the previous discussion, we observe that x ≤ |x|. In the next figure we show the values of x which satisfy the inequality. These being the values lying between −k and k, and including the two numbers. The set [−k, k] = {x ∈ R | − k ≤ x ≤ k} is called a closed interval, since it contains the numbers k and −k. The numbers −k, k are called the endpoints of the interval. [ −k | O ] k Similarly, the inequality |x| ≥ k is equivalent to x ≥ k or −x ≥ k. In the next figure, the values of x that satisfy the inequalities are the values falling before −k and −k itself, or k and those values to the right of k. The set (−k, k) = {x ∈ R | − k < x < k} is called an open interval, since it does not contain either k or −k, that is, an open interval is an interval that does not contain its endpoints. With this definition, we see that the set of values x that satisfy |x| ≥ k are the values x ∈ / (−k, k). ] −k | O [ k Example 1.2.1. Find in the Cartesian plane1 the area enclosed by the graph of the relation |x| + |y| = 1. For |x| + |y| = 1 we have to consider four cases: (a) x ≥ 0 and y ≥ 0; this implies x + y = 1, that is, y = 1 − x. (b) x ≥ 0 and y < 0; this implies x − y = 1, that is, y = x − 1. (c) x < 0 and y ≥ 0; this implies −x + y = 1, that is, y = x + 1. (d) x < 0 and y < 0; this implies −x − y = 1, that is, y = −x − 1. 1 The Cartesian plane is defined as R2 = R × R = {(x, y) | x ∈ R, y ∈ R}. 11 1.2 Absolute value We can now draw the graph of the four straight lines. (0, 1) (−1, 0) (1, 0) (0, −1) The area enclosed by the four straight lines is formed by four isosceles right triangles, each of which has two sides of length equal to 1. Since the area of each of 1 1 these triangles is 1×1 2 = 2 , the area of the square is 4 2 = 2. Example 1.2.2. Solve the equation |2x − 4| = |x + 5|. We have that |2x − 4| =  2x − 4, −2x + 4, if if x≥2 x < 2. We also have that |x + 5| =  x + 5, −x − 5, if if x ≥ −5 x < −5. If x ≥ 2, then 2x−4 = x+5, that is, x = 9. If x < −5, then −2x+4 = −x−5, hence x = 9, but this is impossible since x < −5. The last case that we have to consider is −5 ≤ x < 2. Then, the equation that we have to solve is −2x + 4 = x + 5. Solving for x we get x = − 31 . Therefore, the numbers which satisfy the equation are x = 9 and x = − 13 . Sometimes it is easier to solve these equations without using the explicit form of the absolute value, just by observing that |a| = |b| if and only if a = ±b and making use of the absolute value properties. Observation 1.2.3. If x is any real number then the relation between the square root √ and the absolute value is given by x2 = |x|. The identity follows from |x|2 = x2 and |x| ≥ 0. Properties 1.2.4. If x and y are real numbers the following relations hold:     (a) |xy| = |x||y|. This implies also that  xy  = |x| |y| , if y = 0. (b) |x + y| ≤ |x| + |y|, where the equality holds if and only if xy ≥ 0. Proof. (a) The proof follows directly from |xy|2 = (xy)2 = x2 y 2 = |x|2 |y|2 , and taking the square root on both sides gives the result. 12 Chapter 1. Preliminaries (b) Since both sides of the inequality are positive numbers, it is enough to 2 2 verify that |x + y| ≤ (|x| + |y|) . 2 2 |x + y| = (x + y)2 = x2 + 2xy + y 2 = |x| + 2xy + |y| 2 2 2 2 2 2 ≤ |x| + 2 |xy| + |y| = |x| + 2 |x| |y| + |y| = (|x| + |y|) . In the previous chain of relations there is only one inequality, it follows trivially from the fact that xy ≤ |xy|. Moreover, the equality holds if and only if xy = |xy|, which is true only when xy ≥ 0.  Inequality (b) in Properties 1.2.4 can be extended in a general form as | ± x1 ± x2 ± · · · ± xn | ≤ |x1 | + |x2 | + · · · + |xn |, for real numbers x1 , x2 , . . . , xn . The equality holds when all the ±xi ’s have the same sign. This last inequality can be proved in a similar way, and also using induction2 . Exercise 1.14. If a and b are any real numbers, prove that ||a| − |b|| ≤ |a − b|. Exercise 1.15. Find, in each case, the numbers x that satisfy the following: (i) |x − 1| − |x + 1| = 0. (ii) |x − 1||x + 1| = 1. (iii) |x − 1| + |x + 1| = 2. Exercise 1.16. Find all the triplets (x, y, z) of real numbers satisfying |x + y| ≥ 1, 2xy − z 2 ≥ 1, z − |x + y| ≥ −1. Exercise 1.17 (OMM, 2004). Find the largest number of positive integers that can be found in such a way that any two of them, a and b (with a = b), satisfy the next inequality ab . |a − b| ≥ 100 1.3 Integer part and fractional part of a number Given any number x ∈ R, sometimes it is useful to consider the integer number max{k ∈ Z | k ≤ x}, that is, the greatest integer less than or equal to x. This number is denoted by ⌊x⌋ and it is known as the integer part of x. From this definition, the following properties hold. 2 See Section 3.1. 1.3 Integer part and fractional part of a number 13 Properties 1.3.1. Let x, y ∈ R and n ∈ N, then it follows that: (a) x − 1 < ⌊x⌋ ≤ x < ⌊x⌋ + 1. (b) x is an integer if and only if ⌊x⌋ = x. (c) ⌊x + n⌋ = ⌊x⌋ + n.     = nx . (d) ⌊x⌋ n (e) ⌊x⌋ + ⌊y⌋ ≤ ⌊x + y⌋ ≤ ⌊x⌋ + ⌊y⌋ + 1. Proof. The proof of the first three properties follows immediately from the definition. (d) If we divide ⌊x⌋ by n, we have that ⌊x⌋ = an + b, for an integer number a and for an integer number b such that  0 ≤ b < n.    = an+b = a + nb = a. On the other On the one hand, we have that ⌊x⌋ n n hand, x =  x  since  an+b+c  ⌊x⌋ +c,b+cwith  0 ≤ c < 1, and b + c < n − 1 + 1 = n, we get = = a + = a. Then the equality holds. n n n (e) Since x = ⌊x⌋ + a and y = ⌊y⌋ + b with 0 ≤ a, b < 1, then ⌊x + y⌋ = ⌊x⌋ + ⌊y⌋ + ⌊a + b⌋ by property (c). The inequalities follow if we observe that if 0 ≤ a, b < 1, then 0 ≤ ⌊a + b⌋ ≤ 1.  Example 1.3.2. For any real number x, it follows that   1 ⌊x⌋ + x + − ⌊2x⌋ = 0. 2 If we let n = ⌊x⌋, then x can be expressed as x = n + a with 0 ≤ a < 1, hence     1 1 − ⌊2x⌋ = n + n + a + − ⌊2(n + a)⌋ ⌊x⌋ + x + 2 2   1 − 2n − ⌊2a⌋ =n+n+ a+ 2   1 = a+ − ⌊2a⌋ , 2 1 where the second equality follows1 by property (c). Now, if 0 ≤1 a < 2 , then  1 a + 2 = ⌊2a⌋ = 0, meanwhile, if 2 ≤ a < 1, it follows that a + 2 = ⌊2a⌋ = 1. Example 1.3.3. If n and m are positive integers without common factors, then    n   2n   3n  (m − 1)n (m − 1)(n − 1) . + + + ··· + = m m m m 2 Consider, in the Cartesian plane, the straight line passing through the origin and the point (m, n). Since m and n are relative primes, then on the segment 14 Chapter 1. Preliminaries where points (0, 0) and (m, n) lie, there is no other point with integer coordinates. B = (m, n) C = (0, n) n−1 .. . 1 A = (m, 0) O 1 2 ··· m−1 n n x and passes over the points (j, m The equation of the straight line is y = m j), n n with j = 1, . . . , (m − 1), and such that m j is not an integer. The number m j is equal to the number of points with integer coordinates lying on the straight line n x and y = 1 included. It follows that x = j and between the straight lines y = m the sum is equal to the number of points with integer coordinates that lie in the interior of the triangle OAB, and by symmetry it is equal to half of the points with integer coordinates inside the rectangle OABC. The number of points with integer coordinates in the rectangle is (n − 1)(m − 1), therefore    n   2n   3n  (m − 1)n (m − 1)(n − 1) . + + + ··· + = m m m m 2 Observation 1.3.4. Since the right-hand side of the last inequality is symmetric in m and n, then          n   2n  2m (m − 1)n (n − 1)m m + + + ··· + = + ···+ . m m m n n n For a number x ∈ R we can also consider the number {x} = x − ⌊x⌋, which we call the fractional part of x, and for which the following properties are fulfilled. Properties 1.3.5. Let x, y ∈ R and n ∈ Z, then it follows that: (a) (b) (c) (d) 0 ≤ {x} < 1. x = ⌊x⌋ + {x}. {x + y} ≤ {x} + {y} ≤ {x + y} + 1. {x + n} = {x}. Exercise 1.18. For any real numbers a, b > 0, prove that ⌊2a⌋ + ⌊2b⌋ ≥ ⌊a⌋ + ⌊b⌋ + ⌊a + b⌋. Exercise 1.19. Find the values of x that satisfy the following equation: (i) ⌊x⌊x⌋⌋ = 1. (ii) ||x| − ⌊x⌋| = ⌊|x| − ⌊x⌋⌋. 15 1.4 Notable products Exercise 1.20. Find the solutions of the system of equations x + ⌊y⌋ + {z} = 1.1, ⌊x⌋ + {y} + z = 2.2, {x} + y + ⌊z⌋ = 3.3. Exercise 1.21 (Canada, 1987). For any natural number n, prove that √ √ √ √ √ ⌊ n + n + 1⌋ = ⌊ 4n + 1⌋ = ⌊ 4n + 2⌋ = ⌊ 4n + 3⌋. 1.4 Notable products The area of a square is the square of the length of its side. If the length of the sides is a + b then the area is (a + b)2 . However, the area of the square can be divided in four rectangles as shown in the figure. a a2 ab b ab b2 a b Hence, the sum of the areas of the four rectangles will be equal to the area of the square, that is, (a + b)2 = a2 + ab + ab + b2 = a2 + 2ab + b2 . (1.2) a (a − b)2 (a − b)b Now we give a geometric representation of the square of the difference of two numbers a, b, where b ≤ a. The problem now is to find the area of the square of side a − b. (a − b)b b2 b In the figure we observe that the area of the square of side a is equal to the sum of the areas of the square of sides (a−b) and b, respectively, plus the area of two equal 16 Chapter 1. Preliminaries rectangles with sides of length b and (a − b). That is, a2 = (a − b)2 + b2 + 2b(a − b), hence (1.3) (a − b)2 = a2 − 2ab + b2 . a (a − b)2 (a − b)b Now, in order to find the area of the shaded region of the following figure, (a − b)b b2 a−b b a−b b observe that the sum of the areas of the rectangles covering the regions is a(a − b) + b(a − b), and factorizing this sum we get a(a − b) + b(a − b) = (a + b)(a − b), (1.4) which is equivalent to the area of the large square minus the area of the small square, that is, (1.5) (a + b)(a − b) = a2 − b2 . Another notable product, but now dealing with three variables, is given by (a + b + c)2 = a2 + b2 + c2 + 2ab + 2ac + 2bc. (1.6) The geometric representation of this product is given by the equality between the area of the square with side length a + b + c and the sum of the areas of the nine rectangles in which the square is partitioned, that is, (a+ b + c)2 = a2 + b2 + c2 + ab + ac+ ba+ bc+ ca+ cb = a2 + b2 + c2 + 2ab + 2ac+ 2bc. a2 ba ca a ab b2 cb b ac bc c2 c a c b Next, we provide a series of identities, some of them very well known and some others less known, but all of them very helpful for solving many problems. 17 1.5 Matrices and determinants Exercise 1.22. For every x and y ∈ R, verify the following second-degree identities: (i) x2 + y 2 = (x + y)2 − 2xy = (x − y)2 + 2xy. (ii) (x + y)2 + (x − y)2 = 2(x2 + y 2 ). (iii) (x + y)2 − (x − y)2 = 4xy. x2 + y 2 + (x + y)2 . (iv) x2 + y 2 + xy = 2 x2 + y 2 + (x − y)2 . (v) x2 + y 2 − xy = 2 (vi) Prove that x2 + y 2 + xy ≥ 0 and x2 + y 2 − xy ≥ 0. Exercise 1.23. For any real numbers x, y, z, verify that (x + y)2 + (y + z)2 + (z + x)2 . 2 (x − y)2 + (y − z)2 + (z − x)2 . (ii) x2 + y 2 + z 2 − xy − yz − zx = 2 (iii) Prove that x2 + y 2 + z 2 + xy + yz + zx ≥ 0 and x2 + y 2 + x2 − xy − yz − zx ≥ 0. (i) x2 + y 2 + z 2 + xy + yz + zx = Exercise 1.24. For all real numbers x, y, z, verify the following identities: (i) (ii) (iii) (iv) (v) (vi) (xy + yz + zx)(x + y + z) = (x2 y + y 2 z + z 2 x) + (xy 2 + yz 2 + zx2 ) + 3xyz. (x + y)(y + z)(z + x) = (x2 y + y 2 z + z 2 x) + (xy 2 + yz 2 + zx2 ) + 2xyz. (xy + yz + zx)(x + y + z) = (x + y)(y + z)(z + x) + xyz. (x − y)(y − z)(z − x) = (xy 2 + yz 2 + zx2 ) − (x2 y + y 2 z + z 2 x). (x + y)(y + z)(z + x) − 8xyz = 2z(x − y)2 + (x + y)(x − z)(y − z). xy 2 + yz 2 + zx2 − 3xyz = z(x − y)2 + y(x − z)(y − z). Exercise 1.25. For any real numbers x, y, z, verify that: (i) x2 + y 2 + z 2 + 3(xy + yz + zx) = (x + y)(y + z) + (y + z)(z + x) + (z + x)(x + y).   (ii) xy + yz + zx − x2 + y 2 + z 2 = (x − y)(y − z) + (y − z)(z − x) + (z − x)(x − y). Exercise 1.26. For any real numbers x, y, z, verify that: (x−y)2 +(y −z)2 +(z −x)2 = 2 [(x − y)(x − z) + (y − z)(y − x) + (z − x)(z − y)] . 1.5 Matrices and determinants A 2 × 2 matrix is an array  a11 a21 a12 a22  18 Chapter 1. Preliminaries where a11 , a12 , a21 and a22 are real or complex numbers3 . The determinant of the above matrix, which is denoted by    a11 a12       a21 a22  is the real number defined by a11 a22 − a12 a21 . A 3 × 3 matrix is an array ⎛ a11 ⎜ ⎝ a21 a31 a12 a22 a32 ⎞ a13 ⎟ a23 ⎠ a33 where, again, every aij is a number. The subindex indicates the position of the number in the array. Then, aij is the number located in the ith row and in the jth column. We define the determinant of a 3 × 3 matrix by the rule        a11 a12 a13      a   a     a  21 a22    21 a23    22 a23  .  + a13   a21 a22 a23  = a11   − a12   a31 a32     a31 a33   a32 a33   a31 a32 a33  That is, we move along the first row, multiplying a1j by the determinant of a 2 × 2 matrix obtained by eliminating the first row and the jth column, and then adding all this together, and keeping in mind to place the minus sign before a12 . The result of the determinant is not modified if instead of choosing the first row as the first step we chose the second or third row. In case we choose the second row, we begin with a negative sign and if we choose the third row the first sign is positive, that is,         a11 a12 a13    a    a   a    12 a13   11 a13    11 a12    + a22   − a23  .  a21 a22 a23  = −a21   a32 a33   a31 a33    a31 a32    a31 a32 a33  The signs alternate according to the following diagram:    + − +       − + − .    + − +  There are many properties of the determinants which follow immediately from the definitions. These properties become rules of a sort and the following are the most frequently used. 3 Complex numbers will be treated in Chapter 5. 19 1.5 Matrices and determinants Properties 1.5.1. (a) If we interchange two consecutive rows or two of the determinant does change, for example,     a21  a11 a12 a13         a21 a22 a23  = −  a11     a31  a31 a32 a33  consecutive columns, the sign a22 a12 a32 a23 a13 a33     .   (b) We can factorize the common factor of any row or column of a matrix and the corresponding determinants are related in the following way, for example,      a11 a12 a13   αa11 αa12 αa13          a22 a23  = α  a21 a22 a23  .  a21      a31 a32 a33   a31 a32 a33  (c) If to a row (or column) we add another row (or column), the value of the determinant does not change, as in the following example:      a11 a12 a13   a11 + a21 a12 + a22 a13 + a23          a21 a22 a23  a21 a22 a23  =        a31 a32 a33    a31 a32 a33   ⎞ ⎛  a11 + a12 a12 a13     ⎜  ⎟ = ⎝or  a21 + a22 a22 a23  , ⎠ .    a31 + a32 a32 a33  (d) If a matrix has two equal rows (or two equal columns) the determinant is zero. Example 1.5.2. Using determinants we can establish the identity a3 + b3 + c3 − 3abc = (a + b + c)(a2 + b2 + c2 − ab − bc − ca). (1.7) Note that   a   D= c   b b c a b c a     a     = a  c   b a    c     −b  b  b a     c   + c   b a c      = a3 − abc − abc + b3 + c3 − abc = a3 + b3 + c3 − 3abc. On the other hand, adding to the first   a+b+c b c   D = a+b+c a b   a+b+c c a column the other two,    1 b       = (a + b + c)  1 a    1 c  = (a + b + c)(a2 + b2 + c2 − ab − bc − ca). we have  c   b   a  (1.8) 20 Chapter 1. Preliminaries By properties (b) and (c), the determinants are equal. Observe that the expression a2 + b2 + c2 − ab − bc − ca can be written as4 " 1! (a − b)2 + (b − c)2 + (c − a)2 . 2 From this we obtain another version of identity (1.7), that is, a3 + b3 + c3 − 3abc = ! " 1 (a + b + c) (a − b)2 + (b − c)2 + (c − a)2 . 2 (1.9) Observe that if the above identity satisfies the condition a + b + c = 0 or the condition a = b = c, then the following identity holds: a3 + b3 + c3 = 3abc. (1.10) Reciprocally, if identity (1.10) is satisfied, then it follows that either a + b + c = 0 or a = b = c. √ √ 3 3 Exercise 1.27. Prove that 2 + 5 + 2 − 5 is a rational number. Exercise 1.28. Find the factors of the expression (x − y)3 + (y − z)3 + (z − x)3 . Exercise 1.29. Find the factors of the expression (x + 2y − 3z)3 + (y + 2z − 3x)3 + (z + 2x − 3y)3 . Exercise 1.30. Prove that if x, y, z are different real numbers, then √ √ √ 3 x − y + 3 y − z + 3 z − x = 0. √ 1 Exercise 1.31. Let r be a real number such that 3 r − √ 3 r = 1. Find the values of r − r1 and r3 − r13 . Exercise 1.32. Let a, b, c be digits different from zero. Prove that if the integers (written in decimal notation) abc, bca and cab are divisible by n then also a3 + b3 + c3 − 3abc is divisible by n. Exercise 1.33. How many ordered pairs of integers (m, n) are there such that the following conditions are satisfied: mn ≥ 0 and m3 + 99mn + n3 = 333 ? Exercise 1.34. Find the locus of points (x, y) such that x3 + y 3 + 3xy = 1. Exercise 1.35. Find the real solutions x, y, z of the equation, x3 + y 3 + z 3 = (x + y + z)3 . 4 See Exercise 1.23. 21 1.6 Inequalities 1.6 Inequalities We begin this section with one of the most important inequalities. For any real number x, we have that x2 ≥ 0. (1.11) This follows from the equality x2 = |x|2 ≥ 0. From this result, we can deduce that the sum of n squares is non-negative, n # i=1 x2i ≥ 0 (1.12) and it will be zero, if and only if all the xi ’s are zero. If we make the substitution x = a−b, where a and b are non-negative real numbers, in equation (1.11), we get (a − b)2 ≥ 0. Simplifying, the previous inequality leads to a2 + b2 ≥ 2ab. (1.13) Since a2 + b2 ≥ 2ab if and only if 2a2 + 2b2 ≥ a2 + 2ab + b2 = (a + b)2 , we also have the inequality a2 + b 2 (a + b) ≥ . 2 2 (1.14) In case both a and b are positive numbers, the inequality (1.13) guarantees that b a + ≥ 2. b a (1.15) If we take b = 1 in the previous inequality, then we have that a + a1 ≥ 2, that is, the sum of a > 0 and its reciprocal is greater than or equal to 2, and it will be 2 if and only if a = 1. √ √ Replacing a, b by a, b in (1.13), we obtain that √ a+b √ a + b ≥ 2 ab if and only if ≥ ab. 2 √ Multiplying the last inequality by ab and reordering, we obtain √ ab ≥ 2ab . a+b (1.16) (1.17) 22 Chapter 1. Preliminaries Summarizing, the inequalities (1.14), (1.16) and (1.17), which we have just proved, imply that √ a+b a2 + b 2 2ab ≤ ab ≤ ≤ . (1.18) a+b 2 2 The first expression is known as the harmonic mean (HM ), the second is the geometric mean (GM ), the third is the arithmetic mean (AM ) and the last one is known as the quadratic mean (QM ). Now, we will present a geometric and a visual proof of the previous inequalities. Consider a semicircle with center in O, radius a+b 2 and right triangles ABC, DBA and DAC, as shown in the following figure: A y h e B D E z C O a b These triangles are similar triangles and therefore the following equality holds: DC AD = DB DA h b = a h h2 = ab, √ ab, which according to the hence, the height h of the triangles is given by h = √ diagram is clearly smaller than the radius. Therefore, ab ≤ a+b 2 . To prove the first inequality in (1.18), observe that the triangles DAE and OAD are similar, hence AO AD = , AE AD h2 = y(y + z), 2ab = y, a+b that is,√y represents the harmonic mean. Clearly we have that y ≤ h, therefore 2ab ab. a+b ≤ 23 1.6 Inequalities To prove, geometrically, the last inequality in (1.18), consider the next figure. L A D We have that OD = a+b 2 −a = b−a 2 DL2 = OD2 + OL2 = that is, DL =  a2 +b2 2 O and using the Pythagorean theorem, we obtain $ b − a %2 2 + $ a + b %2 2 which is clearly greater than = a2 + b 2 , 2 a+b 2 . Using Example 1.5.2, we can provide a proof of the arithmetic mean and the geometric mean inequality for three non-negative real numbers. In fact, using the next identity a3 + b3 + c3 − 3abc = ! " 1 (a + b + c) (a − b)2 + (b − c)2 + (c − a)2 , 2 it is clear that if a, b and c are non-negative numbers, then a3 + b3 + c3 − 3abc ≥ 0, that is, a3 +b3 +c3 ≥ 3abc. Moreover, if a+b+c = 0 or (a−b)2 +(b−c)2 +(c−a)2 = 0 the equality holds, and this happens a = b =√c. Now, if x, y and z are √ only when √ non-negative numbers, define a = 3 x, b = 3 y and c = 3 z, then x+y+z √ ≥ 3 xyz 3 (1.19) with equality if and only if x = y = z. Example 1.6.1. For every real number x, it follows that 2 √x +2 x2 +1 ≥ 2. In fact, x2 + 2 x2 + 1 1 √ = √ +√ = 2 2 2 x +1 x +1 x +1 1 x2 + 1 + √ ≥ 2. 2 x +1 The inequality now follows if we apply inequality (1.15). Example 1.6.2. If a, b, c are non-negative numbers, then (a + b)(b + c)(a + c) ≥ 8abc. 24 Chapter 1. Preliminaries As we have seen, (a+b) 2 ≥ √ ab, a+b 2 (b+c) 2 b+c 2 ≥ √ bc and a+c 2 ≥ (a+c) 2 √ ≥ √ ac, then a2 b2 c2 = abc. Example 1.6.3. If x1 > x2 > x3 and y1 > y2 > y3 , which sum is greater? S = x1 y1 + x2 y2 + x3 y3 , S ′ = x1 y2 + x2 y1 + x3 y3 . Consider the difference, S ′ − S = x1 y2 − x1 y1 + x2 y1 − x2 y2 = x1 (y2 − y1 ) + x2 (y1 − y2 ) = −x1 (y1 − y2 ) + x2 (y1 − y2 ) = (x2 − x1 )(y1 − y2 ) < 0, then, S ′ < S. In general, for any permutation {y1′ , y2′ , y3′ } of {y1 , y2 , y3 }, we have that S ≥ x1 y1′ + x2 y2′ + x3 y3′ , (1.20) which is known as the rearrangement inequality5 . Exercise 1.36. Let a, b be real numbers with 0 ≤ a ≤ b ≤ 1, prove that: b−a ≤ 1. (i) 0 ≤ 1 − ab b a (ii) 0 ≤ + ≤ 1. 1+b 1+a Exercise 1.37 (Nesbitt Inequality). If a, b, c ≥ 0, prove that a b c 3 + + ≥ . b+c a+c a+b 2 Exercise 1.38. If a, b, c are the lengths of the sides of a triangle, prove that 3 a3 + b3 + c3 + 3abc ≥ max {a, b, c} . 2 Exercise 1.39. Let p and q be positive real numbers with (i) 5 To 1 1 1 1 ≤ + ≤ . 3 p(p + 1) q(q + 1) 2 (ii) see a general version, consult Example 7.3.6. 1 p + 1 q = 1. Prove that: 1 1 + ≥ 1. p(p − 1) q(q − 1) 25 1.7 Factorization Exercise 1.40. Find the smallest positive number k such that, for any 0 < a, b < 1, with ab = k, it follows that b a b a + + + ≥ 4. b a 1−b 1−a Exercise 1.41. Let a, b, c be non-negative real numbers, prove that (a + b)(b + c)(c + a) ≥ 8 (a + b + c)(ab + bc + ca). 9 Exercise 1.42. Let a, b, c be positive real numbers that satisfy the equality (a + b)(b + c)(c + a) = 1. Prove that ab + bc + ca ≤ 3 . 4 Exercise 1.43. Let a, b, c be positive real numbers that satisfy abc = 1. Prove that (a + b)(b + c)(c + a) ≥ 4(a + b + c − 1). Exercise 1.44 (APMO, 2011). Let a, b, c be positive integers. Prove that it is impossible for all three numbers a2 + b + c, b2 + c + a and c2 + a + b to be perfect squares. 1.7 Factorization One of the most important forms of algebraic manipulation is known as factorization. In this section we study some examples and problems whose solutions depend on factorization formulas. Many of the problems that involve algebraic expressions can be easily solved using algebraic transformations in which the fundamental strategy is to find appropriate factors. We start with some elemental formulas of factorization, where x, y are real numbers: (a) x2 − y 2 = (x + y)(x − y). (b) x2 + 2xy + y 2 = (x + y)2 and x2 − 2xy + y 2 = (x − y)2 . (c) x2 + y 2 + z 2 + 2xy + 2yz + 2zx = (x + y + z)2 . These algebraic identities are cataloged as second-order identities. In fact, we studied these four identities in the section of notable products. However, now we would like, given an algebraic expression, to reduce it to a product of simpler algebraic expressions. 26 Chapter 1. Preliminaries Example 1.7.1. For real numbers a, b, x, y, with x and y different from zero, it follows that b2 (a + b)2 (ay − bx)2 a2 + − = . x y x+y xy(x + y) To obtain the equality, we start with the sum on the left side of the identity and follow the ensuing equalities a2 b2 (a + b)2 a2 y(x + y) + b2 x(x + y) − xy(a + b)2 + − = x y x+y xy(x + y) a2 y 2 + b2 x2 − 2xyab = xy(x + y) (ay − bx)2 . = xy(x + y) An application of the previous identity helps us to prove, in a straightforward manner, the so-called helpful inequality6 of degree 2. This inequality assures that for real numbers a, b and real positive numbers x, y, it follows that a2 b2 (a + b)2 + ≥ . x y x+y The following identities are known as third-degree identities, with x, y, z ∈ R:   (a) x3 − y 3 = (x − y) x2 + xy + y 2 . (b) x3 − y 3 = (x − y)3 + 3xy(x − y). (c) (x + y)3 − (x3 + y 3 ) = 3xy(x + y). (d) x3 − xy 2 + x2 y − y 3 = (x + y)(x2 − y 2 ). (e) x3 + xy 2 − x2 y − y 3 = (x − y)(x2 + y 2 ). To prove the validity of these identities, it is enough to expand one of the sides of the equalities or use the Newton binomial theorem, which we will study in Section 3.2. Another quite important identity of degree 3, previously given as (1.7), is x3 + y 3 + z 3 − 3xyz = (x + y + z)(x2 + y 2 + z 2 − xy − yz − zx), for all x, y, z real numbers. A proof of this identity can be obtained directly expanding the right-hand side of the identity. Different proofs of this equality will be given throughout this book. An equivalent form of the above identity is ' & 1 2 x3 + y 3 + z 3 − 3xyz = (x + y + z) (x − y) + (y − z)2 + (z − x)2 . 2 6 See [6] or [7]. 27 1.7 Factorization   The identities x2 − y 2 = (x + y)(x − y) and x3 − y 3 = (x − y) x2 + xy + y 2 are particular cases of the nth degree identity, xn − y n = (x − y)(xn−1 + xn−2 y + · · · + xy n−2 + y n−1 ), (1.21) for any x, y real numbers. If n is odd, we can replace y by −y in the last formula to obtain the factorization for the sum of two nth odd power numbers, xn + y n = (x + y)(xn−1 − xn−2 y + · · · − xy n−2 + y n−1 ). (1.22) In general, when n is even, the sum of two nth powers cannot be factored, however there are some exceptions when it is possible to complete squares. Let us see the following example. Example 1.7.2 (Sophie Germain identity). For any x, y real numbers, it follows that x4 + 4y 4 = (x2 + 2y 2 + 2xy)(x2 + 2y 2 − 2xy). Completing the square, we have x4 + 4y 4 = x4 + 4x2 y 2 + 4y 4 − 4x2 y 2 = (x2 + 2y 2 )2 − (2xy)2 = (x2 + 2y 2 + 2xy)(x2 + 2y 2 − 2xy). Another example, with even powers is the following. Example 1.7.3. For any x, y real numbers, it follows that x2n − y 2n = (x + y)(x2n−1 − x2n−2 y + x2n−3 y 2 − · · · + xy 2n−2 − y 2n−1 ). To prove this we only have to divide x2n − y 2n by x + y or do the product on the right-hand side and simplify. Example 1.7.4. The number n4 −22n2 +9 is a composite number for any integer n. The idea is to try to factorize the expression. We do it by completing squares, and the common way to do it is as follows: n4 − 22n2 + 9 = (n4 − 22n2 + 121) − 112 = (n2 − 11)2 − 112. While doing this a problem arises: 112 is not a perfect square, therefore the factorization is not immediate. However, we can try the following strategy to complete squares, which is less usual, n4 − 22n2 + 9 = (n4 − 6n2 + 9) − 16n2 = (n2 − 3)2 − 16n2 = (n2 − 3)2 − (4n)2 = (n2 − 3 + 4n)(n2 − 3 − 4n) = ((n + 2)2 − 7)((n − 2)2 − 7), and observe that none of the factors is equal to ±1. The following is another example of how to solve problems using basic factorizations. Example 1.7.5. Find all the pairs (m, n) of positive integers such that |3m −2n | = 1. 28 Chapter 1. Preliminaries When m = 1 or m = 2, it is easy to find the solutions (m, n) = (1, 1), (1, 2), (2, 3). Now, we will prove that these are the only solutions of the equation. Suppose that (m, n) is a solution of |3m − 2n | = 1, with m > 2 and therefore n > 3. We analyze both cases: 3m − 2n = 1 and 3m − 2n = −1. Suppose that 3m − 2n = −1 with n > 3, then 8 divides 3m + 1. However, if we divide 3m by 8 we obtain as a remainder 1 or 3, depending on whether n is odd or even, therefore in this case we do not have a solution. Suppose now that 3m −2n = 1 with m ≥ 3, therefore n ≥ 5, since 2n +1 = 3m ≥ 27. Then 3m − 1 is divisible by 8, hence m is even. Write m = 2k, with k > 1. Then 2n = 32k − 1 = (3k + 1)(3k − 1), therefore 3k + 1 = 2r , for some r > 3. But the previous case tells us we know that this is impossible, therefore in this case there are no solutions. The following formulas are also helpful to factorize. For real numbers x, y, z we have the following equalities: (x + y)(y + z)(z + x) + xyz = (x + y + z)(xy + yz + zx), 3 3 3 3 (x + y + z) = x + y + z + 3(x + y)(y + z)(z + x). (1.23) (1.24) To convince ourselves of the validity of these equalities, just expand both sides in each equality. From these equalities the following observation arises. Observation 1.7.6. (a) If x, y, z are real numbers, with xyz = 1, then (x + y)(y + z)(z + x) + 1 = (x + y + z)(xy + yz + zx). (1.25) (b) If x, y, z are real numbers with xy + yz + zx = 1, then (x + y)(y + z)(z + x) + xyz = x + y + z. (1.26) Exercise 1.45. For all real numbers x, y, z, the following identities hold: (i) (x + y + z)3 − (y + z − x)3 − (z + x − y)3 − (x + y − z)3 = 24xyz. (ii) (x − y)3 + (y − z)3 + (z − x)3 = 3(x − y)(y − z)(z − x). (iii) (x − y)(y + z)(z + x) + (y − z)(z + x)(x + y) + (z − x)(x + y)(y + z) = −(x − y)(y − z)(z − x). Exercise 1.46. For all real numbers x, y, z, prove that (i) If f (x, y, z) = x3 + y 3 + z 3 − 3xyz, then f (x, y, z) = 1 1 f (x + y, y + z, z + x) = f (−x + y + z, x − y + z, x + y − z). 2 4 (ii) If f (x, y, z) = x3 +y 3 +z 3 −3xyz, then f (x, y, z) ≥ 0 if and only if x+y+z ≥ 0, and f (x, y, z) ≤ 0 if and only if x + y + z ≤ 0. 29 1.7 Factorization Exercise 1.47. Prove that for any real numbers x, y, the following identities are satisfied: (i) (x + y)5 − (x5 + y 5 ) = 5xy(x + y)(x2 + xy + y 2 ). (ii) (x + y)7 − (x7 + y 7 ) = 7xy(x + y)(x2 + xy + y 2 )2 . Exercise 1.48. Let x, y and z be real numbers such that x = y and x2 (y + z) = y 2 (x + z) = 2. Find the value of z 2 (x + y). Exercise 1.49. Find the real solutions x, y, z and w of the system of equations x + y + z = w, 1 1 1 1 + + = . x y z w Exercise 1.50. Let x, y and z be real numbers different from zero such that x + 1 y + z = 0 and x1 + y1 + z1 = x+y+z . Prove that for any odd integer n it follows that 1 1 1 1 + n+ n = n . n x y z x + yn + z n Chapter 2 Progressions and Finite Sums 2.1 Arithmetic progressions In antiquity, patterns of points played an important role in the use of numbers and cosmological conceptions. The Pythagoreans used to represent some integers as a set of points arranged in polygonal or polyhedral forms. These integers, presented as spatial arrays, are known as figurate numbers. In this section we will study some of these numbers. Suppose that we want to add the natural numbers 1 + 2 + 3 + · · · + n, where n is any natural number. Call tn the sum of these numbers, for example, t1 = 1, t2 = 1 + 2 = 3, t3 = 1 + 2 + 3 = 6, t4 = 1 + 2 + 3 + 4 = 10. We will represent them as patterns of points in the following form: t1 t2 = 3 t3 = 6 t4 = 10 We can obtain the sum tn from the following figure, where the geometrical arrangement shows that 2tn = n(n + 1). tn ··· ··· tn n-points ··· ··· n + 1-points These numbers are known as triangular numbers. © Springer Internationl Publishing Switzerland 2015 R.B. Manfrino et al, Topics in Algebra and Analysis, DOI 10.1007/978-3-319-11946-5_2 31 32 Chapter 2. Progressions and Finite Sums This sum is also known as the Gauss sum, named after Carl Friedrich Gauss who, when he was only a child, calculated this sum using the following trick. Let tn be the sum of the n numbers that we want to add. Since the sum is commutative, we can arrange the numbers starting from the higher value and ending with the lower. If now we sum, term by term, both arrangements, we get and tn tn 2tn = 1 + 2 + ··· + = n + n− 1 + ··· + = (n + 1) + (n + 1) + · · · + therefore 2tn = n(n + 1), that is, tn = n 1 (n + 1), (2.1) n(n+1) . 2 Remember that even numbers can be represented as 2n, where n = 1, 2, . . . and odd numbers can be written as 2n − 1, where n = 1, 2, . . . . If we want to sum the first n odd numbers, that is, the ones that run from 1 to 2n − 1, and we call cn the sum of these numbers, then we have, following Gauss, that 1 + 3 + · · · + 2n − 1 cn = 1 and cn = 2n − 1 + 2n − 3 + · · · + , (2.2) 2cn = 2n + 2n + ··· + 2n therefore, 2cn = n · 2n = 2n2 , where we have multiplied by n because we have exactly n odd numbers. Therefore, cn = n2 . We can represent these sums using patterns of points in the following way: c1 c2 = 4 c3 = 9 c4 = 16 Then the sum of the first n odd natural numbers corresponds to the representation of the perfect square n2 , that is, ··· n-points .. . ··· n-points Every corridor represents the corresponding odd number we are adding, therefore, the sum of the first n odd numbers is the number of points in the square, that is, n · n = n2 . These numbers are known as the square numbers. 33 2.1 Arithmetic progressions An arithmetic progression is a collection or sequence of numbers such that each term of the sequence can be obtained from the preceding number adding a fixed quantity. That is, a collection {a0 , a2 , . . . } is an arithmetic progression if, for each n ≥ 0, an+1 = an + d, where d is a constant. This common constant is called the difference of the progression and the collection or sequence will be represented by {an }. Proposition 2.1.1. If {an } is an arithmetic progression with difference d, it follows that: (a) The term an is equal to a0 + nd, for n = 0, 1, 2, . . . . n (n + 1) = 2a02+nd (n + 1), for n = 0, 1, 2, . . . . (b) a0 + a1 + · · · + an = a0 +a 2 an−1 +an+1 (c) an = , for n = 1, 2, 3, . . . , that is, each term is the arithmetic mean 2 of its two neighbours. Proof. (a) an −a0 = (an −an−1 )+(an−1 −an−2 )+· · ·+(a1 −a0 ) = d+d+· · ·+d = nd. (b) Similarly as we did with the Gauss sum, if S = a0 + a1 + · · · + an = an + an−1 + · · · + a0 , then 2S = (a0 + an ) + (a1 + an−1 ) + · · · + (an + a0 ) = (a0 + an )(n + 1), a result of the identity aj + an−j = aj+1 + an−j−1 , since aj+1 − aj = an−j − an−j−1 = d. (c) It follows from an − an−1 = an+1 − an .  Example 2.1.2. The following figures are formed with toothpicks and they are composed by equilateral triangles. Figure 1 Figure 2 Figure 3 How many toothpicks are needed to construct the figure with n triangles? To build the first figures we need 3, 5 and 7 toothpicks, respectively. The fourth figure will have 4 triangles, that is, one more than the third figure; but we only need 2 more toothpicks to make an additional triangle. In general, this happens always as we go from figure j to figure j + 1, that is, a new triangle is constructed, but only 2 additional toothpicks are needed. Therefore, the difference of toothpicks going from one figure to the next is 2, that is, if aj and aj+1 are the number of toothpicks necessary to construct the jth figure and the next one, respectively, we have that aj+1 − aj = 2. Then, an = an−1 + 2 = an−2 + 2 · 2 = · · · = a1 + 2 · (n − 1) = 3 + 2n − 2 = 2n + 1. 34 Chapter 2. Progressions and Finite Sums Proposition 2.1.3. The sequence {an } is an arithmetic progression if and only if there exist real numbers m and b such that an = mn + b, for every n ≥ 0. Proof. If {an } is an arithmetic progression with difference d, part (a) of the previous proposition, an = a0 + nd, leads to m = d and b = a0 been the numbers we were searching for. Reciprocally, if an = mn + b, then an+1 − an = m is constant and therefore {an } is an arithmetic progression, with difference m.  An harmonic progression is a sequence {an } which satisfies that the sequence   1 1 is an arithmetic progression. That is, an+1 − a1n is constant, for every natural an number n. ( ) For example, the sequence 1, 21 , 13 , 41 , . . . , n1 , . . . is an harmonic progression since {1, 2, 3, 4, . . . } is an arithmetic progression. Exercise 2.1. Calculate the sum of the first n even numbers. Can you exhibit a geometric representation of them? Exercise 2.2. (i) If {an } and {bn } are arithmetic progressions, then {an + bn } and {an − bn } are arithmetic progressions, this can be shortened saying that {an ± bn } are arithmetic progressions. ( ) (ii) If {an } is an arithmetic progression, then bn = a2n+1 − a2n is an arithmetic progression. Exercise 2.3. If {an } is an arithmetic progression with aj = 0, for every j = 0, 1, 2, . . . , then 1 1 1 n + + ···+ = . a0 a1 a1 a2 an−1 an a0 an Exercise 2.4. Prove that a sequence {an } is an arithmetic progression if and only if there exist real numbers A and B such that Sn = a0 + a1 + · · · + an−1 = An2 + Bn, for every n ≥ 0. Exercise 2.5. A sequence {an} is an arithmetic progression of order 2 if {an+1 − an} is an arithmetic progression. Prove that {an } is an arithmetic progression of order 2 if and only if there exists a degree 2 polynomial P (x) such that P (n) = an , for every n ≥ 0. Exercise 2.6. If {an } ⊂ R+ is an arithmetic progression, then √ √ a1 + an . a1 an ≤ n a1 a2 · · · an ≤ 2 35 2.1 Arithmetic progressions Exercise 2.7. Prove that there are 5 prime numbers which are in arithmetic progression with difference 6. Is the progression unique? Exercise 2.8. For any natural number n, let Sn be the sum of the integers m, with 2n < m < 2n+1 . Prove that Sn is a multiple of 3, for all n. Exercise 2.9. If a < b < c are real numbers which are in harmonic progression, then 4 1 1 1 1 + + = − . b−c c−a a−b c a Exercise 2.10. If a, b, c and d are in harmonic progression, then a + d > b + c. Exercise 2.11. If a, b and c are real numbers, prove that b + c, c + a and a + b are in harmonic progression if and only if a2 , b2 and c2 are in arithmetic progression. Exercise 2.12. An increasing arithmetic progression satisfies that the product of any two terms of the progression is also an element of the progression. Prove that each term of the progression is an integer number. Exercise 2.13. In the following arrangement, all the odd numbers were placed in such a way that in the jth row there are j consecutive odd numbers, 1 3 5 7 9 11 13 15 17 19 .. .. . . (i) Which is the first number (on the left ) in the 100th row? (ii) Which is the sum of the numbers in the 100th row? Exercise 2.14. Consider the following array, in which the numbers from 1 to 9 are placed as indicated. 1 2 3 4 5 6 7 8 9 Observe that the sum of the integers in the two main diagonals is 15. If we construct a similar array with the numbers from 1 to 10000, what is the value of the sum of the numbers in the two main diagonals? 36 Chapter 2. Progressions and Finite Sums Exercise 2.15. Fill the following board in such a way that the numbers in the rows and columns are in arithmetic progression. 74 186 103 0 2.2 Geometric progressions A geometric progression is a sequence of numbers related in such a way that each number can be obtained by multiplying the previous one by a fixed constant, different from zero, which we call the common ratio or the ratio of the progression. That is, the sequence {an } is a geometric progression if the ratio an+1 an is constant. Let this ratio be denoted by r. Note that a0 = 0 and r = 0 are not included. Proposition 2.2.1. If {an } is a geometric progression with ratio r, then: (a) The nth term is an = a0 rn , for n = 0, 1, 2, . . . . 1 − rn+1 (b) The sum a0 + a1 + · · · + an = a0 , for n = 0, 1, 2, . . . . 1 −√r (c) If an are positive numbers then an = an−1 an+1 , for n = 1, 2, 3, . . . . Proof. (a) Since an−1 an an−1 · an−2 (b) Using part (a) we get ·· · ·· aa01 = rn we have, after simplifying, that an = a0 rn . S = a0 + a1 + · · · + an = a0 + a0 r + · · · + a0 r n = a0 (1 + r + · · · + rn ) = a0 (1 − rn+1 ) . 1−r The last equality follows from identity (1.21). √ an (c) Since an+1 an−1 an+1 . an = an−1 , it follows that an =  Example 2.2.2. If {an } and {bn } are geometric progressions, then {an · bn } is a geometric progression. If {an } and {bn } are geometric progressions, then an = a0 rn and bn = b0 sn , for some real numbers r and s. Then, an bn = (a0 b0 )(rs)n is a geometric progression with ratio rs. Exercise 2.16. If {an } and {bn } are geometric progressions, with bn = 0 for all n,   an prove that bn is a geometric progression. 37 2.2 Geometric progressions Exercise 2.17. Find the geometric progressions {an } satisfying that an+2 = an+1 + an , for all n ≥ 0. Exercise 2.18. If {an } is a geometric progression with ratio r, and if the product of a0 , a1 , . . . , an−1 is Pn , prove that: (i) Pn = an0 rn(n−1)/2 . (ii) (Pn )2 = (a0 an−1 )n . Exercise 2.19. If {an } ⊂ R+ is a geometric progression with ratio r, prove that {bn = log an } is an arithmetic progression with difference log r. Exercise 2.20. If a, b and c are in geometric progression, then a3 b3 + b3 c3 + c3 a3 = abc(a3 + b3 + c3 ). Exercise 2.21. Prove that it is possible to eliminate terms of an arithmetic progression of positive integers in such a way that the remaining terms form a geometric progression. Exercise 2.22. The lengths of the sides of a right triangle, given by a < b < c, are in geometric progression. Find the ratio of the progression. Exercise 2.23 (Slovenia, 2009). Let {an } be a non-constant arithmetic progression with initial term a1 = 1. The terms a2 , a5 , a11 form a geometric progression. Find the sum of the first 2009 terms. Exercise 2.24. In the next figure the polygonal line has been constructed between the sides of the angle ABC as follows: the first segment AC of length b is perpendicular to BC, the second segment, of length a, starts where the previous segment ended and it is perpendicular to AB; proceeding in the same manner, the following segments start where the previous one ended, and keeping the perpendicularity to BC and to AB alternately. A b a B C (i) What is the length of the nth segment? (ii) What is the length of the polygonal line of n sides? (iii) What is the length of the polygonal line if it has an infinite number of sides? 38 Chapter 2. Progressions and Finite Sums 2.3 Other sums Now, we proceed to study more complicated sums. For example, let us see how to sum the squares of the first natural numbers, 1 2 + 2 2 + 3 2 + · · · + n2 . Consider the following array of numbers: 1 2 3 4 5 ... k ... n 1 2 3 4 5 ... k ... n 1 .. . 2 .. . 3 .. . 4 .. . 5 .. . 1 2 3 4 5 ... k .. .. . . ... k ... n .. .. . . ... n n-rows We will calculate the sum of the numbers on the array in two different ways. First, add the numbers in each of the rows of the array which, according to the Gauss sum formula, is a triangular number, that is, Rn = 1 + 2 + 3 + · · · + n = n(n + 1) . 2 But since we have n rows, then the sum of all the numbers in the array is ST = n n(n + 1) . 2 On the other hand, if we add each of the corridors marked, we have that the sum of the first corridor is only 1. The sum of the second corridor is 1 + 2 · 2 = 1 + 22 . The sum of the third corridor is 1 + 2 + 3 · 3 = 1 + 2 + 32 . Continuing in the same fashion, the sum of the kth corridor is Ck = 1 + 2 + · · · + (k − 1) + k · k = 3 k k(k − 1) + k2 = k2 − . 2 2 2 Now, if we sum all the corridors, we get the sum of all the numbers in the array 3 2 (1 + 22 + · · · + n2 ) − 2 3 = (12 + 22 + · · · + n2 ) − 2 ST = C1 + C2 + · · · + Cn = 1 (1 + 2 + · · · + n) 2 1 n(n + 1) · . 2 2 Equating both sums ST , we get n 3 1 n(n + 1) n(n + 1) = (12 + 22 + · · · + n2 ) − · . 2 2 2 2 39 2.3 Other sums Then n(n + 1) 1 n(n + 1) 3 2 (1 + 22 + · · · + n2 ) = · +n , 2 2 2 2 therefore 1 2 + 2 2 + · · · + n2 = n(n + 1)(2n + 1) . 6 Now, we will study the sum of the cubes of the first n natural numbers, 1 3 + 2 3 + · · · + n3 . To that purpose, consider the following array of numbers: 6 7 ... 1 2 3 4 5 2 4 6 8 10 12 14 . . . 3 6 9 12 15 18 21 . . . 4 8 12 16 20 24 28 . . . 5 .. . 10 15 20 25 30 35 . . . .. . If we add the numbers in each of the squares, we obtain S1 = 1 S2 = 1 + 2 + 2(1 + 2) S3 = 1 + 2 + 3 + 2(1 + 2 + 3) + 3(1 + 2 + 3) .. .. . . Sn = (1 + 2 + · · · + n) + 2(1 + 2 + · · · + n) + · · · + n(1 + 2 + · · · + n) = n(n + 1) 2 2 . If we now sum all the corridors C1 = 1 C2 = 2 · 2 + 2 · 2 = 22 + 22 = 22 (1 + 1) = 23 C3 = 2(3 + 2 · 3) + 32 = 2 · 3(1 + 2) + 32 = 2 · 32 + 32 = 32 (2 + 1) = 33 .. .. . . Cn = 2(n + 2n + 3n + · · · + (n − 1)n) + n2 = 2n (n − 1)n 2 + n2 = n3 − n2 + n2 = n3 . 40 Chapter 2. Progressions and Finite Sums But, since C1 + C2 + C3 + · · · + Cn = Sn , we get 1 3 + 2 3 + 3 3 + · · · + n3 = n(n + 1) 2 2 . The above sum can also be written as 13 + 23 + 33 + · · · + n3 = (1 + 2 + 3 + · · · + n)2 . Exercise 2.25. Using the corridor technique in the following array 1 2 2 2 3 2 4 2 5 2 . . . k 2 . . . n2 1 2 2 2 3 2 4 2 5 2 . . . k 2 . . . n2 1 2 2 2 3 2 4 2 5 2 . . . k 2 . . . n2 .. .. .. .. .. .. .. .. .. . . . . . . . . . 2 2 2 2 2 ... 2 ... 1 2 3 4 5 k n2 n-rows prove the following equality, n(n + 1) 2 2 = 1 3 + 2 3 + 3 3 + · · · + n3 . 2.4 Telescopic sums When we develop a sum of the form n # k=1 [f (k + 1) − f (k)] , we have that n # k=1 [f (k + 1) − f (k)] = f (2) − f (1) + f (3) − f (2) + · · · + f (n + 1) − f (n), then the terms f (k), for k between 2 and n, cancel out and we obtain that the sum is equal to f (n + 1) − f (1). This type of sums are called telescopic sums. Let us see some examples. Example 2.4.1. Evaluate the sum n # k=1 1 . k(k + 1) 41 2.4 Telescopic sums Observe that n # k=1 1 k(k+1) n # 1 = k(k + 1) k=1 = 1 k − 1 k+1 , therefore 1 1 − k k+1 1 1 1 − + 2 2 3 n 1 = . =1− n+1 n+1 1− = + 1 1 − 3 4 Example 2.4.2 (Canada, 1969). Calculate the sum 1 1 − n n+1 + ··· + *n k=1 k! · k. Observe that k! · k = k!(k + 1 − 1) = (k + 1)! − k!, then we have that n # k=1 k! · k = n # k=1 ((k + 1)! − k!) = (2! − 1!) + (3! − 2!) + · · · + ((n + 1)! − n!) = ((n + 1)! − n!) + (n! − (n − 1)!) + · · · + (2! − 1!) = (n + 1)! − 1. Example 2.4.3. Evaluate the product 1− 1 22 1− 1 32 ··· 1 − 1 20112 . We have that 1− 1 22 = = = = 1 20112 1 1 1 1 1 1+ 1− 1+ 1− ··· 1 + 2 2 3 3 2011 1 1 1 1 1 1+ 1+ ··· 1+ 1− 1− 2 3 2011 2 3 1 2 3 2012 2010 3 4 5 · · ····· · · ····· 2 3 4 2011 2 3 4 2011 2012 1 1006 . = 2 2011 2011 1− 1 32 ··· 1 − Exercise 2.26. Find the sum 1 1 1 1 + + + ···+ . 1 · 4 4 · 7 7 · 10 2998 · 3001 1− 1 2011 ··· 1 − 1 2011 42 Chapter 2. Progressions and Finite Sums Exercise 2.27. Calculate the following sums: n n # # 2k + 1 1 (ii) . (i) k(k + 2) k 2 (k + 1)2 k=1 k=1 Exercise 2.28. Calculate the following sums: n n # # k+1 k (ii) . (i) (k + 1)! (k − 1)! + k! + (k + 1)! k=1 k=1 Exercise 2.29. Find the sum 1+ 1 1 + 2+ 2 1 2 1+ 1 1 + 2 + ···+ 2 2 3 1+ 1 1 + . 2 2011 20122 Exercise 2.30. Find the following product 1+ 1 2 1+ 1 22 ··· 1 + 1 22n . Chapter 3 Induction Principle 3.1 The principle of mathematical induction In Chapter 2 we deduced several formulas for finite sums of numbers. Thus we learned that the sum of the first n natural numbers is given by the identity 1 + 2 + ···+ n = n(n + 1) . 2 (3.1) In fact, this formula is a collection of statements, P(n), which we have proved using algebra, that are valid for every positive integer n. The validity of these series of statements can be proved using what is known as the principle of mathematical induction, which will be developed in this chapter. The principle of mathematical induction claims that a sequence of propositions P(1), P(2), P(3), . . . are valid if: 1. The statement P(1) is true. 2. The statement “P(k) implies P(k + 1)” is true. We can guarantee that this last statement is true assuming that P(k) is true and proving the validity of P(k + 1)7 . Observation 3.1.1. Statement 1 of the principle of mathematical induction is called the induction basis and statement 2 is known as the inductive step. We will prove using this principle the validity of the identity (3.1) for every natural number. Example 3.1.2. The identity 1 + 2 + 3 + · · ·+ n = integer n. 7 To n(n+1) 2 is valid for every positive prove 2. is equivalent to prove that: “not P (k + 1) implies not P (k)”, is true. © Springer Internationl Publishing Switzerland 2015 R.B. Manfrino et al, Topics in Algebra and Analysis, DOI 10.1007/978-3-319-11946-5_3 43 44 Chapter 3. Induction Principle If n = 1 the left-hand side of the identity has a unique term, that is, 1. The right, since this last term is also 1. We have that the formula is valid hand side is 1(1+1) 2 for n = 1. Now, suppose that the statement P(k) is true, that is, the equality 1 + 2 + 3 + ···+ k = k(k + 1) 2 (3.2) is valid. We will prove the corresponding identity for k + 1. According to the formula, the terms on the left-hand side, for k + 1, are 1 + 2 + 3 + · · · + k + (k + 1), and if we use (3.2), we have that k(k + 1) + (k + 1) = (k + 1) 2 (k + 1)(k + 2) = , 2 1 + 2 + 3 + · · · + k + (k + 1) = k +1 2 which is the same as the right-hand side of the formula for k + 1. This proves the validity of P(k + 1). Therefore, by the principle of mathematical induction the identity is valid for every positive integer n Example 3.1.3. For every natural number n, the number n3 − n is a multiple of 6. If n = 1, then n3 − n = 0 is a multiple of 6, since 0 = 6 · 0. Suppose that k 3 − k is a multiple of 6, that is, k 3 − k = 6 r, for some integer number r. We will prove that the statement is true for k + 1, that is, we will show that (k + 1)3 − (k + 1) is a multiple of 6. Now, we have that (k + 1)3 − (k + 1) = k 3 + 3 k 2 + 3 k + 1 − k − 1 = (k 3 − k) + 3(k 2 + k) = 6 r + 3(k 2 + k). Here we have used the induction hypothesis that P(k) holds, that is, k 3 − k = 6r. Since k 2 +k = k(k+1) is the product of consecutive numbers, one of them has to be even, then k 2 + k is even. Then, 3(k 2 + k) is a multiple of 6. Thus, (k + 1)3 − (k + 1) is a sum of multiples of 6. Therefore, by the principle of mathematical induction the result is valid for every positive integer n. Example 3.1.4 (Hanoi Towers). A beautiful legend of the creation of the world tells us that Brahma placed on the Earth three bars of diamond and 64 golden discs. The discs had all different sizes and at the beginning they were located in the first diamond bar following a decreasing order of the diameters from bottom to top. Also Brahma created a monastery where the monks had the fixed task of moving all the discs from the first bar to the third, following some rules. The allowed rules were to move one disc from one bar to any other bar but under the condition that a disc with a greater diameter could never be placed on top of a smaller disc. The 45 3.1 The principle of mathematical induction legend also tells us that when the monks would finish their duty, the world will end. What is the smallest number of necessary moves that the monks have to make to accomplish the work? 1◦ bar 2◦ bar 3◦ bar Let xn be the minimum number of necessary moves to move n discs. If n = 1, there is only one disc and therefore just one move is needed, that is, x1 = 1. With two discs in the first bar, we move the small disc to the second bar, then the big one from the first bar to the third one, and finally the small one from the second bar to the third. Therefore, x2 = 3. With three discs the problem starts to become interesting. Expressing as i → j the move of the top disc from bar i to bar j, the discs can move in the following sequence of moves 1 → 3, 1 → 2, 3 → 2, 1 → 3, 2 → 1, 2 → 3, 1 → 3, that is, x3 = 7. Observing the obtained sequence 1, 3, 7, we can note that if we sum 1 to each term we obtain consecutive powers of 2. Therefore, we conjecture that xn = 2n − 1. To prove this conjecture by induction, we only need to prove the inductive step. Suppose that xn = 2n − 1 moves and let us see what happens with n + 1 discs. By the inductive hypothesis, the n superior discs can be moved to the second bar in 2n − 1 moves. After this, the largest disc moves from the first to the third bar in one move, and finally the n discs in the second bar can be moved to the third bar in 2n − 1 moves. Therefore, in total we made (2n − 1) + 1 + (2n − 1) = 2n+1 − 1 moves. Therefore, 2n − 1 moves are enough, but is this the minimum? We will prove by induction that in fact this is the minimum. For n = 1, this is obvious. Suppose that it is true for n. Now, we want to move n + 1 discs from the first to the third bar, that is, in some moment we have to move the largest disc to the third bar. At this moment the remaining n discs have to be all together in another bar and, by the induction hypothesis, they could not have arrived there in less than 2n − 1 moves. In the same way, when the largest disc moves to its final position in the third bar, the n remaining discs have to be all in another bar, and to move them to the third bar we need at least 2n − 1 moves. Adding, we see that we cannot move n+1 discs in less than (2n −1)+1+(2n −1) = 2n+1 − 1 moves, which completes the proof. With 64 discs, as in the legend, it is necessary to do 264 − 1 moves, and if we suppose that the monks can do one move every second, to finish their task they will need 500 thousand millions of years. 46 Chapter 3. Induction Principle Observation 3.1.5. In many cases, the basis of induction is not for n = 1 but for n = k, for some natural number k. Then, if we prove the induction step, we can conclude that P(n) is true for all natural numbers n ≥ k. Example 3.1.6. For x, y real numbers and for n ≥ 2, it follows that xn − y n = (x − y)(xn−1 + xn−2 y + · · · + xy n−2 + y n−1 ). If n = 2 the result follows because x2 − y 2 = (x − y)(x + y). Suppose that xk − y k can be written as xk − y k = (x − y)(xk−1 + xk−2 y + · · · + xy k−2 + y k−1 ). We will prove the result for k + 1. We start from the following identity, xk+1 − y k+1 = xk+1 − xy k + xy k − y k+1 = x(xk − y k ) + y k (x − y). Using the induction hypothesis, we have that x(xk − y k ) + y k (x − y) = x(x − y)(xk−1 + xk−2 y + · · · + y k−1 ) + y k (x − y) = (x − y)(x(xk−1 + xk−2 y + · · · + y k−1 ) + y k ) = (x − y)(xk + xk−1 y + · · · + xy k−1 + y k ). Therefore, xk+1 − y k+1 = (x − y)(xk + xk−1 y + · · · + xy k−1 + y k ), as we wanted to prove. Example 3.1.7. The sum of the interior angles of an n-sided convex polygon is 180◦(n − 2). The statement makes sense for n ≥ 3, that is, when we have a triangle. For n = 3, the statement holds since the sum of the interior angles of a triangle is 180◦ = 180◦ (3 − 2). Suppose the statement is valid for any convex n-gon. Given a convex (n + 1)-gon with vertices A1 , . . . , An+1 , the diagonal An A1 divides the polygon in a convex n-gon A1 A2 . . . An and a triangle An An+1 A1 . The sum of the interior angles of the (n+ 1)-gon will be the sum of the angles of the n-gon A1 . . . An plus the sum of the interior angles of the triangle An An+1 A1 , that is, 180◦(n−2)+180◦ = 180◦ (n−1). To verify a series of statements P(n), in some cases it is better to work with more general propositions P ′ (n), that is, propositions such that the validity of P ′ (n) will guarantee the validity of P(n). Let us see some examples. Example 3.1.8. For any positive integer n ≥ 2, it follows that 1 1 3 1 + 2 + ···+ 2 < . 22 3 n 4 3.1 The principle of mathematical induction 47 To use the principle of mathematical induction to prove the inequality can be complicated, since if we suppose that 212 + 312 + · · · + n12 < 43 , we would have 1 3 to prove that 212 + 312 + · · · + n12 + (n+1) 2 < 4 , but the margin of manoeuvre is limited. Then we will try to work with the following stronger result Sn ≤ 3 − an , for all n ≥ 2, 4 where Sn is the sum on the left-hand side of the inequality that we want to prove, and {an } are positive numbers that we have to discover. In order to produce a proof by mathematical induction we need to show that the basis of induction is valid, that is 3 1 1 ≤ − a2 or a2 ≤ . 4 4 2 The inductive step consists in showing that the condition Sn ≤ 43 −an implies 1 that Sn+1 ≤ 34 − an+1 . Since Sn+1 = Sn + (n+1) 2 , the above will be true if an and an+1 satisfy 1 − an + an+1 ≤ 0, (n + 1)2 which is equivalent to an − an+1 ≥ Now, the numbers an = 1 n 1 , for all n ≥ 2. (n + 1)2 satisfy the above inequality, since 1 1 1 1 − = ≥ . n n+1 n(n + 1) (n + 1)2 Also, a2 = 12 satisfies the condition of the basis of induction. Therefore, the numbers an = n1 are good candidates to make possible the induction. In fact we have proved that, for every n ≥ 2, it follows that Sn ≤ 34 − n1 , thus 1 1 1 3 + 2 + ···+ 2 < . 22 3 n 4 A similar example is the following. Example 3.1.9. For any positive integer n, it follows that 1 1 1 √ < 2. √ + √ + ···+ (n + 1) n 2 1 3 2 As in the previous example, it is sufficient to prove that 1 2 1 1 √ <2− √ √ + √ + ···+ . (n + 1) n n+1 2 1 3 2 For n = 1, we have 1 √ 2 1 < 2 − √22 , which is true, that is, the induction basis holds. 48 Chapter 3. Induction Principle For the inductive step, it suffices to prove that 2 2 1 √ < √ −√ , (n + 2) n + 1 n+1 n+2 which can be reduced to prove that √ √ √ 1 2 √ < 2( n + 2 − n + 1) = √ , n+2 n+2+ n+1 but this last inequality is obviously true. Another frequently used version of the principle of mathematical induction is the following. Strong induction principle. The set of propositions P(1), P(2), P(3), . . . , P(n), . . . are true if: 1. The statement P(1) is true. 2. The statement “P(1), . . . , P(n) implies that P(n + 1)” is true. Observations 3.1.10. (a) In the strong induction principle, the inductive step is valid if each time that P(k) is valid for k ≤ n then, starting with this hypothesis, we prove that P(n + 1) is valid. (b) It is clear that the strong induction principle implies the simple principle of induction. But in fact both are equivalent, since strong induction is a consequence of simple induction. To see this, it is enough to consider the logical conjunction8 Q(n) of the propositions P(1), . . . , P(n). If P(1) is true also Q(1) is true (since they are exactly the same). If Q(n) is valid so are P(1), P(2), . . . , P(n), and by the strong induction hypothesis also P(n + 1) is valid, this implies that Q(n + 1) is true. Then, by simple induction Q(n), is true for any natural number n and the same is true for P(n). Although strong induction and the simple one are logically equivalent, in some cases it is easier to use one rather than the other. The strong induction is implicit in the definition of sequences by recurrence relations. Example 3.1.11. If a is a real number such that a + is an integer for all n ≥ 1. 1 a is an integer, then an + 1 an For n = 1, the statement is true by the hypothesis that a + a1 is an integer number. Let us see how to solve for n = 2; sometimes this case gives us ideas about how to justify the inductive step.   2 2 Note that a + a1 = a2 + a12 + 2; then a2 + a12 = a + a1 − 2 is an integer number and then our statement for n = 2 is true. 8 The logical conjunction of P(1), . . . , P(n) means that all the propositions are true at the same time. 49 3.1 The principle of mathematical induction Let us analyze again our formula 2 1 1 1 1 1 + a · + · a = a2 + 2 + a0 + 0 , 2 a a a a a     then we get, a2 + a12 = a + a1 · a + a1 − (a0 + a10 ), but this gives us another idea, the statement for n = 2 depends on the statements for n = 1 and for n = 0 (which by the way are also valid). To obtain the statement for n = 3, we will work following the previous idea, a+ 1 a = a+ 1 a a+ a2 + 1 a2 1 a = a2 + · a+ 1 a = a3 + 1 1 +a+ . a3 a It follows that a3 + 1 = a3 a2 + 1 a2 · a+ 1 a − a+ 1 a , the right-hand side is an integer number if a + a1 and also a2 + a12 are integers, but this is already known. It is now clear how we can prove the inductive step. Suppose that the statement is valid for integers less than or equal to n, then from the identity an+1 + 1 = an+1 an + 1 an · a+ 1 a − an−1 + 1 an−1 , (3.3) it follows that the statement for n + 1 is also valid. There are other ways to prove statements inductively. Cauchy’s induction principle. The set of propositions P(1), P(2), . . . , P(n), . . . are all valid if: 1. The statement P(2) is true. 2. The statement “P(n) implies P(n − 1)” is true. 3. The statement “P(n) implies P(2n)” is true. Observation 3.1.12. Let us see why Cauchy’s induction principle implies the principle of mathematical induction. First, note that 1 and 2 guarantee that P(1) is true, that is, the induction basis holds. Since 3 holds, the validity of P(n) implies that P(2n) is true. Now, applying n−1 times the condition 2, we get that P(2n−1), P(2n− 2), . . . , P(n+ 1) are all true. In particular, P(n+ 1) is true. Thus, we have proved the inductive step of the principle of mathematical induction. Therefore all P(n) are true. We apply this inductive process to prove the inequality between the geometric mean and the arithmetic mean. 50 Chapter 3. Induction Principle Example 3.1.13. For real non-negative numbers x1 , . . . , xn the following inequality holds for all n: √ x1 + x2 + · · · + xn ≥ n x1 x2 . . . xn . (3.4) n √ n Denote by A = x1 +x2 +···+x and G = n x1 x2 . . . xn , the arithmetic mean and the n geometric mean of the numbers x1 , . . . , xn , respectively. The proof will be done by induction over n, using the inductive principle just described. 1. For n = 1 we have the equality, and the case n = 2 was proved in Section 1.6. √ 2. Let x1 , x2 , . . . , xn be non-negative numbers and let g = n−1 x1 · · · xn−1 . If we add this number to the numbers x1 , . . . , xn−1 , we obtain n numbers to which we apply P(n) and obtain x1 + · · · + xn−1 + g √ ≥ n x1 x2 · · · xn−1 g = n n g n−1 · g = g. Then, x1 + · · · + xn−1 + g ≥ ng, which in turn leads to therefore P(n − 1) is valid. 3. Let x1 , x2 , . . . , x2n be non-negative numbers, then x1 +···+xn−1 n−1 ≥ g, and x1 + x2 + · · · + x2n = (x1 + x2 ) + (x3 + x4 ) + · · · + (x2n−1 + x2n )  √ √ √ ≥ 2 x1 x2 + x3 x4 + · · · + x2n−1 x2n √ 1 1 √ √ ≥ 2n x1 x2 x3 x4 · · · x2n−1 x2n n = 2n (x1 x2 · · · x2n ) 2n . In the previous sequence we applied several times the statement P(2), which we know is true, and after that the same was done with statement P(n) to the √ √ √ numbers x1 x2 , x3 x4 , . . . , x2n−1 x2n . Example 3.1.14. Let us see another way to prove the previous inequality, but in this case using another variant of the induction principle. Observe first that if some xi = 0, then the inequality is clear. Suppose then that every xi > 0, and that x1 x2 . . . xn = 1. Prove the statements P(n): x1 + x2 + · · · + xn ≥ n. Clearly the basis of induction is true, that is, P(1) : x1 ≥ 1 is true, in fact x1 = 1. Suppose that P(n) is valid for any n positive numbers whose product is 1, and let x1 , . . . , xn , xn+1 be positive numbers whose product is 1. Then, there will be some xi ≥ 1 and some xj ≤ 1. Without loss of generality, we can suppose that x1 ≥ 1 and x2 ≤ 1. Then, by the previous statement, (x1 − 1)(x2 − 1) ≤ 0, then x1 x2 +1 ≤ x1 +x2 . Therefore, x1 +x2 +· · ·+xn +xn+1 ≥ 1+x1 x2 +x3 +· · ·+xn+1 . Now apply the induction hypothesis to the n numbers x1 x2 , x3 , . . . , xn+1 to show that x1 + x2 + · · · + xn + xn+1 ≥ 1 + n, that is, the statement P(n + 1) is true. √ For the general case, if a1 , . . . , an are positive and letting G = n a1 a2 . . . an , if we an a1 consider the identities x1 = G , . . . , xn = G , we have that x1 . . . xn = 1. Then, a1 + · · · + an √ x1 + · · · + xn ≥ n, which is equivalent to ≥ n a1 . . . an . n 51 3.1 The principle of mathematical induction Example 3.1.15. Let x1 , x2 , . . . , xn and y1 , y2 , . . . , yn be natural numbers. Suppose that x1 + x2 + · · · + xn = y1 + y2 + · · · + ym < mn. Then it is possible to cancel out some terms (not all of them) from both sides of the above equality while always preserving the equality. We use induction over k = m + n. Since n ≤ x1 + x2 + · · · + xn < mn, then m > 1, and similarly n > 1, then m, n ≥ 2 and k ≥ 4. For m + n = 4, we have that m = n = 2, and the only possible cases are 1 + 1 = 1 + 1 and 1 + 2 = 1 + 2 (maybe in a different order) and the result is immediate. Suppose that k = m + n > 4 and consider s = x1 + x2 + · · · + xn = y1 + y2 + · · · + ym < mn. Without loss of generality, we can suppose that x1 is the largest term of the set of xi ’s, with i = 1, 2, . . . , n, and y1 is the largest term of the set of yj ’s, with j = 1, 2, . . . , m. We can also assume that x1 > y1 , because if x1 = y1 the problem is solved. Then we have (x1 − y1 ) + x2 + · · · + xn = y2 + · · · + ym . We need to prove that the sum s′ = y2 + · · · + ym satisfies the required condition, s , hence that is, s′ < n(m − 1). Since y1 ≥ y2 ≥ · · · ≥ ym , it follows that y1 ≥ m s m−1 m−1 =s < mn = n(m − 1), m m m and now we can apply the principle of mathematical induction to reach the desired conclusion. s′ = s − y 1 ≤ s − Exercise 3.1. (i) Prove by induction that for q = 1, 1 + q + · · · + q n−1 = 1 − qn . 1−q (ii) Prove that 1 + 21 + 22 + · · · + 2n = 2n+1 − 1. Exercise 3.2. For the Fibonacci sequence, defined by a1 = 1, a2 = 1 and, for n ≥ 3, an = an−1 + an−2 , prove that an+2 = 1 + a1 + a2 + · · · + an . n Exercise 3.3. Prove that 3n+1 divides 23 + 1, for any integer number n ≥ 0. Exercise 3.4. There are 3n coins with identical aspect, but one of the coins is false and its weight is less than the weight of the real coins. Prove how, with a plate weighing scale9 , in n weighings we can identify the false coin. 9 A plate weighing scale is a balance with two plates that will be at the same level if the weight of the objects placed in each one of the plates is the same. 52 Chapter 3. Induction Principle Exercise 3.5. (i) For which integers n does it follow that 7 divides 2n − 1? (ii) For which positive integers n does it follow that 7 divides 2n + 1? Exercise 3.6. Find the values of an , if a1 = 1 and for each n ≥ 2, it follows that a1 + a2 + · · · + an = n2 . Exercise 3.7. For n ≥ 1, prove that       n n n + 1 1 2 3 = + + + ···+ . 2 2 2 2 2 2 Exercise 3.8. Find the values of an , if a1 = 1 and for each n ≥ 2 √ n an+1 √ √ √ . a1 + a2 + · · · + an = 2 Exercise 3.9. Prove the next inequality relating the geometric mean and the arithmetic mean, √ x1 + x2 + · · · + xn ≥ n x1 x2 · · · xn , n by following the indicated steps: (i) Use induction to prove that xn+1 − (n + 1)x + n = (x − 1)2 (xn−1 + 2xn−2 + 3xn−3 + · · · + n). Therefore, for x > 0, it follows that xn+1 − (n + 1)x + n ≥ 0. n+1 and b = (ii) Apply the previous inequality to x = ab , where a = x1 +···+x n+1 x1 +···+xn , and conclude that n x1 + · · · + xn+1 n+1 n+1 ≥ xn+1 x1 + · · · + xn n n = xn+1 bn . (iii) Now use induction again in order to finish the proof. *n Exercise 3.10. that i=1 ei ai  0 < a1 < a2 < · · · < an and ei = ±1. Prove n+1Let has at least 2 different values when the ei vary over the 2n possible elections of the signs. Exercise 3.11. A sequence a1 , a2 , . . . , a2n of numbers 0 or 1 is said to be evenbalanced if a1 +a3 +· · ·+a2n−1 = a2 +a4 +· · ·+a2n . Prove that an arbitrary sequence of numbers 0 or 1, with 2n + 1 elements, has a subsequence10 even-balanced with 2n elements. 10 See Subsection 7.2.7, for the definition of subsequence. 53 3.2 Binomial coefficients Exercise 3.12. Prove that the only infinite sequence {an } of positive numbers such that for each positive integer number n, the equality a31 + a32 + · · · + a3n = (a1 + a2 + · · · + an )2 holds, is the sequence given by an = n, for n = 1, 2, . . . . Exercise 3.13. If a1 < a2 < · · · < an are positive integers, then a31 + a32 + · · · + a3n ≥ (a1 + a2 + · · · + an )2 , with equality if and only if ak = k, for each k = 1, 2, . . . , n. Exercise 3.14. (i) Prove that, for n ≥ 1, it follows that 1+ 1 2 1+ 1 22 ··· 1 + 1 2n < 5 . 2 (ii) Prove that, for n ≥ 1, it follows that 1+ 1 13 1+ 1 23 ··· 1 + 1 n3 < 3. Exercise 3.15. Let a1 , a2 , . . . , an be real numbers greater than or equal to 1. Prove that n # 1 n ≥ . √ n 1 + a 1 + a i 1 · · · an i=1 3.2 Binomial coefficients The factorial of an integer number n ≥ 0, denoted by n!, can be defined by induction as follows: (a) 0! = 1, (b) n! = n(n − 1)!, for n ≥ 1. Observation 3.2.1. If n ≥ 1, then n! = n(n − 1) · · · 2 · 1. 11  all integers n and m, with 0 ≤ m ≤ n, we define the binomial coefficient For n m as n n! = . (3.5) m!(n − m)! m 11 For a combinatorial meaning of the binomial coefficients and the factorial, see [19]. 54 Chapter 3. Induction Principle Properties 3.2.2. For 0 ≤ m ≤ n, it follows that: (a) n 0 = 1 and (b) n m = n n = 1. n . n−m (c) For each m = 1, 2, . . . , n − 1, it follows that n m = n−1 n−1 + . m−1 m (3.6) This identity is known as Pascal’s formula. (d) n m is a positive integer. Proof. To prove (a) and (b) we just apply the definition of binomial coefficient. (c) To prove this property, observe that n−1 n−1 + m−1 m (n − 1)! (n − 1)! + (m − 1)!(n − 1 − m + 1)! m!(n − 1 − m)! m(n − 1)! + (n − m)(n − 1)! n! = = = m!(n − m)! m!(n − m)! = n . m (d) This statement can be justified    by induction over n. For n = 0 and n = 1, it is clear since 00 = 1 and 10 = 11 = 1. n Suppose that for n − 1 the assumption is true, us prove that m is   let nand n = 1 are integers. By = an integer, for m = 0, . . . , n. By (a), we have that n 0  n is a sum of two integers and therefore (c), for m = 1, . . . , n − 1, we have that m it is an integer.  Theorem 3.2.3 (Binomial theorem). Let a and  b be real numbers and let n and m n are the binomial coefficients in the be integers with 0 ≤ m ≤ n. The numbers m binomial development (a + b)n , that is, (a + b)n = n n n n−1 n n−i i n n a + a b + ···+ a b + ···+ b . 0 1 i n Using the sum notation we can write the previous equality as (a + b)n = n # n n−i i a b. i i=0 This identity is known as Newton’s binomial formula. (3.7) 55 3.2 Binomial coefficients Proof. Induction over n yields aprove of this identity.  If n = 0, then (a + b)0 = 1 and 00 a0 b0 = 1. Suppose that n > 0 and that the identity is valid for n − 1, that is, (a + b)n−1 = n−1 # i=0 n − 1 n−1−i i a b i is true, then (a+b)n = (a + b)(a + b)n−1 = (a + b) n−1 # i=0 = = n−1 # n − 1 n−1−i i b a i n−1 n−1 0 n n − 1 n−i i # n − 1 n−i i a b a b + a b + i − 1 n −1 i i=1 i=1 , n−1 + n−1 n n # n−1 n−1 n a + b + an−i bi + 0 n −1 i i − 1 i=1 n−1 n a + 0 n n−1 = n n # n n−i i n n # n n−i i a b. a b + b = a + n 0 i i i=0 i=1 In the last step we have used Pascal’s formula. Example 3.2.4. If n is a positive integer, then n # i=0 (−1)i n i = 0. Apply the binomial theorem with a = 1 and b = −1, to obtain n n # # n n i 0 = (1 − 1) = . (−1)i (−1) = i i i=0 i=0 n Exercise 3.16. Prove the following equalities: n # n (i) j j=0 n =2 . n # n j (ii) 2 = 3n . j j=0 Exercise 3.17. Prove the following equalities: n m n n−r (i) = . m r r m−r (ii) n m = n n−1 . m m−1  56 Chapter 3. Induction Principle Exercise 3.18. Prove the following equalities: n 2 # 2n n = . (i) n j j=0 (ii) r # n k k=0 (iii) m r−k n # m+k k = m+n+1 . n = k=0 (iv) n # k=m k m = n+m . r n+1 . m+1 Exercise 3.19. Calculate the following sums: n # n (i) . j j j=1 (ii) n # j=0 n 1 . j+1 j Exercise 3.20. Prove the following equalities: (i) (ii) (iii) n # (−1)j+1 n j j j=1 n # (−1)j+1 n j(j + 1) j j=1 n # (−1)j n (j + 1)2 j j=0 =1+ 1 1 1 + +···+ . 2 3 n = 1 1 1 + + ··· + . 2 3 n = 1 n+1 1+ 1 1 + ··· + . 2 n Exercise 3.21. Prove that the following relations hold: n n # # n n j = 0. (−1)j j 2 (−1) j = 0. (ii) (i) j j j=1 j=1 Exercise 3.22. Prove that the number of odd integers in the following list n n n , ,..., 0 1 n is a number which is a power of 2. Exercise 3.23. For each prime number p ≥ 3, prove that the number divisible by p2 . 2p−1 p−1 − 1 is 57 3.3 Infinite descent 3.3 Infinite descent The method of infinite descent was frequently used by Pierre Fermat (1601–1665), therefore it is also known as Fermat’s method. In general, it is used to prove that something does not happen. For instance, Fermat used it to prove that there are no integer solutions of the equation x4 + y 4 = z 2 , with xyz = 0. The theoretical basis of his method rests on the fact that there is no such thing as an infinite decreasing collection of positive integers. In other words, we cannot find an infinite collection of positive integers such that n1 > n2 > n3 > · · · . There are two ways to use this idea in order to prove a statement. The first is to start with a statement P(n1 ) which we suppose is true. If from this statement, we can find a positive integer number n2 < n1 such that P(n2 ) is valid and if from this last statement we can find a positive integer number n3 < n2 such that in turn P(n3 ) is valid, and so on and so forth, then an infinite number of positive integers is generated satisfying n1 > n2 > n3 > · · · , but this is not possible, so P(n1 ) is not true. Let us see an example in order to illustrate this method. √ Example 3.3.1. The number 2 is not a rational number. √ √ 1 Suppose that 2 is a rational number, then 2 = m n1 , with m1 and n1 positive √ 1 , we have that integers. Since 2 + 1 = √2−1 √ 2+1= m1 n1 n1 1 , = m1 − n 1 −1 therefore √ 2= 2n1 − m1 n1 −1= . m1 − n 1 m1 − n 1 √ √ 1 Since 1 < 2 < 2, substituting the suppose rational value 2 leads to 1 < m n1 < 2, hence n1 < m1 < 2n1 . From here we have that, 2n1 − m1 > 0 and m1 − n1 > 0. Then, if we define m2 = 2n1 − m1 and n2 = m1 − n1 , we have√that m2 < m1 m2 1 and n2 < n1 , since n1 < m1 and m1 < 2n1 , respectively. Then, 2 = m n1 = n2 , with m2 < m1 and n2 < n1 . Continuing this process we can generate an infinite number of positive integers mi and ni such that √ m1 m2 m3 2= = = = ··· , n1 n2 n3 with m1 >√m2 > m3 > · · · and n1 > n2 > n3 > · · · , but this is not possible. Therefore, 2 is not a rational number. Example 3.3.2. Find all the pairs of positive integers a, b that satisfy the equation a2 − 2b2 = 0. Suppose that there exist positive integers a1 , b1 such that a21 − 2b21 = 0. (3.8) 58 Chapter 3. Induction Principle This implies that a1 is an even number, that is, a1 = 2a2 for some positive integer a2 . Then, since (2a2 )2 − 2b21 = 0, it follows that 2a22 − b21 = 0. Therefore b1 is even, that is, b1 = 2b2 with b2 a positive integer. Substitution in the last equation leads to 2a22 − (2b2 )2 = 0, or a22 − 2b22 = 0. (3.9) This implies that a2 and b2 is another pair of positive integers satisfying equation (3.8). Since a1 = 2a2 , we have that a1 > a2 . Moreover, the above equations imply that a1 > b1 > a2 > b2 . Equation (3.9) proves that a2 is even, that is, a2 = 2a3 for some positive integer a3 . Repeating the above arguments we obtain an infinite sequence of natural numbers in which each term is smaller than the previous one, that is, a1 > b 1 > a2 > b 2 > a3 > b 3 > · · · . However, this sequence cannot exist. Therefore, there is no pair of natural√numbers that satisfy the equation (3.8). Note that this example proves also that 2 is not a rational number. The other way to use the infinite descent method has a more positive character. It can be used to show that a series of propositions P(a) are valid, where a is an element of a set A ⊂ N. To do this we use the following argument: suppose that P(a) is not valid for some a ∈ A and define the set B = {a ∈ A | P(a) is not true}. Since B = ∅, in B there is a first element, say b. Now, using the hypothesis of the problem, we can find a positive integer number c < b such that P(c) is not valid. This leads to a contradiction since b was the minimum element of B. Then, P(a) has to be true for all a ∈ A. Example 3.3.3 (Putnam, 1973). Let a1 , a2 , . . . , a2n+1 be integers such that if we take one out, then the remaining numbers can be divided in two sets of n integers which have the same sum. Prove that all numbers are equal. We can suppose that a1 ≤ a2 ≤ · · · ≤ a2n+1 . If we subtract the smallest number from all these numbers, the new numbers also satisfy the inequality and the conditions of the problem. Then, without loss of generality, we can suppose that a1 = 0. The sum of the 2n remaining numbers different from a1 satisfies the condition of being congruent to 0 modulo 2. Now, let us see that if we choose any two numbers the pair will have the same parity. Let ai and aj be any two such numbers and S = a1 + · · · + a2n+1 . Since S − ai ≡ S − aj ≡ 0 mod 2, we have that ai ≡ aj mod 2. 59 3.3 Infinite descent If we divide by 2 all the numbers, the new collection has the same properties. Using the same arguments we can conclude that a1 ≡ a2 ≡ · · · ≡ a2n+1 ≡ 0 mod 22 . We can continue this argument to conclude that a1 ≡ a2 ≡ · · · ≡ a2n+1 ≡ 0 mod 2k , for each k ≥ 1, but this is possible only in the case in which all the numbers are equal to zero, and therefore the original numbers are equal. Example 3.3.4. Let ABC be an acute triangle. Let A1 be the foot of the altitude from A, A2 the foot of the altitude from A1 over CA, A3 the foot of the altitude from A2 over AB, A4 the foot of the altitude from A3 over CB, and so on and so forth. Prove that all the Ai ’s are different. A A3 B A4 A1 A5 A2 C Observe that each An is a point in one of the sides of the triangle, that is, it cannot be in the extension of the side and it cannot be a vertex of the triangle. This follows from the fact that if An is in the extension of one side of the triangle, then the triangle is obtuse, and if it is a vertex, then the triangle is either obtuse or a right triangle. Note also that An and An+1 do not coincide, because they belong to different sides of the triangle and they are not vertices. Suppose now that An coincides with Am , with n < m, and suppose also that n is the smallest index with the property that An coincides with some Am . We now see that n = 1, since otherwise we would have that An−1 coincides with Am−1 , and therefore n does not satisfy the property of being the smallest index n such that An = Am , for some m > n. Now if A1 coincides with Am , with m ≥ 3, then Am−1 has to be the vertex A, but we already saw that no Am is vertex of a triangle. Then, A1 cannot coincide with any Am . 60 Chapter 3. Induction Principle Exercise 3.24 (Hungary, 2000). Find the prime numbers p for which it is impossible to find integers a, b and n, with n positive, such that pn = a3 + b3 . Exercise 3.25. The equation x2 + y 2 + z 2 = 2xyz has no integer solutions except when x = y = z = 0. Exercise 3.26. Find all the pairs of positive integers (a, b) such that ab + a + b divides a2 + b2 + 1. Exercise 3.27. Prove that for n = 4, it does not exist a regular polygon with n sides such that the vertices are points with integer coordinates. 3.4 Erroneous induction proofs In this section we present some examples showing the necessity of verifying all the steps required in a proof which uses the induction mathematical principle. Sometimes, if we miss proving one detail this can lead us to some absurd situations, as we will see. Example 3.4.1. All non-negative powers of 2 are equal to 1. The induction basis is n = 0. The statement is true because 20 = 1. Suppose this is true for all k ≤ n, that is, 20 = 21 = · · · = 2n = 1. Let us now verify the statement for 2n+1 . Follow the identities and see that 2n+1 = 22n 2n · 2n 1·1 = 1. = = 2n−1 2n−1 1 Then the proof is complete. Error. The inductive step is not valid for n = 0, that is, P (0) ⇒ P (1) is a false statement. Example 3.4.2. All integers greater than or equal to 2 are even. The induction base for n = 2 is clearly valid, since 2 is even. Suppose that for each integer number k with 2 ≤ k ≤ n, the statement is true, that is, such numbers k are even. Let us see now that n + 1 is also an even number. We write n + 1 as n + 1 = k1 + k2 , with k1 , k2 ≤ n. By the induction hypothesis k1 and k2 are even numbers, then the sum n + 1 is also even. This completes the proof. Error. The inductive step is erroneous. Numbers k1 and k2 have to be greater than or equal to 2. But this is not always true, for example, for n = 3. Also, we can justify that in the inductive step we require two previous numbers satisfying the statement. But it turns out that the statement is not valid for the first two numbers, 2 and 3, since 3 is not an even number. 61 3.4 Erroneous induction proofs Example 3.4.3. Consider the statement In : n(n + 1) is odd for all positive integers n. Where is the mistake in the following proof ? Suppose that the statement In is valid for n and we will show that it is true for n + 1. We start from the identity (n + 1)(n + 2) = n(n + 1) + 2(n + 1). On the right-hand side of the identity we have, by the induction hypothesis, that n(n + 1) is odd and, if we add to this last number the even number 2(n + 1), we have that n(n + 1) + 2(n + 1) is also odd, and therefore the statement In+1 is also valid. Error. We have not verified the induction basis. Example 3.4.4. Consider the following statement: Rn : if there are n straight lines, not two of them parallel, then all the lines have a common point. Where is the error in the following proof ? The statement R1 is true. Also R2 is true, since two non-parallel straight lines meet in one point. Suppose the statement valid for n − 1 straight lines, and consider now n straight lines l1 , l2 , . . . , ln , where no two of them are parallel. By the induction hypothesis the n − 1 straight lines l1 , . . . , ln−1 have a common point, call it P . Now, instead of taking out the straight line ln , take out the line ln−1 . Then, by the induction hypothesis, the n − 1 straight lines l1 , . . . , ln−2 , ln have a common point, call it Q. But l1 and l2 have just one common point, then P = Q. Then the n straight lines l1 , . . . , ln have P as a common point. Error. The induction basis has to be proved for n = 3, but the statement R3 is false. Also note that the proof of the inductive step from R2 to R3 does not hold since l2 is eliminated in the second part. Exercise 3.28. Consider the statement: Every function defined on a finite set is constant. Find the error in the following proof. Let f : A → B be a function defined on a finite set A. We will perform the proof by induction over n, the number of elements of the set A. If n = 1, it is clear that f is constant. Suppose that the statement is valid for all the functions defined on sets with n elements and let f be a function defined on a set A = {a1 , . . . , an+1 }, with n + 1 elements. Consider C = A  {an+1 }, which is a set with n elements, by the induction hypothesis, the function f restricted to C is constant. If we now 62 Chapter 3. Induction Principle consider D = A  {an } , which also has n elements, the function f restricted to D is constant, but these two constants coincide with f (a1 ). Then the function f is constant in all the set A and therefore the proof is complete. Exercise 3.29. Consider the statement: For any n ≥ 2, if there are n coins, one of which is false and lighter than the other coins, then the false coin can be identified in at most 4 weighing of the coins in a balance with two plates. Find the error in the following proof. Of course, if we just have 2 coins, weighing the coins just once is enough to identify the false coin, we just put one in each plate of the balance and the plate that raises up has the false coin. Suppose that the result is valid for k coins and consider k + 1 coins, where just one of them is lighter than the others. Now leave one coin out, if in the rest k coins we cannot identify the false coin weighing 4 times we are done; the false coin is the one we took out, otherwise weighing 4 times we found the false coin among the k coins. Exercise 3.30. Consider the statement: Every non-negative integer is equal to 0. Find the error in the following proof. For n ≥ 0, let P(n) be the statement: “n = 0”. Of course P(0) is true. Suppose that for k ≥ 0 the statements P(0), P(1), . . . , P(k) are true. The veracity of P(k + 1) follows from the fact that (k + 1) = k + 1, because if P(k) and P(1) are true, “k = 0”, “1 = 0”, then k + 1 = 0 + 0 = 0 and therefore P(k + 1) is true. By the strong induction principle “n = 0” for all n ≥ 0. Chapter 4 Quadratic and Cubic Polynomials 4.1 Definition and properties Consider an expression of the form P (x) = a3 x3 + a2 x2 + a1 x + a0 , where a0 , a1 , a2 and a3 are constant numbers. We say that P (x) is a cubic polynomial or a polynomial of degree 3 in the variable x if a3 = 0; if a3 = 0 and a2 = 0 we say that P (x) is a quadratic polynomial or a polynomial of degree 2; in case that a3 = a2 = 0 and a1 = 0, we say that P (x) is a linear polynomial or a polynomial of degree 1; finally, if a3 = a2 = a1 = 0 and a0 = 0, we say that P (x) is a constant polynomial and its degree is 0. The degree of the polynomial P (x) is denoted by deg(P ). The constants a3 , a2 , a1 and a0 are called the coefficients of the polynomial. A polynomial with all the coefficients equal to zero is known as the zero polynomial12 . If in a polynomial, the coefficient of the highest power of x is 1, we say that the polynomial is a monic polynomial. We say that a3 x3 + a2 x2 + a1 x + a0 and b3 x3 + b2 x2 + b1 x + b0 , are two equal polynomials if ai = bi , for i = 0, 1, 2, 3. We can evaluate the polynomials by replacing the variable x by a number t, the value of the polynomial P (x), in x = t, is P (t). A zero of a polynomial P (x) is a number r such that P (r) = 0. We also say that r is a root of the polynomial or a solution of the equation P (x) = 0. If the coefficients of a polynomial P (x) are integers, we say that P (x) is a polynomial over the integers or a polynomial with integer coefficients; similarly, if the coefficients are rational numbers, we say that the polynomial is a polynomial over the rationals, etc. 12 In this book, the zero polynomial has no degree. © Springer Internationl Publishing Switzerland 2015 R.B. Manfrino et al, Topics in Algebra and Analysis, DOI 10.1007/978-3-319-11946-5_4 63 64 Chapter 4. Quadratic and Cubic Polynomials In many aspects, the polynomials are like the integers, they can be added, subtracted, multiplied and divided. To see how this is done consider the polynomials P (x) = a0 + a1 x + a2 x2 + a3 x3 , Q(x) = b0 + b1 x + b2 x2 + b3 x3 . We define the sum of polynomials as (P + Q)(x) = P (x) + Q(x) = (a0 + b0 ) + (a1 + b1 )x + (a2 + b2 )x2 + (a3 + b3 )x3 ; the subtraction as (P − Q)(x) = P (x) − Q(x) = (a0 − b0 ) + (a1 − b1 )x + (a2 − b2 )x2 + (a3 − b3 )x3 , and the product of a polynomial by a constant c as (cP )(x) = cP (x) = ca0 + ca1 x + ca2 x2 + ca3 x3 . The product of two polynomials is defined as (P Q)(x) = (P (x))(Q(x)) = a0 b0 + (a0 b1 + a1 b0 )x + (a0 b2 + a1 b1 + a2 b0 )x2 + (a0 b3 + a1 b2 + a2 b1 + a3 b0 )x3 + (a1 b3 + a2 b2 + a3 b1 )x4 + (a2 b3 + a3 b2 )x5 + a3 b3 x6 . Finally, we define the polynomial division. Given the above two polynomials, with the following restrictions P (x) = a3 x3 + a2 x2 + a1 x + a0 3 2 Q(x) = b3 x + b2 x + b1 x + b0 , with a3 = 0, with some coefficient different from zero, there are always S(x) and R(x) such that P (x) = S(x)Q(x) + R(x), with deg(R) < deg(Q) or R(x) = 0. We call the polynomial S(x) the quotient and R(x) the remainder of the division of P (x) by Q(x). If R(x) = 0, we say that Q(x) divides (exactly) P (x) and we write Q(x)|P (x). In the next section, we will see that a number a is a zero of a polynomial P (x) if and only if x − a divides P (x). A polynomial H(x) is the greatest common divisor of P (x) and Q(x) if and only if 1. H(x) divides both P (x) and Q(x), 2. If K(x) is any other polynomial which divides both P (x) and Q(x) then K(x) also divides H(x). It can be proved that H(x) is unique up to a multiplication by a constant number. 65 4.1 Definition and properties 4.1.1 Vieta’s formulas (a) If a monic polynomial P (x) = x2 + px + q has roots a and b, then x2 + px + q = (x − a)(x − b) = x2 − (a + b)x + ab, comparing the coefficients, it follows that p = −(a + b) and q = ab. (4.1) (b) If a monic polynomial P (x) = x3 + px2 + qx + r has roots a, b and c, then x3 + px2 + qx + r = (x − a)(x − b)(x − c) = x3 − (a + b + c)x2 + (ab + bc + ca)x − abc and comparing the coefficients we have that p = −(a + b + c), q = ab + bc + ca, r = −abc. (4.2) Formulas (4.1) and (4.2) are known as Vieta’s formulas. Let us see the following examples. Example 4.1.1 (USSR, 1986). The roots of the polynomial x2 + ax + b + 1 = 0 are natural numbers. Prove that a2 + b2 is not a prime number. If r and s are roots of the polynomial, formula (4.1), assures us that r+s = −a and rs = b + 1. Then, a and b are integers and a2 + b2 = (r + s)2 + (rs − 1)2 = (r2 + 1)(s2 + 1) is the product of two numbers greater than 1. Example 4.1.2 (Germany, 1970). Let p and q be real numbers, with p = 0, and let 3 2 a, b, c be roots of the polynomial  1 1px 1− px + qx + q. Prove that (a + b + c) a + b + c = −1. Since px3 − px2 + qx + q = p(x3 − x2 + pq x + pq ) = p(x − a)(x − b)(x − c), by formula (4.2), we have that (a + b + c) 1 1 1 + + a b c = (a + b + c) ab + bc + ca abc = q p − pq = −1. The generalization of Vieta’s formulas for polynomials of degree greater than 3, will be presented in Chapter 8, dedicated to the theory of polynomials. Exercise 4.1. Find all solutions of m2 − 3m + 1 = n2 + n − 1, where m and n are positive integers. Exercise 4.2. Let P (x) = ax2 + bx + c be a quadratic polynomial with real coefficients. Suppose that P (−1), P (0) and P (1) are integers. Prove that P (n) is an integer number for every integer number n. 66 Chapter 4. Quadratic and Cubic Polynomials Exercise 4.3. If a, b, c, p and q are integers, with q = 0, (p, q) = 1 and of the equation ax2 + bx + c = 0, prove that p divides c and q divides a. p q a root Exercise 4.4. Let a, b, c be real numbers, with a and c non-zero. Let α and β be the roots of the polynomial ax2 + bx + c and let α′ and β ′ be the roots of the polynomial cx2 +bx+a. Prove that if α, β, α′ , β ′ are positive numbers then (α+β)(α′ +β ′ ) ≥ 4. Exercise 4.5. Find all real numbers a such that the sum of the squares of the roots of P (x) = x2 − (a − 2)x − a − 1 is a minimum. Exercise 4.6. If p, q and r are solutions of the equation x3 − 7x2 + 3x + 1 = 0, find the value of 1p + q1 + r1 . Exercise 4.7. The solutions of the equation x3 + bx2 + cx + d = 0 are p, q and r. Find a quadratic equation with roots p2 + q 2 + r2 and p + q + r, in terms of b, c and d. Exercise 4.8. For which positive real numbers m, the roots x1 and x2 of the equation 2m − 1 m2 − 3 = 0, x+ x2 − 2 2 satisfy the condition x1 = x2 − 21 ? Exercise 4.9 (Kangaroo, 2003). Let P (x) be a polynomial such that P (x2 + 1) = x4 + 4x2 . Find P (x2 − 1). Exercise 4.10. The natural numbers a, b, c and d satisfy that a3 + b3 = c3 + d3 and a + b = c + d. Prove that two of these numbers are equal. 4.2 Roots If we divide a degree 3 polynomial P (x) by x − a we get P (x) = (x − a)Q(x) + r, with r∈R and deg(Q) = 2. Let x = a, then it follows that P (a) = r, therefore P (x) = (x − a)Q(x) + P (a). (4.3) It follows, from equation (4.3), that P (a) = 0 if and only if P (x) = (x − a)Q(x), for some polynomial Q(x). This result is known as the factor theorem. (4.4) 67 4.2 Roots If a1 and a2 are two different zeros of P (x), then, by equation (4.4), it follows that P (x) = (x − a1 )Q(x). Since, P (a2 ) = (a2 − a1 )Q(a2 ) = 0 and a2 = a1 , then Q(a2 ) = 0, hence Q(x) = (x − a2 )Q1 (x), for some polynomial Q1 (x). Then, P (x) = (x − a1 )(x − a2 )Q1 (x) with deg(Q1 ) = 1. If deg(P ) = 3 and P (ai ) = 0, for a1 , a2 , a3 , then P (x) = c(x − a1 )(x − a2 )(x − a3 ) with c ∈ R. If there exists m ∈ N and a polynomial Q(x) such that P (x) = (x − a)m Q(x) with Q(a) = 0, (4.5) we say that the multiplicity of the root a is m. Example 4.2.1. If P (x) = ax2 + bx + c is a quadratic polynomial, then its roots are √ √ −b − b2 − 4ac −b + b2 − 4ac and . (4.6) 2a 2a Since a = 0, we can rewrite the polynomial P (x) as b c P (x) = ax2 + bx + c = a x2 + x + a a 2 2 b 2bx b 4c + 2− 2+ = a x2 + 2a 4a 4a 4a . 2  1  2 b − 2 b − 4ac =a x+ 2a 4a ⎡ 2 ⎤ √ 2 2 − 4ac b b ⎦ = a⎣ x + − 2a 2a .. √ √ b b2 − 4ac b2 − 4ac b x+ =a x+ − + 2a 2a 2a 2a  .  . √ √ −b + b2 − 4ac −b − b2 − 4ac =a x− x− . 2a 2a Then, −b + √ b2 − 4ac 2a and −b − √ b2 − 4ac 2a are its roots. The number ∆ = b2 − 4ac is called the discriminant of the quadratic polynomial P (x) = ax2 + bx + c. 68 Chapter 4. Quadratic and Cubic Polynomials Observation 4.2.2. If the discriminant, b2 −4ac of the quadratic polynomial P (x) = ax2 + bx + c is zero, the polynomial P (x) can be written as a constant multiplied by the square of a linear polynomial. In this case the polynomial P (x) has only b . one real root, which is equal to − 2a In the case where the discriminant is greater than zero, that is, b2 − 4ac > 0 then the polynomial P (x) has two different real roots. Finally, when the discriminant is negative, there are two distinct complex roots. This case will be analyzed in Chapter 5. Summarizing, if r and s are the roots of P (x), then the discriminant is zero if and only if r = s. In case the discriminant is different from zero, then r =  s, and P (x) = a(x − r)(x − s). Now, we shall see what is the geometric meaning of the discriminant of a quadratic polynomial. Remember that P (x) = ax2 + bx + c can be written as P (x) = a - b x+ 2a 2 .  1  2 b − 2 b − 4ac = a x + 4a 2a 2 − ∆ . 4a To construct the graph of the previous equation, that is, to locate the set of pairs of points (x, y) = (x, P (x)) in the Cartesian plane, we let y = P (x) and obtain the equation y+ ∆ b =a x+ 4a 2a 2 , (4.7)  b  ∆ which represents the parabola with vertex at the point − 2a , − 4a , where the sign of the coefficient a determines if the parabola opens up (a > 0), or down (a < 0). In fact, the equation (4.7) tells us much more about the quadratic polynomial P (x). Suppose that a > 0, that is, the parabola opens up. The second coordinate ∆ , is positive if and only if −∆ > 0, that is, if the discriminant of of the vertex, − 4a P (x) is negative, which means that the graph of the parabola does not intersect the X-axis. Then P (x) has no real roots. ∆ is negative, the graph of the parabola intersects the X-axis in two If − 4a points x1 and x2 , which are the roots of P (x). Observe that in this case, we have that ∆ is positive, which agrees with the fact that P (x) has two real roots. b , Moreover, the polynomial P (x) reaches the minimum value at the point x = − 2a ∆ and − 4a is the minimum value of P (x). Making a similar analysis if a < 0, we can conclude that when the parabola opens down it will intersect the X-axis if and only if ∆ ≥ 0. Here, the polynomial b where the maximum value P (x) reaches its maximum value at the point x = − 2a ∆ of P (x) is − 4a . 69 4.2 Roots ∆>0 a>0 x1  −b ∆ 2a , − 4a x2  −b ∆ 2a , − 4a  ∆<0 a<0  When the graph of the parabola is tangent to the X-axis, we are in the case in which the discriminant is zero, that is, when both roots are equal. As an application for the quadratic polynomial theory we can prove the following inequality. Example 4.2.3 (Cauchy–Schwarz inequality). If a1 , . . . , an and b1 , . . . , bn are real numbers, it follows that  n   n 1/2 1/2  n #  # #   2 2 ai b i  ≤ . bi ai    i=1 i=1 i=1 *n The expression P (x) = i=1 (ai x + bi )2 is a quadratic polynomial in x. Since P (x) ≥ 0, we have that P (x) cannot have two different real roots, therefore the discriminant cannot be positive. Now, in order to calculate the discriminant of this polynomial, we expand each term of the sum (ai x + bi )2 = a2i x2 + 2ai bi x + b2i , from where the polynomial takes the form  n    n   n # # # 2 2 2 P (x) = bi , ai b i x + ai x + 2 i=1 i=1 i=1 therefore, the discriminant is 4  n # i=1 ai b i 2 −4  n # i=1 a2i  n # i=1 b2i  ≤ 0. Rewriting the above expression and taking the square root, we obtain the desired result, that is,  n   n 1/2 1/2  n #  # #   2 2 ai b i  ≤ bi ai .    i=1 i=1 i=1 70 Chapter 4. Quadratic and Cubic Polynomials Note that the equality holds if the discriminant is zero, that is, when the polynoi mial has just one real root. Observe that this holds if x = −b ai for all i = 1, 2, . . . , n, −bi and this means ai is constant. It is also possible to prove the inequality between the geometric mean and the arithmetic mean of two numbers, by analyzing the discriminant of a certain quadratic polynomial. √ Example 4.2.4. For a, b ≥ 0, it follows that a+b ab and the equality holds if 2 ≥ and only if a = b. √ √ Consider the quadratic polynomial P (x) = (x − a)(x − b), which has two real roots, so$ its discriminant positive or zero. But the discriminant of $is √ % √ % √ √ √ 2 P (x) = (x − a) x − b = x − a + b x + ab is $√ √ √ %2 √ a + b − 4 ab = a + b − 2 ab. √ Since the discriminant is positive or zero, then a + b ≥ 2 ab. Also, the equality √ √ holds if and only if a = b and then a = b. Example 4.2.5. The following construction dates from the Greek period and it is the procedure to find geometrically what in modern terms are the roots of a quadratic polynomial x2 + bx − c2 , with b and c positive numbers. Construct first an isosceles triangle OP Q with base P Q of length b, and the altitude from O of length c. We draw a circumference through the vertices P and Q of the triangle and with center O. O B A c C P b Q D Let AB be the diameter of the circumference parallel to P Q and construct the rectangle ABCD. The positive root of x2 + bx − c2 is equal to the length of the segment CP and the other root (the negative root) is equal to the negative length of the segment CQ. Let us see how to prove this statement. By Vieta’s formulas (4.1) it follows that if α and β are the roots of the polynomial, α + β = −b and αβ = −c2 , then we necessarily have a negative root. Let us consider the right triangles BCP 71 4.2 Roots and QCB. Since ∠P BC = ∠BQC, these triangles are similar. Then we have that CP · CQ = BC 2 = c2 , therefore CP (−CQ) = −c2 . On the other hand, since CQ = CP + b, we have that CP − CQ = −b. Then, CP and −CQ fulfill Vieta’s relations, therefore these last numbers are the roots of the equation. Example 4.2.6. Factorize a3 + b3 + c3 − 3abc. Let us consider the polynomial P (x) = (x − a)(x − b)(x − c) = x3 − (a + b + c)x2 + (ab + bc + ca)x − abc, with roots a, b and c. That is, a3 − (a + b + c)a2 + (ab + bc + ca)a − abc = 0, b3 − (a + b + c)b2 + (ab + bc + ca)b − abc = 0, c3 − (a + b + c)c2 + (ab + bc + ca)c − abc = 0. Adding these three equalities and factoring a + b + c, we get a3 + b3 + c3 − 3abc = (a + b + c)(a2 + b2 + c2 − ab − bc − ca). (4.8) Remember that the expression a2 + b2 + c2 − ab − bc − ca can be written as " 1! (a − b)2 + (b − c)2 + (c − a)2 . 2 From this we get the following factorization, a3 + b3 + c3 − 3abc = " ! 1 (a + b + c) (a − b)2 + (b − c)2 + (c − a)2 . 2 (4.9) Example 4.2.7 (Czechoslovakia, 1969). Let a, b and c be real numbers such that a + b + c > 0, ab + bc + ca > 0 and abc > 0. Prove that a, b, c are positive. Let us consider the cubic and monic polynomial with roots a, b and c, that is, P (x) = (x − a)(x − b)(x − c) = x3 − (a + b + c)x2 + (ab + bc + ca)x − abc. For x ≤ 0, we have that P (x) < 0, then we can guarantee that the roots are positive. Exercise 4.11. For what values λ the polynomial λx2 + 2x + 1 − equal roots? 1 λ does have two 72 Chapter 4. Quadratic and Cubic Polynomials Exercise 4.12. Let a, b and c be positive real numbers. Is it possible, for each of the following polynomials P (x) = ax2 +bx+c, Q(x) = bx2 +cx+a, R(x) = cx2 +ax+b, to have both real roots? Exercise 4.13. For what integer values of k are the solutions of the equation kx2 − (1 − 2k)x + k − 2 = 0 rational numbers? Exercise 4.14 (Czech-Slovakia, 2006). Find all pairs of integers (a, b) such that a + b is a root of the polynomial x2 + ax + b. Exercise 4.15. Find all integer values of x for which the polynomial x2 − 5x − 1 is a perfect square. Exercise 4.16. Solve, using geometry, the equation x2 + bx − c2 , with b and c positive numbers, using the following construction due to R. Descartes. Draw a circumference with center O and radius 2b . Draw QR, a tangent to the circumference through Q, with QR = c. Let S and T be the points where the straight line through R and O cuts the circumference. R S c O b 2 Q T The quadratic polynomial has one positive root and one negative root. In the figure, the length of the segment RS is equal to the positive root and the negative root is equal to the negative length of the segment RT . Exercise 4.17. Let P (x) = ax2 +bx+c be a quadratic polynomial such that P (x) = x does not have real solutions. Prove that P (P (x)) = x has no real solutions either. Exercise 4.18 (Poland, 2007). Let P (x) be a polynomial with integer coefficients. Prove that if P (x) and P (P (P (x))) have a common root, then they have a common integer root. 73 4.2 Roots Exercise 4.19 (Russia, 2009). Let a, b and c be non-zero real numbers such that ax2 + bx + c > cx, for all real numbers x. Prove that cx2 − bx + a > cx − b, for all real numbers x. Exercise 4.20 (Russia, 2009). Two different real numbers a and b are such that the equation (x2 + 20ax + 10b)(x2 + 20bx + 10a) = 0 has no real solutions. Prove that 20(b − a) cannot be an integer number. Exercise 4.21 (Russia, 2011). Let P (x) be a monic quadratic polynomial such that P (x) and P (P (P (x))) have a common root. Prove that P (0)P (1) = 0. Exercise 4.22. Let a, b, c, d, e and f be positive integers such that they satisfy the relation ab + ac + bc = de + df + ef , and let N = a + b + c + d + e + f . Prove that if N divides abc + def then N is a composite number. Exercise 4.23. Let P (x) and Q(x) be two quadratic polynomials with integer coefficients. If both polynomials have an irrational number as a common zero, prove that one of them is a multiple of the other. Exercise 4.24. Determine if there exist polynomials x2 − b1 x + c1 = 0 and x2 − b2 x + c2 = 0, with b1 , c1 , b2 and c2 different, such that the four roots are b1 , c1 , b2 and c2 . Exercise 4.25. Let a, b and c be real numbers. Prove that at least one of the following equations has a real solution: x2 + (a − b)x + (b − c) = 0, x2 + (b − c)x + (c − a) = 0, x2 + (c − a)x + (a − b) = 0. Exercise 4.26. Let a, b and c be real numbers such that a + b + c = 0. Prove that a5 + b 5 + c5 = 5 a2 + b 2 + c2 2 a3 + b 3 + c3 3 . Exercise 4.27. Let a, b and c be real numbers such that a+b+c = 3, a2 +b2 +c2 = 5, a3 + b3 + c3 = 7. Find a4 + b4 + c4 . Exercise 4.28 (OMCC, 2001). Let a, b and c be real numbers such that the equation ax2 +bx+c = 0 has two different real solutions p1 , p2 and the equation cx2 +bx+a = 0 has two different real solutions q1 , q2 . Also the numbers p1 , q1 , p2 , q2 , in this order, form an arithmetic progression. Prove that a + c = 0. 74 Chapter 4. Quadratic and Cubic Polynomials Exercise 4.29. Let a, b and c be real numbers different from zero, with a+ b + c = 0 and a3 + b3 + c3 = a5 + b5 + c5 . Prove that a2 + b2 + c2 = 65 . Exercise 4.30 (Russia, 2010). Let P (x) be a cubic polynomial with integer coefficients such that there exist different integers a, b, and c such that P (a) = P (b) = P (c) = 2. Prove that no integer number d satisfying P (d) = 3 exists. Chapter 5 Complex Numbers 5.1 Complex numbers and their properties The roots of a quadratic equation are not always real numbers. For instance, the roots of the equation x2 + 2x + 10 = 0, which can be calculated using directly equation (4.6) of Section 4.2, are √ √ −2 + −36 −2 − −36 and . 2 2 √ For this reason it is necessary to consider “imaginary” numbers, like −36. A complex number z is an expression of the form x + iy, where x and y are real numbers, and i2 = −1. At this moment we will not worry about the meaning of i, for the time being we will be interested only in the fact that its square is −1. The real part of z, which will be denoted by Re z, is the number x, and the imaginary part of z, denoted by Im z, is the number y. The set C of all complex numbers x + iy can be identified with the set of points (x, y) in the Cartesian plane, and when this is done, due to this representation it is called the complex plane. The X-axis is called the real axis and the Y -axis is known as the imaginary axis. In order to work with complex numbers, we need the following three definitions: the complex conjugate of z, denoted by z̄, is the complex number x − iy; the module or norm of z, denoted by |z|, is the real number x2 + y 2 , which is the distance from the origin to the point (x, y) representing z. Finally, the argument of z = 0 is the angle between the positive real axis and the line through 0 and z, taken in the counterclockwise direction. The argument of z is denoted by arg z and generally it is assigned a value between 0 and 2π. There is another form to write a complex number z, which is known as the polar form of the complex number z. Let r be the norm of z, r = |z|, and θ = arg z, then z = r(cos θ + i sin θ). © Springer Internationl Publishing Switzerland 2015 R.B. Manfrino et al, Topics in Algebra and Analysis, DOI 10.1007/978-3-319-11946-5_5 75 76 Chapter 5. Complex Numbers Y -axis |z | z = x + iy r= y θ X-axis x z̄ = x − iy The set of complex numbers is similar to the set of real numbers, in the sense that there are two operations that can be applied to its elements, the sum and the product of complex numbers. Let z = x + iy and w = u + iv be two complex numbers. Then the sum of these complex numbers is given by z + w = (x + iy) + (u + iv) = (x + u) + i(y + v), and the product by z · w = (x + iy)(u + iv) = xu + xiv + iyu + i2 yv = (xu − yv) + i(xv + yu). The set of real numbers can be seen as a subset of the complex numbers, if we identify each real number x with the complex number (x, 0). Observe that to number i corresponds the number (0, 1) and that (x, y) = (x, 0) + (0, y) = (x, 0) + (0, 1) · (y, 0), and this is also represented as x + iy, that is, to (x, y) corresponds the complex number x + iy. The operations of sum and product of complex numbers satisfy the same properties that these operations satisfy in the set of real numbers, as being commutative, associative and share also the existence of neutral elements for both operations, these being 0 = (0, 0) and 1 = (1, 0), respectively. The sum of complex numbers is exactly the same operation as the sum of vectors in the Cartesian plane; the operation that can be a novelty is the product of complex numbers. Let us see how to find the inverse of a complex number. For that purpose we need an 77 5.1 Complex numbers and their properties important relation that exists between the norm of a complex number z and its conjugate, that is (5.1) z z̄ = |z|2 . In order to see that, note that if z = x + iy, then z z̄ = (x + iy)(x − iy) = x2 + y 2 = |z|2 . From identity (5.1), if z = 0, tells us that its multiplicative inverse, z1 , is equal to |z|z̄ 2 . Exercise 5.1. Let z, w be two complex numbers. Prove that: (i) z + w = z̄ + w̄, zw = z̄ w̄,  z̄ = z. 1 (ii) |z̄| = |z|, |zw| = |z| |w|,  1z  = |z| , if z = 0. (iii) (iv) (v) (vi) Re z = 12 (z + z̄) ≤ |z|. 1 (z − z̄) ≤ |z|. Im z = 2i |z + w| ≤ |z| + |w|. |z + w|2 + |z − w|2 = 2(|z|2 + |w|2 ).   Exercise 5.2. Find the complex numbers z such that Im z + z1 = 0. Exercise 5.3. If z and w are complex numbers with |z + w| = |z − w| and w = 0, iz then is a real number. w Exercise 5.4. If z and w are complex numbers, prove that: (i) |1 − z̄w|2 − |z − w|2 = (1 + |zw|)2 − (|z| + |w|)2 . 2 2 2 2 (ii) |1 + z̄w| − |z + w| = (1 − |z| )(1 − |w| ). 2 2 Exercise 5.5. If z and w are complex numbers such that (1 + |w| )z = (1 + |z| )w, prove that z = w or z̄w = 1. Exercise 5.6. Let z1 , z2 , z3 be complex numbers such that z1 + z2 + z3 = 0 Prove that z12 + z22 + z32 = 0. and |z1 | = |z2 | = |z3 | = 1. Exercise 5.7. Prove that if z1 , z2 are complex numbers with |z1 | = |z2 | = 1 and z1 +z2 z1 z2 = −1, then 1+z is a real number. 1 z2 Exercise 5.8. Let a, b, c and d be complex numbers with the same norm, and such that a + b + c = d. Prove that d is equal to a, b or c. Now, we present the geometric meaning of the complex product. Let us see two examples that will help us to understand such meaning. First, let us consider the transformation z → iz, that is, x + iy → i(x + iy) = ix − y, which in vectorial form means that the vector (x, y) becomes the vector (−y, x), and note that both vectors are perpendicular. Then, the transformation z → iz corresponds to the 78 Chapter 5. Complex Numbers rotation of the complex plane in the counterclockwise direction, around zero, with angle π2 . Y iz z X i2 z = −z If we apply the previous transformation twice, we obtain z → iz → i(iz) = i2 z = −z, which is a rotation with angle π. The previous examples show that complex multiplication implicitly carries a rotation of the Cartesian plane. If instead of taking the product of a complex number z with i, we take the product with another complex number w, certain rotation appears in a natural way. We will see this now. Let z = x+iy = r(cos θ +i sin θ) and w = u+iv = t(cos φ+i sin φ) be two complex numbers written in polar form. Taking its product, we obtain z · w = r(cos θ + i sin θ) · t(cos φ + i sin φ) = r t(cos θ cos φ + i cos θ sin φ + i sin θ cos φ + i2 sin θ sin φ) = r t (cos θ cos φ − sin θ sin φ + i(cos θ sin φ + sin θ cos φ)) (5.2) = r t (cos(θ + φ) + i(sin θ + φ)) , where we used the sine and cosine formula for the sum of two angles. Thus, arg(z · w) = arg z + arg w, modulo 2π. By identity (5.2), we conclude that geometrically the product of two complex numbers, z and w, is the complex number whose norm is the product of the norms of z and w, and its argument is the sum of the arguments of the same two numbers. Y zw w z θ θ 1 X 79 5.2 Quadratic polynomials, complex coefficients Using formula (5.2) repeatedly, for z = cos θ + i sin θ, we obtain the so-called de Moivre’s formula, where for every integer n we have (cos θ + i sin θ)n = cos nθ + i sin nθ. (5.3) Exercise 5.9. Prove that, for complex numbers a, b and c, the following are equivalent: (i) The points a, b and c are collinear. c−a ∈ R. (ii) b−a (iii) cb̄ − cā − ab̄ ∈ R.    1 a ā    (iv)  1 b b̄  = 0.  1 c c̄  Conclude that the equation of the line through b and c is Im( z−c z−b ) = 0. Exercise 5.10. Find the complex numbers z such that z, i, iz are collinear. Exercise 5.11. Let z, w be two vertices of a square, find the other two vertices in terms of z and w. Exercise 5.12. Prove by induction de Moivre’s formula (5.3). Exercise 5.13. Prove that if z + integer n ≥ 1. 1 z = 2 cos θ, then z n + 1 zn = 2 cos nθ, for every 5.2 Quadratic polynomials with complex coefficients Now, let us see that it is possible to obtain explicitly the roots of every polynomial of degree 2 with complex coefficients. In order to do that, first we need to be able to solve the following type of equations (x + iy)2 = a + ib, (5.4) where a and b are real numbers. That is, we need to find the values of x and y, which means to find the square root of a + ib. First, observe that since (x + iy)2 = x2 − y 2 + i2xy, equation (5.4) is equivalent to a = x2 − y 2 and b = 2xy, (5.5) 80 Chapter 5. Complex Numbers the second equation implies that the sign of b determines if the signs of x and y are equal or different. Now, taking the norm on both sides of equation (5.4), it follows that (5.6) x2 + y 2 = a2 + b2 . Adding this last equation to the first equality of (5.5), we obtain 2x2 = a + √ a2 + b2 , that is, x2 = 1$ a+ 2 a2 + b 2 % or x = ± 1$ a+ 2 % a2 + b 2 .   √  Similarly, we obtain y = ± 12 −a + a2 + b2 . Note that x, y are well defined √ √ since a + a2 + b2 ≥ a + |a| ≥ 0 and −a + a2 + b2 ≥ −a + |a| ≥ 0. Once we know how to calculate the square roots of complex numbers, we can calculate the zeros of every quadratic polynomial. In fact, if we consider the quadratic polynomial P (z) = az 2 + bz + c, with a, b, c ∈ C, then, by using formula (4.6) we can always find both zeros of the polynomial. For instance, if the discriminant b2 − 4ac is a complex number, we can obtain its square root and then calculate the two roots, which are now complex numbers too. As a consequence, every quadratic polynomial with complex coefficients has at least one complex root, and therefore can be written as the product of two linear factors with complex coefficients. Observe also that if P (z) is a polynomial with real coefficients, then for every complex number w we have that P (w̄) = P (w), since the conjugate of each coefficient is the same number, just because it is a real number. Thus, if w is a zero of P (z), then w̄ is also a zero. Hence, for a polynomial with real coefficients, the complex roots, if they exist, appear in conjugate pairs. Example 5.2.1. Solve the equation z 8 + 4z 6 − 10z 4 + 4z 2 + 1 = 0. Dividing the expression by z 4 , we obtain that z4 + 1 z4 + 4 z2 + 1 z2 − 10 = z+ 1 z 4 − 6 − 10 = 0. (5.7) Making the change of variable u = z + 1z we get equation u4 = 16, which has solutions u1 = 2, u2 = −2, u3 = 2i, u4 = −2i. From equation u = z + 1z we have that z = u2 ± u2 /4 − 1. By substituting the √ four values of u√we obtain the eight solutions: z1,2 = 1, z3,4 = −1, z5,6 = i(1 ± 2), z7,8 = −i(1 ± 2). Example 5.2.2 (Romania, 1999). Let p and q be complex numbers with q = 0. Prove that if the roots of the quadratic polynomial x2 + px + q 2 = 0 have the same norm, then pq is a real number. 81 5.3 The fundamental theorem of algebra Let x1 , x2 be the roots of the given equation and let r = |x1 | = |x2 |. Then (x1 + x2 )2 x1 x2 x1 x̄2 x2 x̄1 2 p2 = = + + 2 = 2 + 2 + 2 = 2 + 2 Re(x1 x̄2 ) q2 x1 x2 x2 x1 r r r is a real number. Moreover, Re(x1 x̄2 ) ≥ −|x1 x̄2 | = −r2 and then Thus, pq is a real number. p2 q2 ≥ 0. Exercise 5.14. Find all the complex numbers z such that |z| = 1 and |z 2 + z̄ 2 | = 1. Exercise 5.15. Let a, b, c be complex numbers with |a| = |b| = |c| = 0. (i) Prove that if a root of the equation az 2 + bz + c = 0 has norm 1, then b2 − ac = 0. (ii) If each of the equations az 2 + bz + c = 0 and bz 2 + cz + a = 0 have a root of norm 1, then |a − b| = |b − c| = |c − a|. Exercise 5.16 (Romania, 2003). If the complex numbers z1 , z2 , z3 , z4 , z5 all have *5 *5 2 norm 1 and satisfy i=1 zi = i=1 zi = 0, prove that these numbers are the vertices of a regular pentagon. Exercise 5.17 (Romania, 2007). Let a, b, c be complex numbers of norm 1. Prove that there exist numbers α, β, γ ∈ {−1, 1}, such that |αa + βb + γc| ≤ 1. Exercise 5.18 (Romania, 2008). Let a, b, c be complex numbers that satisfy a |bc|+ √ b |ca| + c |ab| = 0. Prove that |(a − b)(b − c)(c − a)| ≥ 3 3 |abc|. Exercise 5.19 (Romania, 2009). Find the complex numbers a, b, c, all with the same norm, such that a + b + c = abc = 1. 5.3 The fundamental theorem of algebra One of the goals of polynomial theory is to find the roots or the factors in which a polynomial can be decomposed. In this spirit we have the result known as the fundamental theorem of algebra. Theorem 5.3.1 (The fundamental theorem of algebra). Every polynomial P (z) = an z n + an−1 z n−1 + · · · + a1 z + a0 , where n ≥ 1, ai ∈ C and an = 0, has at least one root in C. For the proof of this theorem, see Section 5.5. Let us first remember the factor theorem. 82 Chapter 5. Complex Numbers Theorem 5.3.2 (Factor theorem). If a is a zero of a polynomial P (z), then z − a is a factor of P (z). This theorem has the following interpretation: to know the zeros of the polynomial is to know the polynomial. This is made precise in the following result. Corollary 5.3.3. Every polynomial P (z) of degree n with coefficients in C, can be written in the form P (z) = c(z − z1 )(z − z2 ) · · · (z − zn ), zi ∈ C, c ∈ C. That is, the polynomial has exactly n complex roots. The numbers z1 , z2 , . . ., zn are not necessarily distinct. If it is difficult to find the roots of the polynomial, then it is a good idea to find another polynomial for which it could be easier to find the roots. Let us see an example. Example 5.3.4 (USA, 1975). Let P (x) be a polynomial of degree n such that P (k) = k/(k + 1), for k = 0, 1, 2, . . . , n. Find P (n + 1). The condition P (k) = k/(k + 1) does not say anything about the roots of P (x). Then, we can consider the polynomial of degree n + 1, Q(x) = (x + 1)P (x) − x. Clearly, the roots of Q(x) are 0, 1, 2, . . . , n, therefore we can write (x + 1)P (x) − x = Cx(x − 1)(x − 2) · · · (x − n), where C is a constant that is going to be determined. Evaluating in x = −1, we n+1 obtain that 1 = C(−1)(−2)(−3) · · · (−n)(−(n + 1)), from where C = (−1) (n+1)! . Finally, if x = n + 1, we get (n + 2)P (n + 1) − n − 1 = (−1)n+1 , hence P (n + 1) = n+1+(−1)n+1 . n+2 5.4 Roots of unity Using de Moivre’s formula (5.3) it follows immediately that the polynomial z n −1 = 2π 0 has roots 1, w, w2 , . . . , wn−1 , where w = cos 2π n +i sin n . These roots are known as the nth roots of unity and in the complex plane they can be identified as the vertices of a regular n-sided polygon inscribed in the unit circle with center at the origin13 . By the factor Theorem 5.3.2, we have the decomposition z n − 1 = (z − 1)(z − w)(z − w2 ) · · · (z − wn−1 ). 13 More generally, the equation z n − a = 0, for any complex number a = 0, has n different  complex solutions called the nth roots of a. These solutions are n |a|w j , for j = 0, 1, . . . , n − 1 + i sin 2π . and w = cos 2π n n 83 5.4 Roots of unity For instance, for the case n = 3, the roots of z 3 − 1 = (z − 1)(z 2 + z + 1) = 0 are 1, √ √ −1−i 3 −1+i 3 1 2 and w = w = w̄ = , and they are known as the cubic roots w= 2 2 of unity. Note that w satisfies w3 = 1 and 1 + w + w2 = 0. 2π We have seen that w = cos 2π n +i sin n generates all the nth roots of unity, that is, 2 n−1 n , w = 1}. We say that one element u ∈ Un is a primitive Un = {w, w , . . . , w root of unity if um = 1 for all positive integers m < n. Now, we can state the following result. Theorem 5.4.1. (a) If n divides q, then any root of z n − 1 = 0 is a root of z q − 1 = 0. (b) The common roots of z m − 1 = 0 and z n − 1 = 0 are the roots of z d − 1 = 0, where d is the greatest common divisor of m and n, which is denoted by d = (m, n). (c) The primitive roots of z n − 1 = 0 are wk , where 0 ≤ k ≤ n and (k, n) = 1. Proof. (a) If q = pn and w is a root of z n − 1, it follows that wq − 1 = wpn − 1 = (wn )p − 1 = (1)p − 1 = 0, then w is root of z q − 1. (b) If w is a root of z m − 1 and z n − 1, and d = (m, n), we have that d = am+bn for some integers a and b. Hence, wd −1 = wam+bn −1 = wam wbn −1 = (wm )a (wn )b − 1 = 1a 1b − 1 = 0, therefore, w is a root of z d − 1. Conversely, since d divides m and n, by property (a), if w is a root of z d − 1 then it is root of z m − 1 and z n − 1. (c) To prove this part, let us see the following lemma. 2π Lemma 5.4.2. Let w = cos 2π n + i sin n . The smallest positive integer m such that n k m (w ) = 1 is m = (k,n) , where (k, n) is the greatest common divisor of k and n. As a consequence of this lemma part (c) follows and wk is a primitive root if and only if (k, n) = 1. Proof of the lemma. First, note that ws = 1 if and only if s = an. Let m be the smallest positive integer such that (wk )m = 1, then km = an. If d = (k, n), then k1 dm k1 m k = k1 d and n = n1 d, with (k1 , n1 ) = 1, hence a = km n = n1 d = n1 is an integer. Thus n1 |m, and since wkn1 = wk1 dn1 = wk1 n = 1, it follows that m = n1 . n Finally, we have that n = n1 d = md, therefore m = nd = (k,n) .  Example 5.4.3. Prove the identity sin 2π (n − 1)π n π · sin · · · · · sin = n−1 . n n n 2 Consider the polynomial P (z) = (1−z)n −1, which can be written as wn −1, where 2πk w = 1 − z. The roots of wn = 1 are the nth roots of unity, wk = cos 2πk n + i sin n , for k = 0, 1, . . . , n − 1. Then, the roots of P (z) are zk = 1 − wk . 84 Chapter 5. Complex Numbers Looking at the polynomial P (z), we observe that it can be written as P (z) = z(−n+Q(z)), where Q(z) is a polynomial of degree n−1. Hence, if we let P (z) = 0 we see that the roots should satisfy, by Vieta’s formula (8.4) Section 8.3, that 3n−1 3n−1 (−1)n n = k=1 zk , and from this n = k=1 |zk |. Now, we need to calculate |zi |. |zk | = |1 − wk | = 1 − cos 2 2πk n = 1 − 2 cos 2πk n + cos2 = 2 − 2 cos 2πk n = 2πk n 4 sin2 + sin 2πk n + sin2 2πk n πk n = 2 sin 2 πk , n where we used that cos2 x + sin2 x = 1 and 1 − cos(2x) = 2 sin2 x. The identity now follows. Example 5.4.4 (AMC, 2002). Find the number of ordered pairs of real numbers (a, b) such that (a + ib)2002 = a − ib. √ Let z = a + ib, z̄ = a − ib, and |z| = a2 + b2 . The relation given above is equivalent to z 2002 = z̄. Note that |z|2002 = |z 2002 | = |z̄| = |z|, thus |z|(|z|2001 − 1) = 0. Hence, |z| = 0 and therefore (a, b) = (0, 0) or |z| = 1. In the latter case we have that z 2002 = z̄, which is equivalent to z 2003 = z̄ · z = |z|2 = 1. Since the equation z 2003 = 1 has 2003 different solutions, then there are 1 + 2003 = 2004 ordered pairs that satisfy the equation. Exercise 5.20. Solve the equation x4 + x3 + x2 + x + 1 = 0. Exercise 5.21. Find the solutions of the equation x6 + 2x5 + 2x4 + 2x3 + 2x2 + 2x + 1 = 0. Exercise 5.22 (Romania, 2007). Let n be a positive integer. Prove that there exists a complex number with norm 1 that is a solution of the equation z n + z + 1 = 0 if and only if n = 3m + 2, for some positive integer m. Exercise 5.23. If w = 1 is an nth root of unity, prove that: (i) 1 + w + w2 + · · · + wn−1 = 0. (ii) 1 + 2w + 3w2 + · · · + nwn−1 = n w−1 . 5.5 Proof of the fundamental theorem of algebra 85 Exercise 5.24. If w = 1 is a nth root of unity: (i) Prove that (1 − w)(1 − w2 ) . . . (1 − wn−1 ) = n. 1 1 1 + 1−w (ii) Find the value of 1−w 2 + · · · + 1−w n−1 . Exercise 5.25. (i) Prove that if ω = 1 is a cubic root of unity (that is, ω 3 = 1), then for a, b, c ∈ C, it follows that a2 + b2 + c2 − ab − bc − ca = (a + bω + cω 2 )(a + bω 2 + cω). (ii) Use (i) and equation (4.8) to obtain the following identity: a3 + b3 + c3 − 3abc = (a + b + c)(a + bω + cω 2 )(a + bω 2 + cω). (5.8) Exercise 5.26. Two regular polygons are inscribed in the same circle. The first polygon has 1982 sides and the second one has 2973 sides. If the polygons have some vertices in common, how many vertices in common do they have? Exercise 5.27. Find the positive integers n for which x2 +x+1 divides x2n +xn +1. Exercise 5.28. Let S be the set of integers x that can be written in the form x = a3 + b3 + c3 − 3abc, for some integers a, b, c. Prove that if x, y ∈ S , then xy ∈ S . Exercise 5.29 (USA, 1976). If P (x), Q(x), R(x), S(x) are polynomials such that P (x5 ) + xQ(x5 ) + x2 R(x5 ) = (x4 + x3 + x2 + x + 1)S(x), prove that x − 1 is a factor of P (x). Exercise 5.30. Find all the polynomials P (z) of degree at most 2 with coefficients in C, that satisfy P (z)P (−z) = P (z 2 ). 5.5 Proof of the fundamental theorem of algebra ⋆ As we have seen, the fundamental theorem of algebra states that every nonconstant polynomial with complex coefficients has at least one complex root. This theorem includes polynomials with real coefficients. The fundamental theorem of algebra can also be stated saying that every polynomial of degree n, with complex coefficients, has exactly n roots, counting the multiple roots as many times as they appear. The equivalence of both theorems can be proved using repeatedly the factor theorem. 86 Chapter 5. Complex Numbers In spite of the name of the theorem, there are no purely algebraic elementary proofs, since these proofs use the fact that the real numbers are complete and this is not an algebraic concept. This theorem was considered fundamental for algebra when the study of this discipline was concentrated in finding the roots of the polynomial equations with real or complex coefficients. Some proofs of the theorem only show that every non-constant polynomial, with real coefficients, has a complex root. This is sufficient to establish the theorem in the general case, since given a non-constant polynomial P (z) with complex coefficients, the polynomial Q(z) = P (z)P (z̄) has real coefficients and if z is a root of Q(z), then z or its conjugate z̄ is also a root of P (z). The proof that we present here uses a result known as the growth lemma. Lemma 5.5.1 (Growth lemma). Given a polynomial P (z) = an z n + an−1 z n−1 + · · · + a1 z + a0 of degree n ≥ 1, with complex coefficients, there is a real number R > 0 such that if |z| ≥ R then 1 |an ||z n | ≤ |P (z)| ≤ 2|an ||z n |. 2 *n−1 k Proof. Let r(z) = k=0 |ak ||z| . By the triangle inequality, it follows that for every complex number z, |an ||z|n − r(z) ≤ |P (z)| ≤ |an ||z|n + r(z). Now, if |z| ≥ 1 and m < n, it follows that |z|m ≤ |z|n−1 , therefore r(z) ≤ *n−1 M |z| , where M = k=0 |ak |. Taking R =max {1, 2M |an|−1 }, for |z| ≥ R, it follows that n−1 |P (z)| ≤ |an ||z|n + r(z) ≤ |an ||z|n + M |z|n−1 = |z|n−1 (|an ||z| + M ) ≤ |z|n−1 (|an ||z| + |an ||z|) = 2|an ||z|n , where the last inequality follows from the fact that |z| ≥ R ≥ Again, for |z| ≥ R, we have that 2M |an | > M |an | . |P (z)| ≥ |an ||z|n − r(z) ≥ |an ||z|n − M |z|n−1 1 ≥ |an ||z|n − |an ||z|n 2 1 = |an ||z|n , 2 where the last inequality holds if and only if 12 |an ||z|n ≥ M |z|n−1 , which is true if 2M , and the proof is complete.  and only if |z| ≥ |a n| The proof of the fundamental theorem of algebra that we present here is based only on advanced calculus. Calculus provide a very useful result presented as the following lemma, which can be consulted in [17]. 5.5 Proof of the fundamental theorem of algebra 87 Lemma 5.5.2. If f : D → R is a continuous function on D, a closed and bounded subset of R2 , then f attains its minimum and maximum values in points of D. Theorem 5.5.3 (The fundamental theorem of algebra). Every polynomial P (z) = an z n + an−1 z n−1 + · · · + a1 z + a0 , where n ≥ 1, ai ∈ C and an = 0, has at least one root in C. The proof is based on the following two lemmas. Lemma 5.5.4. Let P (z) be a polynomial with complex coefficients. Then |P (z)| attains its minimum value in some point z0 ∈ C.   Proof. Let s = |P (0)| = |a0 | and R1 = max R, n 2s|an |−1 , where the number R is given by the growth lemma. If |z| > R1 , then it follows that |P (z)| ≥ 1 1 1 2s |an ||z n | > |an ||R1n | ≥ |an | = s. 2 2 2 |an | Thus, for all z such that |z| > R1 , it follows that |P (z)| > |P (0)|. In particular, if R2 > R1 , then for every z such that |z| ≥ R2 , the same inequality holds. In this way we have found a closed disk D of radius R2 with center at 0 such that |P (z)| > |P (0)|, for all |z| > R2 . Since |P (z)| is a continuous function with real values, it follows by Lemma 5.5.2 that |P (z)| attains its minimum value in D.  Lemma 5.5.5. Let P (z) be a non-constant polynomial with complex coefficients. If P (z0 ) = 0, then |P (z0 )| is not the minimum value of |P (z)|. Proof. Let P (z) be a non-constant polynomial with complex coefficients, and let z0 be a point such that P (z0 ) = 0. Making the change of variable z + z0 for z moves z0 to the origin, and then we can assume that P (0) = 0. Now, we multiply P (z) by P (0)−1 so that we may assume that P (0) = 1. Then, we must show that 1 is not the minimum value of |P (z)|. Let k be the smallest non-zero power of z in P (z). Then we can assume that P (z) has the form P (z) = 1 + az k + g(z), with g(z) a polynomial of degree greater than k. Let α be a kth root of −a−1 . Then, making one last change of variable, αz by z, we obtain that P (z) has the form P (z) = 1 − z k + z k+1 g(z), for some polynomial g(z). For real positive values of z we obtain, using the triangle inequality, that |P (z)| ≤ |1 − z k | + z k+1 |g(z)|. 88 Chapter 5. Complex Numbers Since z k < 1, for |z| < 1, then |P (z)| ≤ 1 − z k + z k+1 |g(z)| = 1 − z k (1 − z|g(z)|). For z small, z|g(z)| is also small, then we can choose z1 so that z1 |g(z1 )| < 1. It follows that z1k (1 − z1 |g(z1 )|) > 0, and then |P (z1 )| < 1 = |P (0)|, and this finishes the proof.  Using these two lemmas we obtain the proof of the fundamental theorem of algebra. Proof. Let P (z) be a non-constant polynomial with complex coefficients. By Lemma 5.5.4, |P (z)| has a minimum value in some point z0 ∈ C. Then, by Lemma 5.5.5, it follows that |P (z0 )| = 0. Therefore, P (z) has a complex root.  Chapter 6 Functions and Functional Equations 6.1 Functions The concept of function is one of the most important in mathematics. A function is a relation between elements of two sets X and Y , which we denote by f : X → Y , that satisfies: (a) Every element x ∈ X is related to some element y ∈ Y , and we write y = f (x). (b) Every element x ∈ X is related to one and only one element of Y , that is, if f (x) = y1 and f (x) = y2 , then necessarily y1 = y2 . The set X is called the domain, the set Y the codomain and the relation y = f (x) the correspondence rule. We define the image or the range of a function f : X → Y as Img f = {y ∈ Y | there exists x ∈ X with f (x) = y}. We say that f and g are equal functions if they have the same domain X, the same correspondence rule (f (x) = g(x), for all x ∈ X) and the same codomain. We define the graph of a function f : X → Y as Γ(f ) = {(x, y) ∈ X × Y | y = f (x)}, where X × Y = {(x, y) | x ∈ X, y ∈ Y } is the Cartesian product of X and Y . Two simple but important functions are the constant function and the identity function, which are defined as follows: if f : X → Y is such that f (x) = b for all x ∈ X, with b ∈ Y fixed, then f is called the constant function equal to b, while the image is the set with only one element {b}; the identity function has the same domain and codomain, that is, Id : X → X, and it is defined as Id(x) = x for all x ∈ X. For functions f : R → R, the geometric representation of the graph in the Cartesian plane is useful. For instance, the geometric representation of the constant function b and the identity function IdR are the following: © Springer Internationl Publishing Switzerland 2015 R.B. Manfrino et al, Topics in Algebra and Analysis, DOI 10.1007/978-3-319-11946-5_6 89 90 Chapter 6. Functions and Functional Equations y y IdR b x x Some of the problems that appear in the mathematical olympiad contests, which make reference to functions, ask to find all the functions that satisfy a given property, or to find a specific value of some function. Often, these are difficult tasks, therefore it is important to understand the general behavior of the function before, so as to be able to decide correctly which functions satisfy or not the property. In this first section we will offer several examples of the kind of problems that can appear, and we will point out a series of facts that we can ask about a function. Example 6.1.1. Let f : N → N be given by f (n) = n(n + 1). Let us find the values of m and n such that 4f (n) = f (m), where m and n are natural numbers. Suppose that 4f (n) = f (m), then 4n2 + 4n = m2 + m. If we complete the square on the left side of the equation, we obtain 4n2 + 4n + 1 = m2 + m + 1, (2n + 1)2 = m2 + m + 1, but m2 + m + 1 cannot be the square of an integer because m2 < m2 + m + 1 < (m+1)2 . Therefore, there are no natural numbers m and n satisfying the condition. Example 6.1.2. Let f : N → N be a function such that f (3n) = n + f (3n − 3), for every positive integer n greater than 1 and such that f (3) = 1. Find the value of f (12). It is natural to use the fact that 12 = 3 · 4 in order to find f (12) = f (3 · 4). Using the formula or relation of the hypothesis, we have that f (12) = f (3 · 4) = 4 + f (3 · 3). We can repeatedly apply these substitutions to get f (12) = 4 + f (3 · 3) = 4 + 3 + f (3 · 2) = 4 + 3 + 2 + f (3 · 1) = 4 + 3 + 2 + 1 = 10. Observe that we can find f (3n) for all n, in the following way: f (3n) − f (3(n − 1)) = n f (3(n − 1)) − f (3(n − 2)) = n − 1 .. .. . . 91 6.1 Functions .. .. . . f (6) − f (3) = 2 f (3) = 1. Adding these equations results in f (3n) = 1 + 2 + 3 + · · · + n = n(n+1) . 2 Functions can be combined to form new functions. For instance, if we have functions with the same domain and codomain, we can add, subtract or multiply them to obtain new functions. For two functions f , g : X → Y , with Y ⊂ C, we define the sum and the difference of the functions f and g as (f ± g)(x) = f (x) ± g(x), for all x ∈ X. We define the product of the functions f and g as (f · g)(x) = f (x) · g(x), for all x ∈ X. Finally, if g(x) = 0 for all x, we define the quotient of the functions f and g as f g (x) = f (x) , for all x ∈ X. g(x) Example 6.1.3. Find all the functions f : R → R that satisfy f (y + x) − f (y − x) = 4yx, for all x, y ∈ R. (6.1) If we let y = x, we get that f (2y)−f (0) = 4y 2 and taking f (0) = c, we obtain f (2y) = 4y 2 + c. Now, if we let 2y = x, we get that the solution of the functional equation is of the form f (x) = x2 + c. It is easy to see that these functions satisfy equation (6.1). Example 6.1.4. Find all the functions f : R+ → R+ satisfying f (xy) = f x y , for all x, y ∈ R+ . In this case, if we let y = x we have that f (x2 ) = f (1). This makes sense since the function is defined on the positive real numbers. On the other hand, if we let y = x2 , then f (y) = f (1) for all y ∈ R+ . Hence, the functions solving the functional equation are the constant functions. It is easy to see that these functions satisfy the functional equation. In the last two examples we made the substitution y = x, which directly gave us the solution of the given functional equations. Next we present an example where, after substituting the variables for numerical values, we get information about the function. 92 Chapter 6. Functions and Functional Equations Example 6.1.5. Find all the functions f : R → R such that f (x + y) + f (x − y) = f (x) + 6xy 2 + x3 , for all x, y ∈ R. Let y = 0 to see that 2f (x) = f (x) + x3 . Then f (x) = x3 is the function we are looking for. We can directly check that f (x) = x3 satisfies the equation. Example 6.1.6. Find all the functions f : N → R such that f (1) = 3, f (2) = 2 and f (n + 2) + 1 = 2, f (n) for all n ∈ N. (6.2) Observe that the original equation gives us f (3) = 2 − 1 1 5 =2− = f (1) 3 3 and f (4) = 2 − 1 1 3 6 =2− = = . f (2) 2 2 4 Hence, we can conjecture that f (n) = n+2 n is true, for all natural numbers n. It is true by hypothesis for the cases n = 1 and n = 2, and we already verified the result for n = 3 and n = 4. We will finish the proof using induction, that is, assuming the result holds for n, and then proving it for n + 2. Suppose that f (n) = n+2 n , then 1 n 1 = 2 − n+2 = 2 − f (n) n + 2 n n+4 2n + 4 − n = . = n+2 n+2 f (n + 2) = 2 − Hence, we have the result for all n ∈ N and, in fact, the function satisfies the condition. Example 6.1.7 (India, 2010). Find all functions f : R → R satisfying, f (x + y) + xy = f (x)f (y), for all x, y ∈ R. (6.3) Let x = y = 0 in equation (6.3), then f (0) = f (0)2 , hence f (0) = 0 or f (0) = 1. If f (0) = 0, we let y = 0 in equation (6.3) to get f (x) = 0, for all x ∈ R. But the function f (x) = 0, for all x ∈ R, (the constant function zero) does not satisfy condition (6.3) when xy = 0. Suppose that f (0) = 1. If we let x = 1, y = −1, we get that f (1)f (−1) = f (1 − 1) − 1 = f (0) − 1 = 0, then f (1) = 0 or f (−1) = 0. If f (−1) = 0, letting y = −1 we get f (x − 1) − x = 0, then f (x − 1) = x, and using y = x − 1 in this last equality, we get f (y) = y + 1. If f (1) = 0, letting y = 1 we obtain f (x + 1) + x = f (x)f (1) = 0 so that f (x + 1) = −x. Finally, if we take y = x + 1 we obtain f (y) = 1 − y. In this way, the only solutions are f (x) = x + 1 and f (x) = 1 − x. It is easy to check that these functions satisfy the functional equation (6.3). 93 6.1 Functions Another way to create a new function given two functions f : X → Y and g : Y → Z is the composition of f and g, which is the function g ◦ f : X → Z defined for all x ∈ X by (g ◦ f )(x) = g(f (x)). Observe that the composition g ◦f is defined only if the codomain of f is contained in the domain of g. Example 6.1.8 (IMO, 1977). Consider f : N → N such that f (n + 1) > f (f (n)), for every positive integer n. Prove that f (n) = n, for every n ∈ N. Let A = {f (1), f (2), . . . } be the image of f . By hypothesis, note that for every n ≥ 2, it follows that f (n) > f (f (n − 1)). Hence for n ≥ 2, f (n) cannot be the minimum of the image of f . It follows that the minimum of A is f (1) and that f (n) > f (1) for n ≥ 2. Observe that if m ≥ p, then f (m) ≥ p. For p = 2, the result follows from the discussion above. Suppose the result true for p ≥ 2 and let us show it holds for p + 1. Let m ≥ p + 1, then m − 1 ≥ p and f (m − 1) ≥ p; now, by hypothesis, f (m) > f (f (m − 1)) ≥ p, hence f (m) ≥ p + 1. Now, let Ap = {f (p), f (p + 1), f (p + 2), . . . }. For every n ≥ p + 1, it follows that f (n) > f (f (n−1)). Since the observation guarantees that f (f (n−1)) belongs to the set Ap , then f (n) cannot be the minimum of Ap , hence the minimum must be f (p). Therefore, f (n) > f (p) for all n ≥ p + 1. Finally, from the last paragraph, it follows that f (1) < f (2) < · · · < f (p) < f (p + 1) < . . . . (6.4) Now, let us show that f (n) ≥ n. We have that f (1) ≥ 1 and f (2) > f (1), hence f (2) ≥ 2. Similarly, from f (2) ≥ 2 and f (3) > f (2), it follows that f (3) ≥ 3, and using induction we can prove that f (n) ≥ n. Finally, if for some n we have that f (n) > n, then f (n) ≥ n + 1 and, by (6.4), f (f (n)) ≥ f (n + 1), contradicting the hypothesis. Thus, f (n) = n, for every n ∈ N. Exercise 6.1. Find all functions f : R\{0, 1} → R satisfying the functional equation 1 = x, for all x = 0, 1. f (x) + f 1−x Exercise 6.2 (Canada, 1969). Let f : N → N be a function with the following properties: (i) f (2) = 2. (ii) f (mn) = f (m)f (n), for all m and n. (iii) f (m) > f (n), for m > n. Prove that f (n) = n. 94 Chapter 6. Functions and Functional Equations Exercise 6.3. Find all functions f : R\{0} → R such that xf (x) + 2xf (−x) = −1, for x = 0. Exercise 6.4. Find all functions f : R\{0} → R that satisfy the equation 1 f (−x) + f x 1 x = x, for x = 0. Exercise 6.5 (Ireland, 1995). Find all functions f : R → R for which xf (x) − yf (y) = (x − y)f (x + y), for all x, y ∈ R. Exercise 6.6 (Ukraine, 1997). Find all functions f : Q+ ∪ {0} → Q+ ∪ {0}, that satisfy the following conditions: (a) f (x + 1) = f (x) + 1, for all x ∈ Q+ ∪ {0}. (b) f (x2 ) = f (x)2 , for all x ∈ Q+ ∪ {0}. Exercise 6.7. Find all functions f : R → R such that xf (y) + yf (z) + zf (x) = yf (x) + zf (y) + xf (z), for x, y, z real numbers. 6.2 Properties of functions In this section, we will study important properties that a function may or may not have. A good knowledge of these properties can help us to detect what kind of function we have. 6.2.1 Injective, surjective and bijective functions We say that a function f : X → Y is injective (also known as one-to-one) if for any x1 , x2 ∈ X, with x1 = x2 , it follows that f (x1 ) = f (x2 ). The condition is equivalent to saying that if f (x1 ) = f (x2 ), then x1 = x2 . We say that a function f : X → Y is surjective (also known as onto) if Img f = Y , that is, if for every y ∈ Y there exists x ∈ X such that f (x) = y. Finally, we say that a function is bijective if it is injective and surjective. For functions from the real numbers to the real numbers knowing the graph of the function can be very useful. The graph can help us to determine if the function is injective, surjective or both. More precisely, a function is injective if any parallel line to the x-axis intersects the graph of the function in at most one point. A function is surjective, if any horizontal line y = y0 , with y0 in the codomain of the function, intersects the graph in at least one point. The function shown in the following graph is 95 6.2 Properties of functions not injective, since any parallel line above the x-axis, intersects the graph in two points. Moreover, it is not surjective since a horizontal line below the x-axis never intersects the graph of the function. y = f (x) Example 6.2.1. A function f : N → N that satisfies f (f (m) + f (n)) = m + n for all m, n ∈ N, is injective. The function is injective, because if f (m) = f (n), then f (m)+ f (n) = f (n)+ f (n), and from here it follows that m + n = f (f (m) + f (n)) = f (f (n) + f (n)) = n + n, thus m = n. Example 6.2.2. The functions f : R+ → R+ that satisfy the condition f (xf (y)) + f (yf (x)) = 2xy, for all x, y ∈ R+ are injective. Letting x = y, we have that f (xf (x)) = x2 , in particular f (f (1)) = 1. Letting x = f (1) in the last equation, we get that f (1)2 = f (f (1)f (f (1))) = f (f (1)) = 1, hence f (1) = 1. If one takes y = 1 in the original equation, we obtain that f (x) + f (f (x)) = 2x. With this last equality we can show that f is injective. If f (x) = f (y), then 2x = f (x) + f (f (x)) = f (y) + f (f (y)) = 2y, hence x = y. Example 6.2.3. A function f : R → R that satisfies f (f (x) + y) = 2x + f (f (y) − x), for all x, y ∈ R is surjective. If we take y = −f (x), it follows that f (f (x) − f (x)) = 2x + f (f (−f (x)) − x), that is, f (f (−f (x)) − x) = f (0) − 2x. Now, if y ∈ R we need to find x0 such 96 Chapter 6. Functions and Functional Equations that f (0) − 2x0 = y, but x0 = surjective. f (0)−y 2 satisfies f (f (−f (x0 )) − x0 ) = y, thus f is Example 6.2.4. A function f : Q+ → Q+ that satisfies f (xf (y)) = f (x) , for all x, y ∈ Q+ y is bijective. If x = 1, then f (f (y)) = f (1) y , this will help us to show that f is injective. If f (y1 ) = f (y2 ), then f (f (y1 )) = f (f (y2 )), that is, fy(1) = fy(1) . Hence y1 = y2 , 1 2 + and therefore f is injective. It is left to show that f is surjective. Let m n ∈ Q , . If this happens, then we want to show that there is x ∈ Q+ such that f (x) = m n   m f (1) f (f (x)) = f m , hence , and solving for x we get x = ff (1) = f , which n x n (m n ) is a rational number. That is, f is surjective, and therefore f is bijective. Observation 6.2.5. In the previous examples of this section, we were faced with a situation where f (g(x)) = h(x). Notice that if h is injective then g is injective. Also, if h is surjective then f is surjective. 6.2.2 Even and odd functions The functions that satisfy f (x) = f (−x) are called even functions and the functions such that f (x) = −f (−x) are called odd functions. A function f : R → R is even if its graph is symmetric with respect the y-axis, whereas the graph of an odd function is symmetric with respect to the origin. y = f (x) Graph of an even function. y = f (x) Graph of an odd function. 97 6.2 Properties of functions Example 6.2.6. Find all the functions f : Q → Q, that satisfy f (x + y) + f (x − y) = 2f (x) + 2f (y), for all x, y ∈ Q. Letting x = y = 0, we have f (0) = 0. If x = y, then f (2x) = 4f (x), and by induction f (nx) = n2 f (x) for every positive integer n. With x = 0, we have that f (y) + f (−y) = 2f (y), then f (y) = f (−y), hence f is even and f (nx) = n2 f (x) for every integer number n and every rational number x. If r = pq is a rational $ % $ % $ % number, with p ≥ 1, then p2 f (1) = f (p) = f q pq = q 2 f pq , and then f pq = $ %2 p f (1). Hence f (r) = cr2 , for all r ∈ Q with c = f (1). It is easy to check that q these functions satisfy the given condition. 6.2.3 Periodic functions Periodicity plays an important role in mathematics and for this reason we include some examples about this topic. We say that a function f : R → R is periodic if there exists a = 0 ∈ R such that f (x + a) = f (x), for all x ∈ R. The number a is called a period of f . It is clear that for all n = 0, the number na is also a period of f . a −a Example 6.2.7. A function f : R → R is periodic, if for some a ∈ R and every x ∈ R, it is true that 1 + f (x) . f (x + a) = 1 − f (x) From the equation f (x + a) = 1+f (x) 1−f (x) , evaluating in x − a, we obtain that 1+f (x−a) f (x) = 1−f (x−a) . After solving for f (x) from the original equation, we get that f (x+a)−1 (x−a) f (x) = f (x+a)+1 . Equating the last two equations, we get that 1+f 1−f (x−a) = f (x+a)−1 −1 f (x+a)+1 and after simplifying f (x + a) = f (x−a) . Evaluating this last equation in x + a, we get that f (x + 2a) = f−1 (x) , and then f (x + 4a) = −1 −1 = −1 = f (x), f (x + 2a) f (x) that is, f is periodic with period 4a. 98 Chapter 6. Functions and Functional Equations Example 6.2.8 (Belarus, 2005). (a) Consider a function f : N → N that satisfies f (n) = f (n + f (n)) for all n ∈ N. Prove that, if the image of f is finite, then f is periodic. (b) Find a non-periodic function f : N → N such that f (n) = f (n + f (n)), for all n ∈ N. To prove (a) we do as follows. Since f (n) = f (n + f (n)), then f (n + f (n) + f (n)) = f (n + f (n) + f (n + f (n))) = f (n + f (n)) = f (n). Therefore, f (n) = f (n + 2f (n)) and, by induction, f (n) = f (n + kf (n)) for all k ∈ N. Let A = f (N) = {a1 , . . . , at } and T = a1 · · · · · at . Let us see that T is a period of f . Since f (n) = f (n + kf (n)), for all k, it follows that f (n) = f n+ T f (n) f (n) = f (n + T ), T where k = f (n) . That is, T is a period of f . (b) We would like to find a non-periodic function f that satisfies the equation. For n = 2k m with k ∈ N ∪ {0} and m odd, we define f (n) = 2k+1 . Now, let us see that the function satisfies the equation, f (n + f (n)) = f (2k m + 2k+1 ) = f (2k (m + 2)) = 2k+1 = f (n) and moreover, f is not periodic, otherwise its range would be finite, however, its image is the set of powers of 2. 6.2.4 Increasing and decreasing functions We say that a function is increasing if for x < y it follows that f (x) < f (y). We say that a function is non-decreasing if for x < y it follows that f (x) ≤ f (y). Similarly, we say that a function is decreasing if for x < y it follows that f (x) > f (y), and it is non-increasing if for x < y it happens that f (x) ≥ f (y). Another way to guarantee the injectivity of a real function, whose domain is the real numbers, is to know if the function is increasing or decreasing. Example 6.2.9 (IMO, 1992). Find all functions f : R → R that satisfy f (x2 + f (y)) = y + f (x)2 , for all x, y ∈ R. Let a = f (0). With x = 0, we have that f (f (y)) = a2 + y. With x = y = 0, then f (a) = a2 , hence f (x2 + a) + a2 = f (x)2 + f (a). Applying f to both sides of the equation it follows that f (f (x2 + a) + a2 ) = f (f (x)2 + f (a)). The left-hand side of the equality is f (f (x2 + a) + a2 ) = x2 + a + f (a)2 = x2 + a + a4 , (6.5) and the right-hand side of the equality is f (f (x)2 + f (a)) = a + f (f (x))2 = a + (a2 + x)2 = a + a4 + 2a2 x + x2 . (6.6) 6.2 Properties of functions 99 Comparing both equations (6.5) and (6.6), we need to have that 2a2 x = 0, hence a = 0. From here we conclude that f (f (y)) = y, for all y ∈ R and f (x2 ) = f (x)2 , for all x ∈ R. The last equation guarantees that if x ≥ 0, then f (x) ≥ 0. Since f (f (y)) = y we have that f is injective, hence f (x) = 0 if and only if x = 0. In this way when x > 0, then f (x) > 0. Since f (f (x)2 + f (y)) = f (f (x))2 + y = x2 + y, it follows that f (x2 + y) = f (f (f (x)2 + f (y))) = f (x)2 + f (y) = f (x2 ) + f (y). √ If y < x, then x − y > 0 and x = x − y + y. Applying the last equality to x − y and y, it follows that f (x) = f (x − y + y) = f (x − y) + f (y) > f (y), that is, f is increasing. But f non-decreasing and f (f (x)) = x, guarantee that f (x) = x. In fact if f (x) > x, then x = f (f (x)) > f (x) and if f (x) < x, then x = f (f (x)) < f (x). Therefore, the only function that satisfies the functional equation is f (x) = x. 6.2.5 Bounded functions We say that a function f : A ⊂ R → R is bounded above in A, if there exists M ∈ R such that f (a) ≤ M , for all a ∈ A. We say that a function f : A ⊂ R → R is bounded below in A, if there exists N ∈ R such that f (a) ≥ N , for all a ∈ A. We say that a function f : A ⊂ R → R is bounded in A, if there exists M > 0 such that |f (x)| ≤ M , for all x ∈ A, or equivalently −M ≤ f (x) ≤ M , for all x ∈ A. Example 6.2.10. Let m ≥ 2 be an integer number. Find all the bounded functions f : [0, 1] → R such that, for x ∈ [0, 1], it follows that  4 $x% 2x (m − 1) x 1 +f + ···+ f . f (x) = 2 f (0) + f m m m m If |f (x)| ≤ M for x ∈ [0, 1], then, using the triangle inequality, it follows that  m−1   mM j M 1 #  f x  ≤ . = |f (x)| ≤ 2  2 m j=0 m m m Hence |f (x)| ≤ M m , for x ∈ [0, 1]. With this new bound, we can repeat the last M argument to show that |f (x)| ≤ m 2 , for x ∈ [0, 1]. An inductive argument guaranM tees us that for all n ∈ N, it follows that |f (x)| ≤ m n , for x ∈ [0, 1], and when we let n go to infinity, we have that f (x) = 0, for all x ∈ [0, 1], and this is the only function that satisfies the equation. 6.2.6 Continuity When the values of a function f (x) get closer to b as x tends to a, we say that b is the limit of f (x) as x tends to a. In mathematical language this is usually written 100 Chapter 6. Functions and Functional Equations as follows: ∀ ǫ > 0, ∃ δ > 0 such that 0 < |x − a| < δ ⇒ |f (x) − b| < ǫ and we write limx→a f (x) = b. Also, we can consider limx→∞ f (x) = b, which is defined as: for all ǫ > 0 there exists M > 0 such that if x > M then |f (x) − b| < ǫ. One characterization of the limit concept when dealing with sequences is the following theorem. Theorem 6.2.11. Let f be a function, then limx→a f (x) = b if and only if for every sequence {an } with limn→∞ an = a, it follows that limn→∞ f (an ) = b. The proof of this theorem will be given in Chapter 7. Observation 6.2.12. The previous theorem also states that limx→a f (x) is not b, if there exists a sequence {an } such that limn→∞ an = a and limn→∞ f (an ) is not b. Example 6.2.13 (IMO, 1983). Find all functions f : R+ → R+ that satisfy: (a) f (xf (y)) = yf (x), for all positive real numbers x, y. (b) limx→∞ f (x) = 0. If x = 1, then f (f (y)) = yf (1), hence the function is bijective. In fact, if f (x) = f (y), then xf (1) = f (f (x)) = f (f (y)) = yf (1), and since f (1) = 0 we have + that x = y, therefore f is injective; let us %%that f is surjective, let c ∈ R $ see $ now c c = f (f (y)) = yf (1) = c, hence , then it follows that f f f (1) and take y = f (1) f is surjective. Thus, f is bijective. In particular, there is a y0 such that f (y0 ) = 1. Since f (xf (y0 )) = y0 f (x) for all x > 0, taking x = 1, we have that f (1) = y0 f (1), and then y0 = 1, hence x = 1 is a fixed point. If x = y, then f (xf (x)) = xf (x), and xf (x) is also a fixed point of f . If we show 1 that the only fixed point  1 can  conclude that f (x) = x .  1 of f is 1, we From the equation f a f (a) =  af  a and the fact that f is injective, we have that f (a) = a if and only if f a1 = a1 . If there is a fixed point different from 1, hence there is a fixed point a greater than 1. But f (a) = a implies, by induction, that f (an ) = an , (for instance, f (a2 ) = f (af (a)) = af (a) = a2 ). Since an → ∞ and f (an ) = an → ∞, this contradicts the fact that limx→∞ f (x) = 0. Therefore, the only fixed point is 1. The function f (x) = x1 satisfies the conditions of the problem. We say that a function f is continuous at some point a if when we let x tend to a, f (x) tends to f (a), that is, limx→a f (x) = f (a). Also, we say that f is continuous on a set A if f is continuous at every point a ∈ A. One characterization of the continuity property is given by the following result. 6.2 Properties of functions 101 Theorem 6.2.14. A function f is continuous at a if and only if for every sequence {an } with limn→∞ an = a, it follows that limn→∞ f (an ) = f (a). A set D ⊂ R is dense in the set of real numbers if every open interval in R has points in D. Theorem 6.2.15. The set of rational numbers is dense in the set of real numbers. Theorem 6.2.16. If a function f is continuous on R and f is zero in a dense subset of the real numbers, then f is identically zero in R. As a consequence, if the functions f and g are continuous and coincide on a dense subset of R, then they coincide on all R. The proofs of these three theorems will be given in Section 7.4. Example 6.2.17 (Nordic, 1998). Find all the continuous functions f : R → R that satisfy the equation, f (x + y) + f (x − y) = 2(f (x) + f (y)), for all x, y ∈ R. We have seen in Example 6.2.6 that f (x) = f (1)x2 , for x ∈ Q and, by Theorem 6.2.16, f (x) = f (1)x2 for all x ∈ R. Exercise 6.8. Let f , g : R → R be two functions that satisfy f (g(x)) = g(f (x)) = −x, for any real number x. Prove that f and g are odd functions. Exercise 6.9. Find all surjective functions f : R → R that satisfy f (f (x − y)) = f (x) − f (y), for all x, y ∈ R. Exercise 6.10. Find all continuous functions f : R → R that satisfy f (xf (y)) = xy, for all x, y ∈ R. Exercise 6.11 (Belarus, 2005). Find all functions f : N → N that satisfy f (m − n + f (n)) = f (m) + f (n), for all m, n ∈ N. Exercise 6.12 (IMO, 1990). Find a function f : Q+ → Q+ that satisfies the equation f (x) , for all x, y ∈ Q+ . f (xf (y)) = y Exercise 6.13. Find all continuous functions f : R → R that satisfy xf (y) + yf (x) = (x + y)f (x)f (y), for all x, y ∈ R. 102 Chapter 6. Functions and Functional Equations Exercise 6.14 (IMO, 1968). Let f : R → R be a function with the property f (x + a) = 1 + 2 f (x) − f (x)2 , for all x ∈ R and a a fixed number. (i) Prove that f is periodic. (ii) In case that a = 1, give an example of a function of this type. Exercise 6.15. Let a, b > 0, find the values of m such that the equation |x − a| + |x − b| + |x + a| + |x + b| = m(a + b), has at least one real solution. 6.3 Functional equations of Cauchy type The functional equations of Cauchy type are: (C1 ) f (x + y) = f (x) + f (y). (C2 ) (C3 ) f (x · y) = f (x) + f (y). f (x + y) = f (x) · f (y). (C4 ) f (x · y) = f (x) · f (y). In order to establish what functions satisfy certain functional equations we should take into account the domain and the codomain where we want to solve the equation. For instance, if we take equation (C2 ), and we want to solve it in all R, considering y = 0, we obtain that the solution of the equation is f (x) = 0, for all x, which means the equation sought for was very simple. Therefore, it is more adequate for this functional equation to consider the set of real positive numbers as its domain. 6.3.1 The Cauchy equation f (x + y) = f (x) + f (y) The first of the equations of Cauchy type is the most important. With this equation we will illustrate how some functional equations are solved. First, we will see how to determine some of the values that the functions take, and this will allow us to find, in a natural way, other values until we learn how the functions behave in the set of rational numbers. Letting x = y = 0, we have that f (0) = 2f (0), then f (0) = 0. If y = −x, we have that 0 = f (0) = f (x + (−x)) = f (x) + f (−x), and then f (−x) = −f (x), which tells us that the function f should be odd. 103 6.3 Equation Cauchy type With x = y, we have that f (2x) = 2f (x). Now, using induction, we can conclude that f (nx) = nf (x), for any positive integer n. In fact, f ((n + 1)x) = f (nx + x) = f (nx) + f (x) = nf (x) + f (x) = (n + 1)f (x). Recalling that f (−x) = −f (x), we get f (nx) = nf (x) for all n ∈ Z. Now, x x x 1 since f (x) = f ( m m x) = f (m m ) = mf ( m ), we have that f ( m ) = m f (x), therefore x x n n f ( m x) = f (n m ) = nf ( m ) = m f (x). Hence, f (rx) = rf (x), for all r ∈ Q and all x ∈ R. Letting c = f (1), we get f (r) = cr for all r ∈ Q. We conclude that a function f : Q → R that satisfies equation (C1 ) should have the form f (r) = cr, for all r ∈ Q, with c = f (1) a fixed constant. And a function of this type f (x) = cx satisfies such a type of Cauchy equation, since c(x + y) = cx + cy, for any x, y ∈ Q. Additional hypothesis to the Cauchy equation f (x + y) = f (x) + f (y) We would like to determine functions f : R → R that satisfy the first of the Cauchy type equations, which are also known as additive functions. We will see that with an additional hypothesis (we will analyze several of them), the conclusion is that f should be linear, that is, of the form f (x) = ax, for all x ∈ R and with a = f (1). (H1 ) The function is continuous in all R. We know that f (r) = cr, for all r ∈ Q. Since f (x) and cx are continuous in R and coincide in Q, we have, by Theorem 6.2.16, that the functions coincide in all R. Hence f (x) = cx for all x ∈ R. Example 6.3.1 (Jensen’s equation). Find all continuous functions f : R → R that satisfy the equation, f x+y 2 = f (x) + f (y) , for all x, y ∈ R. 2 Note that by letting x = y = 0, we do not obtain information about what is the value of f (0). Define g(x) = f (x) − f (0), which is also continuous. A straightforward calculation shows that x+y 2 g = g(x) + g(y) , 2 but now the function g satisfies g(0) = 0. Taking y = 0 in the new equation, we have that $ x % g(x) , = g 2 2 and after substituting in this last equation x = u + v, we get g u+v 2 = g(u + v) . 2 104 Chapter 6. Functions and Functional Equations Hence, we can affirm that g(u + v) = g(u) + g(v), for all u, v ∈ R, that is, g is a continuous function that satisfies the first equation of Cauchy. Therefore, g(x) = ax, with a = g(1), and then f (x) = ax + b, for all x ∈ R and b = f (0). (H1′ ) The function is continuous only in x = 0. To reduce this to the previous case, it will be enough to show the following result. Lemma 6.3.2. If f : R → R is an additive function, that is, if it satisfies equation (C1 ) and it is continuous at 0, then it is continuous at every real number a. Proof. Let {an } be a sequence with limn→∞ an = a, then the sequence {an − a} satisfies limn→∞ (an −a) = 0. Since f is continuous in 0, Theorem 6.2.14 guarantees that limn→∞ f (an − a) = f (0) = 0. But equation (C1 ) implies that f (an ) = f (an − a) + f (a), then limn→∞ f (an ) = limn→∞ f (an − a) + limn→∞ f (a) = f (a), which implies f is continuous in a.  (H2 ) The function is monotone. If the function f , besides being additive, is monotone (without loss of generality, we can assume that it is non-decreasing), then f (x) should have the form f (x) = cx. To support this claim consider a real number x. Let {rn } and {sn } be sequences of rational numbers converging to x, with rn < x < sn for all n. By the monotonicity of the function f , crn = f (rn ) ≤ f (x) ≤ f (sn ) = csn . Taking the limit, we obtain cx = limn→∞ crn ≤ f (x) ≤ limn→∞ csn = cx. Thus, f (x) = cx. The non-increasing case is similar. Moreover, we can change ≤ to < and reach the same conclusion, and similarly in the case ≥. (H3 ) The function is positive (for positive numbers). If f (x) > 0 for x > 0 and if in addition it is additive, then it is increasing. In fact, if x < y then y − x > 0, and f (y − x) > 0. Hence, f (x) < f (x) + f (y − x) = f (x + (y − x)) = f (y). Similarly, we can consider the decreasing, non-decreasing and non-increasing cases, and in each one of them we can conclude that f is linear. (H4 ) The function is bounded. If the additive function f is bounded in an interval of the form [a, b], that is, if there exists a constant M > 0 such that |f (x)| ≤ M for all x ∈ [a, b], then the function must have the form f (x) = cx. First note that x ∈ [0, b − a] if and only if x + a ∈ [a, b] and for x ∈ [0, b − a] we have |f (x)| = |f (x + a) − f (a)| ≤ |f (x + a)| + |f (a)| ≤ 2M . This guarantees that f is bounded by 2M in [0, b − a]. Let α = b − a, c = f (α) α and g(x) = f (x)− cx. We then have (a) g(x + y) = f (x + y) − c(x + y) = f (x) − cx + f (y) − cy = g(x) + g(y), that is, g is additive. 6.3 Equation Cauchy type 105 (b) g(α) = f (α) − cα = 0. (c) g(x + α) = g(x) + g(α) = g(x), the function g is periodic with period  α.  f (α)  (d) For x ∈ [0, α], we have |g(x)| = |f (x) − cx| ≤ |f (x)|+|cx| ≤ 2M +  α  |α| ≤ 3M , that is, g is bounded in the interval [0, α] and then, because it is periodic, it is bounded in all R. If g(x0 ) = 0 for some real number x0 , then since g(nx0 ) = ng(x0 ) for every integer number n, we can make |g(nx0 )| as large as we wish, then g would not be bounded, which would be a contradiction to the part (d). Therefore, g(x) = 0 for any real number x and then f (x) = cx, for all x in R. (H4′ ) The function is bounded on a neighborhood of 0. By (H1′ ), it will be enough to show that f is continuous in 0. Let {an } be a sequence that converges to 0. We will use Theorem 6.2.14 to show that f (an ) converges to 0. Let ǫ > 0, we will see that |f (an )| < ǫ for large n. If M > 0 is the bound for f in the interval (−a, a), let us choose an integer N such that M N < ǫ. Since limn→∞ an = 0, there exists n0 such that |an | < Na , for all n ≥ n0 . Since |N an | < a, it follows that |f (N an )| ≤ M for n ≥ n0 . But since f (N an ) = N f (an ), we have that |f (an )| ≤ M N < ǫ, for n ≥ n0 , as we wanted. There are several conditions that can be added to the equation of Cauchy to make sure that the function that satisfies the equation is a linear function. Many of these conditions are the source of problems of the kind that appear in the mathematical olympiad. Let us see an example. Example 6.3.3. Find all the functions that satisfy the following equations: (a) f (x + y) = f (x) + f (y), for all x, y ∈ R. (b) f (xy) = f (x)f (y), for all x, y ∈ R. First note that if x ≥ 0, then √ √ √ √ √ f (x) = f ( x · x) = f ( x)f ( x) = (f ( x))2 ≥ 0. But then, by (H3 ), we have that f is linear, that is, it has the form f (x) = cx with c = f (1). Taking x = y = 1 in the equation (b), we get that c = c2 , hence c = 0 or c = 1. Then, f (x) = 0 or f (x) = x, are the only solutions to the problem. 6.3.2 The other Cauchy functional equations ⋆ The Cauchy equation f (x · y) = f (x) + f (y) We will find continuous solutions to this functional equation. If y = 0 belongs to the domain of f , then f (x) = 0. Now, suppose that the function is defined for x = 0. If we take x = y = 1 in the equation, we have that f (1) = 0. Also, 106 Chapter 6. Functions and Functional Equations considering x = y = −1, we get that f (−1) = 0. Now, taking y = −1, we obtain f (−x) = f (x), that is, the function must be even and it will be determined by its behavior when x is positive. But if x, y are positive, there are u, v ∈ R such that x = eu , y = ev , and with them the equation14 can be expressed as f (eu · ev ) = f (eu ) + f (ev ). If we let g(u) = f (eu ), then g(u + v) = g(u) + g(v), which is the first Cauchy equation, and we know that its solution is g(u) = cu, with c = g(1) = f (e), then f (x) = g(u) = f (e) log x for x > 0, and f (x) = f (e) log |x| for x = 0. The Cauchy equation f (x + y) = f (x) · f (y) First note that if for some y, f (y) = 0, then f is constant. This follows from f (x) = f (x − y + y) = f (x − y)f (y) = 0. If f is never zero, then it is positive since f (x) = f ( x2 + x2 ) = (f ( x2 ))2 > 0. Hence, since f is always positive we can take logarithms on both sides of the equation in order to satisfy the functional equation, log f (x + y) = log f (x) + log f (y), which is a functional equation of the first type, then log f (x) = cx, with c = log f (1). Applying the exponential function, we have that f (x) = elog f (1)x = f (1)x , for all x ∈ R. Note that here we have found only the continuous solutions. The Cauchy equation f (x · y) = f (x) · f (y) As in the previous equation, if for some y = 0, f (y) = 0, then f is constant. This follows since f (x) = f ( xy ·y) = f ( xy )f (y) = 0. If f is never zero, then for x positive, √ √ √ f (x) is positive, since f (x) = f ( x · x) = (f ( x))2 > 0. For x = 1, we have that f (1) = (f (1))2 , therefore f (1) = 0 or f (1) = 1. The first option has been studied before, and therefore f (1) = 1. Since f (x2 ) = (f (x))2 , f (−1) = ±1 and taking y = −1 in the original equation, we have that f (−x) = ±f (x). Then it will be enough to see what happens with x > 0. After that we will have two options to extend to the negative real numbers, that is, whether making the function even or odd. Since the function f is positive for x > 0, we can apply the logarithmic function on both sides of the equality to get log f (x · y) = log f (x) + log f (y). Considering g(x) = log f (x), we have that g satisfies the second equation of Cauchy, then g(x) = g(e) log x, hence f (x) = xg(e) = xlog f (e) . Thus the continuous solutions are f (x) = 0, f (x) = ±xlog f (e) . Observe that the point x = 0 remains outside the analysis we have made (when we take a logarithm), but at the end we include 0 and necessarily it has to happen that f (0) = 0, in order to have 14 For the definition of the exponential function, ex , and the function logarithm, log x, see [21]. 107 6.3 Equation Cauchy type continuity there. However, there exists an exception, if f (e) = 1 there are two solutions. One of them is f (x) = 1, which is continuous, and the other solution is f (x) = sign (x), which is not continuous at x = 0. Exercise 6.16. Find all functions f : R → R that satisfy f (x2 ) − f (y 2 ) = (x + y)(f (x) − f (y)), for all x, y ∈ R. Exercise 6.17. Find all functions f : R+ → R+ that satisfy f (x)f (y) − f (xy) = x y + , for all x, y ∈ R+ . y x Exercise 6.18. (i) Find all functions f : R+ → R+ that satisfy the condition f (xf (y)) + f (yf (x)) = 2xy, for all x, y ∈ R+ . (ii) (Short list IMO, 2002) Find all functions f : R → R such that f (f (x) + y) = 2x + f (f (y) − x), for all x, y ∈ R. Exercise 6.19. Let f be a function such that for some number a ∈ R it satisfies f (x + a) = f (x) − 3 , for all x ∈ R. f (x) − 2 Prove that f is periodic. Exercise 6.20. Let f : R → R be a periodic function such that the set {f (n) | n ∈ N} has an infinite number of elements. Prove that the period of f is an irrational number. Exercise 6.21 (Long list IMO, 1977). Determine all the real continuous functions f (x) defined on the interval (−1, 1), that satisfy the functional equation f (x + y) = f (x) + f (y) , 1 − f (x)f (y) for x + y, x, y ∈ (−1, 1). Exercise 6.22. Find all the continuous functions f : R → R such that f (x) + f (y) = f x+y 1 − xy , with x, y = 1. Exercise 6.23. Find all the continuous functions f : R → R that satisfy f (x + y) = f (x) + f (y) + f (x)f (y), for all x, y ∈ R. (6.7) 108 Chapter 6. Functions and Functional Equations 6.4 Recommendations to solve functional equations Next we will present a series of recommendations of the things we need to do in order to find solutions of functional equations. Moreover, we will show some examples where we use these observations. Substituting the variables for values. One of the first steps that we need to follow is to see if it is possible to determine some values of the function we are looking for, for instance f (0), f (1), etc. In some cases, the values can be found through direct substitution. But sometimes we may need to make a variable interchange. For instance, if we found something like f (x + y), it is natural to make y = −x, to obtain f (0). Mathematical induction. We should have in mind that the principle of mathematical induction can help us. In these cases it is important to remember the induction basis. For instance, to know what is f (1) or f (j), and then later to be able to conjecture something more specific that could be a relation that allows  us to go from n to n + 1. Also, try to find expressions like f n1 , and afterwards expressions of the form f (r), with r ∈ Q. These situations, in general, can arise when dealing with equations with variables in Q or in Z. Basic properties of functions. It is important to know if the function is injective, surjective, bijective, periodic, even, odd, or with some kind of symmetry. This can help us to reduce the cases and to concentrate only on the set of numbers where the equation is valid. Substitutions. Beside substitutions by specific values, we can try other more general substitutions, for instance, x1 , x + 1, x + y, x − y. Symmetry in the variables. If the equation has two (or more) variables, for instance x, y, we will always try to substitute the y by the x (and vice versa), and look always for symmetries in the variables. Compare with the Cauchy equations. If our equation can be reduced or simplified to an equation of Cauchy type, then we have made good progress, since we already know the solutions to this type of equations. Continuity, monotonicity. Investigate if the unknown function is monotone or continuous. This is very useful, since the problem could then be reduced to be solved on the rational numbers or on some dense subset of the real numbers. Other numeral systems. In functional equations where the natural numbers are present, it can help us to work in another numeral system different from the 10-base system, for instance, moving to the binary system or the 3-base system. Check. It is important to always check that the function we proposed as solving the equation, really does it. We should never forget this part. Now, we will exhibit examples where these recommendations are put to use. 6.4 Recommendations to solve functional equations 109 Example 6.4.1. Find all functions f : Q → R that satisfy the following conditions, f (xy) = xf (y) + yf (x) and f (x + y) = f (x2 ) + f (y 2 ), for x, y ∈ Q. If in the first equation we set x = y = 0, we obtain f (0) = 0, and taking x = y = 1, we have that f (1) = 2f (1), that is, f (1) = 0. On the other hand, taking x = 0 in the second equation, we get f (y) = f (y 2 ), then the second equation becomes f (x + y) = f (x) + f (y) and we know, by the Cauchy equation, that the function should be such that f (x) = f (1)x, for all x ∈ Q. Moreover, since f (1) = 0 the only solution is f (x) = 0. Example 6.4.2 (Short list IMO, 1988). Let f : N → N be a function that satisfies f (f (m) + f (n)) = m + n, for all m, n. Find all possible values of f (1988). In Example 6.2.1, we showed that the function is injective. Moreover, for l < n, we have that f (f (m + l) + f (n − l)) = m + l + n − l = m + n = f (f (m) + f (n)). (6.8) The injectivity property tells us that f (m + l) + f (n − l) = f (m) + f (n), for l, m, n ∈ N and l < n. (6.9) Now, by induction we will see that f (n) = n. First, for n = 1 let us see that f (1) = 1. If b = f (1), then the following two equalities are true: f (2b) = f (f (1) + f (1)) = 2 and f (b + 2) = f (f (1) + f (2b)) = 1 + 2b. Then, b = 2 is not possible, since, on the one hand we would have that f (2 · 2) = 2 and on the other hand f (2 + 2) = 1 + 2 · 2 = 5. Neither is b > 2 possible, since using f (2b) = 2, f (1) = b and equation (6.9), it follows that b + 2 = f (1) + f (2b) = f (1 + b − 2) + f (2b − (b − 2)) = f (b − 1) + f (b + 2) = f (b − 1) + 1 + 2b, then f (b − 1) = 1 − b < 0, which is not possible. Therefore, b = f (1) = 1. Suppose now that f (n) = n, then from the original equation and from the induction hypothesis, it follows that n + 1 = f (f (n) + f (1)) = f (n + 1). Therefore, the only possible value of f (1988) is 1988. 110 Chapter 6. Functions and Functional Equations Example 6.4.3. Find all increasing or decreasing functions f : R → R such that f (x + f (y)) = f (x) + y, for x, y ∈ R. Letting x = y = 0, we get f (f (0)) = f (0). Since f is increasing or decreasing, it follows that f is injective, then f (0) = 0. Taking x = 0, we get f (f (y)) = y for all y ∈ R. Letting a = f (y), it follows that f (f (y)) = y = f (a), then f (x + y) = f (x + f (a)) = f (x) + a = f (x) + f (y), hence, f satisfies the additive Cauchy equation. Moreover, with the condition f (f (y)) = y, we have that the only solutions are f (x) = x and f (x) = −x, which verify the functional equation. Example 6.4.4. Find all continuous functions f : R → R that satisfy the equation f (x + y) + f (x − y) = 2f (x), for x, y ∈ R. Letting x = y, it follows that f (2x) = 2f (x) − f (0). Then, from the original equation we obtain f (x + y) + f (x − y) = f (2x) + f (0); subtracting 2f (0) on both sides, we get f (x + y) − f (0) + f (x − y) − f (0) = f (2x) − f (0). Hence, f (x) − f (0) is additive, since taking u = x + y, v = x − y in the last equation, leads to f (u) − f (0) + f (v) − f (0) = f (u + v) − f (0). Since f (x)−f (0) is continuous, it follows that f (x) = f (0)+ax, for all x ∈ R. Example 6.4.5. Let f : R → R be a continuous function such that f (x) = x has no real solutions. Then f n (x) = x has no real solutions, where f n is the composition of f with itself, n times, for any n ∈ N. Since f (x) = x has no real roots, that is, there is no x ∈ R such that f (x) − x = 0, then it is true that either f (x) > x or f (x) < x, for all x ∈ R. In fact, since f (x) − x is continuous, and if f were positive on a point d and negative on another point e, then, by the intermediate value theorem15 , there would be a point x0 between d and e such that f (x0 ) − x0 = 0, which is impossible. Therefore, f (x) > x for all x ∈ R or f (x) < x for all x ∈ R. If f (x) > x for all x ∈ R, then x < f (x) < f (f (x)) < · · · < f (f (· · · f (x) · · · )) < · · · , and therefore f (f (· · · f (x) · · · )) = x has no real solutions. Similarly, if f (x) < x, we get that f (f (· · · f (x) · · · )) = x has no real solutions. Exercise 6.24. Prove that there are no functions f : N → N that satisfy f (f (n)) = n + 1, for all n ∈ N. 15 See [21]. 111 6.5 Difference equations Exercise 6.25 (IMO, 1986). Find functions f : R+ ∪ {0} → R+ ∪ {0} such that satisfy: (a) f (xf (y))f (y) = f (x + y), for x, y ≥ 0. (b) f (2) = 0. (c) f (x) =  0, for all x such that 0 ≤ x < 2. Exercise 6.26. Find all functions f : R → R such that f (x − y) = f (x + y)f (y), for all x, y ∈ R. 6.5 Difference equations. Iterations In this section we will study two kinds of functional equations: those relating the values of f (x) and f (x+h) or, more generally, with those of f (x+nh) for some n ∈ N, which are called difference equations, and the functional equations relating f (x) with its iterations, that is, with f 2 (x) = f (f (x)), . . . , f n (x) = f (f (. . . f (x) . . . )).    n times For the difference equations, we will use the difference operator, denoted by ∆, and which is defined, for a function f : R → R, as ∆f (x) = f (x + h) − f (x), (6.10) for x ∈ R, and where h is a fixed real number. Also, we will use the operator E defined by Ef (x) = f (x+ h), and the identity operator I defined by If (x) = f (x), so that ∆ = E − I, that is, ∆f (x) = Ef (x) − If (x) = f (x + h) − f (x). The following properties of the operators are seen to hold trivially. Properties 6.5.1. (a) ∆(af (x) + g(x)) = a∆f (x) + ∆g(x), for a fixed real number a. (b) ∆(f (x) · g(x)) = Ef (x)∆g(x) + g(x)∆f (x). g(x)∆f (x) − f (x)∆g(x) f , if g(x) = 0. (c) ∆ (x) = g g(x)Eg(x) (d) ∆m f (x) = ∆m−1 (∆f (x)), and also ∆m ∆n = ∆n ∆m = ∆m+n . Lemma 6.5.2. For each integer number n ≥ 1, it follows that ∆n xn = hn n! and ∆m xn = 0 for m > n. Proof. The proof is by induction on n. If n = 1, we have that ∆x = (x + h) − x = h and ∆2 x = ∆h = 0, then m ∆ x = 0 for m > 1. 112 Chapter 6. Functions and Functional Equations Suppose that the result is true for all j < n, then ∆n xn = ∆n−1 (∆xn ) = ∆n−1 ((x + h)n − xn ) n−1  n−1 # n # n = ∆n−1 hi ∆n−1 (xn−i ) xn−i hi = i i i=1 i=1 = n h∆n−1 (xn−1 ) = 1 n hhn−1 (n − 1)! = hn n!, 1 and since ∆ applied to a constant is zero, we have that ∆m xn = 0 for m > n, so the induction step is true and the result holds.  Example 6.5.3. If P (x) = a0 + a1 x + · · · + an xn is a polynomial of degree n, it follows that ∆n P (x) = an n!hn and ∆m P (x) = 0 for m > n. In fact, by the previous lemma ∆n P (x) = ∆n (a0 + a1 x + · · · + an xn ) = ∆n (a0 ) + ∆n (a1 x) + · · · + ∆n (an xn ) = an ∆n (xn ) = an n!hn . It is clear that if m > n, ∆m P (x) = ∆m−n (∆n P (x)) = ∆m−n (an n!hn ) = 0. In general, we have the following theorem. Theorem 6.5.4. For any function f , it follows that ∆n f (x) = (E − I)n f (x) = n # j=0 (−1)j n f (x + (n − j)h). j Proof. The proof is by induction. The case n = 1 has been already validated. Suppose now the result holds true for n and let us see what happens for n + 1. ∆n+1 f (x) = ∆n (f (x + h) − f (x)) n n # # n n (−1)j (−1)j = f (x + (n − j)h) f (x + (n − j + 1)h) − j j j=0 j=0 = n # (−1)j j=0 n # + j=0 n f (x + (n − j + 1)h) j (−1)j+1 n f (x + (n + 1 − (j + 1))h) j 113 6.5 Difference equations n # n n (−1)j f (x + (n + 1 − j)h) f (x + (n + 1 − j)h) + j − 1 j j=1 j=0 + , n # n n n j + (−1) = f (x + (n + 1)h) + j j−1 0 j=1 = n # (−1)j f (x + (n + 1 − j)h) + (−1)n+1 = n+1 # (−1)j j=0 n f (x) n n+1 f (x + (n + 1 − j)h). j  When difference equations are applied to functions with variables among the non-negative integers and with h = 1, we get expressions of the form ∆f (0) = f (1) − f (0), ∆f (1) = f (2) − f (1), ∆f (2) = f (3) − f (2), . . . , which are known as sequences in finite differences or recurrent sequences, notions that will be studied more carefully in Chapter 7. Let us see an example dealing with iterations in the functional equation. Example 6.5.5. Find functions f : N → N that satisfy f (f (n)) + f (n)2 = n2 + 3n + 3, for n ∈ N. It is easy to verify that f (n) = n + 1 satisfies the equation. Let us see that this function is the only one that satisfies the equation. If f (n) > n + 1, then f (n)2 ≥ (n + 2)2 , hence f (f (n)) = n2 + 3n + 3 − f (n)2 ≤ 2 n + 3n + 3 − (n + 2)2 = −n − 1 < 0 which is absurd. Therefore f (n) ≤ n + 1. If f (n) < n + 1, we have that f (n)2 < (n + 1)2 , and then f (f (n)) = n2 + 3n + 3 − f (n)2 > n2 + 3n + 3 − (n + 1)2 = n + 2 > f (n) + 1. Hence, f (f (n)) > f (n) + 1, which is impossible as follows from the previous case. Therefore f (n) < n + 1 cannot hold and f (n) = n + 1 is the only solution. Example 6.5.6. Find continuous functions f : R → R that satisfy the following: For each x ∈ R, there exists an integer n ≥ 1 such that f n (x) = x. (6.11) First, let us see that the function is bijective. Suppose that for x, y ∈ R, we have that f (x) = f (y), by property (6.11), there exist n, m ∈ N with f n (x) = x and f m (y) = y. It is clear that f nm (x) = x and f nm (y) = y. But if f (x) = f (y), then f nm (x) = f nm (y), hence x = y. Therefore, f is injective. Next, we will prove that the function is surjective. For each x, there exists n ∈ N with x = f n (x) = f (f n−1 (x)), remember that f 0 (x) = x. Now, we will show that f is increasing or decreasing, which is highlighted in the following more general lemma, not just for the function in the example. 114 Chapter 6. Functions and Functional Equations Lemma 6.5.7. If f : R → R is continuous and bijective, then f is increasing in all R or f is decreasing in all R. Proof. Let us see that in every open interval the function is increasing or decreasing. Let a, b ∈ R with a < b. Since f is injective, then f (a) < f (b) or f (a) > f (b). If f (a) < f (b), we will prove that f is increasing in (a, b), consider x, y with a < x < y < b. (a) It must happen that f (a) < f (x) < f (b), otherwise f (x) < f (a) or f (b) < f (x). In the first case, since f (x) < f (a) < f (b), by the intermediate value theorem there exists x1 ∈ (x, b) with f (x1 ) = f (a), which is a contradiction to the fact that f is injective. Similarly, if f (a) < f (b) < f (x), by the intermediate value theorem there exists x2 ∈ (a, x) with f (x2 ) = f (b) which is impossible, since f is injective. Thus, f (a) < f (x) < f (b). (b) Similarly, we have that for y with x < y < b, it follows that f (x) < f (y) < f (b). Therefore, f is increasing in (a, b). The case when f (a) > f (b) is similar, except that in this situation f will be decreasing.  Let us come back to the example. We will show that if f is increasing, then f (x) = x for all x. In fact, if for some x0 , we have that f (x0 ) = x0 , then f (x0 ) > x0 or f (x0 ) < x0 . But, since f is increasing we have that or x0 < f (x0 ) < f 2 (x0 ) < · · · < f n (x0 ) < . . . x0 > f (x0 ) > f 2 (x0 ) > · · · > f n (x0 ) > . . . , in any case, f n (x0 ) = x0 for every n ≥ 1, which contradicts (6.11). But if f is decreasing, then f 2 (x) = x for all x, in fact, if f is decreasing, we have that f 2 is increasing and then f 2 (x) = x. Finally, we point out some properties that the function has: f 2 (x) = x, for all x ∈ R, and it can be proved that the number n in (6.11) is 1 or 2. Exercise 6.27. Find the sum n # k=0 (−1)k k n n . k Exercise 6.28. Find all functions f : N ∪ {0} → N ∪ {0} that satisfy f (f (f (n))) + f (f (n)) + f (n) = 3n, for all n ∈ N ∪ {0}. Exercise 6.29. Find all the continuous functions f : [0, 1] → [0, 1] that satisfy f (0) = 0, f (1) = 1 and f n (x) = x, for all x ∈ (0, 1) with n ∈ N fixed. Exercise 6.30. Find all the continuous functions f : R → R such that there is a natural number n ≥ 1 with f n (x) = −x, for all x ∈ R. Chapter 7 Sequences and Series 7.1 Definition of sequence A sequence of numbers {an } can be thought of as a function f defined on the set of positive integers and whose images are a set of numbers A. This set can be: natural, integer, rational, real or complex numbers, that is, f : N n → A → f (n) = an . Sometimes it is useful to start the sequence with a0 . We call every element an of the sequence a term of the sequence. We can also think of a sequence as an infinite collection of ordered numbers. In the mathematical olympiad, the problems related to sequences are of different kinds. In some of them it is asked to find specific terms of the sequence, in others to prove that the terms are related in some particular way or that they satisfy certain identities. Also, there are some problems that require one to find a closed formula of the nth term or to prove that the nth term satisfies some property. In the following examples, we present a variety of these situations, and in the next section we will give some properties and characteristics that will help us to achieve the goal we have just described. The set of points an , with n = 1, 2, . . . , is called the range of the sequence. The range of the sequence can be finite or infinite. Example 7.1.1. ) ( (a) For the sequence an = n1 , the range is infinite; ( ) (b) If an = n2 , then this sequence has infinite range;   n , its range is infinite. (c) If an = 1 + (−1) n (d) The sequence {an = (−1)n } has finite range. © Springer Internationl Publishing Switzerland 2015 R.B. Manfrino et al, Topics in Algebra and Analysis, DOI 10.1007/978-3-319-11946-5_7 115 116 Chapter 7. Sequences and Series In this first example, the sequences exhibit certain orders or patterns, but not all sequences are like these. In Chapter 2 we studied the arithmetic and geometric progressions, which are examples of sequences that have a pattern or a rule that can be given in an explicit way, but a sequence {an } such that an is the nth digit in the decimal expression of π has no explicit rule. Let us analyze several examples to get more familiar with sequences. Example 7.1.2. The sequence a0 , a1 , a2 , . . . , is defined as a0 = 0, a1 = 1 and, for 2n . Find the value of a1000 . m ≥ n ≥ 0, am+n + am−n = a2m +a 2 If n = 0, we have that 2am = a2m2+a0 , then a2m = 4am . If m = 1 and n = 0, 2 = 4a22+4 = 10, a2 = 4a1 = 4 = 22 . If m = 2 and n = 1, then a3 + a1 = a4 +a 2 2 hence a3 = 9 = 3 . This suggests that an = n2 . We will use induction to prove this claim. In fact, it is only left to check the inductive step, 4an + a2 a2n + a2 = 2 2 = 2n2 + 2 − (n − 1)2 = (n + 1)2 . an+1 + an−1 = an+1 Thus, a1000 = 10002. Example 7.1.3. Define the sequence {an } as a1 = a2 = 1 and, for n ≥ 1, an+2 = an+1 an + 1. Which elements of the sequence are even and which ones are multiples of 4? By induction we can prove that an is a positive integer. If an−1 and an−2 are positive integers, then an = an−1 an−2 + 1 is also a positive integer. Let us see which terms are even. We have that a3 = a2 a1 + 1 = 2 is even, but a4 and a5 are not even; since by definition each of them is the sum of an even number and 1, then a4 and a5 are odd. However, a6 is even and the formula for a7 and a8 , tell us both are odd. Then, the sequence modulo 2 is 1, 1, 0, 1, 1, 0, . . . . The recursive relation an+2 = an+1 an + 1 generates an odd number if one of the factors is even, and an even number if both factors are odd. Then, the terms of the form a3k are even. If now we consider the sequence modulo 4, we see that it is given by 1, 1, 2, 3, 3, 2, 3, 3, . . . ; after the third term in the sequence, the numbers 2, 3, 3 are repeated, which shows that there are no multiples of 4. Example 7.1.4. The sequence {an } is defined by a1 = a2 = a3 = 1 and, for n ≥ 3, n an−1 . Then every element of the sequence is an integer. by an+1 = 1+aan−2 Observe that for n ≥ 3, the elements of the sequence satisfy an+1 an−2 = 1 + an an−1 , therefore an+2 an−1 = 1 + an+1 an . Subtracting the first equation from the second, we get an+2 an−1 − an+1 an−2 = an+1 an − an an−1 . 117 7.1 Definition of sequence After factoring and rearranging the terms, we obtain (an+2 + an )an−1 = (an + an−2 )an+1 an+2 + an an + an−2 = . an+1 an−1 n−2 If we define bn = ana+a , it follows that bn+2 = bn . That is, it happens that the n−1 even terms of {bn } are all equal and the odd terms are also equal to each other. Then, since b3 = a3 + a1 =2 a2 and b4 = we have that an =  a4 + a2 = a1 1+a3 a2 a1 + a2 a1 = 1+1 1 +1 = 3, 1 3an−1 − an−2 , if n is even 2an−1 − an−2 , if n is odd. By induction, we can conclude that an is an integer number. Example 7.1.5. The sequence {an } defined by a1 = 1 and an+1 = a2n + an , for 1 1 + · · · + 1+a < 1. n ≥ 1, satisfies that for any n, 1+a 1 n 1 Since an+1 = a2n + an , it follows that an+1 = an (a1n +1) = 1 1 1 equivalent to an +1 = an − an+1 . Adding, we obtain n # j=1 1 = 1 + aj 1 1 − a1 a2 + ···+ 1 1 − an an+1 1 an =1− − an1+1 , which is 1 < 1. an+1 Exercise 7.1 (Croatia, 2009). The sequence {an } is defined by a1 = 1, a2 = 3, an = an−1 + an−2 , for n ≥ 3.  n Prove that an < 74 , for all n. Exercise 7.2 (Croatia, 2009). The sequence {an } is defined by a1 = 1, an = 3an−1 + 2n−1 , for n ≥ 2. Find a formula for the general term an in terms of n. Exercise 7.3. The sequence {an } is defined by a1 = 1 and an+1 = 1 + a1 a2 . . . an , for n ≥ 1. Prove that for all n ≥ 1, 1 a1 + ··· + 1 an < 2. 118 Chapter 7. Sequences and Series Exercise 7.4. The sequence {an } is defined by a1 = a2 = 1 and an+1 = a2n + 1 , an−1 for n ≥ 2. Prove that every term of the sequence is a positive integer. Exercise 7.5 (MEMO, 2008). Let {an } be a sequence of positive integers such that an < an+1 for n ≥ 1. Suppose that for all 4-tuples (i, j, k, l) of indices, such that 1 ≤ i < j ≤ k < l and i + l = j + k, it follows that ai + al > aj + ak . Find the smallest possible value of a2008 . Exercise 7.6 (China, 2008). A sequence of real numbers {an } is defined by a0 = 0, 1, a1 = 1 − a0 and an+1 = 1 − an (1 − an ), for n = 1, 2, . . . . Prove that for each positive integer n, (a0 a1 . . . an ) 1 1 1 + + ···+ a0 a1 an = 1. Exercise 7.7. Let {xn } and {yn } be sequences defined by the equations xn+1 = x3n − 3xn and yn+1 = yn3 − 3yn . If x20 = y0 + 2, prove that x2n = yn + 2, for all n. Exercise 7.8. The sequence {an } is defined by a1 = 1, a2 = 12, a3 = 20 and an+3 = 2an+2 + 2an+1 − an , for n ≥ 1. Prove that 1 + 4an an+1 is a perfect square, for n ≥ 1. 7.2 Properties of sequences In this section we study some properties of the sequences that are useful to find specific relations among terms of the sequences, find closed formulas, etc. 7.2.1 Bounded sequences We say that a sequence {an } is bounded if there exist K > 0 such that |an | ≤ K, for all n ∈ N. That is, we say that a sequence is bounded if its range is bounded. For instance, it is clear that the sequences {an = n1 } and {an = (−1)n } are bounded by 1. However, there are sequences for which the bound has to be found. The most important example of a sequence that is not bounded is the following. 119 7.2 Properties of sequences Example 7.2.1. The sequence {an = n} is not bounded, since the set of natural numbers is not bounded. Suppose that N is bounded above. Then, there exists M > 0 such that n ≤ M , for all n ∈ N. Take ⌊M ⌋ the greatest integer less than or equal than M , then the integer ⌊M ⌋ + 1 satisfies that it is a positive integer with M < ⌊M ⌋ + 1, hence M is not an upper bound for N, which is a contradiction. Example 7.2.2. The sequence {an } given by 0 < a0 < a0 + a1 < 1 and an − 1 = 0, an−1 an+1 + for n ≥ 1, is a bounded sequence. Let us find a few terms of the sequence: a0 + a1 − 1 1 − a0 1 − a1 a2 = , a3 = , a4 = , a5 = a0 , a6 = a1 . a0 a0 a1 a1 Therefore, we see that the terms of the sequence are repeated every five terms, then it is bounded. In the last example, we can observe that the term a5 is equal to the term a0 , and in general we have that an+5 = an , for all n ∈ N. The sequences with this property have a special name, which will be studied next. 7.2.2 Periodic sequences A sequence {an } is periodic, with period p ≥ 1, if it satisfies that an+p = an , for all n ∈ N. If a sequence is periodic with period p, then we can find all the values of the sequence if we know the values of the first p terms of the sequence. Actually, if {an } is a sequence with period p and n is a positive integer, by Euclid’s algorithm, we can express n as n = pq + r, with 0 ≤ r < p. Then an = ar if r = 0, and an = ap if r = 0. Also, observe that every periodic sequence is bounded, moreover, the sequences with finite rank are clearly bounded. Example 7.2.3. The sequence {an } is defined by an+2 = a2013 . 1+an+1 . an Find the value of We analyze the first terms of the sequence a3 = 1 + a2 , a1 a4 = 2 1 + 1+a 1 + a3 1 + a1 + a2 a1 = = , a2 a2 a1 a2 1 +a2 1 + 1+a (1 + a1 )(1 + a2 ) 1 + a4 1 + a1 a1 a2 = = , a5 = = 1+a2 a1 a2 (1+a2 ) a3 a2 a a1 1 1 1 + 1+a 1 + a5 a2 a6 = = a1 , = 1+a1 +a 2 a4 a a 1 2 a7 = 1 + a6 1 + a1 = 1+a1 = a2 . a5 a 2 120 Chapter 7. Sequences and Series Then, the sequence is periodic with period 5, that is, an+5 = an for all n. Therefore, 2 a2013 = a3 = 1+a a1 . In some of the examples we have seen so far, we can notice that in order to define the general term of a sequence it is necessary to know some of the previous terms. We will come to this in the following section. 7.2.3 Recursive or recurrent sequences Some of the sequences we have thus far studied satisfy the condition that the term an+1 is a function of some of the previous terms, that is, an+1 = f (a1 , . . . , an ). Sequences of this sort are known as recurrent sequences or recursive sequences. More precisely, we will say that {an } satisfies the recursive equation an+1 = f (a1 , . . . , an ), (7.1) if for every n, the terms of {an } satisfy the last identity. Note that the function f is not the same for each n, for instance, if f (a1 , . . . , an ) = a1 + a2 + · · · + an we have that a2 = f (a1 ) = a1 , a3 = f (a1 , a2 ) = a1 + a2 , . . . . The simplest examples of recursive sequences are the arithmetic progressions an = a1 + (n − 1)d that satisfy the recurrent equation an+1 = an + d and the geometric progressions an = rn−1 a1 , that solve the recurrent equation an+1 = ran . Example 7.2.4. A sequence that generalizes the arithmetic and geometric progressions is the sequence that solves the recursive equation, an+1 = rn an + dn , where {rn } and {dn } are sequences independent of the terms an . Let us find a closed formula for an . It is evident that if for every integer n the equality an = rn−1 an−1 + dn−1 holds, then an = rn−1 an−1 + dn−1 rn−1 an−1 = rn−1 rn−2 an−2 + rn−1 dn−2 rn−1 rn−2 an−2 = rn−1 rn−2 rn−3 an−3 + rn−1 rn−2 dn−3 .. .. . . rn−1 · · · r2 a2 = rn−1 · · · r2 r1 a1 + rn−1 · · · r2 d1 . After adding and canceling terms, we get that an = (rn−1 · · · r1 ) a1 + n−2 # j=1 rn−1 · · · rj+1 dj + dn−1 . In particular, for an+1 = ran +d, it follows that an = rn−1 a1 +(1+r+· · ·+rn−2 )d. In Example (3.1.4) of the Hanoi’s towers, we notice that the number of necessary movements hn to move n disks from one stick to another, satisfies the 121 7.2 Properties of sequences recursive formula hn+1 = 2hn + 1. This formula confirms the value that we found for hn , because it implies that, hn = 2n−1 · h1 + (1 + 2 + · · · + 2n−2 ) · 1 = 2n−1 + 2n−1 − 1 = 2n − 1. We say that {an } is a recurrent linear sequence of order k ≥ 1, if it satisfies the recursive equation an+k = c1 an+k−1 + c2 an+k−2 + · · · + ck an , where c1 , . . . , ck are constant numbers. For instance, the Fibonacci sequence {fn }, defined as the sequence that satisfies the Fibonacci recursion formula fn+1 = fn−1 + fn , with f1 = f2 = 1, is a recurrent linear sequence of order 2. A geometric progression is a recurrent linear sequence of order 1, since an+1 = ran , and these sequences are the only ones of order 1, where r can be any number. An arithmetic progression satisfies an+1 = an + d, which is not a linear recursion because of the constant term d. However, since an+2 = an+1 + d, it follows that an+2 − an+1 = an+1 − an , so that an+2 = 2an+1 − an . Thus the arithmetic progressions are recurrent linear sequences of order 2. Our next objective is to solve the linear recursions of second order, that is, we want to recognize the sequences that satisfy the recursive equation an+2 = ban+1 + can , (7.2) where b and c are fixed constants. For instance, the recursive equation an+1 = 5an − 6an−1 is solved by the sequences {2n } and {3n }, which shows that there is not always only one sequence that solves the equation. Moreover, if {an } and {bn } are sequences that solve a recursive linear equation, then also {Aan + Bbn } solves the equation, for any numbers A and B. Hence there could be several sequences solving equation (7.2). However, if we have that the first two terms of each of two sequences that solve a linear recursion of order 2 are equal, then the two solutions coincide. This follows because, if the first two terms of each solution coincide, then the third terms coincide too and, by induction, all the terms of the two sequences coincide. An application of this remark can be found in the following example. Example 7.2.5. If {fn } is the Fibonacci sequence, then fm+n = fm fn−1 + fm+1 fn , for m ≥ 0 and n ≥ 1, where f0 = 0. Define the sequences am = fm+n and bm = fm fn−1 + fm+1 fn , for m ≥ 0 and with n ≥ 1 fixed. It is easy to show that am and bm satisfy the Fibonacci recursion formula. For instance, am+2 = fm+2+n = fm+1+n + fm+n = am+1 + am and bm+2 = fm+2 fn−1 + fm+3 fn = (fm+1 + fm )fn−1 + (fm+2 + fm+1 )fn = (fm+1 fn−1 + fm+2 fn ) + (fm fn−1 + fm+1 fn ) = bm+1 + bm . 122 Chapter 7. Sequences and Series On the other hand, a0 = fn , a1 = fn+1 , b0 = f0 fn−1 + f1 fn = fn and b1 = f1 fn−1 + f2 fn = fn−1 + fn = fn+1 . Since both sequences satisfy the same linear recurrence of order 2 and coincide in the first two terms, we have that the sequences are equal. Therefore the identity holds. Let us go back to see how to solve the linear recursions of order 2. Following the idea that the linear recursions of order 1 are solved by sequences of the form an = Aλn , let us see what a sequence of the form {an = Aλn } should satisfy in order to be a solution of (7.2). Substituting in equation (7.2), we get Aλn+2 = bAλn+1 + cAλn . If A = 0, it is clear that the constant sequence (7.2). If A = 0, we   an = 0 satisfies can cancel A and after factoring we get λn λ2 − bλ − c = 0. Now, if for some integer n we have that λn = 0, that is, λ = 0 and, therefore an = 0, which we know solves the equation. Now, suppose λ = 0, hence λ2 − bλ − c = 0, so that λ = an = Aλn . √ b± b2 +4bc 2 (7.3) are the only possible values if the solution is of the form Equation (7.3) is known as the characteristic equation of the recursion formula (7.2) and the polynomial on the left is known as the characteristic polynomial. To conclude we analyze two cases. The first corresponding to the roots of the characteristic equation being different and then the case when they are equal. Case A. λ1 and λ2 are the solutions of equation (7.3), with λ1 = λ2 . In this case, we notice that an = Aλn1 + Bλn2 solves equation (7.2). Now, let us see that, if {bn } is a sequence that satisfies the equation, then bn = Aλn1 + Bλn2 for some numbers A and B. For this, we know that it is enough to see that a0 = b0 and a1 = b1 . Then we have to solve a0 = A + B a1 = Aλ1 + Bλ2 . But this system of two equations with two unknowns can be solved in a unique way for A and B. In fact, A= a0 λ2 − a1 λ2 − λ1 and B= a1 − λ1 a0 , λ2 − λ1 and this is the only solution of the system when λ2 − λ1 = 0. 123 7.2 Properties of sequences Case B. The roots of the characteristic polynomial λ1 and λ2 coincide. In this case an = Aλn1 + Bλn2 is not a general solution anymore, since an = (A + B)λn1 , and it is not always possible to choose A and B such that A + B = a0 and (A + B)λ1 = a1 . However, there is another solution of the recursion formula different from λn1 ; this happens to be sequence bn = (n + 1)λn1 . In order to see that this sequence satisfies the recursion, first note that if λ1 = λ2 are the roots of λ2 − bλ − c = 0, by Vieta, b = 2λ1 and c = −λ21 . Then, the recursion is an+2 = 2λ1 an+1 − λ21 an and we can verify that (n + 3)λn+2 = 2λ1 (n + 2)λn+1 − λ21 (n + 1)λn1 , 1 1 which proves that bn = (n + 1)λn1 solves the recursion formula. Now, the two known solutions bn = λn1 and cn = (n + 1)λn1 generate the general solution an = Aλn1 + (n + 1)Bλn1 . In this case the initial conditions determine A and B, that is, there is only one pair of numbers A and B with a0 = A + B a1 = (A + 2B)λ1 . 0 λ1 Actually, A = 2a0 λλ11−a1 and B = a1 −a . Then, in this case, the solution with λ1 initial conditions is unique. We can summarize both cases in the following result. Theorem 7.2.6. (a) If the roots of the equation λ2 − bλ − c = 0 are different (b2 + 4c = 0), then all the solutions an+2 = ban+1 + can , of the recursion formula are of the form an = Aλn1 + Bλn2 , where A and B are any real numbers. (b) If the equation λ2 − bλ − c = 0 has only one double real root equal to λ = 2b , then all solutions of the recursion are of the form an = (A + (n + 1)B)λn , where A and B are any real numbers. (c) If a0 and a1 are given numbers, then A and B are determined by a0 = A + B and a1 = Aλ1 + Bλ2 in case (a), and by a0 = A + B and a1 = (A + 2B)λ in case (b). For instance, the recursion xn+2 = 2xn+1 − xn , has characteristic polynomial λ2 − 2λ + 1, with λ = 1 as the only root. Then, the solutions are of the form xn = (c + dn)1n = c + dn, something we already know as arithmetic progressions. 124 Chapter 7. Sequences and Series Example 7.2.7. Find the solutions of the Fibonacci recursion, f0 = 0, f1 = 1 and fn+2 = fn+1 + fn , for n ≥ 0. The characteristic equation is given by λ2 − λ − 1 = 0 and its roots are √ λ1 , λ2 = 1±2 5 , which are different. Then, the solutions of the recursion are of the $ √ %n $ √ %n form fn = Aλn1 + Bλn2 = A 1+2 5 + B 1−2 5 . Because the first terms are f0 = 0 and f1 = 1, then A = 1 fn = √ 5 √1 5 = −B. Hence the Fibonacci numbers fn are  √ n  √ n 5 1+ 5 1− 5 . − 2 2 Example 7.2.8. Find the solutions of the recursion defined by a0 = 0, a1 = sin α and an+2 = 2 cos α · an+1 − an , for n ≥ 0 and α = nπ. The characteristic polynomial of the given recursion is λ2 − 2 cos α λ + 1 = 0, which has solutions √ 2 cos α ± 4 cos2 α − 4 = cos α ± i sin α. λ1 , λ2 = 2 Hence, the solutions of the recursion are of the form an = Aλn1 + Bλn2 . From the initial conditions we obtain that a0 = A+B = 0 and a1 = Aλ1 +Bλ2 = sin α. From the first equation we have that B = −A, so that A(λ1 − λ2 ) = A(2i sin α) = sin α, 1 . Therefore, and then, using the fact that sin α = 0, A = 2i 1 {(cos α + i sin α)n − (cos α − i sin α)n } 2i 1 = (cos nα + i sin nα − cos nα + i sin nα) 2i = sin nα. an = Example 7.2.9. Analyze the non-linear recurrent equation an+1 = a2n − 2. n n It is clear that if a0 = a + a1 , then an = a2 + a−2 solves the recurrence,  n n 2 n+1 n+1 since a2n − 2 = a2 + a−2 − 2 = a2 + a−2 = an+1 . If |a0 | > 2, then √ 2 a0 + a0 −4 1 a= satisfies a0 = a + a , if |a0 | ≤ 2, so that we can take a0 = 2 cos θ, 2 and therefore a = eiθ = cos θ + i sin θ. 7.2.4 Monotone sequences A sequence {an } of real numbers is monotone increasing if an ≤ an+1 , for all n ∈ N. The sequence is increasing if an < an+1 , for all n ∈ N. 125 7.2 Properties of sequences Similarly, we say that a sequence {an } of real numbers is monotone decreasing if an ≥ an+1 , for all n ∈ N. The sequence is decreasing if an > an+1 , for all n ∈ N. Example 7.2.10. (a) Any arithmetic progression with difference d > 0 is an increasing sequence. (b) Any geometric sequence with a1 > 0 is monotone increasing if r ≥ 1. In (a), if {an } is the arithmetic progression with difference d, then an+1 −an = d > 0, where clearly an < an+1 . In (b), if {an } is a geometric progression with ratio r ≥ 1, then is clear that an+1 = ran ≥ an . Example 7.2.11. The sequence defined by 0 < a1 < increasing. 1 2 and an+1 = 2an (1 − an ) is In order to see that it is increasing, we will need to show that 0 < an and = 2(1 − an ), that is, 0 < an < 12 for all n. Let us proceed by induction. First note that if 0 < an < 21 , then an+1 = 2an (1 − an ) > 0. Now using the geometric mean and the arithmetic mean inequality, we have that 1< an+1 an an+1 = 2an (1 − an ) ≤ 2 an + 1 − an 2 2 = 1 . 2 The inequality is strict since the equality holds when an = 1 − an = 12 , which is not the case in the previous expression. 7.2.5 Totally complete sequences A sequence {an } of positive integers is called totally complete if every positive integer can be expressed as a sum of one or more different terms of the sequence. Clearly the sequence of positive integers {1, 2, 3, . . . , n, . . . } is totally complete. Example 7.2.12. The sequence of powers of 2, {20 , 21 , 22 , . . . , 2n , . . . } is totally complete. We will give a proof using strong induction. The basis cases are evident, since 1 = 20 and 2 = 21 . Suppose that any integer less than n can be written as a sum of different powers of 2. Let 2m be the greatest power of 2 that is less than or equal to n, then 2m ≤ n < 2m+1 . Set d = n − 2m , which is clearly less than or equal to n, and also less than 2m (n < 2m+1 implies that n − 2m < 2m ). By the induction hypothesis, d can be expressed as the sum of different powers of 2 and, since d is less than 2m , this power is not included in the representation of d. Adding 2m to the representation of d we get the representation of n. 126 Chapter 7. Sequences and Series Proposition 7.2.13. A sequence of positive integers {an } that satisfies a1 = 1 and an+1 ≤ 1 + a1 + a2 + · · · + an , for all n = 1, 2, . . . , is totally complete. Proof. We will show, using induction on n, that every integer k less than or equal to a1 + a2 + · · · + an , is the sum of different terms of {a1 , a2 , .., an }. If n = 1, the only k ≤ a1 = 1 is k = 1 and clearly k = a1 = 1. If n = 2, the numbers k to consider are the ones that satisfy k ≤ a2 ≤ 1 + a1 . Since a2 ≤ 1 + a1 = 2, it follows that a2 = 1 or 2. If a2 = 1, k = 1 or 2, then k = 1 = a1 and k = 2 = a1 + a2 . If a2 = 2, k = 1, 2 or 3, then k = 1 = a1 , k = 2 = a2 and k = 3 = a1 + a2 . For the inductive step, suppose the statement true for n and let us prove it for n + 1. Consider a positive integer k that satisfies k ≤ a1 + · · · + an + an+1 . If k ≤ a1 +· · ·+an , by the induction hypothesis, such k is the sum of different elements of {a1 , a2 , . . . , an }. Suppose then that 1+a1 +· · ·+an ≤ k ≤ a1 +· · ·+an +an+1 . Then, since an+1 ≤ 1 + a1 + a2 + · · · + an , it follows that an+1 ≤ k ≤ a1 + · · · + an + an+1 , hence 0 ≤ k − an+1 ≤ a1 + · · · + an . Now, if k − an+1 = 0 the proof is finished, and if 0 < k − an+1 ≤ a1 + · · · + an , then, by the induction hypothesis, k − an+1 is the sum of different elements of the set {a1 , a2 , . . . , an }. Adding an+1 , we have that k is the sum of different elements of {a1 , . . . , an , an+1 }, as we wanted to prove.  Example 7.2.14. The Fibonacci sequence 1, 1, 2, 3, 5, . . . , fn , . . . is totally complete. By the last proposition, it is enough to see that fn+1 ≤ 1 + f1 + f2 + · · · + fn , for every n ≥ 1. For n = 1, it is clear since 1 = f2 ≤ 1 + f1 = 2. For n = 2, the result follows because 2 = f3 ≤ 1 + f1 + f2 = 3. For n ≥ 3, it is enough to show that 1 + f1 + f2 + · · · + fn − fn+1 ≥ 0. But since fn+1 = fn−1 + fn , it follows for n ≥ 3 that 1 + f1 + f2 + · · · + fn − fn+1 = 1 + f1 + f2 + · · · + fn − (fn−1 + fn ) = 1 + f1 + f2 + · · · + fn−2 ≥ 0. 7.2.6 Convergent sequences A sequence {an } converges to or has limit a if for all ǫ > 0 there exists a natural number N such that, for all n ≥ N , it follows that |an − a| < ǫ. This can be written as, ∀ ǫ > 0, ∃ N ∈ N such that, ∀ n ≥ N, |an − a| < ǫ. We say that a is the limit of the sequence and write limn→∞ an = a. A sequence diverges if it does not converge to any point a. 127 7.2 Properties of sequences Example 7.2.15. The sequence {an = 1 n} converges to 0. Given ǫ > 0 we would like to show the existence of N ∈ N such that, for all n ≥ N , it implies that 0 < n1 < ǫ. But if n ≥ N , then n1 ≤ N1 , therefore it is enough to show the existence of N with N1 < ǫ. If such N does not exist, that is if N1 ≥ ǫ, then N ≤ 1ǫ for all N ∈ N. This is a contradiction, because the natural numbers are not bounded. Example 7.2.16. The sequence {an = an } converges to 0 if |a| < 1. If |a| < 1, then 1 |a| > 1, hence 1 |a| n = 1 + p with p > 0. Thus, (1 + p)n = 1 1 1 + np + n(n−1) p2 + · · · ≥ np, and |a| = (1+p) n ≤ np . Choose N ∈ N such that if 2 1 1 n n ≥ N , then n1 < pǫ, so that |a|n = (1+p) n ≤ np < ǫ. Thus {a } converges to 0. 7.2.7 Subsequences Given a sequence {an }, consider a sequence {nk } of positive integers, such that n1 < n2 < n3 < · · · . The sequence {ank } is called a subsequence of {an }. Note that given a sequence, we can obtain an infinite number of subsequences from it. Example 7.2.17. The sequence {an } defined by  1 if n is even, n, an = 2 n , if n is odd, has subsequences that are convergent, not convergent, bounded, not bounded, increasing and decreasing. First note that the sequence is not convergent, and neither bounded nor monotone, but we can find the following subsequences: (a) If we take {a2n }, the sequence is convergent, decreasing and bounded, since 1 for all n ∈ N. the terms are given by 2n (b) If we take {a2n−1 }, the sequence is not convergent, increasing and not bounded, since the terms are given by (2n − 1)2 , for all n ∈ N. Exercise√7.9. Prove that the sequence {an } defined by a0 = 0 and for n ≥ 0, an+1 = 4 + 3an , is a bounded sequence. Exercise 7.10. A sequence {an } is defined by a1 = 1, an+1 = an + 1 , for n ≥ 1. a2n Determine if the sequence is bounded or not and prove that a9000 > 30. Exercise 7.11. The terms of the sequence {an } are positive and a2n+1 = an + 1, for all n. Prove that the sequence contains at least one irrational number. 128 Chapter 7. Sequences and Series Exercise 7.12. Find, in each case, the solutions of the recursive equation of degree 3, an+3 = 3an+2 − 3an+1 + an , if: (i) a1 = a2 = a3 = 1. (ii) a1 = 1, a2 = 2, a3 = 3. (iii) a1 = 1, a2 = 4, a3 = 9. Exercise 7.13. The positive integers a1 , a2 , . . . , are bounded and form a sequence that satisfies the following condition: if m and n are positive integers, then am +an is divisible by am+n . Prove that the sequence is periodic after some terms. Exercise 7.14. Prove that an = n!, the number of permutations of n elements, and dn , the number of permutations of n elements without fixed points, satisfy the recursive equation xn+1 = n(xn + xn−1 ). Why is an = dn for all n? Note: a permutation without fixed points is called a derangement. Exercise 7.15. (i) Use the recursive formula for the number of derangements of a set of n elements, given by dn = (n−1)(dn−1 +dn−2 ), to prove that dn = ndn−1 +(−1)n . (ii) Use Example 7.2.4 to justify the formula dn = n! 1 + (−1)2 (−1)n (−1)1 + + ···+ 1! 2! n! . Exercise 7.16. Lucas’ numbers are defined by the recurrence L1 = 1, L2 = 3 and Ln+1 = Ln + Ln−1 , for n ≥ 2. Prove that  √ n  √ n 1+ 5 1− 5 Ln = + . 2 2 Exercise 7.17. Solve the recursive equation bn+1 = b0 is a fixed positive number. bn 1+bn , for n = 0, 1, 2, . . . , where Exercise 7.18. Prove that the sequence defined by a0 = a = 1 and an+1 = is periodic. 1 1−an , Exercise 7.19. Solve the recursive equation an+1 = 4− a4n . Prove that an converges to 2. Exercise 7.20. Prove that a sequence {an } that satisfies a1 = 1 and an+1 ≤ 2an , for n ≥ 1, is totally complete. Exercise 7.21. Prove that the sequence {1, 2, 3, 5, 7, 11, 13, . . .} of all prime numbers and the number 1, is totally complete. 129 7.3 Series Exercise 7.22. Two brothers inherit n golden pieces with total weight 2n. Each piece has an integer weight and the heaviest of all the pieces does not weigh more than all the others together. Prove that if n is even, then the brothers can divide the golden pieces into two parts with equal weight. Exercise 7.23 (Romania, 2009). A sequence {an } is defined by a1 = 2, an+1 = an + 1 , for n ≥ 1. n Prove that the limit of the sequence exists and is 1. 7.3 Series Given a sequence {an }, to denote the sum ap + ap+1 + · · · + aq with p ≤ q, we use the notation q # an . n=p To a sequence {an } we associate the sequence {sn }, called the sequence of partial sums, given by n # ak . sn = k=1 The infinite sum a1 + a2 + a3 + · · · can be written in short form as ∞ # an = lim n=1 N →∞ N # an = lim sN . n=1 N →∞ This last expression is called infinite series or, more simply, series. If {sn } converges to s, we say that the series converges to s and we write ∞ # an = s. n=1 The number s is called the sum of the series. We say that the series diverges if {sn } diverges. Example 7.3.1. Find the sum ∞ # 1 . n(n + 1) n=1 130 Chapter 7. Sequences and Series The partial sum is 1 1 1 + + ··· + 1·2 2·3 n · (n + 1) 1 1 1 1 1 − − + + ···+ = 1− 2 2 3 n n+1 sn = =1− 1 . n+1 1 Since { n+1 } → 0, when n → ∞, then {sn } → 1. Thus ∞ * n=1 1 = 1. n(n + 1) Exercise 7.24. If {an } is an arithmetic progression with difference d = 0, prove that: n * 1 1 1 1 (i) = − . a · a d a a i i+1 0 n+1 i=0 n * 1 1 1 1 = − . (ii) 2d a0 · a1 an+1 · an+2 i=0 ai · ai+1 · ai+2 ∞ ∞ * * 1 1 1 1 = . (iv) = . (iii) da0 2d · a0 · a1 i=0 ai · ai+1 i=0 ai · ai+1 · ai+2 Exercise 7.25. Let fn be the Fibonacci sequence (f1 = 1, f2 = 1, fn+1 = fn +fn−1 ). Find the sum of the following series: ∞ ∞ * * 1 fn (ii) . (i) n=2 fn−1 fn+1 n=2 fn−1 fn+1 Exercise 7.26. The Koch snowflake is obtained by means of the following process: (i) In step 0, the curve is an equilateral triangle of side 1. (ii) In step n + 1, the curve is obtained from the curve in step n, dividing each one of the sides in 3 equal parts, and constructing externally in the middle part of the divided side, an equilateral triangle erasing the side in which it was constructed. In the following diagram steps 0 and 1 are shown. If Pn and An are the perimeter and the area, respectively, of the curve of step n, find: (i) Pn (ii) An (iii) lim Pn n→∞ (iv) lim An . n→∞ 131 7.3 Series Exercise 7.27. Consider the harmonic progression ∞ (1) * n . The series n=1 as the harmonic series. 1 n is known 1 1 1 1 + + · · · + n+1 > . 2n + 1 2n + 2 2 2 1 1 1 (ii) Prove that for n ≥ 2, + + · · · + 2 > 1. n n+1 n 1 1 3 1 + + > . (iii) Prove that for n ≥ 2, n−1 n n+1 n (iv) Use any of the previous inequalities to conclude that the harmonic series is divergent. (i) Prove that for n ≥ 1, 7.3.1 Power series The following expression is known as formal power series in the variable x with center at zero, f (x) = a0 + a1 x + a2 x2 + · · · + an xn + · · · , (7.4) where a0 , a1 , a2 , . . . is an arbitrary sequence of numbers. The power series can be written as ∞ # f (x) = an xn . n=0 Let us see some examples of how to calculate these series. Example 7.3.2. The sum of the geometric series is given by ∞ # axn = n=0 a , if 1−x |x| < 1. Consider the partial sum sn = a + ax + ax2 + · · · + axn = a(1 + x + · · · + xn ) =a 1 − xn+1 1−x Then, limn→∞ sn = limn→∞ a n n=0 ax = 1−x . *∞ 16 See Example 7.2.16. a(1−xn+1 ) 1−x . = a 1−x , since16 |x| < 1. Therefore 132 Chapter 7. Sequences and Series Example 7.3.3. The following series, known as the derivative series of the geometric series, converges to the given value ∞ # nxn−1 = n=1 1 , for |x| < 1. (1 − x)2 *n Consider the partial sum k=0 kxk = x + 2x2 + 3x3 + · · · + nxn . This partial sum is the sum of the following partial sums: x + x2 + x3 + · · · + xn = x x2 + x3 + · · · + xn = x2 x3 + · · · + xn = x3 .. . .. . xn = xn 1 − xn 1−x 1 − xn−1 1−x 1 − xn−2 1−x 1−x 1−x . Adding these sums, we get x(1 − xn ) + x2 (1 − xn−1 ) + · · · + xn (1 − x) 1−x x + x2 + · · · + xn − nxn+1 = 1−x $ % 1−xn x 1−x − nxn+1 = 1−x x(1 − xn ) nxn+1 . (7.5) − = (1 − x)2 1−x x + 2x2 + 3x3 + · · · + nxn = The value of the infinite sum is ∞ # nxn = lim n→∞ n=1 x(1 − xn ) nxn+1 − (1 − x)2 1−x = x , (1 − x)2 (7.6) since xn+1 and nxn+1 go to zero as n → ∞, because |x| < 1. Therefore, canceling one x on both sides, we have the desired equality. Example 7.3.4. The sum of the following series is given by ∞ # n=0 n(n − 1)xn = 2x2 , for |x| < 1. (1 − x)3 133 7.3 Series Consider the partial sum sn = n # k=0 k(k − 1)xk = 2x2 + 3 · 2x3 + 4 · 3x4 + · · · + n(n − 1)xn = 2x x + 3x2 + 2 · 3x3 + · · · + n(n − 1) n−1 x . 2 We can write the factor in parentheses as x + 3x2 + 6x3 + · · · + n(n − 1) n−1 = x + x2 + x3 · · · + xn−1 x 2 + 2x2 + 2x3 + · · · + 2xn−1 + 3x3 + · · · + 3xn−1 .. . =x = 1 − xn−1 1−x + 2x2 1 − xn−2 1−x + (n − 1)xn−1 + · · · + (n − 1)xn−1 1−x 1−x [x + 2x2 + · · · + (n − 1)xn−1 ] − [xn + 2xn + · · · + (n − 1)xn ] . 1−x By equation (7.5) and the Gauss sum17 , we have that this sum is equal to & ' & $ %' x(1−xn−1 ) (n−1)xn n n(n−1) − x − 2 (1−x) (1−x) 2 . (7.7) 1−x Therefore the value of the series is ⎛& x(1−xn−1 ) ∞ − # (1−x)2 n(n − 1)xn = lim 2x ⎝ n→∞ n=0 (n−1)xn (1−x) ' 1−x & $ %'⎞ − xn n(n−1) 2 ⎠= 2x2 . (1 − x)3 7.3.2 Abel’s summation formula The sums calculated in some of the previous examples in this section can be simplified using what is known as the Abel’s summation formula. Let a1 , a2 , . . . , an and b1 , b2 , . . . , bn be two finite sequences of numbers. Then n # ai b i = i=1 n−1 # i=1 (ai − ai+1 )(b1 + · · · + bi ) + an (b1 + b2 + · · · + bn ), which can be proved simplifying the right-hand side of the identity. 17 See Section 2.1. 134 Chapter 7. Sequences and Series Example 7.3.5. Using the Abel’s summation formula, find the value of the sum *n k−1 , for q = 1. k=1 kq Using the Abel’s summation formula, we obtain that n # kq k−1 = k=1 n−1 # (k − (k + 1))(1 + q + · · · + q k−1 ) + n(1 + q + · · · + q n−1 ) k=1 n−1 # =− k=1 qk − 1 +n q−1 qn − 1 nq n − . = q − 1 (q − 1)2 qn − 1 q−1 =− n−1 1 # k n−1 +n q + q−1 q−1 qn − 1 q−1 k=1 Example 7.3.6 (Rearrangement inequality). If a1 ≤ a2 ≤ · · · ≤ an and b1 ≤ b2 ≤ · · · ≤ bn are two collections of real numbers in increasing order and (b′1 , b′2 , . . . , b′n ) is a permutation of (b1 , b2 , . . . , bn ), then it follows that a1 b1 + a2 b2 + · · · + an bn ≥ a1 b′1 + a2 b′2 + · · · + an b′n . Apply the Abel’s summation formula to the difference of the sums n # i=1 ai b i − n # ai b′i = n # i=1 i=1 = n−1 # i=1 = n−1 # i=1 ai (bi − b′i ) (ai − ai+1 ) i # j=1 ⎛ (ai − ai+1 ) ⎝ bj − i # j=1 i # + an i # j=1 *i n # j=1 j=1 bj − since for every i = 1, . . . , n − 1, ai ≤ ai+1 and b′j ⎞ bj − n # b′j j=1 b′j ⎠ ≥ 0, j=1 bj ≤ *i ′ j=1 bj . Exercise 7.28. Find, using Abel’s summation formula, the value of the sum n # k=1 k 2 q k−1 , with q = 1. Exercise 7.29. Prove that the following series converges to the given value: ∞ # n=0 n2 xn = 2x2 x + , for |x| < 1. 3 (1 − x) (1 − x)2 Exercise 7.30. Find the sum of the following series: n n ∞ ∞ 2 ∞ n ∞ * * * * −1 3 . (ii) . (iii) . (iv) . (i) n n 3 n=1 n=1 3 n=1 2 n=0 4 (v) ∞ n2 * . n n=1 2 7.4 Convergence of sequences and series ⋆ 135 7.4 Convergence of sequences and series ⋆ In this section, we shall present the proofs of the convergence theorems and their properties. However, you can continue reading the book without studying this section. Remember that a sequence {an } converges to or has a limit a if ∀ ǫ > 0, ∃ N ∈ N such that ∀ n ≥ N ⇒ |an − a| < ǫ. If a is the limit of the sequence, we write limn→∞ an = a or briefly an → a. If for any a ∈ R the sequence {an } does not converge to a, we will say that the sequence diverges. Theorem 7.4.1. If a sequence {an } converges to a1 and to a2 , then a1 = a2 . That is, the limit is unique. Proof. If a1 = a2 , take ǫ = 21 |a1 − a2 | > 0. Since {an } converges to a1 and to a2 , there exist N1 and N2 ∈ N such that |an − a1 | < ǫ, if n ≥ N1 and |an − a2 | < ǫ, if n ≥ N2 . For n ≥ N = max(N1 , N2 ), it follows that |a1 − a2 | ≤ |a1 − an | + |a2 − an | < 2ǫ = |a1 − a2 |, which is a contradiction. Hence, a1 = a2 .  Next we present some properties of limits of sequences. Theorem 7.4.2. If limn→∞ an = a, limn→∞ bn = b and α is any real number, then: (a) lim (an + bn ) = a + b. n→∞ (b) lim αan = α a. n→∞ (c) lim an bn = ab. n→∞ (d) If b = 0, then for bn = 0 and n large enough it happens that lim n→∞ 1 1 = , bn b lim n→∞ an a = . bn b Proof. We will prove only parts (a) and (d); the rest is left as an exercise for the reader. (a) Since limn→∞ an = a and limn→∞ bn = b, then there exist N1 , N2 ∈ N such that ǫ ǫ |an − a| < , for n ≥ N1 and |bn − b| < , for n ≥ N2 . 2 2 If N = max(N1 , N2 ), then for n ≥ N it follows that |an + bn − (a + b)| ≤ |an − a| + |bn − b| ≤ ǫ ǫ + = ǫ. 2 2 136 Chapter 7. Sequences and Series (d) Since b = 0, it follows that |b| 2 > 0. Since bn → b, there exists N1 ∈ N such |b| |b| that, for all n ≥ N1 , |bn − b| < |b| 2 , then |b| − |bn | ≤ |bn − b| < 2 and |bn | > 2 . 2 1 Hence |bn | < |b| , for all n ≥ N1 . Let ε > 0, since bn → b, considering all n ≥ N , |bn − b| < Observations 7.4.3. |b|2 2 ε. |b|2 ε 2 there exists N ≥ N1 such that, for Hence, for all n ≥ N ,    1  − 1  = |bn − b| ≤ 2 |bn − b| < ε. 2  bn b |b| |bn | |b|  (a) If {an } converges to a, then any open interval that contains the number a has an infinite number of terms of the sequence. Moreover, outside this interval there are only a finite number of terms of the sequence. (b) If an converges to a, then the sequence is bounded. Part (a) follows since there exists ǫ > 0, with I = (a − ǫ, a + ǫ), contained in the open interval, but in I there are an infinite number of terms of {an }. In order to prove (b), observe that the interval (a − ǫ, a + ǫ) contains all the terms an with n ≥ N for some N , then |an | < |a| + ǫ. Therefore, if we define K = max{|a| + ǫ, |a1 |, . . . , |an |}, it is clear that K is a bound for the sequence, that is, |an | ≤ K for all n ∈ N. Theorem 7.4.4. Let {an }, {bn } and {cn } be three sequences of real numbers. Suppose that there is N ∈ N such that, for all n ∈ N with n ≥ N , an ≤ bn ≤ cn holds. If {an } and {cn } converge to the same limit a, then {bn } converges to a. Proof. Let ǫ > 0, by hypothesis there exist N1 , N2 ∈ N such that: |an − a| < ǫ, for n ≥ N1 and |cn − a| < ǫ, for all n ≥ N2 . If N0 = max(N, N1 , N2 ), then for all n ≥ N0 , it follows that −ǫ < an − a ≤ bn − a ≤ cn − a < ǫ, which implies that |bn − a| < ǫ, that is, {bn } converges to a.  Theorem 7.4.5. A sequence {an } converges to a if and only if any subsequence of {an } converges to a. Proof. If the sequence converges to a, then limk→∞ ak = a, that is, for all ǫ > 0 there exists N ∈ N such that if k > N , then |ak − a| < ǫ. Let nk be an increasing sequence of positive integers and consider the subsequence {ank }. Since nk ≥ k and if k > N , then |ank − a| < ǫ, hence the subsequence converges. Suppose now that the sequence does not converge to a, then there exists ǫ > 0 such that, for all N ∈ N, there exists n > N such that |an − a| > ǫ. For 7.4 Convergence of sequences and series ⋆ 137 N = 1, there is n1 > 1 such that |an1 − a| > ǫ. For n1 , there is n2 > n1 such that |an2 − a| > ǫ. For n2 , there is n3 > n2 such that |an3 − a| > ǫ. Proceeding in the same way, we construct a subsequence {ank } that does not converge to a, which is a contradiction.  Theorem 7.4.6. If {an } is bounded, then there exists a subsequence of {an } that converges. Proof. In order to prove this theorem, we need to construct a convergent subsequence. Since the sequence is bounded, we know that there is M > 0 such that |an | ≤ M , for all n, that is, −M ≤ an ≤ M . Divide the closed interval18 [−M, M ] into two intervals [−M, 0] and [0, M ]. Consider the interval where there are a infinite number of terms of the sequence; suppose that this interval is [0, M ]. To construct the subsequence choose one of the terms in the interval [0, M ], say an1 . M Again, divide the interval [0, M ] into two intervals [0, M 2 ], [ 2 , M ] and choose the interval that contains an infinite number of terms of the sequence. Without loss of generality, we can assume that the interval is [ M 2 , M ]. Choose as a second element of the subsequence one term an2 such that n2 > n1 in the interval [ M 2 , M ]. Continuing this process, we will get a subsequence {ank } and a collection of nested closed intervals of lengths 2Mn . The infinite intersection of these closed intervals is not empty, in fact, it is a unique point19 , say a. This point a is the limit of the M subsequence, since |a − ank | < M  2k , moreover |a − anl | < 2k , for l ≥ k. Theorem 7.4.7. Every upper bounded, increasing sequence of real numbers (monotonically increasing) is convergent. Similarly, every lower bounded decreasing sequence of real numbers (monotonically decreasing) is convergent. Proof. Let {an } be an upper bounded increasing sequence. Since the sequence is bounded, Theorem 7.4.6 implies that there exists a subsequence {ank } that converges to a point a, that is, given ǫ > 0 there exists K ∈ N such that for all k ≥ K, it follows that |ank − a| < ǫ. Let N = nK , we would like to show that for all n ≥ N it follows that |an − a| < ǫ. Since nk ≤ n there is j ≥ k such that nK < · · · < nj ≤ n < nj+1 . Using that the sequence {an } is increasing, we have that anj − a ≤ an − a < anj+1 − a, so that |an − a| < ǫ. The other cases are similar.  Theorem 7.4.8. Let f be a function, then limx→a f (x) = b if and only if for every sequence {an } such that limn→∞ an = a, it follows that limn→∞ f (an ) = b. Proof. Suppose that limn→∞ an = a. Since limx→a f (x) = b, it follows that given ǫ > 0 there exists δ > 0 such that if |x − a| < δ, then |f (x) − b| < ǫ. By the convergence of {an } to a, for δ > 0, there is N ∈ N, such that if n ≥ N , 18 See 19 See Section 1.2, for the definition of interval. [17]. 138 Chapter 7. Sequences and Series |an − a| < δ. Then, since |an − a| < δ if n ≥ N , hence |f (an ) − b| < ǫ, that is, f (an ) → b. Conversely, suppose that limx→a f (x) is not b, that is, there exists ǫ > 0, such that for every δ > 0, there exists x with |x − a| < δ and |f (x) − b| ≥ ǫ. In this way, for all δ = n1 with n ∈ N, there exists an with |an − a| < n1 and |f (an ) − b| ≥ ǫ. Hence, {an } is a sequence that converges to a and it suffices that {f (an )} does not converge to b, which is a contradiction.  Theorem 7.4.9. A function f is continuous at a if and only if for every sequence {an } such that limn→∞ an = a, then limn→∞ f (an ) = f (a). The proof follows directly from the previous theorem, setting b = f (a). Theorem 7.4.10. The set of rational numbers is dense in the set of real numbers. Proof. Let (a, b) be an open interval and let ǫ = b−a > 0. As we proved in Example 7.2.15, there exists a positive integer n with 0< 1 < ǫ. n (7.8) For some m ∈ Z, it follows that a < m n < b, otherwise a and b are between two m+1 m m+1 consecutive numbers of the form m n and n , that is, n ≤ a < b ≤ n . Hence, m 1 m+1  ǫ = b − a ≤ n − n = n , which contradicts inequality (7.8). Lemma 7.4.11. If D ⊂ R is dense, then for every x ∈ R there exists a sequence {an } in D with limn→∞ an = x. Proof. Let x ∈ R, for every n ∈ N it follows, since D is dense, that there exists an ∈ x − n1 , x + n1 ∩D. It is clear that if |an −x| < n1 for all n, then limn→∞ an = x.  Theorem 7.4.12. If f : R → R is continuous and f (x) = 0 for all x ∈ D, where D is dense in R, then f (x) = 0 for all x ∈ R. Proof. By the previous lemma, for x ∈ R there exists a sequence {an } in D with limn→∞ an = x. Since f is continuous in x, by Theorem 7.4.8, it follows that f (x) = limn→∞ f (an ) = 0.  Corollary 7.4.13. If the functions f and g are continuous and coincide in a dense set, then they coincide in all points. The proof of the corollary follows from the last theorem, using f − g. Chapter 8 Polynomials 8.1 Polynomials in one variable A polynomial P (x) in one variable x is an expression of the form P (x) = an xn + an−1 xn−1 + · · · + a1 x + a0 , where a0 , a1 , . . . , an are constants and n ∈ N. Every term of the polynomial is called a monomial or simply a term. The constants ai are known as the coefficients of the polynomial. We will denote by A[x] the set of polynomials with coefficients in A and variable x. Usually, the set A is Z, Q, R or C. In this chapter we will study polynomials with complex coefficients, unless otherwise stated. If an = 0, we say that the polynomial has degree n. In this sense, an xn is the most important term of the polynomial, because it defines the degree and it is called the main term . The number a0 is the constant term. We write deg(P ) to denote the degree of P (x). A polynomial is constant if it has a unique term a0 . If the constant a0 is different from 0 we say that the polynomial has degree zero. If an = 1, we say that the polynomial is monic. There are some special names for polynomials whose degree is small. A polynomial is linear if it has degree 1. We have already studied the quadratic and cubic polynomials, which have degrees 2 and 3, respectively. If the polynomial has degree 4, it is called quartic. In the same way we did for quadratic and cubic polynomials, we say that two polynomials are equal if its coefficients are equal term by term, that is, if the coefficients of the monomials of the same degree are equal. A zero of the polynomial P (x) is a number r such that P (r) = 0. When P (r) = 0, we also say that r is a root or a solution of the equation P (x) = 0. As we did with the quadratic and cubic polynomials, we can add, subtract, multiply and divide polynomials. © Springer Internationl Publishing Switzerland 2015 R.B. Manfrino et al, Topics in Algebra and Analysis, DOI 10.1007/978-3-319-11946-5_8 139 140 Chapter 8. Polynomials Let P (x) = a0 + a1 x + a2 x2 + · · · + an xn , Q(x) = b0 + b1 x + b2 x2 + · · · + bm xm , be any two polynomials with n ≥ m. We define the sum as (P + Q)(x) = (a0 + b0 ) + (a1 + b1 )x + (a2 + b2 )x2 + · · ·+ (am + bm )xm + am+1 xm+1 + · · · + an xn . The difference as (P − Q)(x) = (a0 − b0 ) + (a1 − b1 )x + (a2 − b2 )x2 + · · · + (am − bm )xm + am+1 xm+1 + · · · + an xn . The product of a polynomial P (x) and a constant c is cP (x) = ca0 + ca1 x + ca2 x2 + · · · + can xn . The product of the two polynomials is (P Q)(x) = a0 b0 + (a0 b1 + a1 b0 )x + (a0 b2 + a1 b1 + a2 b0 )x2 + · · · +(a0 br + a1 br−1 + · · · + ai br−i + · · · + ar b0 )xr + · · · + (an bm )xn+m . Example 8.1.1. In order to multiply two polynomials20 we can use the previous definition or it is enough to multiply the coefficients. For instance, if we have the polynomials x4 + 3x3 + x2 − 2x + 5 and 3x3 + 2x2 + 6, its product can be obtained as follows: 1 3 1 −2 5 3 2 6 3 2 6 9 3 3 11 9 7 6 18 6 −12 30 2 −4 10 −6 15 2 6 29 16 −12 30 The product polynomial is 3x + 11x + 9x5 + 2x4 + 29x3 + 16x2 − 12x + 30. Exercise 8.1. Multiply the polynomials P (x) = 4x3 + 2x2 + 7x + 1 and Q(x) = 2x2 + x + 8. Evaluate the two polynomials and their product at x = 2. Exercise 8.2. Let P (x) = (1−x+x2 −· · ·+x100 )(1+x+x2 +· · ·+x100 ). Prove that after simplifying the product, the only terms left are those that have even powers of x. 8.2 The division algorithm Let P (x) = an xn + an−1 xn−1 + · · · + a1 x + a0 , with an = 0, Q(x) = bm xm + bm−1 xm−1 + · · · + b1 x + b0 , with bm = 0 20 See [4], p. 4 141 8.2 The division algorithm be polynomials of degree n and m, respectively, with m ≤ n and complex or real coefficients. The division algorithm says that given polynomials P (x) and Q(x) there exist unique polynomials H(x) and R(x) with real or complex coefficients, according to the case, such that P (x) = Q(x)H(x) + R(x), deg(R) < deg(Q) or R(x) = 0. In order to show how to find H(x) and R(x), let us see an example. These polynomials are known as the quotient and the remainder, respectively. Example 8.2.1. Let P (x) = x5 + x3 + 2x and Q(x) = x2 − x + 1, divide21 P (x) by Q(x) and find H(x), R(x). Dividing P (x) by Q(x) we get x3 + x2 + x x2 − x + 1 + x3 x5 5 −x 4 +2x 3 −x +x x4 +2x −x4 +x3 −x2 x3 −x3 −x2 +2x +x2 −x x 3 2 In this case, H(x) = x + x + x and R(x) = x. If R(x) = 0, we say that Q(x) divides P (x) and we write Q(x)|P (x). Note that the variable x must be the same in all polynomials; thus we can omit it sometimes. The division of a polynomial P (x) of degree n by a polynomial of the form x − a, gives P (x) = (x − a)Q(x) + r, with r∈R and deg(Q) = n − 1. Letting x = a, we get r = P (a), and therefore P (x) = (x − a)Q(x) + P (a) or P (x) − P (a) = (x − a)Q(x). (8.1) It follows from equation (8.1) that P (a) = 0 is equivalent to P (x) = (x − a)Q(x), (8.2) for some polynomial Q(x). Thus, we have proved the following theorem. Theorem 8.2.2 (Factor theorem). The number a is a root of a polynomial P (x) if and only if the polynomial P (x) is divisible by x − a. 21 See [4] p. 58 142 Chapter 8. Polynomials A polynomial H(x) is the greatest common divisor of P (x) and Q(x) if it satisfies: (a) H(x) divides P (x) and Q(x). (b) If K(x) is any other polynomial that divides P (x) and Q(x), then K(x) divides H(x). It can be proved that H(x) is unique, up to multiplication by a constant. There is a method called Euclid’s algorithm, that is used to find the greatest common divisor of two polynomials and which follows the same ideas of Euclid’s algorithm to find the greatest common divisor of two integers. Let us see an example. Example 8.2.3. Find the greatest common divisor of the polynomials x4 − 2x3 − x2 + x − 1 and x3 + 1. We perform the following divisions of polynomials x−2 x3 + 1 x4 − 2x3 − x2 + x − 1 −x −x4 −2x3 − x2 −1 3 +2 +2x − x2 +1 −x −x2 + 1 −x+ 1 x3 3 −x +1 +x x +1 x+1 − x2 +1 2 x +x x+ 1 −x− 1 0 Then, as when dealing with integers, the greatest common divisor is x + 1, which is the remainder before reaching 0 as a remainder. We can express the greatest common divisor above as a combination of the polynomials x4 − 2x3 − x2 + x − 1 and x3 + 1, following the inverse steps of the divisions as shown: x + 1 = (x3 + 1) − (−x2 + 1)(−x) = (x3 + 1) + x(−x2 + 1) ! " = (x3 + 1) + x (x4 − 2x3 − x2 + x + 1) − (x3 + 1)(x − 2) = x(x4 − 2x3 − x2 + x + 1) + (1 − x(x − 2))(x3 + 1) = x(x4 − 2x3 − x2 + x + 1) + (−x2 + 2x + 1)(x3 + 1). 143 8.2 The division algorithm If a1 and a2 are two different zeros of P (x), then by the factor theorem, P (x) = (x − a1 )Q1 (x), with Q1 (x) a polynomial. Since 0 = P (a2 ) = (a2 − a1 )Q1 (a2 ) and a1 = a2 , then Q1 (a2 ) = 0. Again, by the factor theorem Q1 (x) = (x − a2 )Q2 (x), with Q2 (x) a polynomial. Then, P (x) = (x − a1 )(x − a2 )Q2 (x) with deg(Q2 ) = n − 2. In general, if a1 , a2 , . . . , am are different zeros of P (x) we can write P (x) = (x − a1 )(x − a2 ) . . . (x − am )Q(x), for some polynomial Q(x), with deg(Q) = deg(P ) − m. If a is a zero of P (x), then the factor theorem guarantees that there exists a polynomial Q1 (x) with P (x) = (x − a)Q1 (x). If Q1 (a) = 0, we say that a is a zero of order 1, but if Q1 (a) = 0 we say that a is a zero of order greater than 1. If there is m ∈ N and a polynomial Q(x) such that, P (x) = (x − a)m Q(x), with Q(a) = 0, (8.3) then a is a root or a zero of P (x) of multiplicity m. One of the important consequences of the factor theorem is the following result: Theorem 8.2.4. If the polynomial P (x) = an xn + an−1 xn−1 + · · · + a1 x + a0 has n + 1 distinct roots, then the polynomial is identically zero. Proof. We proceed by induction on n. For n = 1, the result is clear, since a polynomial of degree 1, has only one root. Suppose that the result is true for n − 1; let us show that it is true for n. Suppose that r0 , r1 , . . . , rn are roots of the polynomial P (x). By the factor theorem, P (x) = (x − rn )Q(x), where the polynomial Q(x) = an xn−1 + bn−2 xn−2 + · · · + b0 has n distinct roots r0 , r1 , . . . , rn−1 . By induction, Q(x) is identically zero, hence P (x) also is identically zero.  Observation 8.2.5. The previous theorem guarantees that a polynomial of degree n must have at most n distinct roots. A polynomial P (x) = an xn + an−1 xn−1 + · · · + a1 x + a0 , with an = 0 is called reciprocal if ai = an−i , for all i = 0, 1, . . ., n. Example 8.2.6. The polynomials xn + 1, x5 + 3x3 + 3x2 + 1 and 6x7 − 2x6 + 4x4 + 4x3 − 2x + 6 are reciprocal polynomials. Theorem 8.2.7. A reciprocal polynomial P (x) of degree 2n, can be written as P (x) = xn Q(z), where z = x + x1 and Q(z) is a polynomial in z of degree n. Proof. Let P (x) = a0 x2n + a1 x2n−1 + · · · + a1 x + a0 , then $ a0 % a1 P (x) = xn a0 xn + a1 xn−1 + · · · + n−1 + n x x 1 1 P (x) = xn a0 xn + n + a1 xn−1 + n−1 + · · · + an . x x 144 Chapter 8. Polynomials Using the recursive formula (3.3), xk+1 + 1 = xk+1 xk + 1 xk x+ 1 x − xk−1 + 1 xk−1 , it is clear that we can express each term xk + x1k as a polynomial in z = x+ x1 .  Exercise 8.3. Divide P (x) = x8 − 5x3 + 1 by Q(x) = x3 + x2 + 1. Using the division algorithm, find the polynomials H(x) and R(x). Exercise 8.4. Prove that, for n ≥ 1, (x − 1)2 divides nxn+1 − (n + 1)xn + 1. Exercise 8.5. Let n be a positive integer. Find the roots of the polynomial Pn (x) = 1 + x(x + 1) x(x + 1) . . . (x + n − 1) x + + ···+ . 1! 2! n! Exercise 8.6. Determine the polynomials with real coefficients P (x) that satisfy P (0) = 0 and P (x2 + x + 1) = P 2 (x) + P (x) + 1 for all x ∈ R. Exercise 8.7. Prove that any polynomial P (x) of degree n, with a0 = 0, is reciprocal if and only if 1 xn P = P (x), for every x = 0. x Exercise 8.8. Prove that every reciprocal polynomial P (x) of odd degree is divisible by x + 1 and its quotient is a reciprocal polynomial of even degree. Exercise 8.9. If a is a root of a reciprocal polynomial P (x), prove that root of the polynomial. 1 a is also a Exercise 8.10. Determine for which integers n, the polynomial 1 + x2 + x4 + · · · + x2n−2 is divisible by the polynomial 1 + x + x2 + · · · + xn−1 . Exercise 8.11. Prove that the greatest common divisor of xn − 1 and xm − 1 is x(n,m) − 1, where (n, m) denotes the greatest common divisor of m and n. Exercise 8.12 (USA, 1977). Find all the pairs of positive integers (m, n) such that 1 + x + x2 + · · · + xm divides 1 + xn + x2n + · · · + xmn . Exercise 8.13 (Canada, 1971). Let P (x) be a polynomial with integer coefficients. Prove that if P (0) and P (1) are odd, then P (x) = 0 has no integer solutions. 145 8.3 Roots of a polynomial 8.3 Roots of a polynomial 8.3.1 Vieta’s formulas Vieta’s formulas (4.1) and (4.2) can be generalized for polynomials of higher degree. If a monic polynomial xn + an−1 xn−1 + · · · + a1 x + a0 has n roots x1 , x2 , . . . , xn , then xn + an−1 xn−1 + · · · + a1 x + a0 = (x − x1 )(x − x2 ) · · · (x − xn ) = xn − (x1 + · · · + xn )xn−1 + (x1 x2 + · · · + x1 xn + x2 x3 + · · · + xn−1 xn )xn−2 + · · · + (−1)n x1 · · · xn , hence, an−1 = −(x1 + · · · + xn ) an−2 = (x1 x2 + · · · + x1 xn + x2 x3 + · · · + xn−1 xn ) .. . # an−j = (−1)j xi1 xi2 · · · xij .. . (8.4) 1≤i1 <···<ij ≤n a0 = (−1)n x1 x2 . . . xn . The formulas (8.4) are known as Vieta’s formulas. Example 8.3.1. Consider the polynomial P (x) = xn − (x − 1)n , where n is an odd positive integer. Calculate the sum and the product of its roots. The polynomial P (x) can be written as n(n − 1) n−2 x − · · · + (−1)n ) 2 n(n − 1) n−2 x = xn − (xn − nxn−1 + − · · · − 1) 2 n(n − 1) n−2 = nxn−1 − x + · · · + 1. 2 P (x) = xn − (xn − nxn−1 + Then, the sum of its roots is n(n−1) 2n = n−1 2 and the product of its roots is 1 n. 8.3.2 Polynomials with integer coefficients Consider the polynomial P (x) = an xn + · · · + a1 x + a0 with integer coefficients. The difference P (x) − P (y), can be written as an (xn − y n ) + · · · + a2 (x2 − y 2 ) + a1 (x − y), 146 Chapter 8. Polynomials where every term of the sum is a multiple of x − y. This leads us to the following arithmetic property of the polynomials in Z[x]. Theorem 8.3.2. If P (x) is a polynomial with integer coefficients, then P (a) − P (b) is divisible by a − b, for any pair of different integers a and b. In particular, all integer roots of P (x) divide P (0). There is a similar statement for the rational roots of polynomials P (x) with integer coefficients. Theorem 8.3.3 (Theorem of the rational root). Any rational root pq , with (p, q) = 1, of a polynomial P (x) = an xn +an−1 xn−1 +· · ·+a0 with integer coefficients, satisfies that p divides a0 and q divides an . Proof. Let p q be a root of P (x), then qn P p q = an pn + an−1 pn−1 q + · · · + a0 q n = 0. All the terms of the sum, except possibly the first, are multiples of q, and all the terms of the sum, except possibly the last, are multiples of p. Since p and q divide 0, it follows that q|an pn and p|a0 q n , and then the assertion follows, since (p, q) = 1.  Corollary 8.3.4. If P (x) is a polynomial with integer coefficients that takes values {±1} in three different integers, then P (x) has no integer roots. Proof. Suppose that there are integers a, b, c and d such that P (a), P (b), P (c) ∈ {−1, 1} and P (d) = 0. Then, since the integers a, b and c are different, a − d, b − d and c − d are also different and, by Theorem 8.3.2, all divide 1, which is impossible.  8.3.3 Irreducible polynomials A polynomial P (x) with integer coefficients is called irreducible in Z[x], if it cannot be written as a product of two non-constant polynomials with coefficients in Z. Example 8.3.5. Any quadratic polynomial with at least one non-rational root is irreducible in Z[x]. For instance, x2 − x − 1 is irreducible in Z[x], since it has roots √ given by 1±2 5 . Similarly, we define irreducibility over the set of polynomials with coefficients in Q, R. The next theorem claims that for polynomials with integer coefficients, the fact that the polynomial could be factored in Q[x] is equivalent to the fact that the polynomial could be factored in Z[x]. Moreover, a polynomial with real coefficients always can be expressed as a product of linear polynomials and irreducible quadratic polynomials in R[x]. In the case of a polynomial with complex coefficients, it can always be factored into linear factors over C[x]. 8.3 Roots of a polynomial 147 Lemma 8.3.6 (Gauss’ lemma). If P (x) has integer coefficients and P (x) can be factored over the rational numbers, then P (x) can be factored over the integers as well. Proof. Suppose that P (x) = an xn +· · ·+a0 has integer coefficients and that P (x) = Q(x)R(x), where Q(x) and R(x) are non-constant polynomials with rational coefficients. Let q and r be the smallest natural numbers such that qQ(x) and rR(x) have integer coefficients. Then, if d = qr it follows that P1 (x) = dP (x) = qQ(x)·rR(x) = Q1 (x)R1 (x) is a factorization of the polynomial P1 (x) into two polynomials with integer coefficients Q1 (x) = qk xk + · · · + q0 and R1 (x) = rm xm + · · · + r0 . Let a′j , with 0 ≤ j ≤ n, be the coefficients of P1 (x). Based on this, we will construct the required factorization of P (x). Let p be a prime divisor of d. Then all coefficients of P1 (x) are divisible by p. Now let us show that p divides all coefficients of Q1 (x) or divides all coefficients of R1 (x). If p divides all coefficients of Q1 (x), we are done. Otherwise, let i be such that p|q0 , q1 ,. . ., qi−1 , but p ∤ qi . We have that p|a′i and a′i = q0 ri + · · · + qi r0 ≡ qi r0 mod p, which implies that p|r0 . Moreover, p|a′i+1 and a′i+1 = q0 ri+1 + · · · + qi r1 + qi+1 r0 ≡ qi r1 mod p, and then p|r1 . Proceeding in the same way, we can deduce that p|rj , for all j. Then R1p(x) has integer coefficients. Then we have a factorization of dp P (x) into two polynomials with integer coefficients. Taking all the prime divisors of d, we will eventually finish with a factorization of the polynomial P (x) into two polynomials with integer coefficients.  Example 8.3.7. If a1 , a2 , . . . , an are different integers, then the polynomial P (x) = (x − a1 )(x − a2 ) · · · (x − an ) − 1 is irreducible over Z[x]. Suppose that P (x) = Q(x)R(x), for some non-constant polynomials R(x) and Q(x) with integer coefficients. Since Q(ai )R(ai ) = −1 for i = 1, . . . , n, then Q(ai ) = 1 and R(ai ) = −1 or Q(ai ) = −1 and R(ai ) = 1; in both cases, we have that Q(ai ) + R(ai ) = 0. Also, the polynomial Q(x) + R(x) is not zero (because otherwise P (x) = −Q(x)2 ≤ 0 for every real number x, but if x is very large, P (x) is positive, a contradiction). Moreover, Q(x) + R(x) has n zeros a1 , . . . , an , which is impossible given that its degree is less than n. A polynomial with integer coefficients is primitive, if its principal coefficient is positive and there is no integer number that divides all coefficients of the polynomial. Theorem 8.3.8 (Eisenstein’s irreducibility criterion). Consider a polynomial P (x) = an xn + an−1 xn−1 + · · · + a0 , with integer coefficients. Let p be a prime number such that 148 Chapter 8. Polynomials (a) p does not divide an , (b) p divides every coefficient a0 , a1 , . . . , an−1 , (c) p2 does not divide a0 . Then P (x) is irreducible over Q[x]. Moreover, if P (x) is primitive, then it is irreducible over Z[x]. Proof. Suppose that P (x) is reducible over Q[x]. By Gauss’ lemma, P (x) = Q(x)R(x), where Q(x) = qk xk + · · · + q0 and R(x) = rm xm + · · · + r0 are polynomials with integer coefficients. Since a0 = q0 r0 is divisible by p but not by p2 , exactly one of q0 or r0 is a multiple of p. Suppose that p|q0 and p ∤ r0 . Moreover, since p|a1 = q0 r1 + q1 r0 it follows that p|q1 r0 ; then, p|q1 and so on. We conclude that all coefficients q0 , q1 , . . . , qk are divisible by p, but then p|an since an = qk rm , which is a contradiction.  One of the most important applications of the Eisenstein criterion, is to show the irreducibility of the cyclotomic polynomials, xp−1 + xp−2 + · · · + x + 1, with p a prime number. Note that the roots of this polynomial are the pth roots of unity, 2πi that is, the powers of e p . Example 8.3.9. The polynomial P (x) = xp−1 + xp−2 + · · · + x + 1, with p a prime number, is irreducible over Q[x]. Note that (x−1)P (x) = xp −1. With the substitution x = y +1 in the last product we get yP (y + 1) = (y + 1)p − 1 = y p + p p−1 p p−2 p y + y + ··· + y. 1 2 p−1   , if i < p then, since the prime p is not Since pi = p(p−1)···(p−i+1) i!  a factor of i!, i! divides the product (p − 1) · · · (p − i + 1). This implies that pi is divisible by p. Dividing yP (y + 1) by y, it follows that P (y + 1) satisfies the conditions of the Eisenstein criterion and therefore it is an irreducible polynomial, hence P (x) is also irreducible. Let us see some applications in number theory of the previous example. Let p be an odd prime number and consider the polynomial P (x) = xp−1 − 1 with coefficients22 in Zp . By Fermat’s little theorem23 , each of the numbers 1, 2, . . . , p − 1 is a root of the polynomial, then xp−1 − 1 = (x − 1)(x − 2) · · · (x − p + 1). (8.5) (a) Comparing the constant coefficients in the last identity we get Wilson’s theorem: (p − 1)! ≡ −1(mod p). 22 Z = {0, 1, . . . , p p 23 See [8]. − 1} with the sum and product operations module p. 149 8.3 Roots of a polynomial (b) If we expand the right-hand side of (8.5), the coefficient σj of xp−1−j is the sum of all products of j elements of the set {1, 2, . . . , p − 1}. Comparing coefficients, we get that p divides σj for j = 1, 2, . . . , p − 2. Now assume that p ≥ 3. (c) (Wolstenholme) The numerator of the (reduced) fraction 1 1 m = 1 + + ···+ n 2 p−1 σ p−2 is divisible by p. In fact, m n = (p−1)! and, since p|σp−2 and p is relatively prime to (p − 1)!, it follows that p|m. (d) Let m be as in the previous part. If p ≥ 5, it follows that p2 |m. Since (x − 1) . . . (x − p + 1) = xp−1 − σ1 xp−2 + σ2 xp−3 + · · · + σp−1 , it follows, after evaluating in x = p, that (p − 1)! = pp−1 − σ1 pp−2 + σ2 pp−3 + · · · − σp−2 p + σp−1 . Since σp−1 = (p − 1)!, we can reduce the last identity to σp−2 = pp−2 − σ1 pp−3 + · · · + σp−3 p. This shows that σp−2 is divisible by p2 , since every σj is divisible by p and since p2 and (p − 1)! are relatively prime, it follows that p2 |m. Exercise 8.14. Find the solutions of the system x+y+z =w 1 1 1 1 + + = . x y z w Exercise 8.15. Prove that the polynomial x4 − x3 − 3x2 + 5x + 1 is irreducible over Q[x]. Exercise 8.16. Let P (x) be a polynomial with integer coefficients. Prove that if P k (n) = n, for some integer number k ≥ 1, and for some integer number n, then for such integer n it follows that P (P (n)) = n. Exercise 8.17. Find all polynomials of the form an xn + an−1 xn−1 + · · · + a1 x + a0 with aj ∈ {−1, 1}, such that all its roots are real numbers. Exercise 8.18 (USA, 1974). Let a, b, c be different integers and let P (x) be a polynomial with integer coefficients. Prove that it is impossible that P (a) = b, P (b) = c and P (c) = a. Exercise 8.19. Prove that the polynomial with real coefficients P (x) = xn + 2nxn−1 + 2n2 xn−2 + an−3 xn−3 + · · · + a1 x + a0 , cannot have all its roots real. 150 Chapter 8. Polynomials 8.4 The derivative and multiple roots ⋆ For a polynomial of degree n, P (x) = a0 + a1 x + a2 x2 + · · · + an−1 xn−1 + an xn = n # ak xk , k=0 we define the derivative of P (x) as the polynomial* of degree n−1 given by P ′ (x) = n n−2 n−1 a1 + 2a2 x + · · · + (n − 1)an−1 x + nan x = k=1 kak xk−1 . It can be shown that if P (x) = (x − a)Q(x), for some polynomial Q(x), then P ′ (x) = Q(x) + (x − a)Q′ (x). (8.6) There is an important relationship between the roots of a polynomial P (x) and the roots of its derivative P ′ (x), given by the following theorem. Theorem 8.4.1. If for some positive integer m the polynomial P (x) is divisible by (x − a)m+1 , then the polynomial P ′ (x) is divisible by (x − a)m . That is, if a is a zero of multiplicity m + 1 of P (x), then a is a zero of multiplicity m for P ′ (x). Proof. For the proof we will use induction over m. For m = 1, if P (x) is divisible by (x − a)2 , then P (x) = (x − a)Q(x), where Q(x) is a polynomial divisible by x − a, and P ′ (x) = Q(x) + (x − a)Q′ (x). Therefore, P ′ (x) is divisible by x − a. Suppose the result is true for m − 1. Let P (x) be divisible by (x − a)m+1 . Then P (x) = (x − a)Q(x), where Q(x) is divisible by (x − a)m , and from P ′ (x) = Q(x)+(x−a)Q′ (x), it follows that P ′ (x) is divisible by (x−a)m , where it has been used that Q′ (x) is divisible by (x−a)m−1 , this being the induction hypothesis.  Observe that if a is a zero of P (x) with multiplicity one, then P ′ (a) = 0. Example 8.4.2. If m and n are integers, such that 0 < m < n, then n # (−1)k k m k=1 n k = 0. The problem is equivalent to showing that the polynomial Pm (x) = n # k=0 km n k x , k has x = −1 as a root. Let us prove the following stronger result: Pm (x) has a zero of multiplicity at least n − m in x = −1. For this we will see that (x + 1)n−m divides Pm (x), for 0 ≤ m < n. 151 8.5 The interpolation formula We proceed by induction on m. For m = 0, it follows that P0 (x) = n # k=0 k0 n k x = (1 + x)n , k then the statement is true. Suppose that for m, with 0 < m < n, Pm−1 (x) has −1 as a root of multiplicity ′ n − (m − 1). By the previous theorem, Pm−1 (x) has one root of multiplicity n − m ′ n−m in −1. Then Pm−1 (x) = (x + 1) Q(x), for some polynomial Q(x). But ′  n n # # n k−1 1 ′ k m−1 n Pm−1 (x) = k = Pm (x). = km x x x k k k=0 k=0 ′ Hence, Pm (x) = xPm−1 (x) = x(x + 1)n−m Q(x), therefore (x + 1)n−m divides Pm (x). Proposition 8.4.3. If P (x) is a polynomial such that P (a) = P ′ (a) = · · · = P (m−1) (a) = 0 and P (m) (a) = 0, then a is a zero of P (x) of multiplicity m. That is, P (x) = (x − a)m Q(x) for some polynomial Q(x), with Q(a) = 0. Here P (j+1) (x) is the derivative of P (j) (x) and P (1) (x) = P ′ (x). Proof. Since P (a) = 0, then P (x) = (x − a)Q1 (x) for some polynomial Q1 (x). Since P ′ (x) = Q1 (x) + (x − a)Q′1 (x), it follows that P ′ (a) = Q1 (a) = 0. Then Q1 (x) = (x − a)Q2 (x) for some polynomial Q2 (x), hence P (x) = (x − a)2 Q2 (x). Proceeding in the same way we get that P (x) = (x − a)m Qm (x), where Qm (x) is the polynomial Q(x) we are looking for.  Exercise 8.20. Determine if the polynomial P (x) = x3 − x2 − 8x + 12 has multiple roots. Exercise 8.21. Find all the triplets of real numbers (a, b, c) such that the polynomial P (x) = x3 + ax2 + bx + c is divisible by (x + 1)2 . 8.5 The interpolation formula Given two points in the Cartesian plane, there is a unique straight line that joins these two points. Then, for two pairs of real numbers (α0 , β0 ), (α1 , β1 ), with α0 = α1 , there is a unique polynomial P (x) of degree at most 1, such that P (α0 ) = β0 and P (α1 ) = β1 . This can be generalized as follows. Theorem 8.5.1 (Lagrange interpolation formula). Let α0 , α1 , . . . , αn be different real numbers and let β0 , β1 , . . . , βn be another n + 1 set of real numbers. Then there exists a unique polynomial P (x) of degree at most n such that P (αi ) = βi , for 0 ≤ i ≤ n. 152 Chapter 8. Polynomials Proof. First, let us find a polynomial that satisfies the conditions. Consider the polynomials Dk (x) = (x − α0 )(x − α1 ) · · · (x − αk−1 )(x − αk+1 ) · · · (x − αn ) , (αk − α0 )(αk − α1 ) · · · (αk − αk−1 )(αk − αk+1 ) · · · (αk − αn ) with 0 ≤ k ≤ n, where the numerator and the denominator have n factors. It is clear that Dk (x) has degree n, that Dk (αk ) = 1 and that Dk (αi ) = 0, if i = k. Then, the polynomial that satisfies the conditions is P (x) = n # βk Dk (x). k=0 To show the uniqueness of the polynomial, suppose that there are two polynomials P1 (x) and P2 (x) of degree at most n, such that Pj (αi ) = βi , for 0 ≤ i ≤ n, j = 1, 2. Then the polynomial P (x) = P1 (x) − P2 (x) has degree at most n and has n + 1 distinct roots α0 , α1 , . . . , αn . By Theorem 8.2.4, it follows that P (x) is the zero  polynomial, then P1 (x) = P2 (x). Example 8.5.2. For 1 < m < n, the following identity holds: n # (−1)k k m k=1 n k = 0. Construct a polynomial of degree at most n which takes the values βk = k m in the points αk = k, with k = 0, 1, 2, . . . , n, respectively. By the Lagrange interpolation formula it follows that the polynomial we are looking for is given by P (x) = n # k m Dk (x), k=0 where Dk (x) = x(x − 1)(x − 2) · · · (x − k + 1)(x − k − 1) · · · (x − n) . k(k − 1) · · · 1(−1)(−2) · · · (k − n) The coefficient of xn , in the polynomial Dk (x), is equal to n 1 (−1)n 1 = (−1)k−n = (−1)k k k(k − 1) · · · 1(−1)(−2) · · · (k − n) k!(n − k)! n! and therefore the coefficient of xn , in the polynomial P (x), turns out to be n n (−1)n # (−1)k k m . k n! k=0 8.6 Other tools to find roots 153 On the other hand, the polynomial Q(x) = xm satisfies that Q(k) = k m for every 0 ≤ k ≤ n. Then, by Theorem 8.5.1, P (x) and Q(x) are equal. Since m < n, the coefficient of xn in the polynomial P (x) is equal to 0. Example 8.5.3. Find a polynomial P (x) such that xP (x − 1) = (x + 1)P (x), for all x ∈ R. For x = 0, the condition is equivalent to P (0) = 0. If P (n) = 0, then P (n + 1) = 0 since (n + 1)P (n) = (n + 2)P (n + 1). Therefore P (x) has an infinite number of zeros. Then, P (x) ≡ 0 is the only polynomial that satisfies the equation. Exercise 8.22. Let P (x) be a polynomial of degree n such that P (k) = 2k , for k = 0, 1, . . . , n. Find P (n + 1). 8.6 Other tools to find roots 8.6.1 Parameters In order to study the roots of a polynomial of degree greater than two, sometimes it is useful to consider the independent terms as variables. These independent terms are called parameters. To illustrate this let us see the next example. Example 8.6.1. Find the solutions of the equation x3 (x + 1) = 2(x + a)(x + 2a), where a is a real parameter. Solving the equation is equivalent to solving the quartic equation x4 + x3 − 2x2 − 6ax − 4a2 = 0. This equation is difficult to solve. In some cases it is possible to factorize without any trouble the left-hand side, but in many cases it is not easy. However we can use some algebraic tricks; for instance, we can consider the number a as the variable and x as the parameter or constant. Then, we get the quadratic equation in a, 4a2 + 6ax − x4 − x3 + 2x2 = 0. Using the formula to solve second-degree equations, the discriminant is given by 36x2 + 16(x4 + x3 − 2x2 ) = 4x2 (2x + 1)2 , which is a square number. Solving the equation for a, we obtain the two roots of the equation: a1 = − 21 x2 − x and a2 = 12 x2 − 21 x. Then, we can factorize the 154 Chapter 8. Polynomials equation as 1 1 a − x2 + x 2 2 1 x4 + x3 − 2x2 − 6ax − 4a2 = −4 a + x2 + x 2 = (x2 + 2x + 2a)(x2 − x − 2a). Finally, solving these second-degree equations we obtain ! the "solutions x1,2 = −1 ± √ √ 1 − 2a, x3,4 = 12 ± 12 1 + 8a, which are real if a ∈ − 81 , 12 . Exercise 8.23. √ (i) Solve the equation 5 − x = 5 − x2 . √ (ii) Solve the equation a − x = a − x2 , with a > 0. Exercise 8.24. Solve the equation x = a− √ a + x, where a > 0 is a parameter. 8.6.2 Conjugate The idea we are about to consider is very simple: when in some algebraic expression there is a square root in the denominator of a fraction, sometimes it is useful to multiply the expression by some factor that cancels out the square root. Let us show this procedure with some simple examples. Example 8.6.2. If we want to eliminate the square root in the denominator of the expression 1 √ , a+ b we should multiply it by The number a − √ a−√b a− b in order to obtain 1√ a+ b = √ a− b a2 −b . √ √ b is known as the conjugate of a + b. Example 8.6.3. Solve the equation √ √ 1 + mx = x + 1 − mx, where m is a real parameter. √ √ The equation is equivalent to 1 + mx − 1 − mx = x. √ √ and dividing by 1 + mx + 1 − mx, which is the conjugate of √ Multiplying √ 1 + mx − 1 − mx, it follows that 2mx √ √ = x. 1 + mx + 1 − mx 155 8.6 Other tools to find roots √ √ A solution is x = 0 and, if x = 0, then 2m = 1 + mx + 1 − mx, hence m is positive. Squaring and simplifying, it follows that 2m2 − 1 = 1 − m2 x2 , hence 2m2 − 1 ≥ 0. Squaring again and solving for x we get x = ±2 1 − m2 ' & and, since 1 − m2 ≥ 0, it follows that m ∈ √12 , 1 . Exercise 8.25. Solve the equation   √ √ x+ x− x− x=m x √ , x+ x where m is a real parameter. Exercise 8.26 (Short list OIM, 2009). Find all triplets (x, y, z) of positive real numbers that satisfy the system of equations: x+ y + 11 = y + 76 √ √ y + z + 11 = z + 76 √ √ z + x + 11 = x + 76. Exercise 8.27. Let a, b, c > 0, solve the system: 1 =c xy 1 =a bz − cx + zx 1 cy − az + = b. yz ax − by + 8.6.3 Descartes’ rule of signs ⋆ The estimation of the number of positive roots of a polynomial P (x), with real coefficients, can be achieved counting the number of changes of sign C(P ), in the sequence of non-zero coefficients of P (x). Example 8.6.4. The polynomial P (x) = 3x12 + 4x10 − 2x9 − 4x8 − x6 + 3x5 − 2x4 − 6x2 + 11x, has 4 changes of sign, that is, C(P ) = 4. Note that zero coefficients are neglected. 156 Chapter 8. Polynomials Theorem 8.6.5 (Descartes’ rule of signs). The number of positive real roots of a polynomial P (x), with real coefficients, is less than or equal to the number of changes of sign, C(P ), that are produced among its coefficients (neglecting the zero coefficients and counting multiplicities of the roots). Similarly, the number of negative real roots of the polynomial is less than or equal to the changes of sign that appear among the coefficients of P (−x). Moreover, if the number of positive roots is less than the number of changes of sign, then the number of positive roots differs from the number of changes of sign by an even number. Proof. Suppose that the polynomial P (x) has degree n, is monic and that the constant term is not zero, that is, P (0) = 0. Otherwise, we can factor one term of the form xk , which does not contribute to the positive roots. Let us show first that the number of changes of sign and the number of positive roots have the same parity. The proof is by induction on n. For n = 1, the polynomial has degree 1 and the result is clear, since P (x) = x − a, with a > 0, has one change of sign and the only positive root is x = a. If P (x) = x + a, there is no change of sign and the only solution is x = −a, which is negative. Now suppose that P (x) is a monic polynomial of degree n > 1, with P (0) = 0. There are two cases: Case 1. If P (0) < 0, then the number of changes of sign must be odd since it starts with a positive number because the polynomial is monic, and it finishes with a negative number P (0). Let us see that the number of positive roots of the polynomial is also odd. Since the degree of P (x) is n, the term xn dominates for large values of x. Then for some large and positive value of x, say x0 , it follows that P (x0 ) is positive, then P (x) must have a root24 in the interval (0, x0 ), which is clearly positive. Let k be such a root. Then P (x) = (x − k)Q(x), with Q(x) a polynomial of degree (0) positive. Hence, applying the induction hypothesis to Q(x), n − 1 and Q(0) = P−k we obtain that this polynomial has an even number of positive roots, therefore P (x) has an odd number of positive zeros, the zeros of Q(x) and k. Case 2. If P (0) > 0 and the equation has no positive solutions, we are done, since zero is an even number. When the equation has some positive solution, we consider one of them, say k. As before, we have that P (x) = (x − k)Q(x), with (0) negative. Then Q(x) has an Q(x) a polynomial of degree n − 1 and Q(0) = P−k odd number of changes of sign. Applying the induction hypothesis to Q(x), we obtain that Q(x) has an odd number of positive roots. Therefore, P (x) has an even number of positive roots. Until now we have shown that the number of changes of sign and the number of positive roots of a polynomial have the same parity. It is only left to prove that the number of sign changes is greater than or equal to the number of positive roots, 24 This claim is guaranteed by the Intermediate Value Theorem, see [21]. 8.7 Polynomials that commute 157 that is, the number of sign changes is an upper bound for the number of positive roots. If there were more positive roots than sign changes in the coefficients of P (x), then there must be at least two more positive roots than the number of sign changes, that is, there must be at least C(P ) + 2 positive roots. On the other hand, P ′ (x) has at least one change of sign25 between every two roots of P (x), hence there would be at least C(P ) + 1 roots of P ′ (x). But P ′ (x) has at least as many sign changes as P (x), that is, C(P ), and moreover its degree is n − 1. Under these conditions, the induction hypothesis tells us that such polynomial satisfies the rule of signs, that is, it has more sign changes than positive roots, which is a contradiction. Therefore, there are more changes of sign than positive roots.  Example 8.6.6 (Poland, 2001). Let n ≥ 3 be an integer. Prove that a polynomial of the form an xn + an−1 xn−1 + · · · + a1 x + a0 , with an−1 = an−2 = 0 and at least one ak = 0, cannot have all its roots real. Suppose that the polynomial satisfies that a0 = 0 in order to assure that if it has a real root, it will be positive or negative. Applying Descartes’ rule of signs, it follows that the number of real roots of the polynomial is at most n − 1, hence at least one of them is a complex root. If a0 = 0, we can factorize the highest power of x that divides the polynomial and, in this case, work with the quotient polynomial as in the previous case. 8.7 Polynomials that commute Two monic polynomials P (x) and Q(x), with real coefficients in one variable, commute if, for every real number x, it follows that P (Q(x)) = Q(P (x)). This means that they commute as polynomial functions. In this section we try to characterize all monic polynomials Q(x) that commute with a given polynomial P (x). Let us see a first example. Example 8.7.1. Find all monic polynomials of degree 3 that commute with the polynomial P (x) = x2 − α, for some α. Let Q(x) = x3 + ax2 + bx + c. The equality P (Q(x)) = Q(P (x)) can be written as (x3 + ax2 + bx + c)2 − α = (x2 − α)3 + a(x2 − α)2 + b(x2 − α) + c. Expanding these last expressions, we get that the right-hand side of the equality has only even powers of x, whereas on the left-hand side there is x5 with coefficient 25 See Rolle’s theorem [21]. 158 Chapter 8. Polynomials 2a, forcing a to be zero. Hence the coefficient of x3 in the left-hand side is 2c, so that c is also zero. Therefore, Q(x) = x3 + bx. Canceling the parenthesis and equating coefficients of the corresponding powers of x, we get 2b = −3α, b2 = 3α2 + b, α = α3 + bα. Letting α = 2γ, it follows that b = −3γ. The second equation implies that 9γ 2 = 12γ 2 − 3γ, that is, γ 2 − γ = 0, then γ = 0 or γ = 1. It is easy to verify that each of these values represents a solution of the system. Summarizing, a polynomial Q(x) of degree 3 that commutes with P (x) = x2 − α exists only when α = 0 or α = 2. If α = 0, Q(x) = x3 and if α = 2, Q(x) = x3 −3x. Similarly, it can be shown that the only polynomial of degree 2 that commutes with P (x) = x2 − α is P (x) itself and that the only polynomial Q(x) of degree 1 that commutes with P (x) is Q(x) = x. An important example of polynomials that commute are the so-called Tchebyshev polynomials, defined in the following way. Let k be a non-negative integer, the Tchebyshev polynomial Tk (x) is defined, for −1 ≤ x ≤ 1, in a recursive way as follows: T0 (x) = 1, T1 (x) = x and, for k ≥ 2, Tk (x) = 2xTk−1 (x) − Tk−2 (x). (8.7) The first Tchebyshev polynomials are T2 (x) = 2x2 − 1, T3 (x) = 4x3 − 3x, T4 (x) = 8x4 − 8x2 + 1. The identity (8.7) and the induction principle guarantee us that Tk is a polynomial of degree k. Lemma 8.7.2. The Tchebyshev polynomials satisfy Tk (cos t) = cos (kt). Proof. We proceed by induction. We have that T0 (cos t) = 1 = cos 0 = cos (0 · t), moreover, T1 (x) = x implies that T1 (cos t) = cos t = cos (1 · t). Suppose that Tk−1 (cos t) = cos [(k − 1)t] and Tk−2 (cos t) = cos [(k − 2)t]. Let us show the result for k. Since the equation (8.7) holds, then Tk (cos t) = 2 cos t Tk−1 (cos t) − Tk−2 (cos t) = 2 cos t cos[(k − 1)t] − cos[(k − 2)t], (8.8) where we have used the induction hypothesis. Now, cos(kt) = cos[(k−1)t+t] = cos[(k−1)t]·cos t−sin[(k−1)t]·sin t. On the other hand, since sin[(k−1)t] = sin[(k−2)t+t] = sin[(k−2)t] cos t+sin t cos[(k−2)t], 159 8.7 Polynomials that commute it follows that, sin[(k − 1)t] sin t = sin[(k − 2)t] cos t sin t + cos[(k − 2)t] sin2 t = sin[(k − 2)t] cos t sin t + cos[(k − 2)t](1 − cos2 t) = sin[(k − 2)t] cos t sin t + cos[(k − 2)t] − cos[(k − 2)t] cos2 t = cos t (sin[(k − 2)t] sin t − cos[(k − 2)t] cos t) + cos[(k − 2)t] = cos t (− cos[(k − 1)t]) + cos[(k − 2)t]. Therefore, cos kt = cos[(k − 1)t] cos t − cos[(k − 2)t] + cos[(k − 1)t] cos t = 2 cos t cos[(k − 1)t] − cos[(k − 2)t], as desired.  Also, it follows that the Tchebyshev polynomials commute, Tk (Tm (x)) = Tkm (x) = Tm (Tk (x)). This follows from the simple fact that cos [k(mt)] = cos [kmt] = cos [m(kt)]. However, Tk is not a monic polynomial, its principal coefficient is 2k−1 , which is an inconvenience. But this can be easily fixed if we define Pk (x) = 2Tk ( x2 ). This procedure keeps the commuting property. Example 8.7.3 (IMO, 1976). Let P1 (x) = x2 − 2 and let Pj (x) = P1 (Pj−1 (x)), for j = 2, 3, . . . . Prove that for every positive integer n, the roots of the equation Pn (x) = x are real and distinct. Let us write x(t) = 2 cos t. This function sends the interval [0, π] to the interval [−2, 2]. Now, observe that P1 (x) = P1 (2 cos t) = 4(cos t)2 − 2 = 2 cos 2t, P2 (x) = P1 (P1 (x)) = 4(cos 2t)2 − 2 = 2 cos 4t, .. .. . . Pn (x) = 2 cos 2n t. Equation Pn (x) = x is equivalent to 2 cos 2n t = 2 cos t, with solutions 2n t = ±t + 2πk, with k = 0, 1, . . . . That is, the following 2n values of t, t= 2kπ 2n − 1 and t= 2kπ , 2n + 1 give 2n real distinct values of x(t) = 2 cos t that satisfy Pn (x) = x. Let us see another example. 160 Chapter 8. Polynomials Example 8.7.4. There exists an infinite sequence of polynomials P1 (x), P2 (x), . . . , Pk (x), . . . , such that any two of them commute, the degree of Pk (x) is k, and P1 (x) = x and P2 (x) = x2 − 2. One solution is immediate, considering the Tchebyshev polynomials that, as we already saw, commute. However, we will give a constructive proof. There is only one polynomial Pk (x), of degree k, that commutes with P2 (x) = x2 − 2 (see Exercise 8.28). Let us write the first terms of the sequence we are looking for: P1 (x) = x, P2 (x) = x2 − 2, P3 (x) = x3 − 3x, P4 (x) = x4 − 4x2 + 2, P5 (x) = x5 − 5x3 + 5x, P6 (x) = x6 − 6x4 + 9x2 − 2. Here, P4 (x) = P2 (P2 (x)), P6 (x) = P2 (P3 (x)). Observing the previous polynomials, we note they satisfy the relation Pk+1 (x) = xPk (x) − Pk−1 (x). This makes it natural to define the polynomials using this last recursion, for k > 1, and with P1 (x) = x, P2 (x) = x2 − 2. We can prove, using induction, that these polynomials commute with P2 (x) and, by Exercise 8.30, we obtain that any two of them commute. Exercise 8.28. Prove that there exists at most one polynomial of a given degree that commutes with a given polynomial of degree 2. Exercise 8.29. Find all polynomials of degrees 4 and 8 that commute with a given polynomial of degree 2. Exercise 8.30. Prove that if the polynomials Q(x) and R(x) both commute with the polynomial P (x) of degree 2, then they commute with each other. Exercise 8.31. Prove that two polynomials P (x) and Q(x), of degree 1, commute if and only if either both are monic or both have a common fixed point. Exercise 8.32. Given a polynomial R(x), define the polynomial Ra (x) = R(x − a) + a. Prove that if two polynomials P (x) and Q(x) commute, then Pa (x) and Qa (x) commute as well. 8.8 Polynomials of several variables 161 8.8 Polynomials of several variables If x and y are the solutions of the quadratic equation at2 + bt + c = 0, then c −b a = x + y and a = xy. The expressions x + y and xy are examples of polynomials in two variables x and y. In general, a polynomial in two variables x and y, is a sum of terms of the form cxk y m , where c is a constant, k and m are non-negative integers and we denote it by P (x, y). The number k + m is called the degree of the term, and the degree of the polynomial P (x, y) is equal to the degree of the term with the largest degree. We can add, subtract and multiply polynomials of several variables in the same way as the polynomials in one variable. To simplify the polynomials, the terms of the same degree are grouped together. We will consider two types of polynomials with two variables: symmetric polynomials, that is, the ones that satisfy P (x, y) = P (y, x), and homogeneous polynomials where all the terms have the same degree. Similar definitions can be given for polynomials in three variables x, y and z. A polynomial, in three variables, is any finite sum of terms of the form cxk y n z m , where k, n and m are non-negative integers. The degree of the polynomial is given by the maximum sum of the powers k + m + n. If all the terms have the same degree, we say that the polynomial is homogeneous and if it satisfies that P (x, y, z) = P (z, x, y) = P (y, z, x) = P (x, z, y) = P (y, x, z) = P (z, y, x), then we say that P (x, y, z) is symmetric. Example 8.8.1. (a) The elementary symmetric polynomials σ1 = x + y and σ2 = xy are also homogeneous, the first one of degree 1 and the second one of degree 2. (b) The polynomial x2 + x + y + y 2 is symmetric but it is not homogeneous, meanwhile x2 y + 2y 3 is homogeneous but not symmetric. (c) The sum of powers si = xi + y i , with i = 0, 1, 2, . . . , are symmetric. Theorem 8.8.2. Any symmetric polynomial in x and y can be represented as a polynomial in σ1 = x + y and σ2 = xy. Proof. In fact, sn = xn + y n = (x + y)(xn−1 + y n−1 ) − xy(xn−2 + y n−2 ) = σ1 sn−1 − σ2 sn−2 , where si = xi + y i . Then, we have the recursion s0 = 2, s1 = σ1 , sn = σ1 sn−1 − σ2 sn−2 , for n ≥ 2. Now, the proof for any symmetric polynomial is simple. The terms of the form axk y k do not cause any problem, since axk y k = aσ2k . If the term bxi y k , with i < k, appears in the polynomial, then, by symmetry, the term bxk y i also has to be part of the polynomial. Grouping terms, it follows that bxi y k + bxk y i = bxi y i (xk−i + y k−i ) = bσ2i sk−i . But sk−i can be expressed as a polynomial in terms of σ1 and σ2 .  162 Chapter 8. Polynomials The non-linear systems of symmetric equations in two variables x and y can be simplified using the substitution σ1 = x + y and σ2 = xy. The degree of these equations will be reduced, since σ2 = xy is a second-degree term in x and y. As soon as we find σ1 and σ2 , we can find the solutions x, y of the system of symmetric equations, either solving the quadratic equation z 2 − σ1 z + σ2 = 0 or solving the system x + y = σ1 , xy = σ2 . Example 8.8.3. Solve the system x5 + y 5 = 31, x + y = 1. Take σ1 = x + y, σ2 = xy. Then x5 + y 5 = σ1 s4 − σ2 s3 = (x + y)(x4 + y 4 ) − xy(x3 + y 3 ) = 31. (8.9) Since x4 + y 4 = σ1 s3 − σ2 s2 , making the substitutions recursively, we obtain that equation (8.9) is transformed into σ15 − 5σ13 σ2 + 5σ1 σ22 = 31. Then, the system we need to solve is σ15 − 5σ13 σ2 + 5σ1 σ22 = 31, σ1 = 1. Substituting the value of σ1 in the first equation, we get σ22 − σ2 − 6 = 0. This last equation has as solutions σ2 = 3, −2. Hence, we need to solve x + y = 1, xy = 3, −2. Therefore, the solutions are:  √ √   √  √ 1 + i 11 1 − i 11 1 − i 11 1 + i 11 , , (2, −1), (−1, 2). , , 2 2 2 2 The symmetric polynomials in three variables can be expressed in terms of the symmetric polynomials σ1 = x + y + z, σ2 = xy + yz + zx and σ3 = xyz. The sum of the powers si = xi + y i + z i , with i = 0, 1, 2, . . . , can be expressed with σ1 , σ2 and σ3 , in the following way: s0 = x0 + y 0 + z 0 = 3, s1 = x + y + z = σ1 , s2 = x2 + y 2 + z 2 = σ12 − 2σ2 , s3 = x3 + y 3 + z 3 = σ13 − 3σ1 σ2 + 3σ3 , and, for n ≥ 3, we have the recursive equation sn = σ1 sn−1 − σ2 sn−2 + σ3 sn−3 . The non-linear systems of symmetric equations in three variables x, y and z can be simplified using σ1 , σ2 and σ3 . Once we have the equations in σ1 , σ2 and 163 8.8 Polynomials of several variables σ3 , we write the cubic equation u3 − σ1 u2 + σ2 u − σ3 = 0, where σ1 , σ2 and σ3 are its coefficients. Then, the solutions (x1 , x2 , x3 ) of this cubic equation are the solutions of the system. The other solutions can be obtained by permutation of the variables. Example 8.8.4. Solve the system x + y + z = 4, x2 + y 2 + z 2 = 14, x3 + y 3 + z 3 = 34. Note that x, y, z are roots of the polynomial P (u) = u3 − (x + y + z)u2 + (xy + yz + zx)u − xyz. This becomes P (u) = u3 − σ1 u2 + σ2 u − σ3 , where σ1 = x + y + z, σ2 = xy + yz + zx and σ3 = xyz. Then 4 = s1 = σ1 , 14 = s2 = σ12 − 2σ2 = 16 − 2σ2 , then σ2 = 1. The numbers x, y and z are roots of P (u), then x3 − 4x2 + x − σ3 = 0, y 3 − 4y 2 + y − σ3 = 0, z 3 − 4z 2 + z − σ3 = 0. Adding the equations, we get that σ3 = −6, and so the roots we need to find are the roots of the polynomial P (u) = u3 −4u2 +u+6. Observe that u1 = −1 is a root, then we can factorize P (u) as P (u) = (u + 1)(u2 − 5u + 6). Solving u2 − 5u + 6 = 0, we obtain the other roots, which in this case are u2 = 2 and u3 = 3. Hence, the solution of the system is (x, y, z) = (−1, 2, 3) and all its permutations. A technique used to generate integer roots for a family of quadratic polynomials or for a polynomial of several variables, is known as Vieta’s jumping. This tool is used to find the roots of a quadratic polynomial in a recursive way. In general, when this technique is applied to polynomials of several variables only one variable is taken into account while the other variables are considered as constants. Let us see the following example. Example 8.8.5. If x, y are positive integers such that such integer is equal to 3. x2 +y 2 +1 xy is an integer, then Suppose that (x0 , y0 ) is a pair of positive integers such that x20 + y02 + 1 = k, x0 y0 with k an integer. Using this solution, we will find another solution (x1 , y1 ) of positive integers such that x1 + y1 < x0 + y0 . Suppose that x0 > y0 and let 164 Chapter 8. Polynomials P (x) = x2 − ky0 x + y02 + 1, then x0 is a root of P (x) and if x1 is the other root, by Vieta’s formulas, it follows that x0 + x1 = ky0 and x0 x1 = y02 + 1. From the first equality, it follows that x1 is an integer, and from the second equality, x1 is positive y 2 +1 since x0 and y02 + 1 are positive. Then x1 = 0x0 and, since x0 , y0 are integers with x0 > y0 , it follows that x0 ≥ y0 + 1, so that x20 ≥ (y0 + 1)2 > y02 + 1, and therefore x1 < x0 . In this way, we have found another solution (x1 , y1 ) = (x1 , y0 ) such that x1 + y1 < x0 + y0 . This process can be continued and we can find another solution of positive integers (x1 , y1 ) such that the sum x1 + y1 is less than the sum of the elements of the previous solution. Since this process cannot be done infinitely, we must get a solution (x0 , y0 ) such that x0 = y0 . Then k= 1 y02 + y02 + 1 = 2 + 2, y02 y0 hence 2 < k ≤ 3, from where we deduce k = 3. Exercise 8.33. Solve the system x5 + y 5 = 33, x + y = 3. √ √ Exercise 8.34. Find the solutions of the equation 4 97 − x + 4 x = 5. Exercise 8.35. What is the relation between a, b and c if x + y = a, x2 + y 2 = b, x3 + y 3 = c? Exercise 8.36 (IMO, 1961). What conditions must satisfy a and b in order that x, y, z are different positive real numbers and such that x + y + z = a, x2 + y 2 + z 2 = b2 , xy = z 2 ? Exercise 8.37. Solve the system x + y + z = a, x2 + y 2 + z 2 = b2 , x3 + y 3 + z 3 = a3 . Exercise 8.38. Find the integer solutions of the equation (x + y 2 )(x2 + y) = (x + y)3 . Exercise 8.39 (China, 2010). Find all integers k for which there are positive integers a, b, such that a+1 b+1 + = k. b a Exercise 8.40. Let x, y, z be non-zero integers, and such that x2 + y 2 + z 2 xyz is an integer. Prove that this integer is either 1 or 3. Chapter 9 Problems Problem 9.1. Find the irrational numbers a such that a2 + 2a and a3 − 6a are rational numbers. Problem 9.2 (OMCC, 2010). Let p, q, r be rational numbers different from zero, such that 3 pq 2 + 3 qr2 + 3 rp2 is a rational number different from zero. Prove that 1 3 pq 2 + 1 3 qr2 + 1 3 rp2 , is also a rational number. Problem 9.3 (APMO, 2005). Prove that for every irrational number a there exist irrational numbers b and b′ such that a + b and ab′ are rational numbers and such that a + b′ and ab are irrational numbers. Problem 9.4. Let x1 , x2 , . . . , xn be positive integers less or equal than an integer number m. Prove that if the least common multiple of each pair of integers is greater than m, then the sum of the reciprocals of the n numbers is less than 32 . Problem 9.5 (APMO, 2013). Find all positive integers n such that integer number. 2 √n +1 ⌊ n⌋2 +2 is an Problem 9.6 (IMO shortlist, 1998). Determine all the pairs (a, b) of real numbers such that a ⌊nb⌋ = b ⌊na⌋, for every positive integer n. Problem 9.7. If n and m are positive integers without common factors, prove that  4  n   2n 4  3n 4 (m − 1)n (m − 1) . + + + ··· + = m m m m 2 © Springer Internationl Publishing Switzerland 2015 R.B. Manfrino et al, Topics in Algebra and Analysis, DOI 10.1007/978-3-319-11946-5_9 165 166 Chapter 9. Problems Problem 9.8. Find the real solutions of the system 1 1 + a b 1 1 + √ 18 = √ 3 3 a b 9= 1 1+ √ 3 a 1 1+ √ 3 b . Problem 9.9. The different real numbers a, b, c satisfy the identities a + b + 1c = c + a1 . Find the values that abc can get. 1 b = Problem 9.10. The positive real numbers a, b, c satisfy the identity abc(a+b+c) = 1. Find the minimum value of (a + b)(a + c). Problem 9.11. The real numbers a, b, c different from zero, satisfy the identity $ 1+ a% b 1+ bc ca Find the value of a + b + c. $ c% 1+ = ab 1 1 1 + + −1 a b c 2 . Problem 9.12. Let a, b, c be real numbers different from zero and such that a + b + c = 0. Find the value of b−c c−a a−b + + a b c a b c + + b−c c−a a−b . Problem 9.13. Let a, b, c be non-zero and distinct real numbers, such that b c a + + = 0. b−c c−a a−b Find the value of a b c + + . 2 2 (b − c) (c − a) (a − b)2 Problem 9.14. Let a, b, c be integers that satisfy the equality ab + bc + ca = 1. Prove that the number (a2 + 1)(b2 + 1)(c2 + 1) is a perfect square. Problem 9.15. Let a, b, c be integers that satisfy the equality that the number a2 + b2 + c2 is a perfect square. 1 a + 1b + 1c = 0. Prove Problem 9.16. The real numbers a, b, c satisfy the identity (a+b+c)2 = a2 +b2 +c2 . Prove that: b2 c2 a2 + 2 + 2 = 1. (i) 2 a + 2bc b + 2ca c + 2ab bc ca ab (ii) 2 + + = 1. a + 2bc b2 + 2ca c2 + 2ab 167 Chapter 9. Problems Problem 9.17 (Slovenia, 2005). The real numbers a, b, c satisfy the identity abc = 1. Find the value of the expression b+1 c+1 a+1 + + . ab + a + 1 bc + b + 1 ca + c + 1 Problem 9.18. Let a, b, c be distinct, non-zero integers, such that What is the value of a − b? a−c b + b+c a = 2. Problem 9.19 (Czech-Slovak-Polish, 2005). Let n be a positive integer. Find the real non-negative numbers x1 , . . . , xn that solve the following system of equations: x1 + x22 + x33 + · · · + xnn = n x1 + 2x2 + 3x3 + · · · + nxn = n(n + 1) . 2 Problem 9.20 (Canada, 1971). Let x, y be positive real numbers with x + y = 1. % $  Prove that 1 + x1 1 + y1 ≥ 9. Problem 9.21 (Romania, 2007). For real non-negative numbers x, y, z, prove that x3 + y 3 + z 3 3 ≥ xyz + |(x − y)(y − z)(z − x)|. 3 4 Problem 9.22 (Great Britain, 2010). Let a, b, c be the lengths of the sides of a triangle, that satisfy ab + bc + ca = 1. Prove that (a + 1)(b + 1)(c + 1) < 4. Problem 9.23 (OMCC, 2012). Let a, b, c be real numbers such that 1 a+c = 1 and ab + bc + ca > 0. Prove that a+b+c− 1 a+b + 1 b+c + abc ≥ 4. ab + bc + ca Problem 9.24 (IMO, 1964). Let a, b, c be the lengths of the sides of a triangle. Prove that a2 (b + c − a) + b2 (a + c − b) + c2 (a + b − c) ≤ 3abc. Problem 9.25 (IMO, 1975). Consider two collections of numbers x1 ≤ x2 ≤ · · · ≤ xn , y1 ≤ y2 ≤ · · · ≤ yn and a permutation (z1 , z2 , . . . , zn ) of (y1 , y2 , . . . , yn ). Prove that (x1 − y1 )2 + · · · + (xn − yn )2 ≤ (x1 − z1 )2 + · · · + (xn − zn )2 . Problem 9.26 (IMO, 1978). Let x1 , x2 , . . . , xn be distinct positive integers. Prove that x2 xn 1 1 1 x1 + 2 + ···+ 2 ≥ + + ··· + . 12 2 n 1 2 n 168 Chapter 9. Problems Problem 9.27 (IMO shortlist, 2010). Let x1 , . . . , x100 be non-negative real numbers such that xi + xi+1 + xi+2 ≤ 1, for every i = 1, . . . , 100 (where x101 = x1 , x102 = x2 ). Find the maximum possible value of the sum S= 100 # xi xi+2 . i=1 Problem 9.28 (Czech-Slovak-Polish, 2010). Let a, b, x, y be positive real numbers, with a ≥ b and ab ≥ ax + by. Prove that: (i) x + y ≤ a, √ √ √ (ii) a + b ≥ x + y. Problem 9.29 (Thailand, 2005). Let a, b, c be real numbers, prove that 2a − b a−b 2 + 2b − c b−c 2 + 2c − a c−a 2 ≥ 5. Problem 9.30. Find all triplets of positive real numbers a, b, c such that they satisfy the following inequalities ab + 1 ≤ 2c, bc + 1 ≤ 2a, ca + 1 ≤ 2b. Problem 9.31 (Czech-Slovak-Polish, 2010). Determine all triplets of positive integers (a, b, c), that satisfy the identities: √ a b−c=a √ b c−a=b √ c a − b = c. Problem 9.32 (IMO, 1968). Let a, b, c be real numbers. Prove that the system of equations ax21 + bx1 + c = x2 ax22 + bx2 + c = x3 .. .. . . ax2n−1 + bxn−1 + c = xn ax2n + bxn + c = x1 , has a unique real solution if and only if (b − 1)2 − 4ac = 0. 169 Chapter 9. Problems Problem 9.33 (IMO shorlist, 1967). Solve the following system: x2 + x − 1 = y y2 + y − 1 = z z 2 + z − 1 = x. Problem 9.34 (OMM, 2011). Solve the system, x21 + x1 − 1 = x2 x22 + x2 − 1 = x3 .. .. . . x2n−1 + xn−1 − 1 = xn x2n + xn − 1 = x1 . Problem 9.35. For n ≥ 3, find all the positive solutions of the system x21 = x2 + x3 x22 = x3 + x4 .. .. . = . x2n−1 = xn + x1 x2n = x1 + x2 . Problem 9.36. Let x, y, z be real numbers such that x+y+z = x−1 +y −1 +z −1 = 0. Prove that x6 + y 6 + z 6 = xyz. x3 + y 3 + z 3 Problem 9.37 (Peru, 2009). Let a, b, c, d be integers such that a + b + c + d = 0. Prove that (bc − ad)(ac − bd)(ab − cd) is a perfect square. Problem 9.38. Let a, b and c be real numbers different from zero, with a+b+c = 0. Prove that a2 + b 2 b 2 + c2 c2 + a2 a3 b3 c3 + + = + + . a+b b+c c+a bc ca ab Problem 9.39. Find all triplets of positive integers (a, b, c), such that a3 + b3 + c3 − 3abc = p, where p > 3 is a prime number. Problem 9.40. Find all positive integers that are solutions of the equation x3 −y 3 = xy + 61. 170 Chapter 9. Problems Problem 9.41 (Great Britain, 2008). Find the minimum of x2 + y 2 + z 2 , where x, y, z are real numbers that satisfy x3 + y 3 + z 3 − 3xyz = 1. Problem 9.42 (Shortlist OMCC, 2011). The positive real numbers x, y, z, satisfy that z x y x + = y + = z + = 2. z x y Find the value of x + y + z. Problem 9.43. If {an } ⊂ R+ is an arithmetic progression, prove that 1 1 n 1 √ +√ √ + ···+ √ √ = √ √ . √ a0 + a1 a1 + a2 an−1 + an a0 + an Problem 9.44. The lengths of the sides of a right triangle are a < b < c and they are in arithmetic progression. Prove that their difference d is equal to the radius of the incircle of the triangle. Problem 9.45. Prove that in any partition of the set {1, 2, . . . , 9} into two subsets, it is possible to find, in one of them, an arithmetic progression with three terms. Problem 9.46. Let a, b, c be real numbers in arithmetic progression; prove that 2 (a + b + c)3 = a2 (b + c) + b2 (c + a) + c2 (a + b). 9 Problem 9.47. Prove that in the arithmetic progression {3, 7, 11, . . . , 4k − 1, . . . }, there are an infinite number of primes. Problem 9.48. Is it possible to divide the set of natural numbers in two subsets, such that none of them contains a non-constant arithmetic progression? Problem 9.49. Is there a non-constant arithmetic progression where the terms of the progression are all prime numbers? Problem 9.50. Prove that if an arithmetic progression of positive integers contains a perfect square then it has an infinite number of perfect squares. Problem 9.51 (OMM, 1999). Prove that there do not exist 1999 prime numbers in arithmetic progression, all of them smaller than 12345. Problem 9.52 (OMM, 2005). Let us say that a list of numbers a1 , a2 , . . . , am contains an arithmetic triplet ai , aj , ak , if i < j < k and 2aj = ai + ak . For example, 8, 1, 5, 2, 7 has the following arithmetic triplet (8, 5, 2), but 8, 1, 2, 5, 7 does not. Let n be a positive integer. Prove that the numbers 1, 2, . . . , n can be rearranged in a list, such that this list does not contain an arithmetic triplet. 171 Chapter 9. Problems Problem 9.53 (Czech-Slovak, 2010). The four real solutions of the equation ax4 + bx2 + a = 1, form an increasing arithmetic progression. One of the solutions is also a solution of the equation bx2 + ax + a = 1. Find all possible real values of a and b. Problem 9.54 (APMO, 2013). For 2k real numbers a1 , a2 , . . . , ak , b1 , b2 , . . . , bk , define the sequence of numbers Xn by Xn = k # i=1 ⌊ai n + bi ⌋ , (n = 1, 2, . . . ). *k If the sequence Xn forms an arithmetic progression, prove that the sum i=1 ai has to be an integer number. ( ) Problem 9.55. Prove that in any partition of 1, 2, 22 , . . . , 256 into two subsets, it is possible to find in one of them three terms that are in geometric progression. Problem 9.56. Let n > 1 and consider the collection of numbers a0 , a1 , . . . , an defined by a0 = 1 2 and ak+1 = ak + ak , n2 with k = 0, 1, . . . , n − 1. Prove that an < 1. Problem 9.57. Let a1 < a2 < · · · < an < · · · be an increasing sequence of positive integers such that: (i) For every n ≥ 1, a2n = an + n. (ii) If ap is a prime number, then p is prime. Prove that an = n, for all n ≥ 1. Problem 9.58. An arbitraty set of m + n numbers is divided into two arbitrary sets a1 , a2 , . . . , am , b1 , b2 , . . . , bn . Order the numbers, in each set, in increasing form a1 < a2 < · · · < am , b1 < b2 < · · · < bn . Then, the numbers in each set are divided again into two subsets c1 , c2 , . . . , cm and d1 , d2 , . . . , dn and let us arrange them in increasing order c1 < c2 < · · · < cm , d1 < d2 < · · · < dn . Prove the equality |a1 − c1 | + |a2 − c2 | + · · · + |am − cm | = |b1 − d1 | + |b2 − d2 | + · · · + |bn − dn |. 172 Chapter 9. Problems Problem 9.59 (Poland, 1986). Prove that, for every integer n ≥ 3, the number n! can be represented as the sum of n different divisors of n!. Problem 9.60. For each prime number p > 3, prove that the number divisible by p3 . 2p−1 p−1 − 1 is Problem 9.61 (UK, 2004). Let S be a set of rational numbers that satisfy: (i) 12 ∈ S. (ii) If x ∈ S, then 1 x+1 ∈ S and x x+1 ∈ S. Prove that S contains all rational numbers in the interval 0 < x < 1. Problem 9.62 (IMO, 1988). Prove that, if a, b are positive integers such that ab+1 2 +b2 divides a2 + b2 , then aab+1 is a perfect square. Problem 9.63 (Ireland, 2007). If a, b, c are roots of the polynomial P (x) = x3 − 2007x + 2002, determine the value of a−1 a+1 b−1 b+1 c−1 c+1 . Problem 9.64. Determine all positive rational numbers a, b, c such that a + b + c, abc and a1 + 1b + 1c are integers. Problem 9.65. Let a, b, c be real numbers, not all zero. Prove that one of the equations ax2 + 2bx + c = 0, bx2 + 2cx + a = 0, cx2 + 2ax + b = 0 has a real root. Problem 9.66 (Estonia, 2005). Find all pairs of real numbers (a, b) such that the roots of the polynomials 6x2 − 24x − 4a and x3 + ax2 + bx − 8 are all non-negative real numbers. Problem 9.67. Let P (x) be a polynomial such that |P (x)| ≤ 1, for |x| ≤ 1. (i) (Short list IMO, 1986) If P (x) = ax2 + bx + c, find the maximum value of |a| + |b| + |c|. (ii) (Short list IMO, 1996) If P (x) = ax3 + bx2 + cx + d, find the maximum value of |a| + |b| + |c| + |d|. Problem 9.68. Let a, b, c, d be real numbers, with a and d distinct from zero. Prove that if the roots of the polynomials ax3 + bx2 + cx + d and dx3 + cx2 + bx + a bc are positive, then ad ≥ 9. 173 Chapter 9. Problems Problem 9.69 (IMO, 1963). Find all real solutions of the equation √ 2 x2 − 1 = x, where p is a real number. x2 − p + Problem 9.70 (China, 2008). Let P (x) = ax3 + bx2 + cx + d be a polynomial with real coefficients. If P (x) has three positive real roots and P (0) < 0, prove that 2b3 + 9a2 d − 7abc ≤ 0. Problem 9.71 (India, 2010). Let a, b, c be integers with b even and c odd. Suppose that the equation x3 + ax2 + bx + c = 0 has roots α, β, γ, with α2 = β + γ. Prove that α is an integer and that β = γ. Problem 9.72. Consider all quadratic equations x2 + px + q = 0, where the coefficients p, q belong to the interval [−1, 1]. Find all possible values of the solutions of those equations. Problem 9.73 (Hermite’s identity). Given a positive integer n and a real number x, prove that       1 2 n−1 ⌊nx⌋ = ⌊x⌋ + x + + x+ + ··· + x + . n n n Problem 9.74 (IMO, 1968). For every positive integer n, prove that       n + 2k n+1 n+2 + ···+ + · · · = n. + 2 22 2k+1 Problem 9.75 (USA, 1981). Prove that if x is a positive real number and n is a positive integer, then ⌊nx⌋ ≥ ⌊x⌋ ⌊2x⌋ ⌊nx⌋ + + ··· + . 1 2 n Problem 9.76. The function f assigns to each non-negative integer n, the nonnegative integer f (n), such that: (i) f (nm) = f (n) + f (m), for n, m ≥ 0. (ii) f (n) = 0 if the units digit of n is 3. (iii) f (10) = 0. Find the value of f (1985). Problem 9.77 (Czech-Slovak, 2010). Find all functions f : R+ → R+ that satisfy f (x)f (y) = f (y)f (xf (y)) + 1 , xy with x, y ∈ R+ . 174 Chapter 9. Problems Problem 9.78 (Thailand, 2004). Let f : [0, 1] → R be a function such that: (i) f (0) = f (1) = 0. (ii) |f (x) − f (y)| < |x − y|, for x, y ∈ [0, 1] with x = y. Prove that |f (x) − f (y)| < 12 , for all x, y ∈ [0, 1]. Problem 9.79 (IMO, 1978). The set of all positive integers is the union of two disjoint subsets {f (1), f (2), . . . , f (n), . . . }, {g(1), g(2), . . . , g(n), . . . }, where f (1) < f (2) < · · · < f (n) < . . . , g(1) < g(2) < · · · < g(n) < . . . , and g(n) = f (f (n)) + 1, for all n ≥ 1. Determine f (240). Problem 9.80 (IMO, 1981). The function f (x, y) satisfies f (0, y) = y + 1 f (x + 1, 0) = f (x, 1) f (x + 1, y + 1) = f (x, f (x + 1, y)), (9.1) (9.2) (9.3) for all non-negative integers x, y. Determine the value of f (4, 1981). Problem 9.81 (IMO, 1982). The function f (n) is defined for all positive integers n and takes non-negative values. Also, for all m and n, f (n + m) − f (m) − f (n) = 0 or 1, f (2) = 0, f (3) > 0 and f (9999) = 3333. Find f (1982). Problem 9.82 (IMO, 1983). Find all functions f defined in the set of positive real numbers that satisfy the conditions: (i) f (xf (y)) = yf (x), for all positive numbers x, y. (ii) f (x) → 0, when x → ∞. Problem 9.83 (IMO, 2010). Find all functions f : R → R such that, for all x, y ∈ R, the equality f (⌊x⌋y) = f (x)⌊f (y)⌋ holds, where ⌊x⌋ denotes the greatest integer less than or equal to x. 175 Chapter 9. Problems Problem 9.84 (APMO, 2011). Find all upper bounded functions f : R → R that satisfy f (xf (y)) + yf (x) = xf (y) + f (xy), for x, y ∈ R. (9.4) Problem 9.85 (OIM, 2009). Let {an } be a sequence defined by a1 = 1, a2n = an + 1, a2n+1 = 1 , a2n for n ≥ 1. Prove that, for every rational number r, there is a unique positive integer n with an = r. Problem 9.86. Find the term a1000 of the sequence defined by a0 = 1 and an+1 = an , 1 + nan for n ≥ 0. Problem 9.87 (Short list IMO, 2010). A sequence x1 , x2 , . . . , is defined by x1 = 1 and x2k = −xk , x2k−1 = (−1)k+1 xk , for all k ≥ 1. Prove that x1 +x2 +· · ·+xn ≥ 0, for all n ≥ 1. Problem 9.88 (IMO, 2010). Let a1 , a2 , . . . , an , . . . be a sequence of positive real numbers. Suppose that for some positive integer s, we have that an = max{ak + an−k : 1 ≤ k ≤ n − 1}, for all n > s. Prove that there exist positive integers l and N , with l ≤ s, such that an = an−l + al , for all n ≥ N . Problem 9.89 (Bulgaria, 1987). Let k be an integer greater than 1. Prove that there exist a prime number p and an increasing sequence of positive integers a1 , a2 , . . . , an , . . . , such that the terms of the sequence p + ka1 , p + ka2 , . . . , p + kan , . . . are all prime numbers. Problem 9.90 (Austria, 2005). For real numbers a, b, c, set sn = an + bn + cn , for n ≥ 0. Suppose that s1 = 2, s2 = 6 and s3 = 14. Prove that s2n − sn−1 sn+1  = 8, for all n ≥ 2. Problem 9.91 (IMO, 1982). Consider an infinite sequence of positive real numbers {xn }, such that x0 = 1 and xi+1 ≤ xi , for all i ≥ 0. (i) Prove that, for any sequence that satisfies the given conditions, there exists an integer n ≥ 1 such that x2 x2 x20 + 1 + · · · + n−1 ≥ 3.999. x1 x2 xn 176 Chapter 9. Problems (ii) Find a sequence with the given conditions such that x2 x2 x20 + 1 + · · · + n−1 < 4, x1 x2 xn for all n ≥ 1. Problem 9.92 (Great Britain, 2009). Find all sequences {an } that satisfy the following conditions: (i) an+1 = 2a2n − 1, for all n ≥ 1. (ii) a1 is a rational number. (iii) ai = aj , for some i, j with i = j. Problem 9.93. If a0 = 0, a1 = 1 and an = 2an−1 + an−2 , prove that 2k |an if and only if 2k |n. Problem 9.94 (Russia, 1989). The sequence {an } is such that |am + an − am+n | ≤ 1 , for all m, n ≥ 1. m+n Prove that {an } is an arithmetic progression. Problem 9.95 (Bulgaria, 1996). The sequence {an } is defined by a1 = 1 and an+1 = n an + , for n ≥ 1. n an Prove that ⌊a2n ⌋ = n, for all n ≥ 4. Problem 9.96. Let bn be the units digit of the number 11 + 22 + 33 + · · · + nn . Prove that the sequence {bn } is periodic with period 100. √ Problem 9.97. Prove that ⌊(1 + 3)2n+1 ⌋ is an even integer, for n ≥ 0. Problem 9.98. The sequence of integers {an } is defined by a1 = 3, a2 = 5 and an+2 = 3an+1 − 2an , for n ≥ 1. Prove that an = 2n + 1, for all integers n ≥ 1. Problem 9.99. The sequence of integers {an } is defined by a1 = 1, a2 = 2 and an+2 = an+1 − an , for n ≥ 1. Prove that an+6 = an , for all integers n ≥ 1. Problem 9.100 (Short list IMO, 1986). Let a0 = a1 = 1 and, for n ≥ 0, an+2 = 7an+1 − an − 2. Prove that an is a perfect square, for every integer number n ≥ 0. Problem 9.101 (IMO, 1976). The sequence {un } is defined by u0 = 2, u1 = 2n −(−1)n 3 5 2 and , where ⌊x⌋ is un+1 = un (u2n−1 − 2) − u1 , for n ≥ 1. Prove that ⌊un ⌋ = 2 the integer part of x. √ Problem 9.102. If an = ⌊ 2n⌋, for n ≥ 1, what is the value of a2020 ? 177 Chapter 9. Problems Problem 9.103. Let P (x) be a polynomial of degree n with P (j) = 0, 1, . . . , n. Find P (m), for m > n. j j+1 , for j = 1 j, for j = Problem 9.104. Let P (x) be a polynomial of degree n with P (j) = 20 , 21 , . . . , 2n . Find P (0). Problem 9.105 (Short list IMO, 1981). Let P (x) be a polynomial of degree n with −1  , for j = 0, 1, . . . , n. Find P (n + 1). P (j) = n+1 j Problem 9.106 (IMO, 1993). Let n > 1 be an integer and let P (x) = xn +5xn−1 +3. Prove that P (x) is irreducible over Z[x]. Problem 9.107 (Short list, 1997). Let p be a prime number and let Q(x) be a polynomial of degree n with integer coefficients such that: (i) Q(0) = 0, Q(1) = 1, (ii) For any integer n, Q(n) is congruent to 0 or 1 module p. Prove that n ≥ p − 1. Problem 9.108 (Short list IMO, 1997). Let P (x) be a polynomial with real coefficients and P (x) > 0, for x ≥ 0. Prove that there is a positive integer n such that the coefficients of the polynomial (1 + x)n P (x) are all positive. Problem 9.109 (Short list IMO, 2002). Let P (x) = ax3 + bx2 + cx + d be a cubic polynomial with integer coefficients a, b, c, d and a = 0. If xP (x) = yP (y), for an infinite number of integers x, y, with x = y, prove that P (x) has an integer root. Problem 9.110. Consider the positive real numbers a, b, c. Solve the system of equations: xy = a, yz = b, zx = c. Problem 9.111. Consider the real numbers a, b, c. Solve the system of equations: x(y + z) = a, y(z + x) = b, z(x + y) = c. Problem 9.112. Consider the positive real numbers a, b, c. Solve the system of equations: xyz xyz xyz = c, = a, = b. x+y y+z z+x Problem 9.113 (Balkanic, 2002). Solve the system of equations: a3 + 3ab2 + 3ac2 − 6abc = 1, b3 + 3ba2 + 3bc2 − 6abc = 1, c3 + 3cb2 + 3ca2 − 6abc = 1. 178 Chapter 9. Problems Problem 9.114. Let P (x) be a polynomial that takes integer values in the integers. Prove that there are integers c0 , c1 , . . . , cn such that P (x) = cn where x j = x x x , + · · · + c0 + cn−1 0 n−1 n x(x−1)(x−2)...(x−j+1) . j! Problem 9.115. Let p be an odd prime number and let P (x) = xp − x + p. Prove that P (x) is irreducible over Z[x]. Problem 9.116 (IMO, 2004). Find all polynomials P (x) with real coefficients that satisfy the equality P (a − b) + P (b − c) + P (c − a) = 2P (a + b + c), for all real numbers a, b, c, with ab + bc + ca = 0. Problem 9.117 (IMO, 2006). Let P (x) be a polynomial of degree n > 1, with integer coefficients and let k be a positive integer. Consider the polynomial Q(x) = P (P (. . . P (P (x)) . . . )), with k pairs of parenthesis. Prove that Q(x) has at most n integer fixed points, that is, integers that satisfy the equation Q(x) = x. Problem 9.118. Find all polynomials P (x) such that P (0) = 0 and P (x2 + 1) = P (x)2 + 1, for all real numbers x. Problem 9.119. Find all polynomials P (x) such that P (x)2 − 2 = 2P (2x2 − 1), for all real numbers x. Problem 9.120. Let P (x) and Q(x) be monic polynomials such that P (P (x)) = Q(Q(x)), for all real numbers x. Prove that P (x) = Q(x). Chapter 10 Solutions to Exercises and Problems The first eight sections of this chapter contain all solutions of the exercises in the first eight chapters. In Section 9, you can find the solutions to the problems of Chapter 9. The difficulty of the problems in Chapter 9 is usually greater than the difficulty of the exercises that you find in the first eight chapters. However, solving the problems of this last chapter would be an excellent training in preparation for international mathematical competitions. We recommend that the reader consult this last chapter just in case he or she cannot solve the exercises and problems alone. 10.1 Solutions to exercises of Chapter 1 Solution 1.1. (i) If a < 0, then −a > 0. Also use (−a)(−b) = ab. (ii) (−a)b > 0. (iii) a < b ⇔ b − a > 0, now use property (a). (iv) Use property (a). (v) aa−1 = 1 > 0. (vi) If a < 0, then −a > 0. Solution 1.2. Observe that if a2 + b − (a + b2 ) ∈ Q, then (a − b)(a + b − 1) ∈ Q and, since a + b − 1 ∈ Q \ {0}, then (a − b) ∈ Q. Therefore, if a + b ∈ Q and a − b ∈ Q, then 2a and 2b are in Q. Therefore, a and b are rational numbers. Solution 1.3. If a = 0 or b = 0, then the result is clear. Now suppose ab = 0. Since (a2 + b2 )2 − (a4 + b4 ) = 2a2 b2 , we have that a2 b2 ∈ Q. Note that a6 + b6 = (a2 + b2 )3 − 3a2b2 (a2 + b2 ) ∈ Q, then (a3 + b3 )2 − (a6 + b6 ) = 2a3 b3 ∈ Q. Therefore, a3 + b 3 a3 b 3 ∈ Q. ∈ Q and a + b = a2 b 2 a2 + b2 − ab √ √ Solution 1.4. (i) Suppose p is not an irrational number, that is, p = m n , where m, n are integers with (m, n) = 1, that is, m and n are relatively prime numbers. Squaring both sides of the equality, we get pn2 = m2 , that is, p divides m2 , then p divides m. Therefore, m = ps and pn2 = p2 s2 imply n2 = ps2 , this guarantees that p divides n2 and also divides n. Therefore, p divides m and n contradicting the fact that m and n are relatively prime. ab = © Springer Internationl Publishing Switzerland 2015 R.B. Manfrino et al, Topics in Algebra and Analysis, DOI 10.1007/978-3-319-11946-5_10 179 180 Chapter 10. Solutions to Exercises and Problems √ √ (ii) Suppose m is not an irrational number, that is, m = rs , where r, s are integers with (r, s) = 1. Squaring we have ms2 = r2 . Since m is not a perfect square, it has a factor of the form pα , where p is a prime number and α is a positive odd integer. Then pα divides r2 , which means that the prime p appears an even number of times in the factor decomposition of r2 . Since r and s are relatively prime numbers, p does not divide s, hence p appears an odd number of times as a factor of ms2 , which is a contradiction. Solution 1.5. If a + b = ab = n, then b = n − a and n = a(n − a). The last equation is equivalent to a2 − na + n = 0; solving the equation we have √ √ n ± n2 − 4n n ∓ n2 − 4n a= , from where b = . 2 2 √ For n ≥ 5, we have (n − 3)2 < n2 − 4n < (n − 2)2 , therefore n2 − 4n is an irrational number, and then a and b are irrational numbers. Solution 1.6. Suppose m n is a root, with (m, n) = 1, then m and n cannot both be    2 2 +b m even. On the other hand, since a m n n + c = 0, we have that am + bmn + 2 cn = 0. The right-hand side of the last equation is even and the left-hand side is odd. If m and n are odd numbers, the three terms of the left-hand side of the equation are odd. Now, if one term is even and the other is odd then two terms are even, the third odd and the sum is odd again. This contradiction implies that the equation cannot have rational roots. Second Solution. The discriminant b2 − 4ac has to be a perfect square. But, since a, b and c are odd numbers, we can prove that b2 − 4ac ≡ 5 mod 8. However, the square of an odd number has remainder 1 modulo 8. √ √ Solution 1.7. Let u = a + b and v = a − b, then √ √  √ √ √ √ u+ v u− v a+ b= u= + 2 2 √ √ 2 √ √ ( u + v) ( u − v)2 + = 4 4 √ √ u+v u+v uv uv 2 + 2 − + = 2 2 √ √ √ √ √ √ a+ b+a− b a+ b+a− b + a2 − b − a2 − b 2 2 + = 2 2 √ √ a + a2 − b a − a2 − b + , = 2 2 as we wanted to prove. 181 10.1 Solutions of Chapter 1 Solution 1.8. (i) Let x = a  a √ a a . . ., then x2 = a a  a √ a a . . ., therefore, x2 = ax. Factorizing, x(x − a) = 0. Therefore, since a is positive the solution is x = a. Second Solution. We can give another solution using series. We have 1 1 1 1 1 1 x = a 2 a 4 a 8 · · · = a 2 + 4 + 8 +··· = a, since *∞ 1 j=1 2j = 1, see Section 7.3.1.  √ b a b . . ., then x2 = a √ 3 a2 bx. Since x = 0, x3 = a2 b, then x = a2 b. a (ii) Let x = b  a √ b a . . ., therefore x4 = Second Solution. We can give another solution using series. We have 1 since *∞ 1 j=1 22j Solution 1.9. = 1 3 1 1 1 1 1 2 1 x = a 2 + 8 + 32 +··· b 4 + 16 + 64 +··· = a 3 b 3 , * 2 1 and ∞ j=0 22j+1 = 3 , see Section 7.3.1. (i) If xy, yz and zx are in Q, then (xy)(zx) = x2 ∈ Q. Similarly, y 2 , z 2 ∈ Q. yz Therefore, x2 + y 2 + z 2 ∈ Q. (ii) By (i) we have (x2 )2 + (xy)y 2 + (xz)z 2 = x(x3 + y 3 + z 3 ) ∈ Q, then x ∈ Q. Similarly, y, z ∈ Q. $ √ % √ √ Solution 1.10. Since a − ab = a 1 − √ab , it is sufficient to prove that 1 − √ab is a rational number from zero to claim that a is a rational number. √ √ different √ √ √ b( b− a) b b− √ab √ √ But a− ab = √a(√a− b) = − a is a rational number different from −1 (since a = b), therefore 1 − rational number. √ √b a is a rational number different from 0. Similarly, b is a Solution 1.11. To solve (i), define x = 0.111 . . . , then 10x = 1.11 . . . . Subtracting the first equation from the second, we get 9x = 1, therefore x = 19 . (ii) Let x = 1.141414 . . ., then 100x = 114.141414 . . .. Subtracting the first equation from the second, we get 99x = 113, therefore x = 113 99 . Solution 1.12. (i) First observe that 121b = (1 × b2 ) + (2 × b) + 1 = (b + 1)2 , then 121b is a perfect square in any base b ≥ 2. (ii) Since 232b = 2b2 + 3b + 2 has to be a square and since 3 is one of its digits, b ≥ 4. 182 Chapter 10. Solutions to Exercises and Problems For b = 4, 2324 = 46, for b = 5, 2325 = 67, for b = 6, 2326 = 92 and for b = 7, 2327 = 121. Then, b = 7 is the smallest positive integer such that 232b is a perfect square. Solution 1.13. Suppose that a > b. Then for all integers 0 ≤ k ≤ n, xn xk an bk ≥ xn xk bn ak , where the equality holds only when k = n or xk = 0. In particular, we have a strict inequality for k = n − 1. Adding, this becomes xn an n # xk bk > xn bn k=0 or n # xk ak k=0 xn bn xn an > . An Bn This implies that An−1 xn an xn bn Bn−1 =1− <1− = . An An Bn Bn Bn−1 An−1 An = Bn , and if a < b, using what > BBn−1 . Therefore, AAn−1 < BBn−1 if and n n n On the other hand, if a = b, then clearly we proved before, it follows that only if a > b. An−1 An Solution 1.14. Note that |a| = |a − b + b| ≤ |a − b| + |b|, therefore we have |a|−|b| ≤ |a−b|. Similarly, following the same procedure, we have |b|−|a| ≤ |b−a|. From these two inequalities, we get ||a| − |b|| ≤ |a − b|. Solution 1.15. (i) |x − 1| − |x + 1| = 0 is equivalent to |x − 1| = |x + 1|. Squaring both sides of the previous equation and solving (x − 1)2 = (x + 1)2 , we have 4x = 0, therefore the only solution is x = 0. (ii) |x − 1||x + 1| = 1 is equivalent to |x2 − 1| = 1, hence x2 − 1 = 1 x2 = 2 or −(x2 − 1) = 1, or x2 = 0, √ x = 0, x = ± 2 or √ therefore the solutions are x = ± 2 and x = 0. (iii) If x > 1 we get |x + 1| = x + 1 > 2, therefore there are no solutions. If x < −1 we get |x − 1| = −x + 1 > 2, therefore there are no solutions. If −1 ≤ x ≤ 1, then x − 1 ≤ 0 ≤ x + 1, therefore |x − 1| + |x + 1| = (1 − x) + (x + 1) = 2. Thus, the only values of x that satisfy the equality are −1 ≤ x ≤ 1. 183 10.1 Solutions of Chapter 1 Solution 1.16. From the first and third inequalities we have z ≥ |x + y| − 1 ≥ 0. Therefore, z 2 ≥ (|x + y| − 1)2 . Now, 2xy ≥ z 2 + 1 ≥ (|x + y| − 1)2 + 1 ≥ 0, then 2xy ≥ x2 + 2xy + y 2 − 2|x + y| + 2 ≥ |x|2 + 2xy + |y|2 − 2|x| − 2|y| + 2, cancelling out, 0 ≥ |x|2 + |y|2 − 2|x| − 2|y| + 2 = (|x| − 1)2 + (|y| − 1)2 . Therefore |x| = 1 and |y| = 1. Since x and y have to be −1 or 1, but since xy ≥ 0, both numbers have the same sign. For x = y = 1 or x = y = −1 we get, substituting in the original equations, that 2 − z 2 ≥ 1 and z − 2 ≥ −1. Therefore, z 2 ≤ 1 and z ≥ 1. The only value of z that satisfies both inequalities is z = 1. Therefore, there are two solutions to the problem x = y = z = 1 and x = y = −1, z = 1. Solution 1.17. Suppose that a1 < a2 < · · · < an is a collection with the largest quantity of integers that satisfy the property. It is clear that ai ≥ i, for all i = 1, . . . , n. ab , If a and b are two integers from the collection a > b, since |a − b| = a − b ≥ 100   b 100b we get a 1 − 100 ≥ b; therefore, if 100 − b > 0, then a ≥ 100−b . Note that there are no two numbers a and b in the collection greater than 100; in ab > a, which is false. fact if a > b > 100, then a − b = |a − b| ≥ 100 100b 100a ≥ 100−b if We also have that for integers a and b smaller than 100, we have 100−a and only if 100a − ab ≥ 100b − ab if and only if a ≥ b. It is clear that {1, 2, 3, 4, 5, 6, 7, 8, 9, 10} is a collection whose elements satisfy the property. 100a10 100·10 = 100 ≥ 100−10 Now, a11 ≥ 100−a 9 > 11, which implies that a11 ≥ 12. 10 a12 ≥ 100a11 100−a11 ≥ 100·12 100−12 = 1200 88 > 13, hence a12 ≥ 14. a13 ≥ 100a12 100−a12 ≥ 100·14 100−14 = 1400 86 > 16, hence a13 ≥ 17. a14 ≥ 100a13 100−a13 ≥ 100·17 100−17 = 1700 83 > 20, hence a14 ≥ 21. a15 ≥ 100a14 100−a14 ≥ 100·21 100−21 = 2100 79 > 26, hence a15 ≥ 27. a16 ≥ 100a15 100−a15 ≥ 100·27 100−27 = 2700 73 > 36, hence a16 ≥ 37. a17 ≥ 100a16 100−a16 ≥ 100·37 100−37 = 3700 63 > 58, hence a17 ≥ 59. a18 ≥ 100a17 100−a17 ≥ 100·59 100−59 = 5900 41 > 143, hence a18 ≥ 144. Moreover, as we have already observed, there are no two integers of the collection greater than 100, so the largest quantity is 18. A collection with 18 integers that satisfies the conditions is {1, 2, 3, 4, 5, 6, 7, 8, 9, 10, 12, 14, 17, 21, 27, 37, 59, 144} . 184 Chapter 10. Solutions to Exercises and Problems Solution 1.18. By Example 1.3.2, ⌊2a⌋ = ⌊a⌋ + ⌊a + 12 ⌋ and ⌊2b⌋ = ⌊b⌋ + ⌊b + 12 ⌋, then the inequality that we have to prove is equivalent to     1 1 + ⌊b⌋ + b + ≥ ⌊a⌋ + ⌊b⌋ + ⌊a + b⌋, ⌊a⌋ + a + 2 2     then we only have to prove that a + 21 + b + 21 ≥ ⌊a + b⌋. Let a = n+ y, b = m+ x, with n, m ∈ Z and 0 ≤ x, y < 1. Then 0 ≤ x+ y < 2 and a + b = n + m + x + y. We have two cases: (i) If 1 ≤ x + y < 2, then ⌊a + b⌋ = n + m + 1, and at least one of the numbers x or y is greater than or equal to 21 . Suppose that x ≥ 12 , then ⌊b+ 21 ⌋ = ⌊m+x+ 21 ⌋ = m+1, therefore ⌊a+ 21 ⌋+⌊b+ 21 ⌋ ≥ m+n+1 = ⌊a+b⌋. (ii) If 0 ≤ x+ y < 1, then ⌊a+ b⌋ = n+ m and ⌊a+ 21 ⌋+ ⌊b + 12 ⌋ ≥ m+ n = ⌊a+ b⌋. Solution 1.19. (i) We have that ⌊x⌊x⌋⌋ = 1 if and only if 1 ≤ x⌊x⌋ < 2. If x = m + y, with m ∈ Z and 0 ≤ y < 1, then 1 ≤ m2 + my < 2. Observe that m = 0 is impossible, as well as m ≥ 2 or m ≤ −2. Therefore, it only remains to be proved for m = 1 or m = −1. If m = 1, then 1 ≤ 1 + y < 2, from which 0 ≤ y < 1 and then any x in the interval [1, 2) satisfies the equation. If m = −1, then since 1 ≤ m2 + my < 2, we have 1 ≤ 1 − y < 2, from which 0 ≤ −y < 1 and then y = 0 and x = −1. Therefore, the numbers that satisfy the equation are x = −1 and x ∈ [1, 2). (ii) Since ⌊x⌋ ≤ x ≤ |x|, it follows that |x| − ⌊x⌋ ≥ 0, therefore ||x| − ⌊x⌋| = |x| − ⌊x⌋. On the other hand, by property (c) in 1.3.1 we obtain ⌊|x| − ⌊x⌋⌋ = ⌊|x|⌋ − ⌊x⌋. Using the last equalities, the equation becomes |x| − ⌊x⌋ = ⌊|x|⌋ − ⌊x⌋, which is equivalent to |x| = ⌊|x|⌋; then |x| is an integer number and the values of x that satisfy the equation are all the integers. Solution 1.20. Add the three equations to obtain 2x + 2y + 2z = 6.6, so that x + y + z = 3.3. If you subtract this equation from the original ones, we obtain {y} + ⌊z⌋ = 2.2, {x} + ⌊y⌋ = 1.1, {z} + ⌊x⌋ = 0. For the first equation, we obtain ⌊z⌋ = 2 and {y} = 0.2; the second equation becomes ⌊y⌋ = 1, {x} = 0.1, and the third ⌊x⌋ = 0 and {z} = 0. Therefore, the solution is x = 0.1, y = 1.2 and z = 2. √ √ √ √ 2 Solution 1.21. We have n+ n + √ 1 < 4n + 2 if and only if 2n+1+ 4n + 4n < 2 again,√the last 4n + 2, which is equivalent to 4n + 4n < 2n + 1. Squaring √ 2 2 inequality is equivalent to 4n +4n < 4n +4n+1. This proves that n+ n + 1 < √ √ √ √ 4n + 2, then ⌊ n + n + 1⌋ ≤ ⌊ 4n + 2⌋. 10.1 Solutions of Chapter 1 185 √ √ √  ⌊ 4n + 2⌋. Suppose √ integer n, ⌊√ n + n + 1⌋ = √ that, for some √ positive Let q =√ ⌊ 4n + 2⌋, then n + n + 1 < q ≤√ 4n + 2. Squaring, we obtain 2n+1+ 4n2 + 4n < q 2 ≤ 4n+2, or equivalently, 4n2 + 4n < q 2 −2n−1 ≤ 2n+1. Squaring again we find that 4n2 + 4n < (q 2 − 2n − 1)2 ≤ 4n2 + 4n + 1 = (2n + 1)2 . Since there does not exist a square between two consecutive integers, we have q 2 − 2n − 1 = 2n + 1 or q 2 = 4n + 2, which is equivalent to saying that q 2 ≡ 2 mod 4. But this is a contradiction, since any square number is congruent to 0 or 1 mod 4. Therefore, we get the equality. √ √ √ We now prove that ⌊ 4n + 1⌋ = ⌊ 4n + 2⌋ = ⌊ 4n + 3⌋. √ For the that there exists n such √ first equality, suppose √ √ that m = ⌊2 4n + 1⌋ < m + 1 = ⌊ 4n + 2⌋, therefore m ≤ 4n + 1 < m + 1 ≤ 4n + 2, or m ≤ 4n + 1 < (m + 1)2 ≤ 4n + 2. Therefore, since 4n + 1 and 4n + 2 are two consecutive integers, and since (m + 1)2 > 4n + 1, therefore (m + 1)2 = 4n + 2, and again we found a square number which has remainder 2 when we divide the number by 4, which is impossible. For the second equality, proceed in the same way. Solution 1.22. For the first five parts use equations (1.2), (1.3) and (1.5). To prove (vi), use (iv) and (v). Solution 1.23. For the first two parts (i) and (ii), use equations (1.2) and (1.3). To prove (iii), use (i) and (ii). Solution 1.24. To prove (i) and (ii) just expand the left-hand side of the equations and rearrange the terms. To prove (iii), (iv), (v) and (vi) make the operations on both sides of the equality and observe that they are equal. Solution 1.25. To prove (i) and (ii) expand the right-hand side of the equations and simplify. Solution 1.26. Use equations (1.2) and (1.3), and perform the operations on both sides of the equation. Solution 1.27. Let x = 3 √ √ 3 5 + 2 − 5, then   √ √ 3 3 x − 2 + 5 − 2 − 5 = 0. 2+ By equation (1.7), if a + b + c = 0, then a3 + b3 + c3 = 3abc, therefore $ $ √ % √ % √ %$ √ % $ x3 − 2 + 5 − 2 − 5 = 3x 3 2 + 5 2 − 5 , 186 Chapter 10. Solutions to Exercises and Problems simplifying we have that x3 + 3x − 4 = 0. Clearly a root of the equation is x = 1 and the other roots satisfy the equation x2 + x + 4 = 0 which does not have real √ √ 3 3 solutions. Since 2 + 5 + 2 − 5 is a real number, it follows that   √ √ 3 3 2 + 5 + 2 − 5 = 1, which is a rational number. Solution 1.28. Observe that, if x + y + z = 0, then it follows from equation (1.7) that x3 + y 3 + z 3 = 3xyz. Since (x − y) + (y − z) + (z − x) = 0, we obtain the factorization (x − y)3 + (y − z)3 + (z − x)3 = 3(x − y)(y − z)(z − x). Solution 1.29. Observe that (x+ 2y − 3z)+ (y + 2z − 3x)+ (z + 2x− 3y) = 0, then it follows, from equation (1.7), that (x + 2y − 3z)3 + (y + 2z − 3x)3 + (z + 2x − 3y)3 = 3(x + 2y − 3z)(y + 2z − 3x)(z + 2x − 3y). √ √ √ Solution 1.30. Let a = 3 x − y, b = 3 y − z, c = 3 z − x, and suppose that a+ b + c = 0, then it follows, from equation (1.7), that a3 + b3 + c3 = 3abc, √ but then √ √ 0 = (x − y) + (y − z) + (z − x) = a3 + b3 + c3 = 3abc = 3 3 x − y 3 y − z 3 z − x = 0, which is absurd. √ 1 Solution 1.31. If we define a = 3 r, b = − √ 3 r and c = −1, we have a + b + c = 0, % $ √ 1 (−1) = 3, therefore r− r1 = 4. Similarly, r3 − r13 −43 = then r− r1 −1 = 3 3 r − √ 3 r  1 3r − r (−4) = 12, therefore r3 − r13 = 76. Solution 1.32. It follows from  a b  a3 + b3 + c3 − 3abc =  c a b c   c  100b + 10c + a b  = 100a + 10b + c a 100c + 10a + b b a c   c  bca b  = abc a cab b a c  c  b  . a Solution 1.33. If we rewrite the equation as m3 + n3 + (−33)3 − 3mn(−33) = 0, and using equation (1.9), we get ! " (m + n − 33) (m − n)2 + (m + 33)2 + (n + 33)2 = 0. The equation m + n = 33 has 34 solutions with mn ≥ 0 which are (k, 33 − k), with k = 0, 1, . . . , 33, and the second factor is 0 only when m = n = −33, therefore there are 35 solutions. Solution 1.34. If we rewrite the equation as x3 + y 3 + (−1)3 − 3xy(−1) = 0, and using equation (1.9), we have " ! (x + y − 1) (x − y)2 + (y + 1)2 + (−1 − x)2 = 0. Therefore, the points (x, y) satisfy x + y = 1 or x = y = −1. 187 10.1 Solutions of Chapter 1 Solution 1.35. Substituting the equation of the hypothesis in equation (1.7), we get (x + y + z)3 − 3xyz = x3 + y 3 + z 3 − 3xyz = (x + y + z)(x2 + y 2 + z 2 − xy − yz − zx) = (x + y + z)((x + y + z)2 − 3xy − 3yz − 3zx) = (x + y + z)3 − 3(x + y + z)(xy + yz + zx), from where it is clear that xyz = (x + y + z)(xy + yz + zx), therefore (x + y)(y + z)(z + x) = 0. Or use that (x + y + z)3 = x3 + y 3 + z 3 + 3(x + y)(y + z)(z + x). Therefore, the solutions are (x, −x, z), (x, y, −y), (x, y, −x), with x, y, z any real numbers. Solution 1.36. (i) Since 0 ≤ b ≤ 1 and 1 + a > 0, it follows that b(1 + a) ≤ 1 + a, then b−a ≤ 1. 0 ≤ b − a ≤ 1 − ab, therefore 0 ≤ 1 − ab 1 1 ≤ 1+a , then (ii) The inequality is clear. Since 1 + a ≤ 1 + b, we have 1+b a b a b a+b + ≤ + = ≤ 1. 1+b 1+a 1+a 1+a 1+a Solution 1.37. If we define X = times 1, leads to X= a b+c + b a+c + c a+b , adding and substracting three b+c b a+c c a+b a + + + + + −3 b+c b+c a+c a+c a+b a+b a+b+c a+b+c a+b+c + + −3 b+c a+c a+b 1 1 1 + + −3 = (a + b + c) b+c a+c a+b 1 1 1 1 = ((a + b) + (b + c) + (a + c)) + + 2 b+c a+c a+b = − 3. √ Now, from the arithmetic and geometric mean inequality, we get x+y +z ≥ 3 3 xyz  and x1 + y1 + z1 ≥ 3 3 x1 y1 1z . Therefore, X ≥ 12 · 3 · 3 − 3 = 23 . Solution 1.38. Without loss of generality we can assume that a ≥ b ≥ c; the inequality is equivalent to −a3 + b3 +! c3 + 3abc ≥ 0. But, by equation (1.9), " −a3 + b3 + c3 + 3abc = 12 (−a + b + c) (a + b)2 + (a + c)2 + (b − c)2 ≥ 0, since, by the triangle inequality, a < b + c. Solution 1.39. Observe that implies s ≥ 4. 1 p + 1q = 1 implies p + q = pq = s. Now, (p + q)2 ≥ 4pq 188 Chapter 10. Solutions to Exercises and Problems To prove (i), observe that 1 1 1 1 1 p+q+2 1 + = − + − = 1− p(p + 1) q(q + 1) p p+1 q q+1 (p + 1)(q + 1) s+2 s−1 =1− = . 2s + 1 2s + 1 Therefore, we have to prove s−1 1 1 ≤ ≤ , 3 2s + 1 2 but 2s + 1 ≤ 3s − 3 ⇔ 4 ≤ s and 2s − 2 ≤ 2s + 1 ⇔ −2 ≤ 1. To prove (ii), show that 1 1 1 1 1 1 p+q−2 + = − + − = −1 p(p − 1) q(q − 1) p−1 p q−1 q (p − 1)(q − 1) s−2 − 1 = s − 3 ≥ 1. = s−s+1 Solution 1.40. First, note that a a a + = ≥ 4a, b 1−b b(1 − b) since b(1 − b) ≤ b + (1 − b) 2 2 = 1 . 4 Moreover, the equality holds if and only if b = 21 . Similarly, Therefore, b b + ≥ 4b. a 1−a √ √ a b a b + + + ≥ 4a + 4b ≥ 2 42 ab = 8 k. b a 1−b 1−a With equality if and only if a = b. Then, √ a b a b + + + ≥8 k≥4 b a 1−b 1−a if and only if k ≥ 14 , then the smallest number k is 14 . Solution 1.41. Prove that (a + b)(b + c)(c + a) = (a + b + c)(ab + bc + ca) − abc = 98 (a + b + c)(ab + bc + ca) + 19 (a + b + c)(ab + bc + ca) − abc, and from the arithmetic inequality, we have that (a + b + c)(ab + bcb + ca) ≥ $ % $ and geometric mean % √ 3 3 3 abc 3 (ab)(bc)(ca) = 9abc. 189 10.1 Solutions of Chapter 1 Solution 1.42. Using the arithmetic and geometric mean inequality, and the condition (a + b)(b + c)(c + a) = 1, leads to a+b b+c c+a 3 = , 2 2 2 2 √ √ √ a+b b+c c+a abc = ab bc ca ≤ 2 2 2 a+b+c≥33 = 1 . 8 Now, 1 = (a + b)(b + c)(c + a) = (a + b + c)(ab + bc + ca) − abc ≥ 32 (ab + bc + ca) − 18 , see Exercise 1.24 (iii). 3 ≥ 4. Solution 1.43. By Exercise 1.24 (iii), it is enough to prove ab + bc+ ca+ a+b+c But ab + bc + ca + ab + bc + ca 3 3 =3 a+b+c ≥4 4 ab + bc + ca 3 + 3 3 a+b+c 3 . a+b+c Now use that (ab√+ bc + ca)2 ≥ 3(ab · bc + bc · ca + ca · ab) = 3(a + b + c), and that 3 ab + bc + ca ≥ 3 a2 b2 c2 = 3. Solution 1.44. Without loss of generality we can assume that a ≤ b ≤ c. Therefore c2 < c2 + a + b ≤ c2 + 2c < (c + 1)2 ; this proves that c2 + a + b cannot be a perfect square. Solution 1.45. To prove all the equalities of the exercise, just perform the operations and simplify. Solution 1.46. To prove all the equalities of the exercise, just use the identity (1.7). Solution 1.47. Expand both sides of the identities and compare. Solution 1.48. We have 0 = x2 (y + z) − y 2 (x + z) = xy(x − y) + (x2 − y 2 )z = (x − y)(xy + xz + yz). Since x = y, we have xy + xz + yz = 0. Multiplying by x − z we obtain 0 = (x − z)(xy + xz + yz) = xz(x − z) + (x2 − z 2 )y = x2 (y + z) − z 2 (x + y), then z 2 (x + y) = x2 (y + z) = 2. 190 Chapter 10. Solutions to Exercises and Problems Solution 1.49. See that (x + y + z)(xy + yz + zx) = xyz and by equation (1.23) we get (x + y)(y + z)(z + x) = 0. Therefore, the solutions (x, y, z, w) are of the form (x, −x, z, z), (x, y, −y, x) and (x, y, −x, y), with x, y and z real numbers different from zero. Solution 1.50. By equation (1.23) the condition is equivalent to (x + y)(y + z)(z + x) = 0. Therefore, one factor is zero, say x+ y = 0. Then, since n is odd, xn + y n = 0, and also x1n + y1n = 0. 10.2 Solutions to exercises of Chapter 2 Solution 2.1. Call cn the sum of the first n even numbers, then we have cn cn 2cn = = = 2 + 4 + 2n + 2n − 2 + (2n + 2) + (2n + 2) + ··· + 2n ··· + 2 · · · + (2n + 2) (10.1) therefore, 2cn = n(2n + 2) = 2n(n + 1), then cn = n(n + 1). We can represent this sum as arrangements of points forming rectangles such as those below: Solution 2.2. (i) Let d and d′ be the differences of the progressions {an } and {bn }, respectively. Then, (an+1 − an ) ± (bn+1 − bn ) = d ± d′ . Rearranging the terms we have (an+1 ± bn+1 ) − (an ± bn ) = d ± d′ . (ii) If d is the difference of the progression {an }, we obtain bn+1 − bn = (a2n+2 − a2n+1 ) − (a2n+1 − a2n ) = (an+2 − an+1 )(an+2 + an+1 ) − (an+1 − an )(an+1 + an ) = d(an+2 + an+1 − an+1 − an ) = d(an+2 − an ) = 2d2 . Solution 2.3. If d is the difference of the progression {an }, we have n−1 # j=0 n−1 1# 1 1 1 1 = − = aj aj+1 d j=0 aj aj+1 d = 1 d an − a0 a0 an = 1 d 1 1 − a0 an a0 + nd − a0 a0 an = n . a0 an 191 10.2 Solutions of Chapter 2 Solution 2.4. If {an } is an arithmetic progression, a0 + an−1 ·n 2 d d 2a0 + (n − 1)d · n = n2 + a0 − n, = 2 2 2 Sn = a0 + a1 + · · · + an−1 = and with A = d 2 and B = a0 − d 2 we get the result. Suppose now that Sn = a0 + a1 + · · · + an−1 = An2 + Bn, then Sn+1 = a0 + a1 + · · · + an−1 + an = A(n + 1)2 + B(n + 1). Subtracting the first equation from the second, we have an = A(n + 1)2 + B(n + 1) − An2 − Bn = A(2n + 1) + B = 2An + (A + B) and using Proposition 2.1.3, we get the expected result. Solution 2.5. Suppose that {an } is an arithmetic progression of order 2. Consider Sn = (a1 − a0 ) + (a2 − a1 ) + · · · + (an − an−1 ) = an − a0 . By Exercise 2.4, we have an − a0 = An2 + Bn. Therefore, an = An2 + Bn + a0 = P (n), where P (x) is the polynomial of degree 2, given by Ax2 + Bx + a0 . Suppose now that each term of the progression an is equal to P (n), where P (x) is a polynomial of degree 2, that is, an = An2 + Bn + C. It follows that an+1 − an = A(n + 1)2 + B(n + 1) + C − (An2 + Bn + C) = 2An + (A + B). Therefore, by Proposition 2.1.3, {an+1 − an } is an arithmetic progression and thus {an } is an arithmetic progression of order 2. Solution 2.6. A consequence of the inequality between the geometric mean and √ n . By Proposition 2.1.1, we get the arithmetic mean, is n a1 a2 · · · an ≤ a1 +a2 +···+a n √ a1 + a2 + · · · + an a1 + an 1 a1 + an n = ·n· = . a1 a2 · · · an ≤ n 2 n 2 To prove the left-hand side inequality, we use a similar version of the equality of Exercise 2.3 1 1 1 n−1 + + ···+ = . a1 a2 a2 a3 an−1 an a1 an By the inequality between the harmonic mean and the geometric mean, we get a1 an = 1 a1 a2 n−1 ≤ 1 + · · · + an−1 an n−1 (a1 a2 )(a2 a3 ) · · · (an−1 an ). Then, (a1 an )n−1 ≤ a1 (a2 · · · an−1 )2 an . √ √ Therefore, (a1 an )n ≤ (a1 a2 · · · an )2 , that is, a1 an ≤ n a1 a2 · · · an . 192 Chapter 10. Solutions to Exercises and Problems Solution 2.7. If p, p + 6, p + 12, p + 18, p + 24 are the 5 prime numbers, when we consider these numbers modulo 5, we have that they are congruent to p, p + 1, p+ 2, p+ 3, p+ 4, but between five consecutive numbers we have always one that is divisible by 5, and since p is prime, then p = 5 and hence the numbers are forced to be 5, 11, 17, 23 and 29. Solution 2.8. The numbers m between 2n and 2n+1 are part of an arithmetic progression with difference 1 starting in 2n + 1 and ending in 2n+1 − 1. Then, by Proposition 2.1.1, Sn = 3 · 2n n+1 2n + 1 + 2n+1 − 1 n+1 (2 (2 − 1 − 2n ) = − 1 − 2n ), 2 2 and from this it is clear that 3 divides Sn . Solution 2.9. Since a, b and c are in harmonic progression, in that order, we can 1 = A(A+s) , suppose that a1 = A − s, 1b = A and 1c = A + s, with s = 0. We have b−c s 4 c−a = −2(A2 −s2 ) 1 , a−b s = A(A−s) s and 1 c − 1 a = 2s. Therefore, 4 1 A2 + As − 2A2 + 2s2 + A2 − As 1 1 1 + + = = 2s = − . b−c c−a a−b s c a Solution 2.10. If a, b, c and d are in harmonic progression, then their inverses are in arithmetic progression. Suppose that 1b = a1 + s, 1c = a1 + 2s and d1 = a1 + 3s, 1 1+2as and d1 = 1+3as with s = 0, that is, we have 1b = 1+as a , c = a a . Then we have a a a that, b = 1+as , c = 1+2as and d = 1+3as . Then, a+d= 2a + 3a2 s 1 + 3as and b + c = 2a + 3a2 s . 1 + 3as + 2(as)2 Since 1 + 3as < 1 + 3as + 2(as)2 , we have that a + d > b + c, as desired. Solution 2.11. Note that 1 1 b−a b 2 − a2 − = = , c+a b+c (b + c)(c + a) (b + a)(b + c)(c + a) 1 c−b c2 − b 2 1 − = = , a+b c+a (a + b)(c + a) (b + a)(b + c)(c + a) 1 1 1 1 − b+c = a+b − c+a if and only if b2 − a2 = c2 − b2 . Then, b + c, c + a then c+a and a + b are in harmonic progression if and only if a2 , b2 and c2 are in arithmetic progression. Solution 2.12. Suppose that a0 , a1 , . . . is the progression and that d is the difference of the progression, that is, d = an+1 − an for all n ≥ 0, then an = a0 + nd. 193 10.2 Solutions of Chapter 2 By hypothesis, a0 (a0 + d) and a0 (a0 + 2d) also belong to the progression, therefore a0 (a0 + 2d) − a0 (a0 + d) = n0 d for some integer number n0 ≥ 1, then a0 d = n0 d. Since the progression is increasing, it follows that d > 0 and therefore a0 = n0 . Change a0 for any am in the previous argument and conclude that am is an integer number. Solution 2.13. (i) Observe that in the array, the left-hand side number of each row is as follows: 1◦ 2◦ 3◦ 4◦ 5◦ row row row row row .. . 1 1+2= 3 1+2+4 = 7 1 + 2 + 4 + 6 = 13 1 + 2 + 4 + 6 + 8 = 21 .. . 100◦ row 1 + 2 + · · · + 2 · 99 = 1 + 2(1 + · · · + 99) = 9901. (ii) The sum of the numbers on the 100th row is, 9901 + 9903 + · · · + 10099 = (9900 + 1) + (9900 + 3) + · · · + (9900 + 2 · 100 − 1) = 9900 · 100 + (1 + 3 + · · · + 2 · 100 − 1) = 990000 + 1002 = 106 . Solution 2.14. The array will be a square of size 100 × 100. Let S1 be the sum of the numbers in the diagonal which goes from the top left corner to the bottom right corner and let S2 be the sum of the numbers of the other main diagonal. When we move one column to the right in the same row, the number increases by 1; if we move one column to the left in the same row, the number decreases by 1. When we move down in the same column, the number increases by 100. From the top left corner to the bottom right corner through the diagonal, each number is one column to the right and one row below, that is, it is 1 + 100 = 101 greater than the previous number of the diagonal. That is, the sum we want to calculate is the sum of the progression 1, 1+101, . . . , 1+99·101 that, by Proposition 2.1.1 (b), is 2 · 1 + 99 · 101 100 = 500050. S1 = 2 From the top right corner, through the diagonal, each number is in the previous column, that is, one column to the left and one row below, that is, it is −1 + 100 = 99 greater than the previous number in the diagonal. That is, the sum we are 194 Chapter 10. Solutions to Exercises and Problems looking for is the sum of the progression 100, 100 + 1 · 99, . . . , 100 + 99 · 99 that, by Proposition 2.1.1 (b), is S2 = 2 · 100 + 99 · 99 2 100 = 500050. Therefore, S1 = S2 = 500050. Solution 2.15. Call aij , where i denotes the number of the row and j denotes the number of the column, the corresponding position in the table. Let x0 and x1 be the two numbers neighboring 0, x0 on the right-hand side of 0 and x1 on top of 0, then the last row can be filled up with 0, x0 , 2x0 , 3x0 and 4x0 , and the first column with 4x1 , 3x1 , 2x1 , x1 and 0. If x is the number in the position a32 , the number that occupies the a42 position will be 12 (x + x0 ), but we also know that the number in the a42 position is 21 (x1 + 103). Therefore, 12 (x + x0 ) = 21 (x1 + 103), solving for x we have that x = x1 + 103 − x0 . Now, let y be the number occupying the a44 position, then we have 103 = 12 (y + 1 1 2 (x1 + 103)), solving for y we obtain y = 2 (309 − x1 ). 1 1 The number 2 (309−x1)−103 = 2 (103−x1 ) is the difference of the progression in the fourth row, but this difference, added to the number in the a44 position, gives the number in position a45 , that is, 21 (309 − x1 ) + 12 (103 − x1 ) = 206 − x1 . However, we know that 206 − x1 = 12 (186 + 4x0 ), then 2x0 + x1 = 113. (10.2)   Observe also that 21 74 + 12 (x1 + 103) = x1 + 103 − x0 . Simplifying we obtain 4x0 − 3x1 = 161. (10.3) Solving the system of equations (10.2) and (10.3), we get x0 = 50 and x1 = 13. With these values, now it is easy to complete the table, so the filled board is 52 39 26 13 0 82 74 66 58 50 112 109 106 103 100 142 144 146 148 150 172 179 186 193 200 Solution 2.16. If {an } is a geometric progression with ratio r, we have that an = n a0 rn . Similarly, if {bn } is a geometric progressionwith  ratio s, then bn = b0 s . an a0 r n Therefore, since bn = 0 for all n, we have bn = b0 s . Solution 2.17. Let {an } be a geometric progression with ratio r and having the property that an+2 = an+1 + an . Since an = a0 rn , this property is equivalent to a0 rn+2 = a0 rn+1 +a0 rn . Since a0 = 0 and r = 0, we certainly have that r2 = r +1, 195 10.2 Solutions of Chapter 2 which has as solutions r = $ √ %n   . an = a0 1−2 5 √ 1± 5 2 .  $ √ %n  Then, the solutions are an = a0 1+2 5 and Solution 2.18. (i) Since Pn = a0 · a1 · · · · · an−1 and an = a0 rn , for all n, we have Pn = a0 · a1 · · · · · an−1 = a0 (a0 r)(a0 r2 ) · · · (a0 rn−1 ) = an0 r1+2+···+(n−1) = an0 r (ii) Since Pn = an0 r n(n−1) 2 n(n−1) 2 . , it is clear that $ %2 n  n(n−1) (Pn )2 = an0 r 2 = an0 an0 rn(n−1) = an0 a0 rn−1 = an0 ann−1 . Solution 2.19. Since an+1 = an · r, then bn+1 = log an+1 = log (an · r) = log an + log r = bn + log r, and the result follows. Solution 2.20. Factorizing we have     a3 b3 + b3 c3 + c3 a3 − abc(a3 + b3 + c3 ) = −bc + a2 −ca + b2 −ab + c2 . Solution 2.21. If the arithmetic progression is a, a + d, a + 2d, . . . , it is clear that a + ad = a(1 + d) is an element of the arithmetic progression, and also the integers a(1 + d)n , with n ≥ 1, are part of the progression. But these terms form a geometric progression, then it is clear that these terms have to remain in the sequence and we have to eliminate the remaining terms of the original progression. Solution 2.22. Consider a < b < c, then since the lengths of the sides are in geometric progression, we have b = ar and c = ar2 , with r positive. Since the triangle is a right triangle, it follows that a2 + (ar)2 = (ar2 )2 . Simplifying √ the 1+ 5 2 4 2 2 equation we get 1 + r = r , which can be solved for r . That is, r = 2 .  √ Therefore, r = 1+2 5 . Solution 2.23. Let d be the common difference of the progression. Then a2 = 1+d, a5 = 1 + 4d and a11 = 1 + 10d. Since a2 , a5 , a11 form a geometric progression, we have (1 + 4d)2 = (1 + d)(1 + 10d) or 6d2 = 3d. Since the arithmetic progression is not constant, we conclude that d = 21 and the sum of the first 2009 terms is · 21 = 2009 · 503. 2009 + 2009·2008 2 Solution 2.24. By the similarity of the triangles, we have that Again, using the similarity of the triangles, we have ay = yz . b a = ya , then y = a2 b . 196 Chapter 10. Solutions to Exercises and Problems A y z B Then z = 3 = ab2 . z= a4 b2 a 2 y a b a C , and substituting the value of y in this equation results in (i) Carry on with this process and find that the sides of the polygonal measure    2  3 2 3 4 b, a, ab , ab2 , ab3 , . . ., which can be written as b, a, a ab , a ab , a ab , . . ..  n−2 Therefore, the nth segment measures a ab . (ii) The length of the n-sided polygonal line is then b+a $ a %0 b +a $ a %1 b + ···+ a $ a %n−2 b  n−1  1 − ab =b+a 1 − ab   n−1  1 − ab . = b + ab b−a  (iii) Since the number ab is less than one, then raising this number to the nth power and making n go to infinite leads to 0 as its limit. Then, the length of the polygonal line with an infinite number of sides is    n−1  1 − ab ab b2 =b+ = . lim b + ab n→∞ b−a b−a b−a , Solution 2.25. The sum of each row is $ Rn = 12 + 22%+ · · · + n2 = n(n+1)(2n+1) 6 n(n+1)(2n+1) then the sum of all the array is ST = n . See now that the sum of 6 each corridor is Ck = 12 + 22 + · · · + (k − 1)2 + k 3 = k2 k 8 k3 − + . 6 2 6 Then ST = C1 + C2 + · · · + Cn 1 1 8 = (13 + 23 + · · · + n3 ) − (12 + 22 + · · · + n2 ) + (1 + 2 + · · · + n) 6 2 6 1 n(n + 1)(2n + 1) 8 3 1 n(n + 1) = (1 + 23 + · · · + n3 ) − + . 6 2 6 6 2 197 10.2 Solutions of Chapter 2 Therefore 8 3 n2 (n + 1)(2n + 1) n(n + 1)(2n + 1) 1 (1 + 23 + · · · + n3 ) = + − 6 6 12 6 = 2 n(n + 1) 2 8 6 n(n + 1) 2 . From this the sought for equality follows immediately. Solution 2.26. Observe that the numbers {1, 4, 7, . . . , 2998, 3001} form an arithmetic progression {an } with difference 3 and a1 = 1. Each term of the sum can be seen as 1 1 1 1 1 1 = = − − . ai ai+1 ai+1 − ai ai ai+1 3ai 3ai+1 Therefore, we can calculate the sum as 1 1 − 3a1 3a2 1 1 − + ···+ 3a2 3a3 1 1000 1 = . = − 3 3 · 3001 3001 + Solution 2.27. (i) We have n # k=1 1 k(k+2) = 1 2 $ 1 k − 1 k+2 n % 1 1 − 3 · 3000 3 · 3001 , then 1 1 − k k+2 #1 1 = k(k + 2) 2 k=1 1 1 1 1 1 1 1 1 1 − + − + − + ···+ − 2 1 3 2 4 3 5 n n+2 1 1 1 1 − = 1+ − 2 2 n+1 n+2 2n + 3 3 . = − 4 2(n + 1)(n + 2) = (ii) We have 2k+1 k2 (k+1)2 n # k=1 = 1 k2 − 1 (k+1)2 , n # 2k + 1 = k 2 (k + 1)2 k=1 then 1 1 − k2 (k + 1)2 1 1 1 1 1 1 − 2 + 2 − 2 + ···+ 2 − 2 1 2 2 3 n (n + 1)2 1 . =1− (n + 1)2 = 198 Chapter 10. Solutions to Exercises and Problems Solution 2.28. (i) Since k (k + 1) − 1 k+1 1 1 1 = = − = − (k + 1)! (k + 1)! (k + 1)! (k + 1)! k! (k + 1)! the sum we have to calculate becomes 1 1 − 1! 2! + 1 1 − 2! 3! + ···+ 1 1 − n! (n + 1)! =1− 1 . (n + 1)! (ii) The general term can be written as k+1 k+1 = (k − 1)! + k! + (k + 1)! (k − 1)![1 + k + k(k + 1)] k+1 = (k − 1)!(k + 1)2 k 1 = . = (k − 1)!(k + 1) (k + 1)! By the first part of the exercise, the sum is equal to 1 − 1 (n+1)! . Solution 2.29. For any positive integer n, we have 1+ 1 1 n2 (n + 1)2 + (n + 1)2 + n2 (n2 + n + 1)2 + = = . 2 2 2 2 n (n + 1) n (n + 1) n2 (n + 1)2 Then 1+ 1 1 1 n2 + n + 1 =1+ . + = n2 (n + 1)2 n2 + n n(n + 1) Therefore, the sum is equal to 2011 # n=1 1+ 1 n(n + 1) = 2011 # 1+ n=1 1 1 − n n+1 = 2012 − 1 . 2012 Solution 2.30. Define S as the product we have to calculate, that is, 1 2 1 1 · · · 1 + 2n . 22 2      Multiplying both sides of the equality by 1 − 21 and using that 1 − 21 1 + 12   = 1 − 212 , we get S= 1− 1 2 1+ S= 1− 1+ 1 22 1+ 1 22 ··· 1 + 1 22n . 199 10.3 Solutions of Chapter 3 Proceeding in this way, we arrive at 1 1− 2 Therefore, S=  1− 1 22n 2 %   1 − 221n   =2 1− S= 1 − 12 $ 2  1 22n . 2  . 10.3 Solutions to exercises of Chapter 3 Solution 3.1. (i) If n = 1, then 1 = 1−q 1−q = 1. Suppose that 1 + q + · · · + q n−1 = 1−qn 1−q . 1 + q + · · · + q n−1 + q n = Then, 1 − q n+1 1 − qn + qn = , 1−q 1−q as we wanted to prove. Second Solution. Let S = 1+q+· · ·+q n−1 , then Sq = q+q 2 +· · ·+q n . Substracting n the first equality from the second leads to Sq − S = q n − 1, then S = 1−q 1−q . (ii) The proof is immediate from (i). Solution 3.2. The result is valid for n = 1, since a3 = a2 + a1 = 2 and a3 = 1 + a1 = 2. Suppose that the result is valid for n, that is, an+2 = 1 + a1 + a2 + · · · + an . Then the formula is valid for n + 1, since an+3 = an+2 + an+1 = 1 + a1 + a2 + · · · + an + an+1 . Solution 3.3. For n = 0 the statement is valid, because 3 divides 2 + 1 = 3. The n−1 + 1. induction hypothesis for n − 1 tells us that 3n divides 23 We prove now the result for n. We start with +$ , % n n−1 n−1 2 n−1 + 1) 23 23 + 1 = (23 − 23 +1 . By the induction hypothesis, the first factor is divisible by 3n . The second factor n−1 is divisible by 3, since 23 ≡ −1 mod 3. This proves the statement. Solution 3.4. If n = 1 we have three coins. Place in each plate of the weighing scale one coin, if the plates balance, the false coin is the third coin, which we did not place. If not, the plate that lifts is the one that has the false coin. 200 Chapter 10. Solutions to Exercises and Problems Suppose that it is true for 3n coins. Consider 3n+1 coins. Divide the coins in three groups with 3n coins in each. Put one of the groups in one plate of the balance and another group in the other plate. If the plates balance, the false coin is in the third group. If not, the false coin is in the group of the plate that raises. In both cases the problem will reduce to finding the false coin in a group with 3n coins, but this can be done in n weighings and with the weighing already done we have n + 1 weighings. Solution 3.5. First note that for n ≥ 1, it follows that 2n+3 ± 1 = 2n (23 − 1) + 2n ± 1 = 7 · 2n + (2n ± 1), then 7 | 2n ± 1 if and only if 7 | 2n+3 ± 1. This equivalence shows an inductive step of the form 7 | 2n ± 1 ⇒ 7 | 2n+3 ± 1, and of the form 7 ∤ 2n ± 1 ⇒ 7 ∤ 2n+3 ± 1. Now let us see the induction basis. (i) For n = 1, 2 it follows that 7 does not divide 2n − 1. For n = 3, it follows that 7 divides 23 − 1 = 7. Therefore, the integers n we are looking for are the multiples of 3. (ii) For n = 1, 2, 3, it follows that 7 does not divide 2n + 1 (which are 3, 5 and 9). Thus 7 does not divide 2n + 1 with n ≥ 1. Solution 3.6. Observe that a1 + a2 = 22 . Solving with respect to a2 , we show that a2 = 4 − a1 = 4 − 3 = 3. This suggests to us that an = 2n − 1. Suppose that aj = 2j−1, for all j < n. It follows that a1 +a2 +· · ·+an = 1+3+· · ·+(2n−3)+an = n2 . Since 1 + 3 + · · · + (2n − 3) = (n − 1)2 , it follows that (n − 1)2 + an = n2 . From this we conclude that an = n2 − (n − 1)2 = n2 − (n2 − 2n + 1) = 2n − 1. Solution 3.7. For n = 1, the identity is valid, the left-hand side is ⌊ 12 ⌋ = 0 and the right-hand side is ⌊ 12 ⌋⌊ 22 ⌋ = 0. For n = 2, the identity is also valid; on the one hand we have ⌊ 12 ⌋ + ⌊ 22 ⌋ = 1 and on the other hand we have ⌊ 22 ⌋⌊ 23 ⌋ = 1 · 1 = 1. Now we suppose valid the identity for n and we prove that it is true for n + 2. The left-hand side is     n n + 1 n + 2 2 1 + + + ···+ + 2 2 2 2 2       n n + 1 n+1 n+2 + + = 2 2 2 2 n n + 1 n + 1 n  = +1 + + 2 2 2 2 n n + 1 n + 1 n +1 + + = 2 2 2 2    $ n  % n + 1 n+2 n+3 = +1 +1 = , 2 2 2 2 which is what we expected from the right-hand side. 201 10.3 Solutions of Chapter 3 √ √ 1· a Solution 3.8. Observe that a1 = 2 2 . Solving this equation gives as a result √ 2 = a2 , from where a2 = 22 . This suggests an = n2 . Suppose that aj = j 2 , for all j < n. We then have √ (n − 1) an √ √ (n − 1)n √ = , a1 + a2 + · · · + an−1 = 1 + 2 + · · · + n − 1 = 2 2 √ and therefore n = an , that is, an = n2 . Solution 3.9. (i) For n = 1, we have x2 − 2x + 1 = (x − 1)2 . For n = 2 we get x3 − 3x + 2 = (x − 1)2 (x + 2), since (x − 1)2 (x + 2) = (x2 − 2x + 1)(x + 2) = x3 − 3x + 2. Suppose that xn+1 − (n + 1)x + n = (x − 1)2 (xn−1 + 2xn−2 + · · · + n), then (x−1)2 (xn + 2xn−1 + · · · + (n − 1)x2 + nx + (n + 1))   = (x − 1)2 x(xn−1 + 2xn−2 + · · · + (n − 1)x + n) + (n + 1)   = x (x − 1)2 (xn−1 + 2xn−2 + · · · + (n − 1)x + n + (x − 1)2 (n + 1)   = x xn+1 − (n + 1)x + n + (x − 1)2 (n + 1) = xn+2 − (n + 1)x2 + nx + (n + 1)x2 − 2x(n + 1) + (n + 1) = xn+2 − (n + 2)x + (n + 1). Therefore we have proved that xn+1 − (n + 1)x + n = (x − 1)2 (xn−1 + 2xn−2 + · · · + n). Now, if x > 0 the right-hand side of the above equality is greater tan or equal to zero, then xn+1 − (n + 1)x + n ≥ 0. (ii) If x = ab , with a = x1 +x2 +···+xn+1 n+1 and b = x1 +x2 +···+xn , n we have that an+1 a − (n + 1) + n ≥ 0 bn+1 b  x1 +···+xn+1  an+1 n+1 ≥ (n + 1) −n x1 +···+xn bn+1 n then n(x1 + · · · + xn+1 ) an+1 ≥ −n bn+1 x1 + · · · + xn an+1 n(x1 + · · · + xn ) nxn+1 ≥ + −n n+1 b x1 + · · · + xn x1 + · · · + xn nxn+1 an+1 ≥ , n+1 b x1 + · · · + xn an+1 ≥ xn+1 which is what we wanted to prove. bn+1 x1 +···+xn n = xn+1 bn , 202 Chapter 10. Solutions to Exercises and Problems (iii) Suppose that x1 +x2 +···+xn n x1 + · · · + xn+1 n+1 ≥ √ n x1 x2 · · · xn , then n+1 ≥ xn+1 x1 + · · · + xn n n ≥ xn+1 (x1 · · · xn ), where the first inequality is due to the (ii) part and the second by the induc√ n+1 tive step. Therefore, x1 +···+x ≥ n+1 x1 · · · xn+1 , which is what we wanted n+1 to prove. Solution 3.10.*For n = 1 the result follows. Suppose that the result is true for  n−1 n n − 1. Then different values and the same holds for e a has at least 2 $* % i=1 i i n−1 i=1 ei ai − an . The greater value of these sums is a1 + a2 + · · · + an−1 − an , but it is clear that a1 + a2 + · · · + an−1 − an < a1 + a2 + · · · + an−2 − an−1 + an < a1 + · · · + an−3 − an−2 + an−1 + an .. .. . . < a1 − a2 + a3 + · · · + an < −a1 + a2 + a3 + · · · + an < a1 + a2 + a3 + · · · + an , then we have n additional different sums. Since proved. n 2 +n = n+1 2 , the result is Solution 3.11. For n = 1 the statement is true, because in a sequence of three numbers (chosen from the set of numbers {0, 1}) there are two that are equal. Suppose the statement to be valid for 2n + 1 numbers and consider a sequence with 2n + 3 elements. If for any i we have that ai = ai+1 , then by the induction hypothesis, a1 , . . ., ai−1 , ai+2 ,. . ., a2n+3 has a subsequence even-balanced with 2n elements. To this subsequence add two equal elements and you will have a subsequence evenbalanced with 2(n + 1) elements. On the contrary, we have that ai = ai+1 for all i ∈ {1, 2, . . . , n + 1}, therefore ai = ai+2 for all i. Since the sequence has an odd number of elements, the initial and last numbers have to be the same. Then, if we take out the element that occupies the central place of the sequence, we will have a subsequence with 2n elements that clearly is even-balanced, since the sequence is symmetric with respect to the central element. Solution 3.12. Prove, by induction, that ak = k. For k = 1 the statement is true, since a31 = a21 and a1 > 0, hence a1 = 1. Suppose the result is true for 1, 2, . . . , k and prove that it is valid for k + 1. 10.3 Solutions of Chapter 3 203 We then have that the equality a31 + a32 + · · · + a3n = (a1 + a2 + · · · + an )2 , is satisfied by n = 1, 2, . . . k, k + 1. By the induction hypothesis, we have that a1 = 1, a2 = 2, . . . , ak = k and the condition for n = k + 1 is 13 + 23 + · · · + k 3 + a3k+1 = (1 + 2 + · · · + k + ak+1 )2 . Expanding the right-hand side of the equality, we obtain (13 + 23 + · · · + k 3 ) + a3k+1 = (1 + 2 + · · · + k)2 + 2(1 + 2 + · · · + k)ak+1 + a2k+1 . The first terms on both sides of the equality can be canceled out, and from there we get a3k+1 = 2(1 + 2 + · · · + k)ak+1 + a2k+1 ; but this is equivalent to a3k+1 = ak+1 + a2k+1 . Dividing by ak+1 = 0, we obtain a2k+1 − ak+1 − k(k + 1) = 0. 2 k(k+1) 2 This is a quadratic equation in ak+1 with roots −k and k+1. Since ak+1 is positive, we end up with ak+1 = k + 1, which ends the proof. Solution 3.13. We will do the proof using induction. For n = 1, since a1 ≥ 1, we have that a31 ≥ a21 , moreover, a31 = a21 if and only if a1 = 1. Suppose the inequality is true for n = k and consider k + 1 positive integers such that a1 < a2 < · · · < ak < ak+1 . Then, we have ak+1 ≥ ak + 1, therefore (ak+1 − 1)ak+1 ak (ak + 1) ≥ = 1 + 2 + · · · + ak . 2 2 Note that the sum 1 + 2 + · · · + ak contains all positive integers less than or equal k+1 ≥ to ak , then it is greater than or equal to a1 + a2 + · · · + ak , hence (ak+1 −1)a 2 2 a1 + a2 + · · · + ak , which multiplied by 2ak+1 , is equivalent to (ak+1 − ak+1 )ak+1 ≥ 2(a1 + a2 + · · · + ak )ak+1 , that is, a3k+1 ≥ 2(a1 + a2 + · · · + ak )ak+1 + a2k+1 . On the other hand, the induction hypothesis implies that a31 + a32 + · · · + a3k ≥ (a1 + a2 + · · · + ak )2 . Adding the last two inequalities, we obtain a31 + a32 + · · · + a3k + a3k+1 ≥ (a1 + a2 + · · · + ak + ak+1 )2 , from where the inequality is true for n = k + 1. It is not difficult to deduce, from the previous proof, that the equality follows if and only if ak+1 = ak = 1 and a31 + a32 + · · · + a3k = (a1 + a2 + · · · + ak )2 . In the last identity, the induction hypothesis implies that if the equality holds, then a1 = 1, a2 = 2, . . . , ak = k. Therefore, ak+1 = ak + 1 implies that ak+1 = k + 1. That is, the sequence is ai = i, for i = 1, 2, . . . , k + 1. Reciprocally, we have equality for a1 = 1, a2 = 2, . . . , ak = k, ak+1 = k + 1 thanks to the classical formula. 204 Chapter 10. Solutions to Exercises and Problems Solution 3.14. (i) The original inequality for n = 1 can be verified directly. For n ≥ 2, it is enough to prove by induction that 1+ 1 2 1+ 1 22 ··· 1 + 1 2n 5 2 1− 1− 2n+1 ≤ 1 2n . For n = 2 we have the equality. The inductive step is reduced to verifying that 5 2 1− 1 2n 1 1+ 2n+1 ≤ 5 2 1 . To see this, observe that the previous inequality is equivalent to 1− 1− 1 2n 1+ 1 2n+1 ≤1− 1 2n+1 1 1 1 1 + n+1 − 2n+1 ≤ 1 − n+1 2n 2 2 2 2 1 1 ≤ n + 2n+1 2n+1 2 2 1 1 1 ≤ n 1 + n+1 n 2 2 2 , which is clearly true. (ii) As in the previous part, it will be easier to see that 1+ 1 13 1+ 1 23 ··· 1 + 1 n3 <3− 1 . n For the inductive step, it is necessary to verify that 3− 1 n 1+ 1 (n + 1)3 ≤3− 1 . n+1 Performing all the operations and simplifying leads to 3− 1 3 1 1 + − ≤3− n (n + 1)3 n(n + 1)3 n+1 3 1 1 1 ≤ + 1+ (n + 1)3 n+1 n (n + 1)3 1 3 1 1 +1 ≤ 1+ n + 1 (n + 1)2 n (n + 1)3 n(n2 + 2n + 4) ≤ n3 + 3n2 + 3n + 2 0 ≤ n2 − n + 2, and the last inequality follows. 205 10.3 Solutions of Chapter 3 Solution 3.15. Define P (n) as the statement we want to prove. If n = 1 the equality follows and then P (1) is true. To see that P (2) is valid, consider the following set of equivalences: 1 2 1 + ≥ √ 1 + a1 1 + a2 1 + a1 a2 √ ⇔ (2 + a1 + a2 )(1 + a1 a2 ) ≥ 2(1 + a1 )(1 + a2 ) √ √ ⇔ 2 a1 a2 + (a1 + a2 ) a1 a2 ≥ a1 + a2 + 2a1 a2 √ √ √ ⇔ 2 a1 a2 (1 − a1 a2 ) + (a1 + a2 )( a1 a2 − 1) ≥ 0 √ √ ⇔ ( a1 a2 − 1)(a1 − 2 a1 a2 + a2 ) ≥ 0. √ √ √ But the last inequality is true, since a1 a2 ≥ 1 and ( a1 − a2 )2 ≥ 0. We now see that P (2n ) ⇒ P (2n+1 ). n+1 2# i=1 n n+1 2 2# # 1 1 1 = + 1 + ai 1 + a 1 + ai i i=1 i=2n +1 2n 2n + √ √ n 2 1+ a1 · · · a2n 1+ a2n +1 · · · a2n+1   2 n ≥2 √ √ 1 + 2n a1 · · · a2n 2n a2n +1 · · · a2n+1 ≥ = 2n 2n+1 . √ 1 + 2n+1 a1 · · · a2n+1 For the first inequality we used P (2n ) twice and for the second we applied P (2) √ √ to the numbers 2n a1 · · · a2n and 2n a2n +1 · · · a2n+1 . that Now, let us see that P (n + 1) ⇒ P (n). √ If we apply P (n + 1) to the numbers a1 , . . . , an , an+1 = n a1 · · · an , we have 1 1 1 + ··· + + ≥ 1 + a1 1 + an 1 + an+1 1+ n+1 √ a1 · · · an n a1 · · · an n+1  = n+1 1+ 1 1+ (a1 · · · an ) n n+1 . = 1 + an+1 Hence n+1 1 1 n n + ··· + ≥ = . √ n 1 + a1 1 + an 1 + an+1 1 + a1 · · · an Solution 3.16. To prove (i) and (ii) apply the binomial Theorem 3.2.3 to (1 + 1)n and (1 + 2)n , respectively. 206 Chapter 10. Solutions to Exercises and Problems Solution 3.17. To prove (i) just apply formula (3.5) and for (ii) take r = 1 in (i). Solution 3.18. (i) In equation (1 + x)n (1 + x)n =(1 + x)2n , the coefficient of the term xn on the right-hand side of the equation is 2n n . If we expand the left-hand side of the equation we find that + , n n n j n n + x + ···+ x + ··· + x 0 1 j n + , n n n k n n + x + ···+ x + ···+ × x . 0 1 k n     Therefore the term xn will appear when we multiply nj xj and nk xk , with j +k = n, and then # n n 2n = . j k n 0≤j,k≤n j+k=n But # 0≤j,k≤n j+k=n since n j =  n n−j n j n k n # n = j j=0 n n−j n # n = j j=0 2 ,  . (ii) In the same way as in (i), it is enough to compare the coefficient of xr on both sides of the identity (1 + x)n (1 + x)m = (1 + x)n+m which turns out to be # 0≤j,k≤r j+k=r n k m j = n+m . r Then, r # n m n+m = . k r−k r k=0   m+1 on the left-hand side of the equality and using (iii) Changing m 0 by 0 Pascal’s formula (3.6), we have that m+1 m+1 m+2 m+n + + + ··· + 0 1 2 n m+2 m+2 m+n = + + ···+ 1 2 n 207 10.3 Solutions of Chapter 3 m+3 m+3 m+n + + ···+ 2 3 n .. . = = m+n m+n + n−1 n m+n+1 . n = =   m+1 (iv) Changing m m by m+1 on the left-hand side of the equality and using Pascal’s formula (3.6), we obtain m+1 m+1 m+2 n + + + ···+ m+1 m m m m+2 m+2 = + + ···+ m+1 m m+3 m+3 = + + ···+ m+1 m .. = . n n + m+1 m = = n m n m n+1 . m+1 Solution 3.19. (i) Using part (ii) of Exercise 3.17, it follows that j n j =n n−1 , j−1 then n # j=1 j n j = n # n j=1 =n n−1 # j=0 n # n−1 j−1 j=1 n−1 j−1 =n n−1 j = n · 2n−1 . (ii) By part (i) of Exercise 3.17, it follows that from where n+1 j+1 = n+1 n , j+1 j 1 n j+1 j = n+1 1 n+1 j+1 208 Chapter 10. Solutions to Exercises and Problems hence, n # j=0 n 1 j+1 j = n # j=0 = 1 n+1 n+1 j+1 n 1 # n+1 n + 1 j=0 j + 1 n+1 1 # n+1 j n + 1 j=1 ⎡⎛ ⎞ ⎤ n+1 # n+1 ⎠ n+1 ⎦ 1 ⎣⎝ = − j 0 n+1 j=0 = = 1 [2n+1 − 1]. n+1 Solution 3.20. (i) First, using repeatedly Pascal’s formula (3.6) and part (ii) of Exercise 3.17, it follows that + , n−1 n−1 1 n 1 1 n 1 n−1 = + + = j j j j n j j j j−1 + , 1 n−2 n−2 1 n = + + j n j j j−1 n−1 1 1 n 1 n−2 + + , = j n−1 n j j j and carrying on in this way, we obtain 1 n j j = n−1 1 n 1 1 j + + ···+ . n j n−1 j j j Therefore n # j=1 n j j j+1 1 (−1) = n # j+1 (−1) j=1 -n−j # i=0 n−i 1 n−i j . . If we change the sum’s order, our previous identity changes to ⎡ ⎤ n−1 n−i # # n−i ⎦ ⎣ (−1)j+1 1 . n − i j i=0 j=1 Set k = n − i, then the right-hand side of the identity becomes ⎤ ⎡ n # k n k n # # # 1 k k ⎦ #1 1⎣ (−1)j+1 . (−1)j+1 = = j k j k j=1 k j=1 k=1 k=1 k=1 10.3 Solutions of Chapter 3 209 (ii) Observe that the left-hand side of the equality is 1 n (−1)n+1 n 1 n − + ···+ 1·2 1 2·3 2 n(n + 1) n 1 1 n n n 1 1 1 = 1− − − − + · · · + (−1)n+1 2 2 3 n n+1 1 2 n , + + 1 n 1 n n 1 n 1 n + · · · + (−1)n+1 − − + ··· = − 2 2 n n 2 1 3 2 1 , (−1)n+1 n + . (10.4) n+1 n Let U be equal to the terms inside the first square bracket. By the previous statement we have that U = 1 + 12 + · · · + n1 , and for the terms inside the second n n+1 1 1 square bracket we use j+1 j = n+1 j+1 . Then the above identity (10.4) takes the form 1 n 1 n (−1)n+1 n − + ···+ 1·2 1 2·3 2 n(n + 1) n , + 1 n+1 n+1 n+1 n + 1 =U− − + · · · + (−1) n+1 n+1 2 3 + n+1 n+1 n+1 1 − =U− + + ··· n+1 0 1 2 , n+1 − (1 − (n + 1)) +(−1)n+1 n+1 n 1 [n] = U − =U− n+1 n+1 n+1−1 1 1 =U− = + ··· + . n+1 2 n+1 (iii) Call T the left-hand side of the equality, then T = 1 n 1 n (−1)n n − + · · · + . 12 0 22 1 (n + 1)2 n Multiplying by n + 1, we have n+1 n n+1 n n+1 n − + · · · + (−1)n . 0 1 12 22 (n + 1)2 n n n+1 Using the fact that n+1 j+1 j = j+1 , our identity becomes (n + 1)T = 1 n+1 1 n+1 1 n+1 − + · · · + (−1)n 1 2 n+1 n+1 1 2 1 1 . = 1 + + ···+ 2 n+1   1 1 Therefore, the sum we are looking for is T = n+1 1 + 12 + · · · + n+1 . (n + 1)T = 210 Chapter 10. Solutions to Exercises and Problems Solution 3.21. (i) Using the equality n # (−1)j j j=1 n j = n # n m (−1)j n j=1 =n n−1 j−1 n−1 #   n n−1 m m−1 = =n leads to the set of identities n # (−1)j j=1 n−1 j−1 n−1 j (−1)j+1 j=0 and using Example 3.2.4, we reach the result n n−1 # n−1 j (−1)j+1 j=0 (ii) Again, using the equality n # j=1 (−1)j j 2 n j = n # n m (−1)j n · j j=1 n # = = n · 0 = 0.   , it follows that n n−1 m m−1 n−1 j−1 =n n # (−1)j j j=1 n−1 j−1 n # n−1 n−1 =n (−1)j (−1) (j − 1) +n , j−1 j − 1 j=1 j=1 =n n # j=2 j (−1)j (j − 1) n # n−1 n−1 , (−1)j +n j−1 j−1 j=1 since in the above identity the first term of the first sum is zero. Finally, by Example 3.2.4, and using the previous part of this exercise, we find n n n # # n−1 n−1 (−1)j (−1)j (j − 1) +n j−1 j−1 j=1 j=2 = n · 0 + n · 0 = 0. Solution 3.22. Consider the following   array, where in the mth row, for m ≥ 0, we have the binomial coefficients m j modulo 2. On the left column is shown the number of the row written in base 2 and in the right column the number of odd binomial coefficients written in base 2. 0 1 2 3 = 2 + 20 4 = 22 5 = 22 + 20 6 = 22 + 21 7 = 22 + 21 + 20 1 1 1 1 1 1 1 1 1 0 1 0 1 0 1 1 0 0 1 1 1 0 0 1 1 0 1 1 1 1 1 0 1 1 1 1 20 21 21 22 21 22 22 23 211 10.3 Solutions of Chapter 3 This array suggests to us that the number of odd binomial coefficients will be 2k , with k the number of non-zero digits when we write n in base 2. Note that if n = 2α1 + 2α2 + · · · + 2αr , with α1 > α2 > · · · > αr ≥ 0, then α1 (1 + x)n = (1 + x)2 α1 αr · · · (1 + x)2 αr ≡ (1 + x2 ) · · · (1 + x2 ) mod 2. The previous identity follows because if we expand the binomial of the form (1 + x)2a the first and the last binomial coefficients are 1 and the rest are even. α1 αr It is clear that if we expand (1 + x2 ) · · · (1 + x2 ), there are 2r terms.   *n  2 Solution 3.23. The proof is based on the identity j=0 pj = 2p p , as seen in Exercise 3.18 part (i).   Since pj is divisible by p for all j = 1, 2, . . . , p−1, each term of the sum is divisible 2 by p2 , with 2p exception of the first and the last, which are equal to 1. Therefore, p divides p − 2. To finish the proof, observe that 2p − 1 1 −1= 2 p−1 2p −2 . p Solution 3.24. For p = 2, we have that 21 = 13 + 13 and for p = 3, we get 32 = 13 + 23 . Let us see now that there is no prime number p > 3, for which there exist a, b and n such that a3 + b3 = pn . Suppose that we can find such numbers and that n is the smallest integer number that fulfills the conditions of the problem. Since p ≥ 5, one of the numbers a or b is greater than 1, then a3 + b3 ≥ 5. Since a3 + b3 = (a + b)(a2 − ab + b2 ) and a2 − ab + b2 = (a − b)2 + ab ≥ 2, then p must divide a + b and a2 − ab + b2 . But then, p divides (a + b)2 − (a2 − ab + b2 ) = 3ab. Since p ≥ 5, p has to divide a or b, but since p|a + b, it follows that p divides a and it also divides b. Then, a3 + b3 ≥ 2p3 , hence n > 3. Since pn−3 = a3 + b 3 pn = = p3 p3 a p 3 + b p 3 , it follows that n − 3 also satisfies the condition; then n is not a minimum. Solution 3.25. Consider the equation x2 + y 2 + z 2 = 2xyz. The left-hand side of the equation has exactly one even term or all three terms are even. If exactly one 212 Chapter 10. Solutions to Exercises and Problems term is even, then the right-hand side of the equation is divisible by 4 and the left-hand side is divisible only by 2, so we have a contradiction. Then all terms are even, that is, x = 2x1 , y = 2y1 , z = 2z1 and x21 + y12 + z12 = 4x1 y1 z1 . (10.5) From equation (10.5), following the same reasoning leads to x1 = 2x2 , y1 = 2y2 , z1 = 2z2 and (10.6) x22 + y22 + z22 = 8x2 y2 z2 . Again from equation (10.6), it follows that x2 , y2 , z2 are even, and so on and so forth. Then x = 2x1 = 22 x2 = 23 x3 = · · · = 2n xn = · · · , y = 2y1 = 22 y2 = 23 y3 = · · · = 2n yn = · · · , z = 2z1 = 22 z2 = 23 z3 = · · · = 2n zn = · · · , that is, if (x, y, z) is a solution, then x, y and z are divisible by 2n for all n. This is impossible, unless x = y = z = 0. Solution 3.26. The solutions are a = b = 1 or a and b consecutive square numbers. We can write the divisibility condition as k(ab + a + b) = a2 + b2 + 1, (10.7) for some integer number k. If k = 1, then the equation (10.7), is equivalent to (a − b)2 + (a − 1)2 + (b − 1)2 = 0, from where a = b = 1. If k = 2, then equation (10.7) can be written as 4a = (b − a − 1)2 , from where we deduce that a has to be a square number, that is, a = d2 . Then b − d2 − 1 = ±2d, that is, b = ±(d ± 1)2 and, a and b are consecutive square numbers. Suppose now that k ≥ 3, and let (a, b) be a solution with a the minimum and a ≤ b. Write equation (10.7) as a quadratic equation in b, b2 − k(a + 1)b + (a2 − ka + 1) = 0. Since root b is an integer, the other root r satisfies b + r = k(a + 1) and it is also an integer. Since equation (10.7) has to be true if we substitute b by r, note that k(ar + a + r) = a2 + r2 + 1 > 0 implies ar + a + r > 0, and then we can conclude that r > 0. And since a ≤ b and the product of the roots a2 − ka + 1 is less than a2 , we have r < a. But (r, a) is also a solution of (10.7), which contradicts a being a minimum. Solution 3.27. First we prove that we cannot find an equilateral triangle such that the vertices are points with integer coordinates. Suppose we can find such a triangle. Let a be the length of the sides of the triangle such that the vertices 213 10.3 Solutions of Chapter 3 √ are points with integer coordinates. The area of the triangle is a2 43 which is an irrational number, since a2 is an integer number. On the other hand, the area of any polygon whose vertices are points with integer coordinates is a rational number26 . The vertices of a regular hexagon P1 P2 P3 P4 P5 P6 cannot be points with integer coordinates, since P1 P3 P5 would be an equilateral triangle whose vertices have integer coordinates. Let n = 3, 4, 6. Suppose that P1 P2 P3 . . . Pn is a regular n-gon with vertices having integer coordinates. Through the points P1 , P2 , . . . , Pn draw the parallels −−−→ −−−→ −−−→ to P2 P3 , P3 P4 , . . . , P1 P2 , respectively, as shown in the figure. P1 P5 P2 P4 P3 The points of intersection of the parallels are also points with integer coordinates and form a regular n-gon inside the first one. With the new n-gon we can proceed in the same form. This process can continue to generate an infinite number of n-gons. The square of the length of the sides of these polygons are integers that decrease in each step, but this is impossible. Solution 3.28. Error. The given arguments assume that the set has at least 3 elements, and we use that a1 , an , an+1 are different. We can say that the statement P1 ⇒ P2 is not valid. Solution 3.29. Error. Statement P(n) is: for n coins among which one is false, it is enough to weigh 4 times to identify the false coin. When we take out one coin there are two cases: (a) the coin that we took out is genuine, (b) the coin we took out is false. In the first case, the inductive step works, but in the second case it does not, because among the coins that remain we do not have a false coin. Solution 3.30. Error. Statement P(0) implies P(1) is false. 26 See [13]. 214 Chapter 10. Solutions to Exercises and Problems 10.4 Solutions to exercises of Chapter 4  2 2  Solution 4.1. The equation can be written as m − 23 − 54 = n + 12 − 45 or  2  2 m − 32 − n + 12 = 0, that is, (m + n − 1)(m − n − 2) = 0. Since m and n are positive integers, then m + n − 1 > 0, therefore m and n are solutions of the equation if and only if m − n − 2 = 0, that is, the solutions are (m, n) = (a + 2, a), where a is any positive integer. Solution 4.2. Since P (−1) = a − b + c, P (0) = c and P (1) = a + b + c are integers, we have that 2a, 2b y c are integers. For n = 2m, with m an integer number, we have that P (n) = P (2m) = a(4m2 )+b(2m)+c = (2m2 )(2a)+m(2b)+c is an integer, and for n = 2m+1, with m an integer number, we have that P (n) = P (2m+ 1) = a(2m+ 1)2 + b(2m+ 1)+ c = (2m2 + 2m)(2a) + (2a) + m(2b) + (a + b + c) is also an integer. Solution 4.3. If pq is a solution of ax2 + bx + c = 0, it follows, multiplying by q 2 , that ap2 + bpq + cq 2 = 0. Then p | cq 2 and q | ap2 , but since (p, q) = 1, it follows that p | c and q | a. Solution 4.4. Note that cx2 + bx + a = x2 (c + b x1 + a x12 ), then the roots are the inverse of the roots of ax2 + bx + c, therefore (α + β)(α′ + β ′ ) = (α + β) 1 1 + α β ≥ 4. Another way, by Vieta’s formulas (4.1), α + β = − ab and α′ + β ′ = − bc , then b2 (α + β)(α′ + β ′ ) = ac . But b2 − 4ac ≥ 0, as they are real roots and since a = αβ, ′ ′ c = α β are positive. Solution 4.5. Let x1 , x2 be the zeros of P (x). Then, by Vieta’s formulas (4.1), we have x1 + x2 = a − 2 and x1 x2 = −a − 1. Substitute in the identity x21 + x22 = (x1 + x2 )2 − 2x1 x2 the values of the sum and the product of x1 and x2 to obtain (a − 2)2 + 2(a + 1) = a2 − 2a + 6 = (a − 1)2 + 5 ≥ 5, with equality for a = 1. Then, a = 1 is the only number. Solution 4.6. Observe that p1 + 1q + qr + pr + pq = 3 and pqr = −1. Then, 1 r 1 p = qr+pr+pq . By Vieta’s formula (4.2), pqr + q1 + r1 = −3. Solution 4.7. Since p, q and r are roots of the given cubic equation, by Vieta’s formula (4.2), it follows that p + q + r = −b and pq + qr + rp = c. Since (p+q +r)2 = p2 +q 2 +r2 +2(pq +qr+rp), we obtain (−b)2 = p2 +q 2 +r2 +2c, and rearranging terms b2 − 2c = p2 + q 2 + r2 . Therefore, a quadratic equation with the desired roots is (x − (−b))(x − (b2 − 2c)) = x2 + (b − b2 + 2c)x + (2bc − b3 ) = 0. 10.4 Solutions of Chapter 4 215   2 x + m 2−3 = 0, Solution 4.8. Since x1 and x2 are solutions to equation x2 − 2m−1 2 2 it follows that x1 + x2 =2m−1 and x1 x2 = m 2−3 . We need x1 = x2 − 12 , then it 2 2m−1 1 1 is necessary that x2 − 2 + x2 = 2x2 − 2 = 2 , that is, x2 = m 2 . Similarly, we   x2 m2 3 m2 −3 1 2 want to have x2 − 2 x2 = 2 , then x2 − 2 = 2 − 2 . Hence, substituting m2 m m2 3 x2 = m 2 , we get 4 − 4 = 2 − 2 , that is (m + 3)(m − 2) = 0. Then, it follows that m = −3 or m = 2. Since m has to be positive, then m = 2. Therefore, x1 = 12 and x2 = 1. Solution 4.9. Since P (x2 + 1) = x4 + 4x2 , the polynomial P (x) has to be of degree 2 and it is monic, that is, P (x) = x2 + bx + c, then (x2 + 1)2 + b(x2 + 1) + c = x4 + 4x2 . Expand the equation to obtain x4 + 2x2 + 1 + bx2 + b + c = x4 + 4x2 , hence 2 + b = 4 and 1 + b + c = 0, then b = 2, c = −3. Substitution leads to P (x) = x2 + 2x − 3, therefore P (x2 − 1) = (x2 − 1)2 + 2(x2 − 1) − 3 = x4 − 2x2 + 1 + 2x2 − 2 − 3 = x4 − 4. Solution 4.10. Note that a3 + b3 = c3 + d3 if and only if (a + b)(a2 − ab + b2 ) = (c + d)(c2 − cd + d2 ), but since a + b = c + d = 0, we can cancel these factors and obtain a2 − ab + b2 = c2 − cd + d2 . On the other hand, from a + b = c + d we get, squaring this equality, that a2 + 2ab + b2 = c2 + 2cd + d2 . Subtracting these two last identities, it follows that ab = cd. Then, the two quadratic polynomials x2 − (a + b)x + ab and x2 − (c + d)x + cd coincide, in particular their roots are the same. But, by Vieta’s formula (4.1) we know that the roots of the polynomials are {a, b} and {c, d}, respectively. Therefore, {a, b} = {c, d}. Solution has   equal roots if the discriminant is zero, that is,  4.11. The polynomial 4 − 4λ 1 − λ1 = 0, then λ 1 − λ1 = 1. Thus λ = 2 is the only possibility. Solution 4.12. It is not possible. Otherwise, if the three polynomials had two real roots, the discriminants, b2 − 4ac and c2 − 4ab, a2 − 4bc would be positive. Hence, b2 > 4ac, c2 > 4ab, a2 > 4bc, and multiplying the inequalities we would have a2 b2 c2 > 64a2 b2 c2 , which is false. Solution 4.13. The solutions of the equation are given by x= 1 − 2k ± √ (1 − 2k)2 − 4k(k − 2) 1 − 2k ± 1 + 4k = . 2k 2k The number x will be rational if 1 + 4k is a perfect square, that is, k has to be 2 an integer of the form k = n 4−1 , with n a positive integer. Since we want k to be an integer, n2 − 1 has to be divisible by 4, but n2 − 1 = (n + 1)(n − 1) is 2 divisible by 4 if and only if n is odd. Then, for k = n 4−1 , with n odd, the roots of kx2 − (1 − 2k)x + k − 2 = 0 are rational numbers. 216 Chapter 10. Solutions to Exercises and Problems Solution 4.14. If a + b is a root of the polynomial P (x) = x2 + ax + b, then 0 = (a + b)2 + a(a + b) + b = b2 + (3a + 1)b + 2a2 , and so b has to be a root of the polynomial Q(x) = x2 + (3a + 1)x + 2a2 . But Q(x) will have an integer root if its discriminant (3a + 1)2 − 4 · 2a2 = (a + 3)2 − 8 is a perfect square. But two square numbers have difference 8 if and only if they are 1 and 9, then (a + 3)2 = 9, therefore a = −6 or a = 0. If a = −6, then b = 8 or 9 and if a = 0 then b = 0 or −1. Hence, the only possible pairs (a, b) are: (−6, 8), (−6, 9), (0, 0) and (0, −1). Solution 4.15. We want to solve equation x2 − 5x − 1 = n2 , that is, x2 − 5x − (n2 + 1) = 0. The solutions of the equation are given by √ 5 ± 25 + 4n2 + 4 . (10.8) x1,2 = 2 For x to be an integer number it is necessary that 4n2 + 29 = t2 . Then, t2 − 4n2 = (t − 2n)(t + 2n) = 29, that is, t + 2n = ±29 and t − 2n = ±1 or t + 2n = ±1 and t − 2n = ±29. Solving these equations, it follows that 4n = ±28, then n = 7 and t = 15 or n = −7 and t = −15. Substituting n in equation (10.8), we obtain x1 = 10 and x2 = −5. Solution 4.16. We present the solution due to R. Descartes. Using Vieta’s formulas (4.1), we find that if α and β are the polynomial roots, then α + β = −b and αβ = −c2 , then there is a negative root. R S c O b 2 Q T Consider the triangles RQS and RT Q, which are similar, since they share the angle of the vertex R and ∠RT Q = ∠RQS. Then, we have that RT · RS = RQ2 = c2 , therefore, RS · (−RT ) = −c2 . On the other hand, since RT = RS + b, we have RS +(−RT ) = −b. Then, −RT and RS satisfy Vieta’s relations, therefore those numbers are the roots of the equation. Solution 4.17. If P (x) = x does not have real solutions, then P (x) > x, for all x or P (x) < x, for all x. Hence, P (P (x)) > P (x) > x or P (P (x)) < P (x) < x, for all x, therefore it is impossible to have P (P (x)) = x. 10.4 Solutions of Chapter 4 217 Solution 4.18. If P (P (P (x0 ))) = P (x0 ) = 0 for some x0 , then P (P (0)) = P (P (P (x0 ))) = 0. Hence, P (0) is a zero of P (x) and it is an integer because P (x) is a polynomial with integer coefficients. Moreover, P (P (P (P (0)))) = P (P (0)) = 0, then P (0) is also a root of P (P (P (x))). Solution 4.19. Note that ax2 +bx+c > cx is equivalent to P (x) = ax2 +(b−c)x+c > 0; this guarantees that a > 0, c = P (0) > 0 and (b − c)2 − 4ac < 0. On the other hand, to prove that cx2 − bx + a > cx − b is equivalent to proving that cx2 − (b + c)x + a + b > 0, but since c > 0, it is enough that the discriminant be negative; such discriminant is (b + c)2 − 4c(a + b) = (b − c)2 − 4ac, but we have already proved that it is negative. Solution 4.20. Suppose that 20(b − a) is an integer number. By symmetry we can also suppose that b > a, and then 20(b − a) ≥ 1. Since there are no real solutions, the discriminant of the polynomial x2 + 20bx + 10a is negative, therefore 1 1 . Hence 0 < b − a < b < 10 10b2 − a < 0. Then, we have 10b2 < a < b and b < 10 and, then 20(b − a) < 2, but if 20(b − a) is an integer number we have 20(b − a) = 1 1 1 2 1 and then b = a + 20 . Thus, 10b2 − a = 10(a + 20 ) − a = 10a2 + 40 > 0, which is a contradiction. Therefore, 20(b − a) can never be an integer number. Solution 4.21. If P (x) = x2 + bx + c satisfies P (P (P (x0 ))) = P (x0 ) = 0, for some x0 , then P (P (0)) = P (P (P (x0 ))) = 0. Therefore, 0 = P (P (0)) = P (c) = c2 + bc + c = c(c + b + 1) = P (0)P (1). Solution 4.22. Expand the following polynomial and use the relation ab+ac+bc = de + df + ef , to get that (x + a)(x+b)(x + c) − (x − d)(x − e)(x − f ) = x3 + (a + b + c)x2 + (ab + ac + bc)x + abc − x3 + (d + e + f )x2 − (de + df + ef )x + def = N x2 + abc + def. Then, if we let x = d, the above expression becomes (d + a)(d + b)(d + c) = N d 2 + (abc + def ). Then, if N divides abc + def , then N divides (d + a)(d + b)(d + c). Let p be a prime number such that p divides N , then p divides at least one of the factors d + a, d + b or d + c. Then, p ≤ max(d + a, d + b, d + c) < N , that is, p is a factor of N and N is a composite number. 218 Chapter 10. Solutions to Exercises and Problems Solution 4.23. Since P (x) and Q(x) have integer coefficients, we can divide by the main coefficient and assume that P (x) = (x − α)(x − r) and Q(x) = (x − α)(x − s), where α ∈ R\Q, that is, P (x) and Q(x) are monic polynomials with rational coefficients. In a quadratic polynomial, if one root is an irrational number the other root is also irrational, since the sum of both roots has to be a rational number. Then, r, s ∈ R\Q. Note that α + r, αr, α + s and αs are all rational numbers since they are the coefficients of P (x) and Q(x). Then (α + r) − (α + s) = r − s ∈ Q r αr = ∈ Q. αs s Let r − s = pq and rs = m equation, it follows that n . Solving for r from the second m  p p m m · s, from where we get · s − s = , that is, s r= m n n q n − 1 = q . If n − 1 = 0, it follows that p s= which is a contradiction, then m n q m n −1 ∈Q = 1. Thus, r = s. Solution 4.24. Multiplying both equations, it follows that x4 − (b1 + b2 )x3 + (c1 + c2 + b1 b2 )x2 − (b1 c2 + b2 c1 )x + c1 c2 = 0. On the other hand, since b1 , c1 , b2 and c2 are roots, we have by, Vieta’s formulas (4.1.1) and equating the coefficients, that b 1 + b 2 + c1 + c2 = b 1 + b 2 b 1 b 2 + b 1 c1 + b 1 c2 + b 2 c1 + b 2 c2 + c1 c2 = c1 + c2 + b 1 b 2 b 1 b 2 c1 + b 1 b 2 c2 + b 1 c1 c2 + b 2 c1 c2 = b 1 c2 + c1 b 2 b 1 b 2 c1 c2 = c1 c2 . From the first equation, we obtain c1 = −c2 , then from the second equation it follows that c1 c2 = c1 + c2 = 0, from where we get c1 = c2 = 0, which contradicts the fact that c1 and c2 are different numbers. Hence, those polynomials do not exist. Solution 4.25. Since (a − b) + (b − c) + (c − a) = 0, then some of the terms of the sum a − b, b − c or c − a are less than or equal to zero. Suppose, without loss of generality, that a − b ≤ 0. Then, the discriminant of the third equation is (c − a)2 − 4(a − b) ≥ 0, that is, the third equation has a zero that is a real number. Solution 4.26. Let P (x) = x3 − (a + b + c)x2 + (ab + bc + ca)x − abc be the monic polynomial with zeros a, b and c. Let A = ab + bc + ca, B = abc and 219 10.4 Solutions of Chapter 4 Tn = an + bn + cn . Then, T0 = 3, T1 = 0, T2 = (a + b + c)2 − 2(ab + bc + ca) = −2A. The equation P (x) = 0 is equivalent to x3 = −Ax + B, from where xn+3 = −Axn+1 + Bxn . Then, it follows that Tn+3 = −ATn+1 + BTn . Now find T3 , T4 and T5 . Solution 4.27. Using (a+b+c)2 = a2 +b2 +c2 +2(ab+bc+ca), we get ab+bc+ca = 2. Using the identity a3 + b3 + c3 − 3abc = (a + b + c)(a2 + b2 + c2 − ab − bc − ca), we get abc = − 32 . Then, the cubic polynomial with roots a, b and c is P (x) = x3 −3x2 +2x+ 32 ; now, from aP (a)+bP (b)+cP (c) = 0, it follows that a4 +b4 +c4 = 9. Solution 4.28. Each of the equations ax2 +bx+c = 0 and cx2 +bx+a = 0 have two different real solutions, a = 0 and c = 0. Moreover, r is the root of ax2 + bx + c = 0 if and only if r1 is a root of cx2 + bx + a = 0. Therefore, {q1 , q2 } = p11 , p12 . If p1 , q1 , p2 , q2 are in arithmetic progression,   1 1  |p1 − p2 |  , |p1 − p2 | = |q1 − q2 | =  −  = p1 p2 |p1 p2 | from where |p1 p2 | = 1. Using Vieta’s formula (4.1), we have p1 p2 = ac , so |c| = |a| and then a = ±c. If a = c, the two given quadratic equations are equal, and then p1 = q1 , p2 = q2 , which tell us that the difference of the progression is 0. Then, p1 = q1 = p2 = q2 which is a contradiction. Therefore, a = −c or a + c = 0. Solution 4.29. Let Tn = an + bn + cn for each integer number n, then T0 = 3, T1 = 0 and T2 = (a + b + c)2 − 2(ab + bc + ca) = −2(ab + bc + ca). Now, define A = ab + bc + ca and B = abc; then, by Vieta’s formulas (4.2), it follows that a, b and c are the roots of the equation x3 + Ax − B = 0 and T2 = −2A. For n ≥ 0, we substitute a, b and c in xn+3 = −Axn+1 + Bxn and adding we obtain Tn+3 = −ATn+1 + BTn . Then T3 = −AT1 + BT0 = 3B, T4 = −AT2 + BT1 = 2A2 , T5 = −AT3 + BT2 = −5AB. Hence, T5 5 = −AB = T3 3 · T2 2 . Since T3 = T5 , the last equality implies that T2 = 65 . Solution 4.30. Let Q(x) = P (x) − 2, since a, b and c are the roots of Q(x), it is clear that Q(x) = α(x − a)(x − b)(x − c), for some integer number α. If for some integer number d we have P (d) = 3; then, since 1 = P (d) − 2 = Q(d) = α(d − a)(d − b)(d − c), the factors on the right-hand side of the equation have to be −1 or 1, then two of d − a, d − b, d − c are equal, so a, b, c are not different, which is a contradiction. This guarantees that there does not exist an integer number d with P (d) = 3. 220 Chapter 10. Solutions to Exercises and Problems 10.5 Solutions to exercises of Chapter 5 Solution 5.1. For (i) and (ii) apply directly the definition. The proof of (iii) and (iv) is straightforward once you do the operations. To prove (v), after squaring both sides, observe that z w̄ + wz̄ ≤ 2|z w̄|. (vi) Do the operations using |z|2 = z z̄. x−iy 1 = (x+iy)(x−iy) = xx−iy Solution 5.2. Let z = x + iy, then 1z = x+iy 2 +y 2 . Hence,   −y 1 2 2 Im z + z = y + x2 +y2 = 0 is equivalent to y(x + y − 1) = 0, with solutions y = 0 or x2 + y 2 = 1. The solution y = 0 means that the real axis satisfies the original equation and the solution x2 + y 2 = 1 is the unit circle. Second Solution. Write z in its polar form, that is, z = r(cos θ). Then,  θ + i sin 1 1 1 1 1 = (cos (−θ)+i sin (−θ)) = (cos θ−i sin θ). Thus, Im z + ) = (r− z r r z r sin θ = 0, and then r − 1r = 0 or sin θ = 0. Equation r − 1r = 0 implies that r = 1, that is, the complex numbers that satisfy the equation are the ones on the unit circle; meanwhile the complex numbers z that satisfy sin θ = 0 are the complex numbers with argument 0 or π, that is, all the real numbers. Third Solution. Observe that Im z + 1 z 1 1 = z̄ + z z̄ ⇔ (z z̄ − 1)(z − z̄) = 0 ⇔ |z| = 1 or z = z̄ =0 ⇔ z+ ⇔ z is on the unit circle or the real axis. Solution 5.3. Use the fact that z z̄ = |z|2 for every complex number z, and see that 2 |z + w| = (z + w)(z + w) = z z̄ + ww̄ + z w̄ + z̄w, 2 |z − w| = (z − w)(z − w) = z z̄ + ww̄ − z w̄ − z̄w. Then, |z + w| = |z − w| if and only if z w̄+ z̄w = −z w̄− z̄w, that is, 2(z w̄+ z̄w) = 0. Using the fact that Re z = z+z̄ 2 , we obtain 2(z w̄ + z̄w) = 4 Re w̄z = 0. Since 2 2 |w| z z w̄ = |w| w , then w̄z = w z, hence Re w̄z = Re w = 0. Therefore, w is purely iz z imaginary, that is, w = ir for some r ∈ R. Thus, w = −r, which is a real number. Second Solution. A geometric proof is the following. Since w = 0, we can divide the original equation by w, to obtain z  z       + 1 =  − 1 , w w which means that wz is in the perpendicular bisector of the segment that joins 1 and −1, that is, the imaginary axis, then wz is purely imaginary and now we can conclude as above. 221 10.5 Solutions of Chapter 5 Solution 5.4. (i) Use the fact that z z̄ = |z|2 . The left-hand side of the identity is |1 − z̄w|2 − |z − w|2 = (1 − z̄w)(1 − z w̄) − (z − w)(z̄ − w̄) 2 2 2 2 2 2 = 1 + |z| |w| − z̄w − z w̄ − |z| − |w| + z w̄ + wz̄ 2 2 = 1 + |z| |w| − |z| − |w| . In the same way, the right-hand side of the identity is (1 + |zw|)2 − (|z| + |w|)2 = 1 + 2 |zw| + |z|2 |w|2 − |z|2 − |w|2 − 2 |z| |w| 2 2 2 2 = 1 + |z| |w| − |z| − |w| . Therefore, using the above relations we reach the desired conclusions. (ii) Proceed as before: 2 2 |1 + z̄w| − |z + w| = (1 + z̄w)(1 + z w̄) − (z + w)(z̄ + w̄) = 1 + |z|2 |w|2 + z̄w + z w̄ − |z|2 − |w|2 − z w̄ − wz̄ 2 2 2 2 2 2 = 1 + |z| |w| − |z| − |w| = (1 − |z| )(1 − |w| ). Solution 5.5. It is clear that z = 0 if and only if w = 0, thus we can assume that both numbers are non-zero. Also suppose that z = w, then we need to show that z̄w = 1. Simplifying the given identity, we obtain z + zww̄ − w − wz z̄ = 0. The last equality can be written as z − w = zw(z̄ − w̄). Considering the norm on both sides, and using the fact that the norm of a complex number is equal to the norm of its conjugate, it follows that |zw| = 1, that is, zwz̄ w̄ = 1. 2 Multiplying the equality z + zww̄ − w − wz z̄ = 0 by z̄, we obtain |z| + 1 − z̄w − wz̄ |z|2 = 0. Now, from this last equation we have that (|z|2 + 1)(1 − z̄w) = 0, thus z̄w = 1. Solution 5.6. Observe that z12 + z22 + z32 = (z1 + z2 + z3 )2 − 2(z1 z2 + z2 z3 + z3 z1 ). 2 2 2 Then, since % 0, it follows that z1 + z2 + z3 = −2(z1 z2 + z2 z3 + z3 z1 ) = $ z1 + z2 + z3 = −2z1z2 z3 z11 + z12 + z13 = −2z1 z2 z3 (z¯1 + z¯2 + z¯3 ) = 0. Solution 5.7. Since z1 z̄1 = |z1 |2 = 1, then 1 z1 = z̄1 , and also 1 z2 = z̄2 . Then, 1 + 1 z̄1 + z̄2 z1 + z2 = z1 1 z21 = , 1 + z̄1 z̄2 1 + z1 z2 1 + z1 z2 hence z1 +z2 1+z1 z2 is a real number. Solution 5.8. If some of a, b, c is zero, the result is clear. Then, suppose that a, b, c = 0. It is enough to see that (d − a)(d − b)(d − c) = 0, or equivalently, that d3 − (a + b + c)d2 + (ab + bc + ca)d − abc = 0 and, by the hypothesis of the exercise, this is equivalent to showing that (ab + bc + ca)d = abc. 222 Chapter 10. Solutions to Exercises and Problems Since all numbers have the same norm, define r = |a| = |b| = |c| = |d|. On the other hand, it is known that d = a+ b + c, then d¯ = a + b + c = ā + b̄ + c̄. Now, ¯ b̄b dd āa c̄c 1 1 1 1 1 1 2 1 d = a + b + c = r a + b + c , and it follows that d = a + b + c . That is, d(ab + bc + ca) = abc, as we wanted to prove. Solution 5.9. We show first that (i) is equivalent to (ii). If a, b, c are collinear, then arg(b − a) = arg(c − a) or arg(b − both are on $ a) = % arg(c − $ a) + π, because % 1 the same line, then it follows that arg c−a = arg (c − a) · = arg(c − a) − b−a b−a c−a arg(b − a) = 0 or π, which implies that c−a b−a ∈ R. Reciprocally, if t = b−a ∈ R, then c − a = t(b − a) or c = (1 − t)a + tb, which means that c is on the straight line that determines a and b, that is, a, b and c are collinear. Now, we show that (iii) is equivalent to (iv). In order to do this, note that the determinant of the matrix in (iv) is equal to bc̄ − cb̄ − a(c̄ − b̄) + ā(c − b). Then z = cb̄ − cā − ab̄ ∈ R is equivalent to z = z̄, that is, cb̄ − cā − ab̄ = bc̄ − ac̄ − bā, which is the same as bc̄ − cb̄ − a(c̄ − b̄) + ā(c − b) = 0. Finally, observe that c − a b̄ − ā c−a cb̄ − cā − ab̄ + |a|2 = · . = b−a b − a b̄ − ā |b − a|2 2 |a| c−a Since |a−b| 2 is a real number, b−a ∈ R is equivalent to cb̄ − cā − ab̄ ∈ R, that is, (ii) is equivalent to (iii). From the part (ii), it follows that the equation of the line through b and c is Im( z−c z−b ) = 0. Solution 5.10. By Exercise   1  0 =  1  1 5.9, z, i, iz are collinear if and only if  i −i  1−i 1+i z z̄  = z z̄ − z̄ − z, 2 2 iz −iz̄  2  1+i 2   =  . Then, the complex numbers that satisfy which is equivalent to z − 1+i 2 2 the condition of the exercise are the points in the circle with center 1+i 2 and radius  1+i  √2  = 2 2 . Solution 5.11. We will construct first the two squares which have as side length the segment determined by z and w. To do that, consider the points 0 and z − w as two consecutive vertices of a square. Then one possible third vertex is i(z − w), which is the rotation with angle 90◦ of the complex number z − w (or it could be −i(z − w) if we rotate −90◦ ). Finally, the fourth vertex is the sum of the previous two, that is, (z − w) + i(z − w) (or (z − w) − i(z − w) in the other case). Then, to calculate the vertices of the squares that are formed with z and w as consecutive vertices, we should add w to the vertices of the squares found, that is, the vertices 223 10.5 Solutions of Chapter 5 we are looking for are z, w, i(z − w) + w and z + i(z − w) or z, w, −i(z − w) + w and z − i(z − w). In the case where z and w are opposite vertices of the square, again translate one of the vertices to the origin, that is, now the opposite vertices are 0 and z − w. Now, in a square the diagonals intersect each other in a right angle in their midpoints, then we need to consider the two complex numbers orthogonal to z − w and then translate them to the of z − w. That is, we need to consider  midpoint z−w and −i z−w Finally, w to + + z−w the complex numbers i z−w 2 2 2 2 .   adding z−w z−w all the previous vertices, we get the square with vertices w, i + w, z, + 2 2  z−w −i z−w + + w. 2 2 Solution 5.12. Proceed by induction. The basis of induction is n = 2. We know (cos θ + i sin θ)2 = cos2 θ − sin2 θ + i2 cos θ sin θ = cos 2θ + i sin 2θ, where in the last equality we used the identity for the sum of angles for sine and cosine. Suppose then that the identity is true for some n = k, that is, we have (cos θ + i sin θ)k = cos kθ + i sin kθ. Then, (cos θ + i sin θ)k+1 = (cos θ + i sin θ)(cos θ + i sin θ)k = (cos θ + i sin θ)(cos kθ + i sin kθ) = cos θ cos kθ − sin θ sin kθ + i(cos θ sin kθ + cos kθ sin θ) = cos (k + 1)θ + i sin (k + 1)θ. Solution 5.13. Equation z + 1 z = 2 cos θ can be rewritten as z 2 + 1 = 2z cos θ or z 2 − 2z cos θ + 1 = 0, √ then z = cos θ ± cos2 θ − 1 = cos θ ± i sin θ. Using de Moivre’s formula, it follows that z n = cos nθ ± i sin nθ, then 1 1 = cos nθ ∓ i sin nθ. = n z cos nθ ± i sin nθ Adding the last two identities leads to z n + 1 zn = 2 cos nθ. Solution 5.14. Equation |z 2 + z̄ 2 | = 1 is equivalent to |z 2 + z̄ 2 |2 = (z 2 + z̄ 2 )(z̄ 2 + z 2 ) = 1, but using |z| = 1, the last equation is equivalent to (z 4 + 1)2 = z 4 , which can be factored as a difference of squares (z 4 − z 2 + 1)(z 4 + z 2 + 1) = 0. These two quadratic equations can be solved using directly the general formula to obtain 224 Chapter 10. Solutions to Exercises and Problems that the solutions are z 2 = 12 ± we can obtain the values of z. √ 3 2 i and z 2 = − 21 ± √ 3 2 i, and from these equalities Solution 5.15. (i) Let z1 , z2 be the roots of the equation with |z1 | = 1. Since z1 z2 = ac , we have that |z2 | =  ac  |z11 | = 1. Now, since z1 + z2 = − ab and |a| = |b|, it follows that |z1 + z2 |2 = 1. This last equation is equivalent to (z1 + z2 )(z̄1 + z̄2 ) = 1, or (z1 + z2 ) 1 1 + z1 z2 = 1.  2 Hence, (z1 + z2 )2 = z1 z2 , that is, − ab = ac , which can be reduced to b2 = ac. (ii) It follows that b2 = ac and c2 = ab by the previous part. Multiplying both equalities we have that b2 c2 = a2 bc, and then a2 = bc. Therefore, a2 + b2 + c2 = ab + bc + ca. This last identity is equivalent to (a − c)2 = (a − b)(b − c). Taking norms, |c − a|2 = |a − b| · |b − c|. Similarly, we can obtain |b − c|2 = |c − a| · |a − b| and |a − b|2 = |b − c| · |c − a|. Adding the last equalities, we get |b − c|2 + |c − a|2 + |a − b|2 = |b − c| · |c − a| + |c − a| · |a − b| + |a − b| · |b − c|, which is equivalent to (|b − c| − |c − a|)2 + (|c − a| − |a − b|)2 + (|a − b| − |b − c|)2 = 0, and the result follows. Solution 5.16. Let z 5 + az 4 + bz 3 + cz 2 + dz + * e be the polynomial having * as roots 5 the numbers z1 , z2 , z3 , z4 , z5 . By Vieta, a = i=1 zi = 0 and b = i<j zi zj = %2 $ *5 1 *5 − 12 i=1 zi2 = 0. Also, we have z i i=1 2 0= 5 # i=1 * zi = 5 # # 1 1 z1 z2 z3 z4 , = z z1 z2 z3 z4 z5 i=1 i cyclic * then d = 0 and 0 = i<j zi zj = z1 z2 z13 z4 z5 cyclic z1 z2 z3 , hence c = 0. Thus, the polynomial can be reduced to z 5 + e, which has as roots complex numbers that are the vertices of a regular pentagon. Solution 5.17. If a = b, |a − b + c| works; if a = −b, then |a + b + c| works. Suppose that a is different from b and −b. Consider the numbers a + b, a − b, −a + b and −a − b, which are the vertices of a rhombus of side 2. Taking as a center each of these vertices, construct disks of radius 1; these 4 disks cover the circle with center 0 and radius 1. In particular, the point c belongs to one of these disks, then the distance from the center of such a disk to c is less than or equal to 1. Second Solution. The numbers a, b, c are the vertices of a triangle, with orthocenter in a + b + c. If the triangle is acute, then its orthocenter is inside the triangle and then |a + b + c| ≤ 1. If the triangle is obtuse, without loss of generality, we can suppose that the obtuse angle is in a, then −a, b, c are the vertices of an acute triangle with orthocenter −a + b + c, which is inside the triangle, and in this case |−a + b + c| ≤ 1. 225 10.5 Solutions of Chapter 5 Solution 5.18. If at least one of the numbers a, b, c is 0, the result follows. Consider b c a , β = |b| , γ = |c| , then |α| = |β| = |γ| = 1 and if a |bc| + b |ca| + c |ab| = 0 α = |a| is divided by |abc|, then α + β + γ = 0, hence α, β, γ are the vertices of an equilateral triangle. Thus, the angle between two of them is π3 . Using the cosine 2 2 2 2 law, |a − b| = |a| + |b| + |a| |b| ≥ 3 |a| |b|. Similarly, |b − c| ≥ 3 |b| |c| and 2 2 2 2 2 2 2 |c − a| ≥ 3 |c| |a|, then |a − b| |b − c| |c − a| ≥ 33 |a| |b| |c| . Solution 5.19. Since a, b, c have the same norm and |abc| = 1, it is clear that = ab + bc + ca. |a| = |b| = |c| = 1. Then, 1 = a + b + c = a1 + 1b + 1c = ab+bc+ca abc The monic polynomial which has zeros a, b, c is, P (z) = (z − a)(z − b)(z − c) = z 3 − (a + b + c)z 2 + (ab + bc + ca)z − abc = z 3 − z 2 + z − 1 = (z − 1)(z 2 + 1), hence, {a, b, c} = {1, i, −i}. Solution 5.20. Multiplying both sides of the equation x4 + x3 + x2 + x + 1 = 0 by x−1, we get x5 −1 = 0; then to find the roots of x4 +x3 +x2 +x+1 = 0 is equivalent to finding the roots of x5 − 1 = 0, which are different from 1. These  rootsarethe quintic roots of unity, given by w, w2 , w3 , w4 where w = cos 52 π + i sin 52 π . Second Solution. The equation can be solved dividing by x2 , and then making the substitution y = x + x1 , and finally using the general formula to solve a quadratic equation. That is, x2 + x2 + 2 + x+ 1 x2 1 x 1 1 +x+ +1=0 2 x x 1 + x+ −1=0 x 2 + x+ 1 x −1=0 y 2 + y − 1 = 0. The roots of this last equation are y1 = solving the two equations x+ 1 = y1 x √ −1+ 5 , 2 and x+ y2 = √ −1− 5 . 2 It is left to find x 1 = y2 , x which are equivalent to x2 − y1 x + 1 = 0 and x2 − y2 x + 1 = 0. Solving these two equations we find the four roots we are looking for: √ √ √ √ 10 + 2 5 10 + 2 5 −1 + 5 −1 + 5 +i , x2 = −i , x1 = 4 4 4 4 √ √ √ √ −1 − 5 10 − 2 5 10 − 2 5 −1 − 5 x3 = +i , x4 = −i . 4 4 4 4 226 Chapter 10. Solutions to Exercises and Problems As a result of having two different methods for solving equation x4 +x3 +x2 +x+1 = 0, we can conclude that √ √ −1 + 5 2 10 + 2 5 2 π + i sin π = +i . cos 5 5 4 4 Solving for the real and the imaginary part, we get √ √ −1 + 5 10 + 2 5 ◦ ◦ cos 72 = , sin 72 = . 4 4 Solution 5.21. Note that the polynomial x6 + 2x5 + 2x4 + 2x3 + 2x2 + 2x + 1 can be factored as x6 + 2x5 + 2x4 + 2x3 + 2x2 + 2x + 1 = (x + 1)(x5 + x4 + x3 + x2 + x + 1). In this way we have to find the roots of the equation (x + 1)(x5 + x4 + x3 + x2 + x + 1) = 0, where it is clear that x = −1 is one of the roots. The other roots are complex numbers and can be calculated following the trick used in the previous problem. Multiply x − 1 by x5 + x4 + x3 + x2 + x + 1 = 0 to get equation x6 − 1 = 0 and find the roots distinct from 1. These roots are the 6th roots of unity, which can be calculated using de Moivre’s formula (5.3). Solution =  3m+2 for some positive integer m, then the complex number   5.22. If n 2π + i sin is a solution with norm 1. Conversely, if z is a solution with cos 2π 3 3 norm 1, then z̄ = z1 is also a solution. Then, z n + z + 1 = 0 = z n + z n−1 + 1, which implies that z n−2 = 1, z 2 + z + 1 = 0, z 3 = 1 with z = 1, hence n = 3m + 2 for some positive integer m. Second Solution. Let P (z) = z n + z + 1 = 0. If P (w) = 0, with |w| = 1, then w = cos θ+i sin θ, and then, using de Moivre’s formula (5.3), wn = cos nθ+i sin nθ, it follows that 0 = (cos nθ + cos θ + 1) + i(sin nθ + sin θ). Then sin2 nθ = sin2 θ and cos2 nθ = cos2 θ + 2 cos θ + 1, and from this cos θ = − 21 . It follows that w3 = 1 and w2 + w + 1 = 0, therefore wn = w2 , and then n ≡ 2 (mod 3). Conversely, if n ≡ 2 (mod 3), for w = 1, with w a root of unity of order 3, P (w) = 0. Then P (z) = z n + z + 1 = (z 2 + z + 1)Q(z), for some polynomial Q(z) with integer coefficients. Solution 5.23. (i) Let S = 1 + w + w2 + · · · + wn−1 . Multiplying by w both sides of the equality, we get Sw = w + w2 + · · · + wn−1 + wn , and subtracting from S the last equality, we obtain S − Sw = 1 − wn . Therefore n S = 1−w 1−w = 0. 227 10.5 Solutions of Chapter 5 (ii) Let S = 1 + 2w + 3w2 + · · · + nwn−1 . Multiplying by w we get Sw = w + 2w2 + 3w3 + · · · + nwn . Then, S − Sw = 1 + w + w2 + w3 + · · · + wn−1 − nwn = n−nwn (by the first part, 2 1 + w + w2 + w3 + · · · + wn−1 = 0). Therefore, S = −nw 1−w , hence 1 + 2w + 3w + n · · · + nwn−1 = w−1 . Solution 5.24. (i) Observe that if w = 1 is a nth root of unity, then z n − 1 = (z − 1)(z − w) . . . (z − wn−1 ). Hence, zn − 1 = (z − w)(z − w2 ) . . . (z − wn−1 ) = z n−1 + z n−2 + · · · + z + 1. z−1 Now, let z = 1 in the previous equality in order to obtain (1 − w)(1 − w2 ) . . . (1 − wn−1 ) = n. (ii) Consider the polynomial P (z) = (z − w)(z − w2 ) . . . (z − wn−1 ), and since n −1 = z n−1 +z n−2 +· · ·+z+1. z −1 = (z−1)(z−w) . . . (z−wn−1 ), we get P (z) = zz−1 n Note now that P ′ (z) P (z) = 1 z−w + ′ P (1) P (1) . 1 z−w 2 +···+ 1 z−w n−1 , then it is enough to calculate Since P ′ (z) = (n − 1)z n−2 + (n − 2)z n−3 + · · · + 2z + 1, we obtain P ′ (1) = ′ (1) . Now, from P (1) = n we can conclude that PP (1) = n−1 1+2+· · ·+(n−1) = (n−1)n 2 2 . Solution 5.25. (i) Note that (a + bω + cω 2 )(a + bω 2 + cω) = a2 + abω 2 + acω + bcω 2 + abω + b2 + acω 2 + abω + c2 = a2 + b2 + c2 + (ab + bc + ca)ω + (ab + bc + ca)ω 2 = a2 + b2 + c2 + (ab + bc + ca)(ω + ω 2 ) = a2 + b2 + c2 + (ab + bc + ca)(−1). (ii) Substitute the equation obtained in (i). Solution 5.26. The number of common vertices is given by the number of common roots of z 1982 −1 = 0 and z 2973 −1 = 0. Then, by Theorem 5.4.1, it follows that the number we are looking for is the greatest common divisor of (1982, 2973) = 991. Solution 5.27. The roots of x2 + x + 1 are w = ei w3 = 1 and 1 + w + w2 = 0, we obtain 2π 3 and w2 . Using the relations n = 3k ⇒ w6k + w3k + 1 = 1 + 1 + 1 = 3, n = 3k + 1 ⇒ w6k+2 + w3k+1 + 1 = w2 + w + 1 = 0, n = 3k + 2 ⇒ w6k+4 + w3k+2 + 1 = w4 + w2 + 1 = w + w2 + 1 = 0. Therefore the answer is for all n that are not multiples of 3. 228 Chapter 10. Solutions to Exercises and Problems Solution 5.28. Use that x, y are of the form   a b c    c a b ,    b c a in order to see that the product of both numbers is of the same form. Second Solution. Use that x = a3 + b3 + c3 − 3abc = (a + b + c)(a + bω + cω 2 )(a + bω 2 + cω) = P (1)P (ω)P (ω 2 ), for P (z) = cz 2 + bz + a and ω a cubic root of unity. Then, x belongs to S if and only if x = P (1)P (ω)P (ω 2 ), hence, if x = P (1)P (ω)P (ω 2 ) and y = Q(1)Q(ω)Q(ω 2 ) for Q(z) another polynomial of degree 2, we have that xy = R(1)R(ω)R(ω 2 ) with R(z) = P (z)Q(z). Note that R(z) is of degree 4, and after dividing R(z) by z 3 − 1, we get that R(z) = (z 3 − 1)L(z) + R1 (z) with R1 (z) of degree at most 2 and with xy = R1 (1)R1 (ω)R1 (ω 2 ), then xy ∈ S . Solution 5.29. Let w = e2πi/5 , then w5 = 1. Evaluating in the original equation w, w2 , w3 , w4 , we obtain the following four equations: P (1) + wQ(1) + w2 R(1) = 0 P (1) + w2 Q(1) + w4 R(1) = 0 P (1) + w3 Q(1) + wR(1) = 0 P (1) + w4 Q(1) + w3 R(1) = 0. Now, if these equations are multiplied by −w, −w2 , −w3 , −w4 , respectively, we obtain: −wP (1) − w2 Q(1) − w3 R(1) = 0 −w2 P (1) − w4 Q(1) − wR(1) = 0 −w3 P (1) − wQ(1) − w4 R(1) = 0 −w4 P (1) − w3 Q(1) − w2 R(1) = 0. Using 1 + w + w2 + w3 + w4 = 0 and adding the equations, we get 5P (1) = 0, that is, x − 1 divides P (x). Solution 5.30. If z is a root of P (z), then z 2 is also a root. Hence, if |z| > 1 there will be an infinite number of roots, which is impossible since P (z) is a polynomial. If 0 < |z| < 1, the same will happen, and there will be an infinite number of roots. Then, all roots are 0 or they belong to the unit circle. 229 10.6 Solutions of Chapter 6 If P (z) is a constant polynomial, then the constant polynomials P (z) = 1 and P (z) = 0 satisfy the equation. If P (z) = az + b, with a = 0, substituting in the given equation, we get (az + b)(−az + b) = az 2 + b, that is, az 2 + b = −a2 z 2 + b2 . Since a = 0, it follows that a = −1 and b2 = b, then b = 0 or b = 1, and in this case there are two polynomials, P (z) = −z and P (z) = 1 − z. If P (z) = az 2 + bz + c, with a = 0, then P (z)P (−z) = (az 2 + bz + c)(az 2 − bz + c) = a2 z 4 + (2ac − b2 )z 2 + c2 . Comparing with P (z 2 ) = az 4 + bz 2 + c, we obtain a2 = a, 2ac − b2 = b and c2 = c. Since a = 0, it follows that a = 1; for c2 = c we have the solutions c = 0 and c = 1. For each of the values of c, we get two values for b: if c = 0, then b = 0 and b = −1; for c = 1, we get b = 1 and b = −2. Thus, in this case we have 4 polynomials that satisfy the given equation: P (z) = z 2 , P (z) = z 2 −z = −z(1−z), P (z) = z 2 − 2z + 1 = (1 − z)2 and P (z) = z 2 + z + 1. 10.6 Solutions to exercises of Chapter 6 Solution 6.1. Observe that if f (x) + f f 1 1−x +f Hence, 2f (x) = x + x−1 x x−1 x − 1 1−x = = $ 1 1−x 1 1−x % = x, then and f x−1 x + f (x) = x−1 . x −x3 +x−1 x(1−x) . Solution 6.2. Prove by induction that f (n) = n. Let m = n = 1 in order to see that f (1) = 1, and since f (2) = 2, by (ii) and using the induction hypothesis, it follows that f (2k) = 2f (k) = 2k and f (2k + 2) = f (2)f (k + 1) = 2(k + 1) = 2k + 2. Finally, by (iii), 2k = f (2k) < f (2k + 1) < f (2k + 2) = 2k + 2, hence f (2k + 1) = 2k + 1. Then f (n) = n, for all n ∈ N. Solution 6.3. Taking −x instead of x in the original equation, we obtain −xf (−x)− 2xf (x) = −1. Then xf (x) + 2xf (−x) = −xf (−x) − 2xf (x), hence, 3xf (x) = −3xf (−x). This we can substitute in the original equation to obtain xf (x) = 1. 1 Solution 6.4. Taking −x instead of x in the original equation, we obtain −x f (x) + $ %   1 −1 1 2 f −x = −x, then f (x)−xf x = x . Now, taking x instead of x in the original 230 Chapter 10. Solutions to Exercises and Problems   + f (x) = x1 . Adding the last two equalities leads equation, we have that xf −1 x 1 2 to 2f (x) = x + x . Hence, the only function that satisfies the original equation is f (x) = 1 2 1 + x2 . x Solution 6.5. Taking y = −x in the original equation, we get xf (x) + xf (−x) = 2xf (0), for all x. Then f (x) + f (−x) = 2f (0) for all x = 0. Taking x+y and x in the functional equation, we get (x+y)f (x+y)−xf (x) = yf (2x + y), and taking 2x + y, −x, we obtain (2x + y)f (2x + y) + xf (−x) = (3x + y)f (x + y). Then, the last two equations can be rewritten as x (f (x + y) − f (x)) = y (f (2x + y) − f (x + y)) (2x + y)(f (2x + y) − f (x + y)) = x(f (x + y) − f (−x)). Multiplying the first equation by 2x + y and the second one by y, and reducing we get (2x + y)x(f (x + y) − f (x)) = yx(f (x + y) − f (−x)). Canceling out x on both sides of the equation, simplifying and solving for 2xf (x + y), leads to 2xf (x + y) = (2x + y)f (x) − yf (−x). Substituting the value of f (−x), gives us 2xf (x + y) = (2x + y)f (x) − y(2f (0) − f (x)) = 2(x + y)f (x) − 2yf (0). That is, xf (x + y) = (x + y)f (x) − yf (0) xf (x + y) − xf (0) = (x + y)f (x) − yf (0) − xf (0) x(f (x + y) − f (0)) = (x + y)(f (x) − f (0)). Now if x = 1, then f (1+y)−f (0) = (1+y)(f (1)−f (0)). Substituting 1+y = x, we get f (x) = f (0)+ x(f (1)− f (0)). Then, if we define the constants m = f (1)− f (0), b = f (0), the functions that satisfy the equation have the form f (x) = mx + b. Clearly, the functions of the form f (x) = mx + b satisfy the original functional equation. Solution 6.6. First, note that if f (x + 1) = f (x) + 1, by induction it follows that f (x+n) = f (x)+n, for all n ∈ N. Moreover, f (x2 ) = f (x)2 and f (x+n) = f (x)+n imply that f ((x + n)2 ) = f (x + n)2 = (f (x) + n)2 . Then f (x2 + 2xn + n2 ) = f (x2 + 2xn) + n2 = f (x)2 + 2f (x)n + n2 , and then f (x2 + 2xn) = f (x)2 + 2f (x)n. Taking x = 0 and n = 1 in the last equation, we get f (0)2 +f (0) = 0, then $ f (0) = % 0. 2 Moreover, taking x = pq and n = q in the last equation, leads to f pq2 + 2p = $ %2 $ 2 % $ 2% 2 f pq + 2qf ( pq ) = f ( pq2 ) + 2qf ( pq ) and, since f pq2 + 2p = f pq2 + 2p, then f ( qp ) = pq . That is, f (x) = x for all x ∈ Q+ ∪ {0}. 231 10.6 Solutions of Chapter 6 Solution 6.7. Suppose that x, y, z are different numbers. The equation can be rewritten as x(f (y)− f (z))+ yf (z) = f (x)(y − z)+ zf (y); now subtracting on both sides yf (y), we get x(f (y) − f (z)) + y(f (z) − f (y)) = f (x)(y − z) + f (y)(z − y), hence f (y) − f (z) f (x) − f (y) = . x−y y−z Then, the slope between points (x, f (x)) and (y, f (y)) is equal to the slope between points (y, f (y)) and (z, f (z)), then every 3 points of the graph of f are collinear; hence the graph of f is a line and therefore f (x) = mx + b, for some real numbers m and b. In fact, m is the common slope and b = f (0). Clearly, the affine functions f (x) = mx + b satisfy the equation. Second Solution. Taking y = −1, z = 1, we get xf (−1) − f (1) + f (x) = −f (x) + f (−1) + xf (1), and solving for f (x) the result is f (x) = f (1) − f (−1) f (−1) + f (1) x+ . 2 2 Solution 6.8. The equality g(f (x)) = −x, for any real number x, guarantees that g(f (g(x))) = −g(x). If to the equality f (g(x)) = −x, we apply g to both sides, we obtain g(f (g(x))) = g(−x). Then g(−x) = −g(x), hence g is odd. Similarly we can prove that f (−x) = −f (x). Solution 6.9. Taking y = 0 in the original equation, we have f (f (x)) = f (x)−f (0). But since f is surjective, given any real number y there exists x with f (x) = y, then f (y) = y − f (0). Taking y = 0 in this last equation, we get f (0) = 0. Thus f (x) = x for any real number x. It is clear that the function f (x) = x satisfies the equation. Solution 6.10. Taking x = 0 in the original equation gives us f (0) = 0. Now, if x = 1, we have f (f (y)) = y, then f is bijective. Taking f (y) as y in the equation and using f (f (y)) = y, we get f (xy) = xf (y). By symmetry in the variables x, y, also it is true that f (xy) = yf (x), and then xf (y) = yf (x). Hence for x, y f (x) = f (y) is constant and equal to different from 0, it follows that f (x) x y , then x f (1), thus f (x) = f (1)x. Using f (x) = f (1)x in the original equation, it follows that xy = f (1)xf (y) = f (1)2 xy for x, y ∈ R, then f (1)2 = 1. Hence f (x) = x or f (x) = −x are the only continuous solutions of the equation. Solution 6.11. Since m − n + f (n) ≥ 1 holds for all n ∈ N, then f (n) ≥ n. Letting F (n) = f (n) − n, we can rewrite the functional equation as F (m + F (n)) = F (m) + n, for all m, n ∈ N. Taking m = 1 and adding 1 to both sides of the last equation, we have that F (1 + F (n)) + 1 = F (1) + n + 1, for n ∈ N. If now we apply F on both sides and 232 Chapter 10. Solutions to Exercises and Problems we use the new equation, we get F (1) + 1 + F (n) = F (n + 1) + 1, then F (n + 1) = F (n) + F (1), for all n ∈ N. That is, F (n) = F (1)n for all n ∈ N, then (m+nF (1))F (1) = mF (1)+n, for all n, m ∈ N. In this last equation, taking n = m = 1, leads to F (1)(F (1)+1) = F (1)+1, hence F (1) = 1, and then f (n) = 2n for all n ∈ N. Clearly f (n) = 2n satisfies the functional equation. Solution 6.12. In Example 6.2.4, we proved that the function is bijective. With y = 1 and using the injectivity, it follows that $ % f (1) = 1, and then f (f (y)) = y1 . Applying f to both sides, we get 1 f (y) =f 1 y . + For x, y ∈ Q , take z such that f (z) = y, then f (xy) = f (xf (z)) = f (x) = f (x)f (f (z)) = f (x)f (y). z In this way a function that satisfies the functional equation must satisfy the two equations 1 and f (xy) = f (x)f (y). f (f (x)) = x One particular solution can be defined as follows. Let p1 , p2 , . . . be the ordered prime numbers and we define the function on the prime numbers as follows:  pi+1 , if i is odd f (pi ) = 1 pi−1 , if i is even and for a rational number r = pn1 1 · · · pnk k , the function is defined as f (r) = f (p1 )n1 · · · f (pk )nk , where nk ∈ Z. Solution 6.13. Taking x = y in the original equation we get xf (x) = x(f (x))2 , then x(f (x)2 − f (x)) = 0 for any real number x. Hence, for x = 0, we have f (x)2 = f (x), so that for every real number x it follows that f (x) = 0 or 1. If for all x = 0 we have f (x) = 0, then by continuity it follows that f (0) = 0, and then f is identically zero on the real numbers. If for some x0 = 0 it happens that f (x0 ) = 1, then taking x0 in the original equation we obtain x0 f (y)+y = (x0 +y)f (y), hence y = yf (y) for all real numbers y. Then, f (y) = 1 for all y = 0, and by continuity f (0) = 1, which guarantees that f is identically 1 on the real numbers. Therefore, the only functions that satisfy the equation are the constant functions 0 and 1. 233 10.6 Solutions of Chapter 6 Solution 6.14. (i) We should expect that the period is related to a, then it is a good idea to iterate the function. By doing it we get 1 + 2 1 = + 2 f (x + 2a) = 1 = + 2 f (x + a) − f (x + a)2 1 + 2 f (x) − f (x)2 − 1 − f (x) 2 2 1 − 4  1  = + f (x) − 2 f (x) − f (x)2 − (f (x) − f (x)2 )  1  . 2 Since f (x) ≥ 12 for all x, then we have f (x) = f (x + 2a) for all x. Hence, f is periodic with period 2a. (ii) To find an example observe that f (x) ≥ 12 for all x and, on the other hand,  2 the original equation guarantees that f (x + 2) − 12 = f (x + 1)(1 − f (x + 1)) ≤  1 2 2 , where the inequality follows from the geometric and the arithmetic mean inequality. Therefore, a possible example is, for n ∈ Z, the function  1 , if 2n ≤ x < 2n + 1 f (x) = 2 1, if 2n + 1 ≤ x < 2n + 2. Solution 6.15. Suppose that the equation has at least one real solution x. Then m(a + b) = |x − a| + |x − b| + |x + a| + |x + b| ≥ |(x − a) − (x + b)| + |(x − b) − (x + a)| = 2(a + b), and since a + b > 0, it follows that m ≥ 2. Conversely, suppose m ≥ 2, then the equation has at least one real solution. In fact, if we define f (x) = |x − a| + |x − b| + |x + a| + |x + b|, observe that f (0) = 2(a + b) ≤ m(a + b) and f (ma + mb) = 4m(a + b) > m(a + b). By the intermediate value theorem27 , there exists x such that f (x) = m(a + b). Solution 6.16. Without loss of generality we can assume that f (0) = 0, since the function g(x) = f (x) − f (0) satisfies the equation and g(0) = 0. Taking y = 0 in the equation, we have f (x2 ) = xf (x). Using this last equation and taking y = 1, we get xf (x) − f (1) = (x + 1)(f (x) − f (1)), hence f (x) = f (1)x. This means all functions that satisfy the original equation are of the form f (x) = f (1)x + f (0). It is easy to check that the affine functions f (x) = mx + b satisfy the equation. 27 See [21]. 234 Chapter 10. Solutions to Exercises and Problems Solution 6.17. Taking x = y = 1 in the original equation, it follows that f (1)2 − f (1) = 2, then f (1) = 2. Now, if we let y = 1, we get f (x)f (1) − f (x) = x + x1 , then f (x) = x + x1 . And it is clear that f (x) = x + x1 satisfies the equation. Solution 6.18. (i) In Example 6.2.2 we proved that these functions are injective. As we have seen in the example, taking x = y, leads to f (xf (x)) = x2 , in particular f (f (1)) = 1. Taking x = f (1) in the last equation, gives us f (1)2 = f (f (1)f (f (1))) = f (f (1)) = 1, hence f (1) = 1. Letting y = 1 in the original equation we obtain f (x) + f (f (x)) = 2x. If z > 0, taking x = zf (z) and y = 1z in the functional equation, we get f 1 z zf (z)f 1 1 f (zf (z)) = 2zf (z) . z z +f Then, using f (zf (z)) = z 2 , it follows that f 1 z zf (z)f = f (z),   but since f is injective, it follows that f (z)f z1 = 1. If now we take x = z, y = z1 in the original functional equation, we get f but since f 1 z = 1 f (z) , zf f 1 z +f 1 f (z) = 2, z then f Since f (z)f 1 z z f (z) +f f (z) z = 2. = 1, it also follows that z f (z) ·f f (z) z = 1, then f z f (z) =f f (z) z =1 and again the injectivity of f guarantees that f (z) = z. (ii) In Example 6.2.3, we proved that these functions are surjective. Then, there exists a number x0 such that f (x0 ) = 0. Letting x = x0 in the original equation, we obtain f (y) = 2x0 + f (f (y) − x0 ), therefore if we make z = f (y) − x0 we get f (z) = z − x0 . Hence, the functions that satisfy the equation must have the form f (z) = z + c, for some constant c. Solution 6.19. First note that 2 must not be in the image in order to consider the (x)−3 . If for some x, f (x) = 1 then f (x + a) = −2 quotient ff (x)−2 −1 = 2, hence 1 is not in 235 10.6 Solutions of Chapter 6 the image either. Now, observe that 2f (x) − 3 f (x + a) − 3 = f (x + a) − 2 f (x) − 1 f (x + 2a) − 3 = f (x). f (x + 3a) = f (x + 2a + a) = f (x + 2a) − 2 f (x + 2a) = Then, f (x) is periodic of period 3a. Solution 6.20. Let T be the period of f . Suppose that T = pq , where p and q are relatively prime positive integers. Then, qT = p is also a period of f . Let n = kp + r, where k and r are integers and 0 ≤ r < p − 1. Then f (n) = f (kp + r) = f (r), and f (n) ∈ {f (1), f (2), . . . , f (p − 1), f (p)}, for all positive integers n, which is a contradiction with the fact that {f (n) | n ∈ N} has an infinite number of elements. Solution 6.21. Letting x = y = 0, we get f (0) = 2f (0) , 1 − f (0)2 which makes sense if f (0) = ±1. Then f (0)3 + f (0) = 0, hence f (0) = 0. Now take g(x) = arctan f (x), then tan g(x) = f (x) which is well defined for x ∈ (−1, 1). Substituting in the equation (6.7) we obtain tan g(x + y) = tan g(x) + tan g(y) = tan(g(x) + g(y)). 1 − tan g(x) tan g(y) The last equality follows from the tangent formula for the sum of two angles. Now, apply the inverse tangent function on both sides of the equation to obtain g(x + y) = g(x) + g(y) + k(x, y)π, where k(x, y) is a function that only takes integer values. On the other hand, since f (0) = 0 we have g(0) = 0 and then k(0, 0) = 0. But since k is a continuous function, k(x, y) = 0 for all x, y ∈ R, we get the equation g(x + y) = g(x) + g(y), which is the Cauchy equation whose continuous solution is g(x) = αx. Hence, the solution of equation (6.7) is the function f (x) = tan αx. Solution 6.22. Since tan u + tan v = tan(u + v), 1 − tan u tan v 236 Chapter 10. Solutions to Exercises and Problems we can take x = tan u and y = tan v with xy = 1, which is true if and only if tan u tan v = 1, that is, u − v = π2 . The equation becomes f (tan u) + f (tan v) = f (tan(u + v)), then f ◦ tan is additive and continuous, therefore f (tan u) = cu, which implies that f (x) = c arctan x. Solution 6.23. The function f (x) ≡ −1 is a solution of the functional equation. If now we define g(x) = f (x) + 1, substituting we get g(x + y) − 1 = g(x) − 1 + g(y) − 1 + [g(x) − 1][g(y) − 1] = g(x) − 1 + g(y) − 1 + [g(x)g(y) − g(x) − g(y) + 1] = g(x)g(y) − 1. Therefore g(x) satisfies the Cauchy equation g(x + y) = g(x)g(y) and then g(x) = ax , with a ∈ R+ and f (x) = ax − 1. Solution 6.24. Suppose that there exists a function that satisfies f (f (n)) = n + 1. (10.9) Applying f to both sides of the equation, we obtain f (n + 1) = f (f (f (n))) = f (n) + 1. Let us see by induction that f (n + 1) = f (1) + n. The case n = 1 is obvious, since f (n + 1) = f (n) + 1 and after substituting n = 1, we get f (1 + 1) = f (1) + 1. Suppose the result true for n − 1 and prove it for n. Since it is true for n − 1, the following holds: f (n + 1) = f (n) + 1 = f ((n − 1) + 1) + 1 = (f (1) + n − 1) + 1 = f (1) + n, then f (n + 1) = f (1) + n for all n ∈ N. From this last equation and by equation (10.9), we get n + 1 = f (f (n)) = f (n) − 1 + f (1) = n − 1 + f (1) − 1 + f (1) = n − 2 + 2f (1). Thus f (1) = 32 , which is a contradiction, since the image of f are the natural numbers. Therefore, no f exists that satisfies equation (10.9). Solution 6.25. If x ≥ 2, then f (x) = f (x − 2 + 2) = f ((x − 2)f (2))f (2) = 0, this together with (iii), implies that f (x) = 0 if and only if x ≥ 2. For 0 ≤ y < 2, we have f (y) = 0 and 0 = f (2) = f (2 − y + y) = f ((2 − y)f (y))f (y), then f ((2 − y)f (y)) = 0, hence (2 − y)f (y) ≥ 2. 2 , we get f (x + y) = f (xf (y))f (y) = f (2)f (y) = 0, then x + y ≥ 2, Taking x = f (y) but this implies 2 ≥ (2 − y)f (y). Since (2 − y)f (y) ≥ 2, then (2 − y)f (y) = 2, that 2 is, f (u) = 2−y for 0 ≤ y < 2. 237 10.6 Solutions of Chapter 6 Hence, f (y) =  2 2−y , for 0 ≤ y < 2 for y ≥ 2. 0, It is not difficult to see that f satisfies the conditions of the exercise. Solution 6.26. It is clear that f (x) = 0 and f (x) = 1 are solutions; let us see that there are no other solutions. Taking x = y = 0 in the functional equation, it follows that f (0) = [f (0)]2 , then either f (0) = 0 or f (0) = 1. Let us analyze the two cases: (i) f (0) = 0. Letting y = 0 in the functional equation, we get f (x) = f (x)f (0) = 0, then f (x) = 0 for all x ∈ R. ii) f (0) = 1. First, we will see that f (x) = 0 for all x ∈ R. Letting x = y in the original equation, we obtain 1 = f (0) = f (2x)f (x), then f (x) = 0 for all x ∈ R. Substituting x and y by 2u and u, respectively, we obtain f (u) = f (3u)f (u), since f (u) = 0, then f (3u) = 1. Finally, letting 3u = x, we get f (x) = 1 for all x ∈ R. Therefore the only solutions are f (x) = 0 and f (x) = 1. Solution 6.27. By Theorem 6.5.4, for a function f we have ∆n f (x) = n # (−1)k k=0 n f (x + n − k) when h = 1. k Moreover, by Example 6.5.3, if P (x) = a0 +a1 x+· · ·+an xn , then ∆n P (x) = an n!, when h = 1. n . The coefficient Consider f (x) = P (x) = (n − x)n*   of xn in this polynomial n n n n k n is (−1) , then (−1) n! = ∆ P (x) = k=0 (−1) k (k − x) . Therefore, letting x = 0 in the last equation, we get n # k=0 (−1)k k n n k = (−1)n n!. Solution 6.28. We prove that the function f is injective. If f (n) = f (m), then f (f (f (n))) + f (f (n)) + f (n) = f (f (f (m))) + f (f (m)) + f (m), hence 3n = 3m, that is, n = m. Evaluating in n = 0, it follows that f (f (f (0))) + f (f (0)) + f (0) = 0, then f (f (f (0))) = f (f (0)) = f (0) = 0. It is evident that f (n) = n satisfies the equation; let us see that it is the only solution. By induction suppose that f (k) = k for 0 ≤ k < n. Since f is injective, f (n) cannot take any of the values 0, 1, . . . , n − 1, then f (n) ≥ n and also f (f (n)) ≥ n and f (f (f (n))) ≥ n. Thus, f (f (f (n))) + f (f (n)) + f (n) ≥ 3n. 238 Chapter 10. Solutions to Exercises and Problems By hypothesis, the equality must hold, then f (f (f (n))) = f (f (n)) = f (n) = n, which proves that f (n) = n, for all n ∈ N ∪ {0}. Solution 6.29. Let us prove that f (x) = x. First, note that f is injective because if f (x) = f (y), then x = f n (x) = f n (y) = y, hence x = y and f is injective. Now, let us see that f is increasing. Suppose that there exist x1 and x2 such that x1 < x2 and f (x1 ) ≥ f (x2 ); since f is continuous in [0, x1 ], then by the intermediate value theorem28 there is c ∈ [0, x1 ] such that f (c) = f (x2 ), which is a contradiction, since f is injective. Now, assume that x < f (x), then f (x) < f (f (x)) = f 2 (x) < · · · < f n (x) = x, which is a contradiction. Similarly, if we suppose that x > f (x) we reach another contradiction, therefore f (x) = x is the only function that satisfies the conditions. Solution 6.30. Note that f is injective. If for all x, y ∈ R we have that f (x) = f (y), then f n (x) = f n (y), hence −x = −y. Also, f is surjective because −x = f (f n−1 )(x). Since f (−x) = f (f n (x)) = f n (f (x)) = −f (x), we have that f is odd, therefore f (0) = 0. But if f is bijective and continuous, then it is monotone. Let us prove that f cannot be increasing. If x < y implies that always f (x) < f (y), then −x = f n (x) < f n (y) = −y, and y < x, which is a contradiction. Thus, if f is decreasing, then x and f (x) should have different signs for x = 0 (note that x > 0, implies that f (x) < 0, and x < 0, implies that f (x) > 0). Then, for x = 0, xf (x) < 0 and then x, f (x), f 2 (x), . . . , f n (x) alternate signs, but if f n (x) = −x, then n is odd. Let x > 0 and assume that f (x) > −x. Since f is decreasing and odd, we have f (f (x)) < f (−x) = −f (x) < x, again, since f (x) < −f (f (x)), being decreasing and odd f (f (x)) > f (−f (f (x))) = −f (f (f (x))). Continuing in this way, we get x > −f (x) > f 2 (x) > −f 3 (x) > · · · > −f n (x) = x, which is a contradiction, therefore f (x) ≤ −x. Similarly, we can show that for x > 0 it is not possible that f (x) < −x. Then f (x) = −x for x > 0. Now, using that f is odd, we conclude that f (x) = −x, for all x ∈ R. And f (x) = −x satisfies the functional equation. 28 See [21]. 239 10.7 Solutions of Chapter 7 10.7 Solutions to exercises of Chapter 7 Solution 7.1. Proceed by induction. For n = 1, we have a1 = 1 < 47 and for n = 2,  7 2 49 we have a2 = 3 = 48 16 < 16 = 4 , which proves the induction basis.  n−2 For n > 2, suppose the result valid for n − 2 and n − 1, that is, an−2 < 47  n−1 . By direct calculation and using the induction hypothesis, it and an−1 < 74 follows that an = an−1 + an−2 7 4 n−1 < 7 4 n−2 < 7 4 + 49 16 n−2 7 4 = = 7 4 n−2 7 +1 4 = 7 4 n−2 11 4 n . Solution 7.2. Adding 2n to both sides of an = 3an−1 + 2n−1 , we get an + 2n = 3an−1 + 3 · (2n−1 ) = 3(an−1 + 2n−1 ), for all n ≥ 2. Setting bn = an + 2n , we obtain bn = 3bn−1 = · · · = 3n−1 b1 . Since b1 = a1 + 2 = 1 + 2 = 3, it follows that bn = 3n−1 · 3 = 3n , hence an = 3n − 2n . Solution 7.3. Since an+1 = 1 + a1 a2 . . . an , it follows that a1 a2 . . . an = an+1 − 1, then a1 a2 . . . an−1 = an − 1; hence an+1 − 1 = (an − 1)(an ) > 0, therefore 1 an+1 − 1 = 1 1 1 . = − (an − 1)(an ) an − 1 an Finally, we get 1 1 1 1 + ··· + =1+ + ···+ a1 an a2 an 1 1 1 1 =1+ − − + ···+ a2 − 1 a3 − 1 an − 1 an+1 − 1 1 1 1 − = 2− < 2. =1+ a2 − 1 an+1 − 1 an+1 − 1 Solution 7.4. By definition an+1 an−1 = a2n + 1. Consider an+2 an = a2n+1 + 1. If we subtract from this last equation the original identity, we get an+2 an − an+1 an−1 = a2n+1 − a2n , which can be rewritten as an (an+2 + an ) = an+1 (an+1 + an−1 ). Therefore, an+2 + an an+1 + an−1 = . an+1 an 240 Chapter 10. Solutions to Exercises and Problems Now, the sequence bn = an+1 +an−1 an with n ≥ 2 is constant29 and since a3 + a1 b2 = = a2 a22 +1 a1 + a1 a2 = 3, n−1 it follows that an+1a+a = 3, then an+1 = 3an − an−1 , for every n ≥ 2. Now, n the principle of mathematical induction helps us to conclude that every an is an integer and the original equation implies that each an is positive. Solution 7.5. We have a2 − a1 ≥ 1 and an+2 − an+1 ≥ (an+1 − an ) + 1, where the second inequality follows by applying the condition to (n, n + 1, n + 1, n + 2) for all n. By induction, it is possible to show that an+1 − an ≥ n for all n ≥ 1. Therefore, an+1 ≥ n + an and a1 ≥ 1, and again by induction we have an ≥ 12 (n2 − n + 2). Since the sequence an = 12 (n2 − n + 2) satisfies the conditions of the problem (the condition ai + al > aj + ak in this case becomes i2 + l2 > j 2 + k 2 , where we take i = d − y, l = d + y, j = d − x, k = d + x, with 0 ≤ x < y), the smallest value of a2008 is 2 015 029. Solution 7.6. Note that a1 = 1 − a0 implies that a0 = 1 − a1 . Now, a2 = 1 − a1 (1 − a1 ) = 1 − a1 a0 , and then by induction we have an = 1 − a0 a1 . . . an−1 . That is, an+1 = 1 − an (1 − an ) = 1 − an (a0 . . . an−1 ) = 1 − a0 . . . an−1 an . The proof is finished using induction. For n = 0, 1 the identity follows immediately. Now suppose the statement holds for n and consider 1 1 + ···+ a0 an+1 1 1 + ···+ = (a0 . . . an ) a0 an (a0 . . . an+1 ) an+1 + (a0 . . . an+1 ) 1 an+1 = an+1 + a0 . . . an = 1. Solution 7.7. For n = 0, we have x20 = y0 + 2. Now, use induction. Suppose that x2k = yk + 2 and prove that x2k+1 = yk+1 + 2. Indeed, x2k+1 = (x3k − 3xk )2 = (x2k )3 − 6(x2k )2 + 9(x2k ). Using the induction hypothesis, we have x2k+1 = (yk + 2)3 − 6(yk + 2)2 + 9(yk + 2) = yk3 − 3yk + 2 = yk+1 + 2. Solution 7.8. For n = 1, we have 1 + 4a1 a2 = 1 + 4(1)(12) = 49 = 72 . Now, we use induction to show that for n ≥ 2, we have 1 + 4an an+1 = (an+1 + an − an−1 )2 . That is, for n = 2 we get 1 + 4a2 a3 = 1 + 960 = 961 = 312 , and (a3 + a2 − a1 )2 = (20 + 12 − 1)2 = 312 . 29 See Example 7.1.4. 10.7 Solutions of Chapter 7 241 For the inductive step, suppose that 1 + 4an an+1 = (an+1 + an − an−1 )2 . Note that (an+2 + an+1 − an )2 = (2an+1 + 2an − an−1 + an+1 − an )2 = (2an+1 + an+1 + an − an−1 )2 = 4a2n+1 + 4an+1 (an+1 + an − an−1 ) + (an+1 + an − an−1 )2 = 4a2n+1 + 4an+1 (an+1 + an − an−1 ) + (1 + 4an an+1 ) = 4an+1 (2an+1 + 2an − an−1 ) + 1 = 4an+1 an+2 + 1, as we wanted to prove. √ Solution 7.9. Note that a1 = 2, a2 < 4, a3 < 4 + 3 · 4 = 4. We show by induction that a√n < 4 for all √ n ∈ N. For n = 0, 1, 2 and 3, it is clear. Suppose that an < 4, then 4 + 3an < 4 + 3 · 4 = 4, and an+1 < 4. Hence, the sequence is bounded by 4. Solution 7.10. Since an+1 = an + a12 , then a3n+1 = a3n + 3 + a33 + a16 > a3n + 3. n n n Since √ a32 = 1 + 3 + 3 + 1 > 2 · 3, by induction it follows that a3n √ > 3n. Therefore, an > 3 3n and the sequence is not bounded. Moreover, a9000 > 3 27000 = 30. Solution 7.11. Suppose that all terms of the sequence are rational positive numbers, an = pqnn , with (pn , qn ) = 1. Then p2n+1 pn pn + qn = a2n+1 = an + 1 = +1= , 2 qn+1 qn qn 2 2 2 that is, qn+1 (pn + qn ) = qn · p2n+1 . Then, note that qn |qn+1 and qn+1 |qn , therefore 2 qn+1 = qn for all n. n Then, qn+1 = (q1 )1/2 is a positive integer for all n. This happens only if q1 = 1 and then qn = 1 √for all n, meaning that an is an integer for all n. Now, if an = 1, then an+1 = 2, which is a contradiction. Then, an > 1 for all n. It follows that a2n+1 − a2n = an + 1 − a2n = 1 + an (1 − an ) < 0 and an+1 < an for all n, that is, we have an infinite decreasing sequence of positive integers, which is a contradiction. Therefore, the sequence must contain irrational numbers. Solution 7.12. It is not difficult to see that the constant sequences {an )= A}, the ( linear sequences {an = Bn} and the sequences of the form an = Cn2 , with A, B, C fixed numbers, solve the recurrence. Then, also the sequences ) ( an = A + Bn + Cn2 are solutions. )Given the initial conditions, the solutions are {an = 1}, {an = n} ( and an = n2 , respectively. Solution 7.13. Since the sequence is bounded, some terms are repeated infinitely many times. Let K be the greatest number that is repeated infinitely many times in the sequence, and let N be a positive integer such that ai ≤ K for i ≥ N . 242 Chapter 10. Solutions to Exercises and Problems Choose m ≥ N such that am = K. We will prove that m is the period of the sequence, that is, ai+m = ai for all i ≥ N . First, we suppose that ai+m = K for some i. Since ai + am is divisible by ai+m = K, then ai = K = ai+m . Now, if ai+m < K, choose j ≥ N such that ai+j+m = K, then it follows that ai+m + aj < 2K. Since ai+m + aj is divisible by ai+j+m = K, then ai+m + aj = K and therefore aj < K. Since ai+j+m = K, the argument in the previous paragraph implies that ai+j+m = ai+j = K, and then K divides ai + aj . It follows that ai + aj = K, since ai ≤ K and aj < K. Therefore, ai+m = K − aj = ai . Solution 7.14. The sequence an = n! satisfies the given recursion because n(n! + (n − 1)!) = n(n + 1)(n − 1)! = (n + 1)!. The number of derangements of n + 1 elements can be found as follows: Consider the permutations of n+ 1 elements without fixed points; the first element can be any of the n elements different from the first. Since there are n elements left, the dn+1 permutations can be divided into n groups according to which one was in the first place of the n elements different from the first. The groups have the same number of elements. Take one of the groups, say the one where the second element was in the first place. The permutations are divided in two, when 1 goes to 2 and otherwise. In the first case, there are dn−1 derangements and in the second 1 is moved to any place different from 2 and the rest will move freely to a different place from the first, then there are dn such permutations, hence dn+1 = n(dn + dn−1 ). The sequences are different since the first terms are not equal, that is, d0 = 1, d1 = 0 and a0 = a1 = 1. Solution 7.15. (i) Note that dn − ndn−1 = −(dn−1 − (n − 1)dn−2 ) = (dn−2 − (n − 2)dn−3 ) = · · · = (−1)n−2 (d2 − 2d1 ) = (−1)n−2 (1 − 2 · 0) = (−1)n . (ii) A direct application of the formula in Example 7.2.4, leads to dn = n(n − 1) · · · 2 · d1 + = (n!)d1 + n−2 # n−2 # j=1 n(n − 1) · · · (j + 2)(−1)j+1 + (−1)n n! n! (−1)j+1 + (−1)n (j + 1)! n! j=1 (−1)2 (−1)3 (−1)n + + ···+ 2! 3! n! 2 1 (−1) (−1)n (−1) + + ···+ = n! 1 + 1! 2! n! = n! . 243 10.7 Solutions of Chapter 7 Solution 7.16. The characteristic√equation of the recursion is x2 − x − 1 = 0 which √ 1+ 5 has roots r = 2 and s = 1−2 5 . The solutions of the equation have the form $ √ %n $ √ %n Ln = A 1+2 5 + B 1−2 5 . $ √ %2 $ √ %2 √ Since 1 = L1 = A+B + 25 (A − B) and 3 = L2 = A 1+2 5 + B 1−2 5 , 2 $ √ %n $ √ %n then A = B = 1, hence Ln = 1+2 5 + 1−2 5 , for all n. b0 > 0 and by induction bn > 0. Consider Solution 7.17. If b0 > 0, then b1 = 1+b 0 1 an = bn ; the recursive equation takes the form an+1 = an + 1, which has as a b0 solution the sequence an = a0 +n. Then bn = 1+nb is the solution of the equation. 0 Solution 7.18. Notice that an+3 = 1 1 1 = = 1 − an+2 1 − 1−a1n+1 1 − 1− 1 1 = an . 1−an Solution 7.19. Suppose that an = bn+1 bn , then the recursion takes the form bn+2 = 4bn+1 −4bn which is linear of order 2. Its characteristic polynomial is λ2 −4λ+4 = 0, which has as unique solution λ = 2. Then, bn = (A + nB)2n for some numbers A n+1 is the solution of the equation. = (A+(n+1)B)2 and B. Hence an = (A+(n+1)B)2 (A+nB)2n (A+nB) Clearly an converges to 2. Solution 7.20. By Proposition 7.2.13, it is enough to observe the following inequalities, where we use the fact that ak+1 ≤ 2ak , for k = 1, 2, . . . , n, an+1 ≤ 2an = an + an ≤ an + 2an−1 = an + an−1 + an−1 .. . ≤ an + an−1 + · · · + a2 + 2a1 = an + an−1 + · · · + a2 + a1 + 1. Solution 7.21. Denote the sequence by {pn }; prove that pn+1 ≤ 2pn , for any n ≥ 1. For n = 1 it is immediate, since 2 = p2 = 2p1 = 2. For n ≥ 2, we use Bertrand’s postulate30 , which says that given an integer m > 1, there exists a prime number p such that m < p < 2m. For pk with k ≥ 2, it follows, again by Bertrand’s postulate, that there exists a prime number p with pk < p < 2pk . But this prime number is greater than or equal to the prime number after pk , that is pk+1 ≤ p. Then, pk+1 ≤ 2pk and then, using the previous exercise, we have the result. 30 See [15]. 244 Chapter 10. Solutions to Exercises and Problems Solution 7.22. Let a1 , a2 , . . . , an be the integer weights of each of the golden pieces and suppose that a1 ≤ a2 ≤ · · · ≤ an . By hypothesis, a1 + a2 + · · · + an = 2n and an ≤ a1 + a2 + · · · + an−1 . If an = a1 + a2 + · · · + an−1 , we are done. Case a1 ≥ 2 is clear since ai ≥ 2 for all i, and the condition a1 + a2 + · · · + an = 2n, implies that a1 = a2 = · · · = an = 2. Since n is even, we can perform the required partition. Now, suppose a1 = 1 and an < a1 + a2 + · · · + an−1 , then an ≤ 1 + a1 + a2 + · · · + an−1 . Then, it is enough to show that ak+1 ≤ 1 + a1 + a2 + · · ·+ ak , for k = 1, 2, . . . , n− 2. Suppose that the previous statement is not true, that is, let ak+1 > 1 + a1 + a2 + · · · + ak for some k ∈ {1, 2, . . . , n − 2}. Then ak+1 ≥ k + 2, and ai ≥ k + 2 for every i = k + 1, k + 2, . . . , n. Moreover, we know that ai ≥ 1 for i = 1, 2, . . . , k, then 2n = a1 + a2 + · · · + an ≥ k + (n − k)(k + 2) = −k 2 + (n − 1)k + 2n. This implies that k 2 − (n − 1)k ≥ 0, therefore k ≤ 0 or k ≥ n − 1, and both contradict that k ∈ {1, 2, . . . , n − 2}, and so the result follows. Now, to make the division follow the ideas of the proof of Proposition 7.2.13. Solution 7.23. It is possible to show by induction that 1 ≤ an ≤ 2 and that an ≥ an+1 , for all n ≥ 1. Therefore, the limit of the √ sequence is equal to some number L (see Theorem 7.4.7) which satisfies L = L, hence L = 1. Solution 7.24. (i) Observe that n # i=0 1 1 − ai ai+1 = 1 1 1 1 1 1 − + − + ···+ − a0 a1 a1 a2 an an+1 a2 − a1 an+1 − an a1 − a0 + + ···+ a0 a1 a1 a2 an+1 an d d d + + ···+ = a0 a1 a1 a2 an+1 an   n # 1 . =d a a i=0 i i+1 = On the other hand, the series n # i=0 1 1 − ai ai+1 Therefore, *n = *n 1 i=0 ( ai − 1 ai+1 ) is telescopic, hence 1 1 1 1 1 1 1 1 − + − + ···+ − = − . a0 a1 a1 a2 an an+1 a0 an+1 1 i=0 ai ai+1 = 1 d $ 1 a0 − % an+1 . 1 245 10.7 Solutions of Chapter 7 (ii) Observe that n # 1 1 − ai ai+1 ai+1 ai+2 i=0 1 1 1 1 1 1 − + − + ···+ − a0 a1 a1 a2 a1 a2 a2 a3 an an+1 an+1 an+2 a3 − a1 an+2 − an a2 − a0 + + ···+ = a0 a1 a2 a1 a2 a3 an an+1 an+2   n # 2d 2d 2d 1 = . + + ···+ = 2d a0 a1 a2 a1 a2 a3 an an+1 an+2 aa a i=0 i i+1 i+2 $ % * On the other hand, the series ni=0 ai a1i+1 − ai+11ai+2 is telescopic, then = n # i=0 Therefore, 1 1 − ai ai+1 ai+1 ai+2 *n 1 i=0 ai ai+1 ai+2 1 2d = $ = 1 a0 a1 1 1 − . a0 a1 an+1 an+2 − 1 an+1 an+2 % . (iii) Since a1 = a0 + d, a2 = a1 + d = a0 + 2d, . . ., an+1 = a0 + (n + 1)d, it follows from part (i) that ∞ # i=0 1 1 = lim n→∞ d ai ai+1 1 1 − a0 a0 + (n + 1)d = 1 . da0 (iv) Since a1 = a0 + d, a2 = a1 + d = a0 + 2d, . . ., an+1 = a0 + (n + 1)d, we have from part (ii) that ∞ # i=0 1 1 = lim n→∞ 2d ai ai+1 ai+2 1 1 − a0 a1 (a0 + (n + 1)d)(a0 + (n + 2)d) = 1 . 2da0 a1 Solution 7.25. Use the recurrence formula for the Fibonacci sequence in order to obtain the following equalities: (i) ∞ ∞ # # fn+1 − fn−1 fn = = f f fn−1 fn+1 n=2 n=2 n=2 n−1 n+1 ∞ # = lim N →∞ (ii) ∞ # 1 1 − fn−1 fn+1 1 1 1 1 + − − f1 f2 fN fN +1 = 1 1 + = 2. f1 f2 ∞ ∞ # # 1 fn fn+1 − fn−1 = = f f f f f f f f n=2 n−1 n+1 n=2 n−1 n n+1 n=2 n−1 n n+1 = ∞ # n=2 1 1 − fn−1 fn fn fn+1 = lim N →∞ 1 1 − f1 f2 fN fN +1 = 1 = 1. f1 f2 246 Chapter 10. Solutions to Exercises and Problems Solution 7.26. In step 0, the equilateral triangle has perimeter 3. In step 1, the curve is formed by 4 · 3 segments of length 13 , then it has perimeter 31 · 4 · 3. In step 2, the curve is formed by 42 · 3 segments of length 312 . Then its perimeter is 1 4n 2 has perimeter Pn = 31n · 4n · 3 = 3n−1 . 32 · 4 · 3. In general, in step n the curve 4n = ∞, since Pn is a geometric progression In this way, limn→∞ Pn = limn→∞ 3n−1 with ratio r = 43 > 1. √ The equilateral triangle of side 1 has area equal to 43 . In step 1 the curve encloses an area equal to the sum of the area of the original triangle and the area of the are%constructed on every side of the three equilateral triangles of sides 13 , which $ original triangle. That is, A1 = √ 3 4 √ 3 4 +3 · 1 32 . 1 In step 2 another $ 4 × 3 equilateral to the % $triangles % of side 9 are added % $ √structure, √ √ √ √ 3 3 3 3 1 1 then A2 = 4 + 3 4 · 32 + 4 · 3 4 · 34 . In general, An = 4 + 3 43 · 312 + $√ % % $√ 1 4 · 3 43 · 314 + · · · + 4n−1 · 3 43 · 32n . %% $ √ $ 2 n−1 Notice that we can write An = 43 1 + 13 1 + 94 + 942 + · · · + 94n−1 . In this way, we have that the sum inside the second parenthesis is the sum of a$ geometric %% √ $ progression with ratio r = 94 . Therefore, limn→∞ An = 43 1 + 13 1−1 4 = 9 √  √  3 1 + 53 = 2 5 3 . 4 Solution 7.27. (i) For n ≥ 1, it follows that 2n1+j ≥ Since the inequality is strict for j = 2n , we have 2n 1 2n+1 , for all j = 1, 2, . . . , 2n . 1 2n 1 1 1 + n + · · · + n+1 > n+1 = . +1 2 +2 2 2 2 (ii) The sum can be written as 1 1 1 1 + + ··· + 2 = + n n+1 n n + + + Since 1 kn+1 + ··· + 1 (k+1)n ≥ n (k+1)n = 1 k+1 , 1 1 + ···+ n+1 2n 1 1 + ··· + 2n + 1 3n .. . 1 1 + ···+ 2 (n − 1)n + 1 n . for each k = 2, 3, . . . , n − 1, we have 1 1 1 1 1 1 1 + + ···+ 2 ≥ + + + ···+ . n n+1 n n 2 3 n In order to reach the conclusion, observe that for n ≥ 2, 1 1 1 2 + 3 + 6 = 1. 1 n + 1 2 + 1 3 + ··· + 1 n ≥ 247 10.7 Solutions of Chapter 7 1 1 + n1 + n+1 > (iii) Simplifying the inequality n−1 2 2 n − 1 < n , which is always true for n ≥ 1. (iv) From part (i), we get 1 + 1 1 1 2n +2 + · · · + 2n+1 > 2 then 1+ 1 2 > 12 , 1 3 + 1 4 > 3 n, we see that it is equivalent to 1 1 2, 5 + ··· + 1 8 > 21 , . . . , 1 2n +1 + 1 1 1 n+1 + + · · · + n+1 > , 2 3 2 2 hence we can conclude that the harmonic series is divergent. Solution 7.28. Use Abel’s summation formula to obtain n # k 2 q k−1 = k=1 n−1 # (k 2 − (k + 1)2 )(1 + q + · · · + q k−1 ) + n2 (1 + q + · · · + q n−1 ) k=1 n−1 # =− (2k + 1) k=1 n−1 # k qk − 1 q−1 qn − 1 q−1 + n2 n−1 # qk − 1 qn − 1 q −1 −2 (k + 1) + n2 q−1 q−1 q−1 k=1 k=1 n−1 n−1   # # 1 2 k k q −1 − (k + 1)(q − 1) = q−1 q−1 = k=0 + n2 k=0 qn − 1 q−1 n 2 qn − 1 − − = (q − 1)2 q−1 q−1 +n 2 qn − 1 q−1  n # k=1 n(n + 1) kq k−1 − 2  . (10.10) Using Example 7.3.5, we can see that equation (10.10) is equal to 2 qn − 1 − (q − 1)2 q − 1 = nq n qn − 1 − q − 1 (q − 1)2 + −n + n(n + 1) + n2 (q n − 1) q−1 n2 q n 2(q n − 1) (1 − 2n)q n − 1 . + + 3 2 (q − 1) (q − 1) q−1 Therefore, n # k=1 k 2 q k−1 = 2(q n − 1) (1 − 2n)q n − 1 n2 q n . + + 3 2 (q − 1) (q − 1) q−1 248 Chapter 10. Solutions to Exercises and Problems Solution 7.29. Observe that ∞ # n=0 n(n − 1)xn = ∞ # n=0 n2 xn − ∞ # nxn . n=0 Now, use Examples 7.3.3 and 7.3.4, to obtain ∞ # n2 xn = n=0 ∞ # n=0 n(n − 1)xn + ∞ # nxn = n=0 x 2x2 + . (1 − x)3 (1 − x)2 Solution 7.30. In order to prove (i), (ii) y (iii) use the fact that the series are 1 geometric series with ratios 43 , −1 3 and 3 , respectively. Then, their sums are equal 1 to 1−r , where r is the ratio. ∞ n * (iv) Use Example 7.3.3 to obtain = 2. n n=1 2 (v) Use Exercise 7.29 to prove that  2 ∞ 1 # 2 12 8 4 n2 2 = +  1 3  1 2 = + = 6. n 2 2 2 n=1 2 2 10.8 Solutions to exercises of Chapter 8 Solution 8.1. The product of the coefficients of the polynomials is 4 2 2 7 1 1 8 4 8 4 32 16 56 8 2 7 1 14 2 8 8 48 25 57 8 Therefore, the product of the polynomials is R(x) = 8x5 + 8x4 + 48x3 + 25x2 + 57x + 8. Evaluating in x = 2, we have P (2) = 4 · 23 + 2 · 22 + 7 · 2 + 1 = 55, Q(2) = 2 2 · 2 + 2 + 8 = 18 and R(2) = 8 · 25 + 8 · 24 + 48 · 23 + 25 · 22 + 57 · 2 + 8 = 990. Solution 8.2. Each factor of P (x) is a geometric progression, then P (x) = (1 − x + x2 − · · · + x100 )(1 + x + x2 + · · · + x100 ) (−x)101 − 1 x101 − 1 x101 + 1 = −x − 1 x−1 x+1 2 101 (x ) − 1 = 1 + x2 + · · · + x200 . = x2 − 1 = x101 − 1 x−1 249 10.8 Solutions of Chapter 8 Solution 8.3. Apply the division algorithm to obtain H(x) and R(x), then x8 − 5x3 + 1 = (x5 − x4 + x3 − 2x2 + 3x − 9)(x3 + x2 + 1) + 11x2 − 3x + 10. Hence H(x) = x5 − x4 + x3 − 2x2 + 3x − 9 and R(x) = 11x2 − 3x + 10. Solution 8.4. Let P (x) = nxn+1 − (n + 1)xn + 1, evaluating in x = 1, P (1) = n − (n + 1) + 1 = 0, that is, x = 1 is a zero of P (x), then x − 1 divides P (x). In fact P (x) = (x − 1)(nxn − xn−1 − · · · − x − 1). If Q(x) = nxn − xn−1 − xn−2 − · · · − x − 1, then Q(1) = n − n = 0 implies that x − 1 divides Q(x), hence (x − 1)2 divides P (x). Solution 8.5. It is clear that P1 (x) = 1 + x has as the only root −1 and P2 (x) has roots −1 and −2. By induction, we will see that the roots of Pn (x) are −1, −2, . . . , −n. Suppose that Pn (x) has roots −1, −2, . . . , −n, then Pn (x) = (x + 1)(x + 2) · · · (x + n) . n! Hence, x(x + 1)(x + 2) · · · (x + n) (n + 1)! (x + 1)(x + 2) · · · (x + n) x(x + 1)(x + 2) · · · (x + n) + = n! (n + 1)! (x + 1)(x + 2) · · · (x + n)(n + 1) + x(x + 1)(x + 2) · · · (x + n) = (n + 1)! (x + 1)(x + 2) · · · (x + n)(x + n + 1) , = (n + 1)! Pn+1 (x) = Pn (x) + which shows that the roots of Pn+1 (x) are −1, −2, . . . , −(n + 1). Solution 8.6. Since P (0) = 0, then P (1) = P 2 (0) + P (0) + 1 = 1. Now, evaluate in x = 1, the identity P (x2 +x+1) = P 2 (x)+P (x)+1 to obtain P (3) = 3. Evaluating in x = 3, we get that P (32 + 3 + 1) = P 2 (3) + P (3) + 1 = 9 + 3 + 1 = 13, then P (13) = 13. Now, if we define xn+1 = x2n + xn + 1 and P (xn ) = xn , then P (xn+1 ) = 2 P (xn + xn + 1) = P 2 (xn ) + P (xn ) + 1 = x2n + xn + 1 = xn+1 . This process constructs an infinite number of fixed points of P (x) which are different, since xn+1 − xn = x2n + 1 > 0. But a polynomial cannot have an infinite number of fixed points unless P (x) = x. 250 Chapter 10. Solutions to Exercises and Problems Solution 8.7. Let P (x) = an xn + · · · + a1 x + a0 be a polynomial with a0 = 0. Since   n n−1 1 1 1 1 n n + an−1 + · · · + a1 x P an = x + a0 x x x x = an + an−1 xn−1 + · · · + a1 xn−1 + a0 xn ,   we have xn P x1 = P (x) if and only if an + an−1 xn−1 + · · · + a1 xn−1 + a0 xn = an xn + an−1 xn−1 + . . . + a1 x + a0 if and only if the “complementary coefficients” are equal, that is, ai = an−i , for all i = 0, . . . , n. Solution 8.8. Use the previous exercise to see that P (−1) = 0, then P (x) = (x + 1)Q(x), for a polynomial Q(x) of degree n − 1. Since (x + 1)Q(x) = P (x) = xn P it follows that Q(x) = xn−1 Q follows that Q(x) is reciprocal. 1 x = xn 1 +1 Q x 1 x , 1 x . Hence, using again the previous exercise, it Solution 8.9. A reciprocal  since an = a0 = 0,  polynomial does not have 0 as a root, then a = 0. Since an P a1 = P (a) = 0, it follows that P a1 = 0, then a1 is also a zero of P (x). Solution 8.10. If n is odd, it follows that x2n−2 + x2n−4 + · · · + x4 + x2 + 1 can be factored as (xn−1 + xn−2 + xn−3 + · · · + 1)(xn−1 − xn−2 + xn−3 − xn−4 + · · · + 1), which proves that the polynomial in the left is divisible by 1 + x + · · · + xn−1 . If n is even, −1 is a root of 1 + x + x2 + · · · + xn−1 , but it is not a root of 1 + x2 + x4 + · · · + x2n−2 . Then 1 + x2 + x4 + · · · + x2n−2 is not divisible by 1 + x + x2 + · · · + xn−1 . Solution 8.11. Suppose that n ≥ m. Recall that Euclid’s algorithm is used to find the greatest common divisor of (m, n), in the following way: n = ms1 + r1 m = r1 s2 + r2 .. .. . . rj−1 = rj sj+1 + 0, with 0 ≤ ri < ri−1 and r0 = m, hence (m, n) = rj . Notice that xn − 1 = xms1 +r1 − 1 = xr1 (xms1 − 1) + xr1 − 1 = xr1 xms1 − 1 xm − 1 (xm − 1) + xr1 − 1, 251 10.8 Solutions of Chapter 8 then the division of xn −1 by xm −1, leaves remainder xr1 −1, then (xn −1, xm −1) = (xm − 1, xr1 − 1). Proceeding in the same way, we obtain (xn − 1, xm − 1) = (xm − 1, xr1 − 1) = (xr1 − 1, xr2 − 1) = · · · = (xrj−1 − 1, xrj − 1) = xrj − 1. Therefore, (xn − 1, xm − 1) = x(n,m) − 1. Solution 8.12. The problem is equivalent to finding all pairs (m, n) such that (xmn+n − 1)(x − 1) (xm+1 − 1)(xn − 1) is a polynomial. Notice that xn − 1 and xm+1 − 1 are divisors of xmn+n − 1. These factors can only have x − 1 as a common factor, but by the previous exercise, (xm+1 −1, xn −1) = x(m+1,n) −1, therefore it will be enough to have (m+1, n) = 1. Solution 8.13. If a is an integer with P (a) = 0, then P (x) = (x − a)Q(x), for some polynomial Q(x) with integer coefficients, and P (0) = −aQ(0) and P (1) = (1 − a)Q(1). But if a is an integer, then either a or 1 − a is even, and so one of P (0) and P (1) is even, which is a contradiction. Solution 8.14. Consider the polynomial with roots x, y, z, that is, P (u) = (u − x)(u − y)(u − z) = u3 + au2 + bu + c. Then, by Vieta’s formulas, xyz , c = −xyz, a = −x − y − z = −w, b = xy + yz + zx = w therefore, b = − wc = ac . Hence, P (u) = u3 + au2 + bu + ab = (u + a)(u2 + b), and u = −a = w is a root. Without loss of generality, we can assume that it is x, that is, x + y + z = x, from where y + z = 0. This last equality implies that y = −z and, then b = −y 2 , hence the other roots are y and −y. That is, the roots are (x, y, −y) and therefore the solutions of the system are the triplets (x, y, −y) and its permutations, with x, y ∈ R. Solution 8.15. Since the coefficients of the polynomial are integers, it is enough to see that it is irreducible over Z[x]. The polynomial has no integer roots, since a root of x4 − x3 − 3x2 + 5x + 1 must divide 1, and then it must be 1 or −1, but P (1) = 3 and P (−1) = −5. Or, by Exercise 8.13, and because P (0) and P (1) are odd, P (x) = 0 has no integer solutions. Hence, if P (x) can be factored, it must be into two monic quadratic polynomials, as follows: x4 − x3 − 3x2 + 5x + 1 = (x2 + bx + c)(x2 + dx + e), with b, c, d, e integers. Equating coefficients, it follows that: b + d = −1 c + e + bd = −3 be + cd = 5 ce = 1. 252 Chapter 10. Solutions to Exercises and Problems The last identity implies c = e = 1 or c = e = −1. Now, the third identity takes the form b + d = 5 or b + d = −5. In any case, these equalities are in contradiction with the first identity. Second Solution. It is enough to see that the polynomial is irreducible over Z2 [x]. But in Z2 [x], the polynomial can be written as x4 + x3 + x2 + x + 1. It is clear that 0 and 1 are not roots of this polynomial, then if it is reducible it should be decomposed as the product of two irreducible quadratic polynomials. But the only irreducible quadratic polynomial in Z2 [x] is x2 +x+1, and (x2 +x+1)(x2 +x+1) = x4 + x2 + 1 = x4 + x3 + x2 + x + 1. This completes the proof. Solution 8.16. Consider the integers x0 = n and xk+1 = P (xk ), for k ≥ 0. If k = 1, the result is immediate. Suppose now xk = x0 , with k ≥ 2, and define di = xi+1 − xi . Since di = xi+1 − xi divides P (xi+1 ) − P (xi ) = xi+2 − xi+1 = di+1 , for all i = 0, 1, . . . , k − 1, and since dk = d0 = 0, it follows that |d0 | = |d1 | = · · · = |dk |. Suppose that d0 = d1 . In this case d2 = d0 , otherwise x3 − x2 = −(x2 − x1 ), then x3 = x1 and the sequence of iterates takes the form x0 , x1 , x2 , x1 , x2 , . . . . Hence it should not exist xk , with k ≥ 2, that coincides with x0 . Similarly, if dj = d0 , for every j, then xj = x0 + jd0 = x0 , for every j, which is a contradiction. Hence d0 = −d1 , that is, x1 − x0 = −(x2 − x1 ). Therefore x0 = x2 . Solution 8.17. Suppose that an = 1. If r1 , . . . , rn are the roots, by Vieta’s formulas it follows that r12 r22 · · · rn2 = 1 and # r12 + r22 + · · · + rn2 = (r1 + · · · + rn )2 − 2 ri rj = a2n−1 − 2an−2 ≤ 3. 1≤i<j≤n The inequality between the arithmetic and the geometric mean guarantees that  r12 + r22 + · · · + rn2 ≥ n n r12 r22 · · · rn2 = n. Both inequalities imply that n ≤ 3. In the case n = 3, from the eight polynomials of the form x3 ± x2 ± x ± 1, the only ones that have three roots are x3 − x ± (x2 − 1) = (x2 − 1)(x ± 1). In the case n = 2, only the polynomials x2 ± x − 1 have its two roots real. In the case n = 1, the only polynomials are x ± 1. Solution 8.18. If P (a) = b, P (b) = c and P (c) = a, then P (P (P (a))) = P 3 (a) = a, by Exercise 8.16, and P 2 (a) = a. But on the other hand, P (P (a)) = P (b) = c, then c = a, which is a contradiction. Solution 8.19. Suppose that r1 , . . . , rn are all the roots of P (x) and that all are real; by Vieta’s formulas it follows that n # i=1 ri = −2n and # 1≤i<j≤n ri rj = 2n2 . (10.11) 253 10.8 Solutions of Chapter 8 * 2 On the other hand, the Cauchy–Schwarz inequality guarantees that ( ni=1 ri ) ≤ * * *n 2 *n 2 n n −1 2 2 i=1 ri ) . Hence i=1 1 , and then − i=1 ri ≤ n ( i=1 ri # 1≤i<j≤n 1 ri rj = 2 ≤  n # ri i=1 1 1 − 2 2n 2 n 1# 2 r 2 i=1 i  n 2 # n−1 = ri (−2n)2 = 2n(n − 1), 2n i=1 − contradicting equation (10.11). Solution 8.20. Calculate the derivative of the polynomial P (x), that is, P ′ (x) = 3x2 − 2x − 8, which has roots x = 2 and x = − 43 . Since P (2) = 0, then 2 is a multiple root. Solution 8.21. The fact that P (x) is divisible by (x + 1)2 is equivalent to the fact that −1 must be a root of multiplicity at least 2 of P (x), that is, P (−1) = P ′ (−1) = 0. This is equivalent to −1 + a − b + c = 0 3 − 2a + b = 0. Then, the solution’s triplets are (a, b, c) = (t, 2t − 3, t − 2), with t ∈ R. Solution 8.22. The polynomial of the Lagrange interpolation formula is P (x) = = n # k=0 n # (−1)n−k 2k x(x − 1)(x − 2) · · · (x − k + 1)(x − k − 1) · · · (x − n) k(k − 1) · · · (k − k + 1)(k − k − 1) · · · (k − n) (−1)n−k 2k x(x − 1)(x − 2) · · · (x − k + 1)(x − k − 1) · · · (x − n) . k! (n − k)! k=0 Then, P (n + 1) n # (n + 1)n · · · (n + 1 − (k − 1))(n + 1 − k − 1) · · · (n + 1 − n) = (−1)n−k 2k k! (n − k)! k=0 = = n # k=0 n # k=0 n (−1)n−k 2k # (n + 1)! (n + 1)! = (−1)n−k 2k k! (n − k)!(n + 1 − k) k! (n + 1 − k)! k=0 (−1)n−k 2k n+1 . k 254 Chapter 10. Solutions to Exercises and Problems On the other hand, 1 = (2 − 1)n+1 = n+1 # k=0 n+1 k 2k (−1)n+1−k = −P (n + 1) + 2n+1 . Therefore, P (n + 1) = 2n+1 − 1. √ √ Solution 8.23. (i) First observe that x ∈ [− 5, 5], since the left-hand side is non-negative. Squaring both sides of the equation and rearranging, we get x4 − 10x2 + x + 20 = 0, which can be factorized as (x2 + x − 5)(x2 − x − 4) = 0. Then, x2 + x − 5 = x2 −x − 4 = 0. The solutions of these are, for √   0 or √  equations 1 the first x1,2 = 2 −1 ± 21 and for the second x3,4 = 12 1 ± 17 . Only two √ √ of them are in the interval [− √5,  5], and therefore the solutions of the equation √    are 12 −1 + 21 and 12 1 − 17 . (ii) As in the previous part, squaring both sides of the equation, it follows that x4 − 2ax2 + x + a2 − a = 0, which is a quadratic equation in a, a2 − (2x2 + 1)a + x4 + x = 0. The discriminant of the quadratic equation is (2x − 1)2 , so that the roots of the equation are a1 = x2 + x and a2 = x2 − x + 1. It follows that, a2 − (2x2 + 1)a + x4 + x = (a − x2 − x)(a − x2 + x − 1) = 0. 2 Now, solving the quadratic equations x2 + x − √ a = 0 and x − x + 1 − a = 0, √ 1± 1+4(a−1) give us the roots x = −1± 2 1+4a and x = . We can show that the √2 √ √ √ −1− 1+4a −1+ 1+4a root is not. On the other hand, 2 √ is always in [− a, a] while 2 √ √ 1+ 1+4(a−1) 3 would be in [− a, a], if 4 ≤ a ≤ 1 and the root x = the root 2 √ √ √ 1− 1+4(a−1) belongs to [− a, a] only when a ≥ 1. 2 √ Solution 8.24. It is clear that x ≥ 0, a + x ≥ 0 and a − a + x ≥ 0. Squaring both sides of the equation leads to √ √ a + x = a − x2 . x2 = a − a + x which is equivalent to 255 10.8 Solutions of Chapter 8 Then, a − x2 ≥ 0, and after squaring, it follows that a + x = (a − x2 )2 , which is equivalent to the equation P (x) = x4 − 2ax2 − x + a2 − a = 0. In order to solve this equation, consider a as the variable and x as the parameter, then a2 − (2x2 + 1)a + x4 − x = 0. The discriminant of the equation is (2x2 + 1)2 − 4(x4 − x) = (2x + 1)2 , then the roots of the equation are a1 = x2 − x and a2 = x2 + x + 1. This implies that we can factorize P (x) as P (x) = (x2 − x − a)(x2 + x + 1 − a). The positive roots of the equation x2 − x − a = 0 do not satisfy the condition √ a − x2 ≥ 0, since a − x2 = −x. The roots of x2 + x + 1 − a = 0 are x1 = −1+ 24a−3 √ and x2 = −1− 24a−3 . Only x1 can be non-negative, and this happens when a ≥ 1. Solution 8.25. First, observe that x is a positive number. Taking conjugates on the left-hand side of the equation leads to √ √ 2 x m x √ √ = √ , x+ x+ x− x x+ x √ then m is also a positive number. Dividing by x and simplifying the equation, √ √ gives us (2 − m) x + √ x = m x√− x. Then 2 − m ≥ 0, and hence, √ after squaring, (2−m)2 (x+ x) = m2 (x− x) which is equivalent to (2−m)2 ( x+1) = √ √ √ 2 −2m+2 m2 ( x − 1). Solving for x, we get x = m2(m−1) , hence m > 1 and the solution to the equation is x = (m2 −2m+2)2 4(m−1)2 , for 1 < m ≤ 2. Solution 8.26. From the first equation, we get x= y + 76 − y + 11 which is equivalent to x( y + 76 + y + 11) = 65. Then, 65 √ x= √ . y + 76 + y + 11 65√ 65√ and z = √x+76+ . Without loss of Similarly, we can obtain y = √z+76+ z+11 x+11 generality, we can assume that x ≤ y ≤ z. From these inequalities and since the √ function x is increasing, it follows that √ √ √ √ x + 76 + x + 11 ≤ y + 76 + y + 11 ≤ z + 76 + z + 11. 256 Chapter 10. Solutions to Exercises and Problems Then, taking inverses and multiplying by 65, it follows that y ≤ x ≤ z. Hence, x = y. From this equality, we obtain 65 65 √ √ = √ = z. x= √ y + 76 + y + 11 x + 76 + x + 11 Thus x = y = z. Then, in order to find the triplets, we need to find the solutions to the equation √  √ x x + 76 + x + 11 = 65, (10.12)  √ √ where x is a positive real number. Since the function x x + 76 + x + 11 is monotone increasing for positive real numbers, there is at most one solution to the equation (10.12). Since 5 is a solution, the only triplet that solves the system is (5, 5, 5). Solution 8.27. Consider a, b and c as the unknowns, then we have a system of linear equations. Multiplying the first equation by x and then by y, next substituting cx in the second equation and cy in the last equation, we obtain 1 1 − xz y 1 1 a(xy − z) − b(y 2 + 1) = − − . x yz a(x2 + 1) − b(xy + z) = (10.13) (10.14) Now, multiply equation (10.13) by y 2 + 1 and equation (10.14) by −(xy + z) and, add them. It follows that a(x2 + y 2 + z 2 + 1) = x2 + y 2 + z 2 + 1 , xz 1 1 1 hence a = xz . Similarly, we obtain b = yz and c = xy . Since a, b, c are positive, x, 1 1 y, z must have the same sign. Therefore, abc = (xyz)2 , hence xyz = ± √abc . That is, the solutions to the system are √ b a c ,√ ,√ abc abc abc and a c b , −√ , −√ −√ abc abc abc . Solution 8.28. Let Q(x) = xk + a1 xk−1 + a2 xk−2 + · · · + ak−1 x + ak and P (x) = x2 + px + q. Consider the equality (xk + a1 xk−1 + · · · + ak )2 + p(xk + a1 xk−1 + · · · + ak ) + q −(x2 + px + q)k − a1 (x2 + px + q)k−1 − · · · − ak = 0. Calculating the coefficients of x2k , x2k−1 , . . . , x1 , x0 , we get a system of equations for a1 , a2 , . . . , ak , p, q. It is quite difficult to write this system, and to solve it even more. However, some useful observations can be made without solving the system. 257 10.8 Solutions of Chapter 8 When the polynomials are expanded, notice that the coefficients b1 , b2 , . . . , bk of the powers of x2k−1 , x2k−2 , . . . , xk , are b1 = 2a1 + R1 (p, q) = 0, b2 = 2a2 + R2 (p, q, a1 ) = 0, .. .. . . bk = 2ak + Rk (p, q, a1 , . . . , ak−1 ) = 0, where each Ri is some algebraic expression in terms of p, q, a1 , . . . , ak−1 . The first of these equations implies that a1 can be expressed in terms of p and q; the second equation implies that a2 can be expressed in terms of p, q and a1 , and therefore, only in terms of p and q. Similarly, we can conclude that all coefficients of the polynomial Q(x) that commute with P (x) can be expressed in a unique way in terms of p and q, which is what we wanted to prove. Solution 8.29. First, let us see that the polynomial Q(x) = P (P (x)) commutes with P (x). In fact, Q(P (x)) = P (P (P (x))) = P (Q(x)). This polynomial Q(x) has degree 4 and, by the previous exercise, it is the only polynomial of degree 4 that commutes with P (x). Similarly, it can be shown that the only polynomial of degree 8 that commutes with P (x) is R(x) = P (P (P (x))). Solution 8.30. Let S(x) = Q(R(x)) and T (x) = R(Q(x)). Since P (x) commutes with both Q(x) and R(x), it follows that P (S(x)) = P (Q(R(x))) = Q(P (R(x))) = Q(R(P (x))) = S(P (x)). Therefore, P (x) commutes with S(x). Similarly, it can be shown that P (x) commutes with T (x). Since S(x) and T (x) are monic polynomials of the same degree (if Q(x) and R(x) have degrees k and l, respectively, then S(x) and T (x) have degrees kl), by Exercise 8.28, it follows that S(x) = T (x), that is, Q(R(x)) = R(Q(x)). Solution 8.31. Let P (x) = ax + b and Q(x) = cx + d. The condition P (Q(x)) = Q(P (x)) implies that acx + ad + b = acx + bc + d, that is, d(a − 1) = b(c − 1), and from here we proceed by cases. First, if a = 1, then b(c − 1) = 0, and b = 0 or c = 1. If b = 0, it follows that P (x) = x and Q(x) = cx + d commute and they have the common fixed point d . Now, if c = 1, then P (x) = x + b and Q(x) = x + d, which clearly commute. − c−1 Second if a = 1, then d = b(c−1) a−1 and we have the polynomials, P (x) = ax + b, b and Q(x) = cx + b(c−1) with fixed point a−1 with fixed point − a−1 , if c = 1. If c = 1, then d = 0, hence P (x) = ax + b and Q(x) = x, a case that was already considered. b , − a−1 It is clear that if P (x) = x + α and Q(x) = x + β, then they commute. Now, if there is x0 such that P (x0 ) = Q(x0 ) = x0 , then P (x) = a(x − x0 ) + x0 and Q(x) = b(x − x0 ) + x0 . A direct calculation proves that the last two polynomials commute, which is the end of the proof. 258 Chapter 10. Solutions to Exercises and Problems Solution 8.32. Notice that Pa (Qa (x)) = Pa (Q(x − a) + a) = P (Q(x − a) + a − a) + a = P (Q(x − a)) + a. Similarly, Qa (Pa (x)) = Q(P (x − a)) + a. Then, Pa (x) and Qa (x) commute if and only if P (Q(x − a)) = Q(P (x − a)), which is the case. Solution 8.33. The system can be rewritten as x5 + y 5 = σ15 − 5σ13 σ2 + 5σ1 σ22 = 33 σ1 = 3. Substituting the value of σ1 in the last equation, we obtain the equation 15σ22 − 5 · 27σ2 + 9 · 27 = 33. Simplifying the equation, we get σ22 − 9σ2 + 14 = 0. The solutions are σ2 = 2 and σ2 = 7. Now, in order to obtain the values that we are looking for, we need to solve, for σ2 = 2 and σ2 = 7, the system x + y = 3 and xy = σ2 . √ √ Solution 8.34. Define y = 4 x and z = 4 97 − x, then the equation can be rewritten as y + z = 5. We have to solve the system of equations y+z =5 4 y + z 4 = 97 − x + x = 97. Now, y 4 + z 4 = σ14 − 4σ12 σ2 + 2σ22 = 97. Substituting the value of σ1 , we have to solve the quadratic equation 2σ22 − 100σ2 + 54 = 97. The solutions are σ2 = 44 and σ2 = 6. To obtain the values of y and z, we must find the solutions to the systems of equations y+z =5 y+z =5 yz = 44, yz = 6. Solution 8.35. We have x3 + y 3 = σ13 − 3σ1 σ2 = c. (10.15) From the second equation, we have σ12 − 2σ2 = b. Solving for σ2 , we get σ2 = since σ1 = a. Substituting in equation (10.15) a3 − 3a a2 − b 2 =c a3 − 3ab + 2c = 0. a2 −b 2 , 259 10.8 Solutions of Chapter 8 Solution 8.36. Substituting the value of z 2 in the second equation, it follows that x2 + y 2 + xy = b2 . From the first equation, we get x + y − a = −z. (10.16) Squaring both sides leads to (x + y)2 − 2a(x + y) + a2 = z 2 (x + y)2 − 2a(x + y) + a2 = xy 2 2 (10.17) 2 x + xy + y − 2a(x + y) = −a . Since x2 + y 2 + z 2 = x2 + y 2 + xy = b2 , substituting in equation (10.17) and 2 +b2 . Substituting this value in (10.16), we solving for x + y, results in x + y = a 2a 2 2 2 2 2 2 2 −b ) +b −b , that is, xy = (a 4a . Observe, since z and a and x + y = a 2a get z = a 2a 2 2 2 are positive, that a > b . By Vieta’s formulas, x and y are roots of the equation w2 − a2 + b 2 2a w+ (a2 − b2 )2 = 0. 4a2 (10.18) But the solutions of equation (10.18) are a2 +b2 2a w1,2 = ±  a2 +b2 2 2a −4 $ (a2 −b2 )2 4a2 % . 2 Since we are looking for real solutions, the discriminant must be positive, that is ∆= a2 + b 2 2a 2 − 1 4(a2 − b2 )2 = 2 (3a2 − b2 )(3b2 − a2 ) > 0. 4a2 4a Since 3a2 > b2 , then 3b2 − a2 > 0. Hence, 3b2 > a2 > b2 , which means |b| < a < √ 3|b|. Solution 8.37. Take σ1 = x + y + z, σ2 = xy + yz + zx and σ3 = xyz. Then 1 2 (a − b2 ), σ3 = 2 where the third equation was obtained from the tion (4.8). Solve now, σ1 = a, σ2 = 1 a(a2 − b2 ), 2 given factorization in equa- 1 1 u3 − au2 + (a2 − b2 )u − a(a2 − b2 ) = 0 2 2 + , 1 2 2 2 (u − a) u + (a − b ) = 0. 2   2 2 b2 −a2 Its solutions are u1 = a, u2 = b −a and u = − 3 2 2 . 260 Chapter 10. Solutions to Exercises and Problems Solution 8.38. The equation is equivalent to xy + x2 y 2 = 3xy 2 + 3x2 y, which is equivalent to xy(1 + xy − 3x − 3y) = 0. If (x, y) = (m, 0) or (x, y) = (0, m), then they are solutions for any integer m. If xy = 0, 1 + xy − 3x − 3y = 0. Since x = 3 or y = 3 does not satisfy the equation, then we can divide by x − 3 or y − 3. Solving for y, y= 8 3x − 1 =3+ , x−3 x−3 so that y is an integer if x − 3 divides 8. Then, x − 3 = ±1, x − 3 = ±2, x − 3 = ±4 and x − 3 = ±8. Therefore, x ∈ {−5, −1, 1, 2, 4, 5, 7, 11}, that is, the solutions are (m, 0), (0, m), for any integer m, and (x, y) ∈ {(−5, 2), (2, −5), (−1, 1), (1, −1), (4, 11), (11, 4), (5, 7), (7, 5)}. Solution 8.39. Let k be a fixed number and consider the solutions of the equation (a, b), with a, b ∈ N, a ≥ b, and from this set of solutions choose the one for which a + b is minimum. 2 and, since a is a If we can show that a = b, then 1 + a1 = k2 , hence a = k−2 positive integer, k = 3 or 4. Let us see that b = a in the following way. Suppose a > b and notice that a is a b+1 solution of the equation x+1 b + x = k, or equivalently, it is a root of the quadratic 2 2 equation x − (kb − 1)x + b + b = 0. If a1 is the other root, by Vieta’s formulas, it 2 follows that a+a1 = kb−1 and aa1 = b2 +b. Hence, a1 +b = b a+b +b. Since a > b, it 2 follows that a ≥ b+1 so that b ≥ b a+b = a1 . Therefore, the pair (b, a1 ) is a solution of the original equation. Since (a, b) is the solution with minimum sum a + b, it 2 follows that a+b ≤ b+a1 = b+ b a+b so that b2 +b ≥ a2 ≥ (b+1)2 = b2 +2b+1, which implies b + 1 ≤ 0. But this contradicts the fact that b is positive, therefore a = b. Solution 8.40. Consider the case k > 3. Suppose that the integers a, b, c satisfy equation a2 + b2 + c2 = kabc; then at least one of them is positive, and the other two are both positive or both negative. In the negative case, we can change the sign to both numbers to obtain a solution where all are positive. Then, without loss of generality, suppose that all three numbers are positive. Now, let us prove that the three numbers are distinct. Suppose that the previous statement is false, for instance a = b. Then 2a2 + c2 = ka2 c, that is, c2 = a2 (kc − 2). Therefore, kc − 2 is a perfect square and then there is an integer number d ≥ 1 such that kc = 2 + d2 . Substituting the value of kc in the equation 2a2 + c2 = ka2 c, it follows that c2 = d2 a2 or c = da. Now, d2 = kc − 2 = k(da) − 2, hence 2 = d(ka − d) and this implies d divides 2, that is, d = 1 or 2. In both cases ka = 3, contradicting the fact that k > 3. Then, suppose that a > b > c ≥ 1. The triplet (kbc − a, b, c) is also a solution of the equation x2 + y 2 + z 2 = kxyz, with kbc − a a positive integer, since a(kbc − a) = b2 + c2 and a > 0. 10.9 Solutions of Chapter 9 261 Consider now the polynomial P (x) = x2 − (kbc)x + b2 + c2 . The roots of this polynomial are a and kbc − a, moreover P (b) = 2b2 + c2 − kb2 c ≤ 2b2 + c2 − kb2 < 3b2 − kb2 = (3 − k)b2 < 0. The interval where P (x) is negative is the interval with end points the roots of the polynomial. Then b is between a and kbc − a, and since b < a, it follows that kbc − a < b. Hence max (kbc − a, b, c) = b < a = max (a, b, c). Repeating this construction, we obtain a decreasing sequence of positive integers, which is something impossible, then there are no solutions of the original equation for k > 3. Now, let k = 2. Suppose that a2 + b2 + c2 = 2abc, where a, b, c are integers. Since a2 + b2 + c2 is even, not all numbers a, b, c are odd. If exactly one of them is even, reducing modulo 4, we get 2 ≡ 0 (mod 4), a contradiction. Therefore, the three numbers are even, and they can be written as a = 2a′ , b = 2b′ and c = 2c′ , so that a′2 +b′2 +c′2 = 4a′ b′ c′ . The last equation is the case k = 4, which has no integer solutions except (0, 0, 0), and from this we get (a, b, c) = (2a′ , 2b′ , 2c′ ) = (0, 0, 0). For k = 3, a solution is (1, 1, 1) and, for k = 1, consider multiples of 3 to reduce it to the case k = 3. 10.9 Solutions to problems of Chapter 9 √ Solution 9.1. Note that b = a2 +2a+1 = (a+1)2 ∈ Q and b > 0, then a = −1± b. On the other hand, √ √ a3 − 6a = (−1 ± b)3 − 6(−1 ± b) √ √ √ = −1 ± 3 b − 3b ± b b + 6 ∓ 6 b √ = 5 − 3b ± (b − 3) b. √ Since a3 − 6a and 5 − 3b are√rational numbers, we have (b − 3) b ∈ Q. If b = 3, √ b ∈ Q and, √ then a = −1 ± b ∈ Q is a contradiction. Therefore b = 3 and then a = −1 ± 3. √ Moreover, it is clear that if a = −1± 3, the numbers a2 +2a = 2 and a3 −6a = −4 are rational. Solution 9.2. First, we make the following substitution to simplify the notation, a = 3 pq 2 , b = 3 qr2 and c = 3 rp2 . We have to prove that a1 + 1b + 1c = ab+bc+ca abc 262 Chapter 10. Solutions to Exercises and Problems is a rational number. Since abc = pqr is a rational number, it is enough to prove that ab + bc + ca is a rational number. Notice that (a + b + c)3 = a3 + b3 + c3 + 3(ab + bc + ca)(a + b + c) − 3abc. (10.19) Since a + b + c is a rational number, so is (a + b + c)3 and clearly a3 = pq 2 , b3 = qr2 and c3 = rp2 are rational numbers. It is now straightforward, from equation (10.19) and since a + b + c = 0, that ab + bc + ca is a rational number. Solution 9.3. Since a = a(2 − a) − a(1 − a), we have that the numbers a(2 − a) and a(1 − a) cannot be both rational numbers, then one of them is an irrational number; this one will define the number b, that is, b is 2 − a or 1 − a. Notice that a + (2 − a) = 2 and a + (1 − a) = 1 are rational numbers, and a(2 − a) or a(1 − a) is an irrational number (if −a(1 − a) is an irrational number, then also a(1 − a) is an irrational number).     Similarly, since a1 = a + a2 − a + a1 , the numbers a + a2 and a + a1 cannot be both rational numbers; then one of them is an irrational number and b′ is one of the numbers a2 or a1 . Notice that, ab′ = 1 or 2 is a rational number and a + b′ = a + a2 or a + a1 is an irrational number. Solution 9.4. For each i, there exist among 1, 2, . . . , m, ⌊m/xi ⌋ multiples of xi . None of them is a multiple of xj for j = i, since the least common multiple of xi and xj is greater than m. Then, there exist ⌊m/x1 ⌋ + ⌊m/x2 ⌋ + · · · + ⌊m/xn ⌋ different numbers in {1, 2, . . . , m}, and these numbers are divisible by some of the numbers x1 , x2 , . . . , xn . None of these last numbers can be 1 (unless n = 1, and in this case, the result is immediate). Therefore,       m m m + + ··· + ≤ m − 1. x1 x2 xn Since m xi < ⌊ xmi ⌋ + 1 for each i, we have m 1 1 1 + + ···+ x1 x2 xn < m + n − 1. If we prove that n ≤ (m + 1)/2, then 1 3 1 1 n−1 < . + + ···+ <1+ x1 x2 xn m 2 To prove that n ≤ (m + 1)/2, observe that the largest odd divisors of x1 , x2 , . . . , xn are all different, because otherwise, if two numbers have the same greatest odd divisor, one of them will be a multiple of the other, which will be a contradiction to the hypothesis. Therefore, n is less than or equal to the quantity of odd numbers among 1, 2, . . . , m, and the inequality follows. 263 10.9 Solutions of Chapter 9 √ Solution 9.5. Let m = ⌊ n⌋ and a = n − m2 . We have m ≥ 1, since n ≥ 1. Now, from n2 + 1 = (m2 + a)2 + 1 ≡ (a − 2)2 + 1 mod (m2 + 2), it follows that the condition of the problem is equivalent to the fact that (a − 2)2 + 1 is divisible by m2 + 2. Since 0 < (a − 2)2 + 1 ≤ max{22 , (2m − 2)2 } + 1 ≤ 4m2 + 1 < 4(m2 + 2), then (a − 2)2 + 1 = k(m2 + 2), for k = 1, 2 or 3. Now, we prove that none of these cases occur. Case 1. When k = 1, we have (a − 2)2 − m2 = 1, and this implies that a − 2 = ±1 and m = 0, but this contradicts the fact that m ≥ 1. Case 2. When k = 2, we have (a − 2)2 + 1 = 2(m2 + 2), but any perfect square is congruent to 0, 1, 4 modulo 8, and therefore (a − 2)2 + 1 ≡ 1, 2, 5 mod 8, meanwhile 2(m2 + 2) ≡ 4, 6 mod 8, then this case does not occur. Case 3. When k = 3, we have (a − 2)2 + 1 = 3(m2 + 2). Since any perfect square is congruent to 0, 1 modulo 3, we have (a − 2)2 + 1 ≡ 1, 2 mod 3, meanwhile 3(m2 + 2) ≡ 0 mod 3, then this case is also impossible. Solution 9.6. It is easy to prove that when a = 0 or b = 0 or a = b or a and b are both integers, the identity follows. Suppose now that a, b do not accomplish any of the above conditions. We a have, for n = 1, that ab = ⌊a⌋ ⌊b⌋ , then b is a rational number different from zero. p a Suppose that b = q , with (p, q) = 1. If p is different from 1 and −1, then p divides ⌊na⌋, for all n, in particular it divides ⌊a⌋, therefore a = kp + ǫ, for some k ∈ N and 0 ≤ ǫ < 1. Notice that ǫ = 0, otherwise a = kp and b = kq = ⌊b⌋ are integers. Then, since there exists n ∈ N, with 1 ≤ nǫ < 2, we have that ⌊na⌋ = ⌊knp + nǫ⌋ = knp + 1 is not divisible by p, which is a contradiction. Similarly, it cannot happen for q to be different from 1 and −1. Therefore p, q ∈ {±1}, but since a = b, we have that b = −a, then ⌊−a⌋ = − ⌊a⌋, which is only possible if a is an integer number. Therefore, there are no more pairs of numbers (a, b) that satisfy the conditions the problem. Solution 9.7. As we proved in Example 1.3.3, we have    n   2n   3n  (m − 1)n (m − 1)(n − 1) + , + + ··· + = m m m m 2 and we want to find the value of the sum  4  n   2n 4  3n 4 (m − 1)n + + + ···+ = X. m m m m Adding both equations, term by term, we get 2n 3n (m − 1)n (m − 1)(n − 1) n + + + ···+ =X+ , m m m m 2 (10.20) 264 Chapter 10. Solutions to Exercises and Problems n on the left-hand side of the equation (10.20) and using Gauss addifactorizing m tion formula, we have that the sum on the left-hand side is (m − 1)m 2 n n (1 + 2 + · · · + (m − 1)) = m m = n(m − 1) . 2 Replacing this value in equation (10.20) and solving for X, we have X= n(m − 1) (m − 1)(n − 1) m−1 (m − 1) − = (n − n + 1) = . 2 2 2 2 Solution 9.8. Since (a + b + c)3 = a3 + b3 + c3 + 3(a + b)(b + c)(c + a), it follows that 1 1 √ +1 + √ 3 3 a b 3 = 1 1 √ +√ 3 3 a b 1 1 + +1+3 a b 1 √ +1 3 b 1 1+ √ 3 a = 10 + 3(18) = 64, therefore 1 √ 3 a + 1 √ 3 b = 3, then the solutions are (a, b) = 1 8, 1   1 , 1, 8 . Solution 9.9. From the first identity, we get a − b = 1c − 1b = b−c bc . Similarly, a−b a−b c−a b − c = ca and c − a = ab . Therefore a − b = (abc)2 and, since a and b are different, we have (abc)2 = 1, so that abc = ±1. It is clear that the numbers (a, b, c) = (1, − 12 , −2) and (a, b, c) = (−1, 12 , 2) satisfy the identities and, with these triplets, we obtain the two possible values of abc. 1 Solution 9.10. Since (a+ b)(a+ c) = a(a+ b + c)+ bc = bc + bc ≥ 2 and the equality holds when bc = 1,√ it follows that the minimum value is 2, and it is reached when b = c = 1 and a = 2 − 1. Solution 9.11. The equation is equivalent to (bc+a)(ca+b)(ab+c) = (ab+bc+ca− abc)2 . We expand the equation and cancel out terms to obtain abc(a2 +b2 +c2 +1) = 2abc(a + b + c) − 2abc(ab + bc + ca), which is equivalent to (a + b + c − 1)2 = 0. Therefore, a + b + c = 1. Solution 9.12. First prove that b−c c−a a−b + + a b c =− (b − c)(c − a)(a − b) abc and do the same for a b c + + b−c c−a a−b = −9 Therefore, the value we are looking for is 9. abc . (b − c)(c − a)(a − b) 265 10.9 Solutions of Chapter 9 Solution 9.13. Notice that a 1 1 b c 1 + + + + b−c c−a a−b b−c c−a a−b a b c = + + (b − c)2 (c − a)2 (a − b)2 a+b b+c c+a + + + (b − c)(c − a) (c − a)(a − b) (a − b)(b − c) 0= and that a+b b+c c+a + + = 0. (b − c)(c − a) (c − a)(a − b) (a − b)(b − c) Hence, the value we are looking for is 0. Solution 9.14. Notice that a2 + 1 = a2 + ab + bc + ca = (a + b)(a + c). Similarly, b2 + 1 = (b + c)(b + a) and c2 + 1 = (c + a)(c + b). Then (a2 + 1)(b2 + 1)(c2 + 1) = ((a + b)(b + c)(c + a))2 . Solution 9.15. The condition a1 + 1b + 1c = 0, implies that abc = 0 and ab+bc+ca = 0. Since (a+b+c)2 = a2 +b2 +c2 +2(ab+bc+ca), it is clear that a2 +b2 +c2 = (a+b+c)2 . Solution 9.16. Notice that the identity of the hypothesis implies that ab+bc+ca = a2 a a2 = a2 −2(ab+ca) = 3a−r , where r = 2(a + b + c). Then, 0, so a2 +2bc a2 a2 b2 c2 a b c + 2 + 2 = + + + 2bc b + 2ca c + 2ab 3a − r 3b − r 3c − r 3 = since (3a − r)(3b − r)(3c − r) = 27abc + From the equality a2 a2 +2bc a2 27abc + r2 = 1, (3a − r)(3b − r)(3c − r) r3 2 . bc + 2 a2 +2bc = 1, we can also conclude that bc ca ab + 2 + 2 = 1. + 2bc b + 2ca c + 2ab Solution 9.17. Notice that a+1 b+1 c+1 + + ab + a + 1 bc + b + 1 ca + c + 1 b+1 c+1 1 c+1 a(1 + bc) + + =1+ + = a(b + 1 + bc) bc + b + 1 ca + c + 1 bc + b + 1 ca + c + 1 1 c+1 1 + b(c + 1) =1+ + =1+ = 2. b(c + 1 + ca) ca + c + 1 b(c + 1 + ca) 266 Chapter 10. Solutions to Exercises and Problems Solution 9.18. The equation is equivalent to a2 − 2ab + b2 − ac + bc = 0. Defining y = a − b and factorizing, we obtain y 2 − cy = 0. The roots of this quadratic equation are 0 and c, but 0 is not possible, since a and b are different. Hence, a − b = c. Solution 9.19. If the numbers x1 , . . . , xn solve the system, then 1 0 = x1 + x22 + · · · + xnn − n − x1 + 2x2 + · · · + nxn − n(n + 1) 2 = (x22 − 2x2 + 2 − 1) + (x33 − 3x3 + 3 − 1) + · · · + (xnn − nxn + n − 1). But, using the inequality between the geometric and the arithmetic mean, for each k ≥ 2 and x ≥ 0, we have √ k xk + k − 1 = xk + 1 + · · · + 1 ≥ k xk = kx, with equality if and only if x = 1.   Then, since each term of the sum xkk − kxk + k − 1 ≥ 0 and the total sum is zero, we have that each term of the sum is zero, and this happens if each xk = 1. Then, x2 = · · · = xn = 1 and, recalling the first equation, we also have that x1 = 1. Solution 9.20. Observe that 1+ 1 x 1+ 1 y ≥9 ⇔ (x + 1)(y + 1) ≥ 9xy ⇔ 2 ≥ 8xy ⇔ (x + y)2 ≥ 4xy ⇔ (x − y)2 ≥ 0. Solution 9.21. The inequality is equivalent to x3 + y 3 + z 3 − 3xyz ≥ 9 |(x − y)(y − z)(z − x)|. 4 But since x3 + y 3 + z 3 − 3xyz = it is enough to prove that " ! 1 (x + y + z) (x − y)2 + (y − z)2 + (z − x)2 , 2 " 9 ! 1 (x + y + z) (x − y)2 + (y − z)2 + (z − x)2 ≥ |(x − y)(y − z)(z − x)|. 2 4 Let p = |(x − y)(y − z)(z − x)|, using the inequality between the geometric and the arithmetic mean, we have that (x − y)2 + (y − z)2 + (z − x)2 ≥ 3 3 p2 . (10.21) 267 10.9 Solutions of Chapter 9 Now, since |x − y| ≤ x + y, |y − z| ≤ y + z, |z − x| ≤ z + x, we get 2(x + y + z) ≥ |x − y| + |y − z| + |z − x|. Applying again the inequality between the geometric and the arithmetic mean, we obtain √ (10.22) 2(x + y + z) ≥ 3 3 p. Hence, the result follows from inequalities (10.21) and (10.22). Solution 9.22. If a ≥ 1, since b + c > a ≥ 1, we have that ab + bc + ca = a(b + c) + bc > 1 + bc > 1, which is a contradiction to the fact that ab+bc+ca = 1, therefore a < 1. Similarly, we can prove that b < 1 and c < 1. Thus, (1 − a)(1 − b)(1 − c) > 0, then 1 + ab + bc + ca > a + b + c + abc. Adding 2 on both sides of the last inequality, and using the fact that ab+bc+ca = 1, we obtain 3 + ab + bc + ca > 2 + a + b + c + abc, that is, 4 > 1 + a + b + c + ab + bc + ca + abc = (a + 1)(b + 1)(c + 1). Solution 9.23. Notice that a+b+c− abc (a + b)(b + c)(c + a) = ab + bc + ca ab + bc + ca 2 a + b2 + c2 + 3(ab + bc + ca) , = ab + bc + ca 1 1 1 + b+c + a+c = 1 is equivalent the last equality is valid, since the condition a+b to (a + b)(b + c)(c + a) = (a + b)(b + c) + (b + c)(c + a) + (c + a)(a + b) = a2 + b2 + c2 + 3(ab + bc + ca). Then, the result to be proved is equivalent to a2 + b2 + c2 ≥ ab + bc + ca, which is valid using the inequality between the geometric mean and the arithmetic mean. Solution 9.24. Since the expression is a symmetric function in a, b and c, we can assume, without loss of generality, that c ≤ b ≤ a. In such a case, a (b + c − a) ≤ b (a + c − b) ≤ c (a + b − c). For example, the first inequality can be justified as follows: a (b + c − a) ≤ b (a + c − b) ⇔ ab + ac − a2 ≤ ab + bc − b2 ⇔ (a − b) c ≤ (a + b) (a − b) ⇔ (a − b) (a + b − c) ≥ 0. 268 Chapter 10. Solutions to Exercises and Problems By the rearrangement inequality, see Example 7.3.6, we have a2 (b + c − a) + b2 (c + a − b) + c2 (a + b − c) 2 ≤ ba(b + c − a) + cb(c + a − b) + ac(a + b − c) a (b + c − a) + b2 (c + a − b) + c2 (a + b − c) ≤ ca(b + c − a) + ab(c + a − b) + bc(a + b − c). ! " Therefore, 2 a2 (b + c − a) + b2 (c + a − b) + c2 (a + b − c) ≤ 6abc. Solution 9.25. Squaring and rearranging the given inequality, it is equivalent to n # i=1 But, since *n x2i − 2 i=1 yi2 = n # xi yi + i=1 *n 2 i=1 zi , n # i=1 yi2 ≤ n # i=1 x2i − 2 n # xi zi + i=1 n # zi2 . i=1 the inequality we have to prove is equivalent to n # i=1 xi zi ≤ n # xi yi , i=1 which is the rearrangement inequality (see Example 7.3.6). Solution 9.26. Let (a1 , a2 , . . . , an ) be a permutation of (x1 , x2 , . .%. , xn ) with a1 ≤ $ 1 1 1 , that is, bi = a2 ≤ · · · ≤ an , and let (b1 , b2 , . . . , bn ) = n2 , (n−1) 2 , · · · , 12 1 (n+1−i)2 , for the indices i = 1, . . . , n. Consider the permutation (a′1 , a′2 , . . . , a′n ) of (a1 , a2 , . . . , an ), defined by a′i = xn+1−i , for i = 1, . . . , n. Using the rearrangement inequality (see Example 7.3.6), we have x2 xn x1 + 2 + · · · + 2 = a′1 b1 + a′2 b2 + · · · + a′n bn 2 1 2 n ≥ an b1 + an−1 b2 + · · · + a1 bn = a1 bn + a2 bn−1 + · · · + an b1 a2 an a1 = 2 + 2 + ···+ 2. 1 2 n Since 1 ≤ a1 , 2 ≤ a2 , . . . , n ≤ an , we get 1 x1 x2 xn a1 a2 an 1 2 n 1 1 + +···+ 2 ≥ 2 + 2 +···+ 2 ≥ 2 + 2 +···+ 2 = + +···+ . 12 22 n 1 2 n 1 2 n 1 2 n Solution 9.27. First, define x2i = 0, x2i−1 = 12 , for all i = 1, . . . , 50. Then, we  2 25 have S = 50 · 21 = 25 2 . Now we prove that always S ≤ 2 . 269 10.9 Solutions of Chapter 9 Let 1 ≤ i ≤ 50, under the conditions of the problem, we have x2i−1 ≤ 1 − x2i − x2i+1 and x2i+2 ≤ 1 − x2i − x2i+1 . Using the inequality between the geometric mean and the arithmetic mean, we get x2i−1 x2i+1 + x2i x2i+2 ≤ (1 − x2i − x2i+1 )x2i+1 + x2i (1 − x2i − x2i+1 ) (x2i + x2i+1 ) + (1 − x2i − x2i+1 ) 2 = (x2i + x2i+1 )(1 − x2i − x2i+1 ) ≤ 2 = 1 . 4 Adding these inequalities, for i = 1, 2, . . . , 50, we obtain S= 50 # 1 25 . (x2i−1 x2i+1 + x2i x2i+2 ) ≤ 50 · = 4 2 i=1 Solution 9.28. (i) From bx + by ≤ ax + by ≤ ab, it follows that x + y ≤ a. (ii) We have √ x+ √ ax + a y= by ≤ b ax + by ≤ ab 1 1 + a b 1 1 + a b = √ a + b. The first inequality is given by the Cauchy–Schwarz inequality (see Example 4.2.3), and the second one follows from the hypothesis ax + by ≤ ab. Solution 9.29. Let x = a a−b , y= b b−c , z= (x − 1)(y − 1)(z − 1) = = c c−a , b a−b a a−b = xyz. then c b−c b b−c a c−a c c−a When we expand and cancel out terms, we obtain x + y + z = xy + yz + zx + 1. Then 2a − b a−b 2 + 2b − c b−c 2 + 2c − a c−a 2 = (x + 1)2 + (y + 1)2 + (z + 1)2 = 3 + x2 + y 2 + z 2 + 2(x + y + z) = 3 + x2 + y 2 + z 2 + 2(xy + yz + zx + 1) = 5 + (x + y + z)2 ≥ 5. 270 Chapter 10. Solutions to Exercises and Problems Solution 9.30. Suppose a ≤ b ≤ c, then a2 ≤ bc, therefore a2 + 1 ≤ bc + 1 ≤ 2a. (10.23) On the other hand, since (a−1)2 ≥ 0, we have a2 ≥ 2a−1. The last two inequalities imply that a2 = 2a − 1, that is, a = 1. The original second inequality can be rewritten, using that a = 1, as bc ≤ 1. But since 1 ≤ b ≤ c, we also have that bc ≥ 1, therefore bc = 1. The first and third inequalities becomes b + 1 ≤ 2c and c + 1 ≤ 2b, therefore (b + 1)(c + 1) ≤ 4bc = 4. If we expand and cancel out some terms, we get b + c ≤ 2. The inequality between the geometric mean and the arithmetic mean, and the 2  ≤ 1, then the equality holds, previous inequality, guarantee that 1 = bc ≤ b+c 2 which is true only if b = c. Since bc = 1, then b = c = 1. Therefore, a = b = c = 1 is the only solution. Solution√9.31. Without √ loss of generality, we can assume that a = max{a, b, c}. Then c( b − 1) ≤ a( b − 1) = c, hence b ≤ 4. √ √ √ We also have b( c − 1) = a ≥ b, then c ≥ 4. Now, 4 ≤ c ≤ c( c− 1) ≤ c( a− 1) = b ≤ 4, then b = c = 4 and also a = 4. Therefore, there is a unique triplet that satisfies the equations, that is, (4, 4, 4). Solution 9.32. If {x1 , . . . , xn } is a real solution of the system, it is clear that also {x2 , x3 , . . . , xn , x1 } is a solution. But, by hypothesis, we only have one solution, then x1 = x2 = · · · = xn . The system reduces to only one equation ax2 + (b − 1)x + c = 0, which has a unique solution if (b − 1)2 − 4ac = 0. Reciprocally, if (b − 1)2 − 4ac = 0, the polynomial P (x) = ax2 + (b − 1)x + c = 2  a x + b−1 has only one solution. 2a *n Adding the equations of the system, we have i=1 P (xi ) = 0, but since either all the numbers have the same sign or are zero, then P (xi ) = 0 for every   , and the system has as a unique solution x1 = x2 = · · · = xi . Hence xi = − b−1 2a   xn = − b−1 2a . Solution 9.33. If one of the variables x, y or z is equal to 1 or −1, then we obtain the solutions (1, 1, 1) or (−1, −1, −1), respectively. Now we will see that these are the only solutions of the system. Let f (t) = t2 + t − 1. If one of the variables x, y or z is greater than 1, for example x > 1, then we have x < f (x) = y < f (y) = z < f (z) = x, which is not possible. Therefore x, y, z ≤ 1. If one of the variables x, y or z is less than x < −1. Since ! −1, choose " f (t) = t + 12 − 45 ≥ − 45 , we then get x = f (z) ∈ − 45 , −1 . But, f + 5 − , −1 4 = −11 −1, 16 + 5 ⊂ (−1, 0) and f ((−1, 0)) = − , −1 , 4 271 10.9 Solutions of Chapter 9  ! then it follows that y = f (x) ∈ (−1, 0), z = f (y) ∈ − 45 , −1 and x = f (z) ∈ (−1, 0), which is a contradiction. Then, −1 ≤ x, y, z ≤ 1. If −1 < x, y, z < 1, then x > f (x) = y > f (y) = z > f (z) = x, which is not possible. Therefore, there are no other solutions. Solution 9.34. Add the n equations to obtain n # i=1 x2i + n # i=1 xi − n = n # xi , i=1 *n from where i=1 x2i = n. On the other hand, rewrite the equations as follows: x21 + x1 = x2 + 1 x22 + x2 = x3 + 1 .. .. . . x2n−1 + xn−1 = xn + 1 x2n + xn = x1 + 1 and multiply the equations to obtain n 6 i=1 xi (xi + 1) = n 6 (xi + 1), i=1 3n so that i=1 xi = 1 if xi = −1, for all 1 ≤ i ≤ n. *n 3n From the two equations, i=1 x2i = n and i=1 xi = 1, we obtain, using the inequality between the geometric mean and the arithmetic mean, that 7 8 n *n 2 86 x n i=1 i n 1= = x2i = 1, ≥ 9 n n i=1 and it follows that x21 = x22 = · · · = x2n = 1. Then, a possible solution is (1, 1, . . . , 1). If some xi = −1, by the symmetry of the equations, we can assume that x1 = −1, and then it is easy to see that all xi = −1. Hence the only other solution is (−1, −1, . . . , −1). Solution 9.35. The only solution, with all the xi ’s equal, is clearly (2, 2, . . . , 2). If there is another solution, let m and M be the minimum and maximum value of the xi , respectively. Then m < M , and for some indexes j, k (taken modulo n), we would have m2 = xj + xj+1 ≥ 2m and M 2 = xk + xk+1 ≤ 2M , from where 2 ≤ m < M ≤ 2, which is absurd. Therefore (2, 2, . . . , 2) is the only solution. 272 Chapter 10. Solutions to Exercises and Problems Solution 9.36. The condition x + y + z = 0 implies that x3 + y 3 + z 3 = 3xyz. The condition x−1 + y −1 + z −1 = 0 guarantees that x−3 + y −3 + z −3 = 3x−1 y −1 z −1 . Now, since x6 + y 6 + z 6 = (x3 + y 3 + z 3 )2 − 2x3 y 3 z 3 (x−3 + y −3 + z −3 ), we have x6 + y 6 + z 6 = 9x2 y 2 z 2 − 6x2 y 2 z 2 = 3x2 y 2 z 2 . Thus the result follows if we use again x3 + y 3 + z 3 = 3xyz. Solution 9.37. Since the sum of the numbers is zero, d = −a − b − c, therefore bc − ad = bc + a(a + b + c) = a(a + b) + c(a + b) = (a + c)(a + b) ac − bd = ac + b(a + b + c) = b(a + b) + c(a + b) = (b + c)(a + b) ab − cd = ab + c(a + b + c) = c(c + a) + b(c + a) = (b + c)(c + a). Then, (bc − ad)(ac − bd)(ab − cd) = (a + b)2 (b + c)2 (c + a)2 . Solution 9.38. We have # a2 + b 2 # a4 # a2 + b 2 # a4 + ab(a2 + b2 ) # a3 − = + = bc a+b abc c abc cyclic cyclic cyclic cyclic cyclic 1 4 = (a + b4 + c4 + ab(a2 + b2 ) + bc(b2 + c2 ) + ca(c2 + a2 )) abc 1 (a + b + c)(a3 + b3 + c3 ) = 0. = abc Solution 9.39. Since a3 + b3 + c3 − 3abc = and since a + b + c > 2, then ! " 1 (a + b + c) (a − b)2 + (b − c)2 + (c − a)2 2 a + b + c = p and ! " (a − b)2 + (b − c)2 + (c − a)2 = 2. Suppose that !a ≥ b ≥ c. If a > b > c, then a" − b ≥ 1, b − c ≥ 1 and a − c ≥ 2, and this leads to (a − b)2 + (b − c)2 + (c − a)2 ≥ 6 > 2, which is absurd, therefore a = b = c + 1 or a − 1 = b = c. Thus, we have that the prime number is of the form p = 3c + 2, in the first case or of the form p = 3c + 1, in the second case. Then the p+1 p−2 p+2 p−1 p−1 triplet is ( p+1 3 , 3 , 3 ) in the first case, and ( 3 , 3 , 3 ) in the second case. Solution 9.40. The equation is equivalent to (3x)3 + (−3y)3 + (−1)3 − 3(3x)(−3y)(−1) = 1646, which can be factorized as (3x − 3y − 1)(9x2 + 9y 2 + 1 + 9xy + 3x − 3y) = 2 · 823. Now, the first factor on the left-hand side is smaller than the second factor and, since 823 is a prime number and 3x − 3y − 1 ≡ 2 mod 3, we get 3x − 3y − 1 = 2 and 9x2 + 9y 2 + 1 + 9xy + 3x − 3y = 823. Solving the system for positive real numbers leads to x = 6 and y = 5. 273 10.9 Solutions of Chapter 9 Solution 9.41. Using the identity (4.8), the condition x3 + y 3 + z 3 − 3xyz = 1 is equivalent to (10.24) (x + y + z)(x2 + y 2 + z 2 − xy − yz − zx) = 1. Let A = x2 + y 2 + z 2 and B = x + y + z. Observe that B 2 − A = 2(xy + yz + zx). By identity (4.9) we have that B > 0. The equation (10.24) becomes B A− B2 − A 2 = 1, then 3A = B 2 + B2 . Since B > 0, we apply the inequality between the geometric mean and the arithmetic mean to obtain 3A = B 2 + B2 = B 2 + B1 + B1 ≥ 3, that is, A ≥ 1. The minimum A = 1 is reached, for example, with (x, y, z) = (1, 0, 0). Solution 9.42. Equations x + y z zx + y = 2z, = 2, y + z x = 2, z + xy + z = 2x, x y = 2, imply that yz + x = 2y and that xyz + y 2 = 2yz, xyz + z 2 = 2zx, xyz + x2 = 2xy. Therefore, xy + yz + zx = x + y + z, 2 2 2 3xyz + (x + y + z ) = 2(xy + yz + zx). We also have that yzx = (2 − x)(2 − y)(2 − z) 1= zxy = 8 − 4(x + y + z) + 2(xy + yz + zx) − xyz. (10.25) (10.26) (10.27) If we define a = x + y + z, we have by equation (10.25), that xy + yz + zx = a and we also get that x2 + y 2 + z 2 = (x + y + z)2 − 2(xy + yz + zx) = a2 − 2a. Now, from equation (10.26), it follows that 3xyz = −a2 + 4a. Finally, by equation (10.27), we can conclude that 1 = 8 − 4a + 2a − −a2 + 4a . 3 Hence, we get the equation a2 − 10a + 21 = 0, which has roots a = 3 and a = 7. Therefore, x + y + z is equal to 3 or 7. But, if x + y + z = 7, since x + yz + y + xz + z + xy = 6, we have that yz + xz + xy = −1, which is not possible for x, y, z positive numbers. The sum x + y + z = 3, can be achieved with x = y = z = 1, which are also solutions of the equations, hence the only possible value for x + y + z is 3. 274 Chapter 10. Solutions to Exercises and Problems Solution 9.43. Let d be the common difference of the progression {an }. Note that, for j = 0, 1, . . . , n − 1, we have that √ √ √ √ aj−1 − aj aj − aj−1 1 1 . √ =√ √ ·√ √ = √ aj−1 + aj aj−1 + aj aj−1 − aj aj − aj−1 Use the fact that aj − aj−1 = d, for all j, in order to get √ √ √ n √ n # # aj − aj−1 an − a0 1 = . √ = √ aj−1 + aj d d j=1 j=1 Finally, observe that √ √ an − a0 an − a0 a0 + nd − a0 n = √ = √ =√ √ √ √ . d d( a0 + an ) d( a0 + an ) a0 + an Solution 9.44. Let d > 0 be the common difference of the progression. Suppose that a = b − d, b and c = b + d are the lengths of the sides of the triangle. Since c2 = a2 + b2 , we have (b + d)2 = (b − d)2 + b2 , therefore b = 4d. On the other 2 (b−d)4d 2 hand, the area of the triangle is a·b = 12d 2 = 2 2 = 6d , and it is also equal to 3d+4d+5d 31 = 6d. From this, the inradius r multiplied by the semiperimeter s = 2 r · 6d = 6d2 , and then r = d. Solution 9.45. Suppose that {1, 2, . . . , 9} has been divided into two subsets A and B such that neither of them contains an arithmetic progression. Suppose that 5 ∈ A. It is clear that 1 and 9 cannot both be in A. Then, we have the following cases: (i) If 1 ∈ A and 9 ∈ B. Since {1, 5} ⊂ A, we have that 3 ∈ B; 3, 9 ∈ B imply that 6 ∈ A; 5, 6 ∈ A imply that 4, 7 ∈ B; 3, 4 ∈ B imply that 2 ∈ A; 7, 9 ∈ B imply that 8 ∈ A. But, {2, 5, 8} ⊂ A is an arithmetic progression, which is absurd. (ii) If 9 ∈ A and 1 ∈ B. This case is analogous to (i). (iii) If 1, 9 ∈ B. Then we have two subcases: (1) If 7 ∈ A. In this case, 5, 7 ∈ A imply 6 ∈ B and 3 ∈ B. Therefore, {3, 6, 9} ⊂ B, which is absurd. (2) If 7 ∈ B. Since 7, 9 ∈ B, we have that 8 ∈ A; 1, 7 ∈ B imply 4 ∈ A; 4, 5 ∈ A imply 3 ∈ B; 1, 3 ∈ B imply that 2 ∈ A. Therefore, again we have an arithmetic progression, that is, the progression {2, 5, 8} is in A, which is a contradiction. Solution 9.46. Observe that 2 (a + b + c)3 − a2 (b + c) − b2 (c + a) − c2 (a + b) 9 1 = (a + b − 2c) (2a − b − c) (a − 2b + c) . 9 31 See [5]. 10.9 Solutions of Chapter 9 275 Solution 9.47. Suppose the number of prime numbers is not infinite. Let p be the greatest prime number of the progression. Consider the number n = 4p! − 1, which belongs to the progression. Since n > p, the number is a composite number and it does not have prime divisors of the form 4k − 1 (the factors of this form belong to p!), then its prime divisors are of the form 4k + 1. But the product of factors of the form 4k + 1 is also a number of the form 4k + 1, and then n must be of this form as well, which is a contradiction. Solution 9.48. Divide the set of natural numbers N = {1, 2, 3, . . . } in the following way 1 4 5 6 11 . . . 2 3 7 8 9 10 ... The sets we are looking for are A = {1, 4, 5, 6, 11, 12, 13, 14, 15, 22, . . .} and B = {2, 3, 7, 8, 9, 10, 16, 17, 18, 19, 20, 21, 29, . . .}. In fact, each one of them has “gaps” between numbers as large as we want. Therefore, it is not possible to have in some of them an arithmetic progression, since the elements of the progression have a constant difference d, which will be overtaken by a proper gap. Solution 9.49. The answer is no. If there is an arithmetic progression with difference d, we have that the d consecutive integers (d + 1)! + 2, (d + 1)! + 3, . . . , (d + 1)! + (d + 1) are composite numbers. But among them there has to be an element of the progression, because this progression has difference d, which is a contradiction. Second Solution. Let m > 1 be a number in the progression. Then m + md = m(d + 1) is also an element of the progression and is not a prime number. Solution 9.50. Suppose that the arithmetic progression with difference d contains a perfect square, say a2 . Then the numbers a2 , a2 + d, a2 + 2d, . . . are in the progression, and then the numbers a2 + (2a + d)d = (a + d)2 are also in the progression. Now, it is clear that the square numbers of the form (a + kd)2 , for every k ∈ N, are also in the progression. Solution 9.51. Suppose that there are 1999 prime numbers, smaller than 12345, in arithmetic progression. Let p be the first prime number in the progression and let r be the difference of the progression. Then the progression is p, p + r, p + 2r, . . . , p + 1998r. The prime number p cannot be one of the prime numbers 2, 3, . . . , 1997, because if it is one of them, then p + pr, which is in the progression, is not a prime number. Therefore, p ≥ 1999. Since p is an odd number and p + r is prime, then r is even. All the even numbers are of the form 6n, 6n + 2 or 6n − 2. Let us see now that r cannot be of the 276 Chapter 10. Solutions to Exercises and Problems form 6n + 2 nor can it take the form 6n − 2. In fact, since p is prime, it is of the form 6k + 1 or 6k − 1. In any of those four cases, there is in the progression a multiple of 3, p + r = (6k + 1) + (6n + 2), p + 2r = (6k + 1) + 2(6n − 2), p + 2r = (6k − 1) + 2(6n + 2), p + r = (6k − 1) + (6n − 2). Therefore r is of the form 6n and then the progression is p, p + 6n, . . . , p + 1998(6n). But p ≥ 1999 and n ≥ 1 imply that p+1998(6n) ≥ 1999+11988 = 13987 > 12345. Hence, the numbers p + jr cannot be all smaller than 12345. Solution 9.52. For n < 3, there does not exist a rearrangement with an arithmetic triplet. For n = 3, the list 2, 1, 3 achieves the task. We will construct an example for n, using the examples of the previous values of n. On one side of the list, we put the even numbers between 1 and n, and on the other side the odd numbers. If the even numbers are j, we rearrange them using the example for the j numbers and then we have them multiplied by 2. If there are k odd numbers, to order them we use the example for the k multiplying those numbers by 2 and subtracting 1. In this way we obtain a rearrangement of the numbers from 1 to n. If on the even side, 2a, 2b and 2c form an arithmetic triplet, then a, b and c is also an arithmetic triplet for the case j, which is absurd. If in the odd side, 2a − 1, 2b − a and 2c − 1 is an arithmetic triplet, then a, b and c is also an arithmetic triplet in the example for the case of k, which is also absurd. Finally, one term on the even side and one term on the odd side satisfy that their sum is odd, then there is not a third term in between them such that its double would be this sum. Then, the constructed rearrangement does not have arithmetic triplets. Solution 9.53. Since there are 4 solutions for the first equation, a = 0. Let x0 be the common solution to both equations. Taking the difference of the equations, it follows that ax40 − ax0 = 0, which can also be written as ax0 (x30 − 1) = 0. Then, the common solution is x0 = 0 or x0 = 1. If x0 = 0 in the first equation, we obtain a = 1, therefore x0 is a solution with multiplicity at least 2, but this is not possible because we know that there are four different roots. Then the common solution is x0 = 1. When we substitute in the first equation, we obtain 2a + b = 1, and then this equation can be rewritten as ax4 + (1 − 2a)x2 + a − 1 = 0, which has 1 and −1 as solutions. Therefore (x − 1)(x + 1)(ax2 − a + 1) = 0. The quadratic equation ax2 − a + 1 = 0 must have 2 different real solutions, say r and −r, with r > 0. 277 10.9 Solutions of Chapter 9 There are two cases, a > 1 and a < 0. If a > 1, then 0 < r < 1 and the roots −1, −r, r, 1 are in arithmetic progression 1 5 9 only when r = 13 . In such case a = 1−r 2 = 8 and b = 1 − 2a = − 4 . If a < 0, then r > 1, and the numbers −r, −1, 1, r are in arithmetic progression 5 1 1 only if r = 3. In such case a = 1−r 2 = − 8 and b = 4 . *k *k Solution 9.54. Write A = i=1 ai and B = i=1 bi . Now we add over i the corresponding terms in the inequalities ai n + bi − 1 < ⌊ai n + bi ⌋ ≤ ai n + bi , to obtain An + B − k < Xn ≤ An + B. Now, suppose that {Xn } is an arithmetic progression with common difference d, then nd = Xn+1 −X1 and A+B−k < X1 ≤ A + B. Combine the above inequalities to obtain A(n + 1) + B − k < nd + X1 ≤ A(n + 1) + B or An − k ≤ An + (A + B − X1 ) − k < nd < An + (A + B − X1 ) < An + k, from which we conclude that |A − d| < nk , for any integer number n; then A = d. Since {Xn } is a sequence of integers, d has to be also an integer number, therefore we conclude that A is an integer number. Solution 9.55. Consider a partition of {1, . . . , 256} into two subsets, A and B. Divide {1, . . . , 9} in two subsets A1 and B1 , in the following way: k ∈ A1 (resp. B1 ) if and only if 2k−1 ∈ A (resp. B). Clearly, A1 ∩ B1 = ∅. If A1 = ∅ (or B1 = ∅), then B1 (or A1 ) has three numbers in arithmetic progression a, b and c. Then 2a−1 , 2b−1 and 2c−1 is a geometric progression in B (or A). If A1 = ∅ and B1 = ∅, then A1 ∪ B1 is a partition of {1, . . . , 9}. By Problem 9.45, one of the sets, say A1 , contains an arithmetic progression of three terms a, b and c. Then 2a−1 , 2b−1 and 2c−1 is a geometric progression in A. Solution 9.56. Remember that the nth term in a geometric progression is an = a0 · q n , where a0 is the first term  of the progression and q is the ratio. Since ak+1 = ak 1 + n12 , the given collection of numbers coincide with the first terms of a geometric progression whose ratio is 1 + n12 and whose first term is given by 12 . The nth term of the collection is given by an = 1 2 1+ 1 n2 n .  n We have to prove that 1 + n12 < 2, for n > 1. By Newton’s binomial theorem (Theorem 3.2.3), we have that 1+ 1 n2 n = n # n i i=0 1 n2 i = n # i=0 Ti , where Ti = n i 1 n2 i . 278 Chapter 10. Solutions to Exercises and Problems Now, notice that Ti = Ti+1 n! i!(n−i)! n! (i+1)!(n−i−1)! · 1 n2i 1 n2i+2 = (i + 1) 2 · n > 1, n−i hence the sequence Ti is decreasing, then Ti < T1 = 1+ 1 n2 1 n. Therefore n = 1 + T1 + T2 + T3 + · · · + Tn < 1 + nT1 = 2. Solution 9.57. Suppose that a1 = a. By induction we will see that an = a + n − 1. Suppose that an = a + n − 1 and prove that an+i = a + n + i − 1, for 1 ≤ i ≤ n. Since a2n = an + n, the hypothesis leads us to an = a + n − 1, then a2n = a + 2n − 1. Now, since the sequence is increasing, it follows that a + n − 1 = an < an+1 < · · · < an+i < · · · < a2n = a + 2n − 1. Hence an+i = a + n + i − 1 for 1 ≤ i ≤ n. (10.28) If a1 = 1, then we get an = n, for all n ≥ 1. What remains to be proved is the induction basis, that is, a1 = 1. Suppose that a1 > 1. Let p be the least prime number greater than (a1 + 1)! + a1 + 1. Of course, this prime number p is an element of the sequence, that is, p = an = a1 + (n − 1), for some n. By equation (10.28), the an are consecutive numbers. Moreover, by property (ii), n = p − a1 + 1 is also a prime number. Since a1 > 1, p − a1 + 1 < p, and by the way we have chosen p, we have that p−a1 +1 ≤ (a1 +1)!+a1 +1 < p. Then (a1 +1)!+2 ≤ p−a1 +1 ≤ (a1 +1)!+a1 +1. But this is a contradiction, since among the numbers (a1 + 1)! + 2, (a1 + 1)! + 3, . . . , (a1 + 1)! + a1 + 1 there are no prime numbers, that is, all the numbers are composite numbers, since j divides ((a1 + 1)! + j). The contradiction proves that a1 = 1. Solution 9.58. The proof is by induction over n + m. The statement is clear if m + n = 2. Suppose that the statement is true for m + n < k, and consider m + n = k arbitrary numbers a1 , a2 , . . . , am , b1 , b2 , . . . , bn . Define the sets A = {a1 , a2 , . . . , am }, B = {b1 , b2 , . . . , bn }, C = {c1 , c2 , . . . , cm }, D = {d1 , d2 , . . . , dn }. We have two cases: 1. If A ∩ C = ∅ or B ∩ D = ∅. For example, suppose that A ∩ C = ∅. This implies that ai = cj , for some indices i, j ∈ {1, 2, . . . , m}. Without loss of 279 10.9 Solutions of Chapter 9 generality, let i ≤ j. If i = j, one of the terms in the equality we want to prove is zero and, then we can apply directly the induction hypothesis. If i < j, then ci < · · · < cj = ai < ai+1 < · · · < aj , then |ai − ci | + |ai+1 − ci+1 | + · · · + |aj − cj | = (ai − ci ) + (ai+1 − ci+1 ) + · · · + (aj − cj ). If we change the order of the terms, the value of the sum does not change, and this value is equal to (ai+1 − ci ) + (ai+2 − ci+1 ) + · · · + (aj − cj−1 ) + (ai − cj ) = |ai+1 − ci | + |ai+2 − ci+1 | + · · · + |aj−1 − cj−2 | + |aj − cj−1 |, since ai − cj = 0. Then the result follows from the induction hypothesis, for the m + n − 1 = k − 1 numbers a1 < a2 < · · · < ai−1 < ai+1 < · · · < am , b1 < b2 < · · · < bn , c1 < c2 < · · · < cj−1 < cj+1 < · · · < cm , d1 < d2 < · · · < dn . 2. If A ∩ C = B ∩ D = ∅. In this case, we have that a1 is in D and b1 is in C. Without loss of generality, suppose that a1 < b1 . Then a1 has to be equal to d1 . We will prove that b1 has to be equal to c1 . If b1 = ci , for some i > 1, then b1 > c1 and, since c1 = bj , for some j > 1, we have that b1 > bj , which is a contradiction. Therefore, b1 = c1 , and taking into account that ai = d1 , this implies |a1 − c1 | = |b1 − d1 |. Now, we use the induction hypothesis for the numbers a2 < a3 < · · · < am , b 2 < b 3 < · · · < b n , c2 < c3 < · · · < cm , d2 < d3 < · · · < dn . Solution 9.59. First, observe that for n ≥ 2, we have n−1 # j=1 n−1 # j = (j + 1)! j=1 1 1 − j! (j + 1)! =1− 1 . n! (10.29) Multiplying by n! the equation (10.29), we obtain the desired representation 1+ n−1 # j=1 j · n! = n!. (j + 1)! Solution 9.60. Remember that every integer number relatively prime to p has a multiplicative inverse modulo p. Denote the inverse of x modulo p by x−1 . Observe that p−1 # p 2 p−1 # p p−1 2 2p −2= . = p k k k−1 k=1 k=1 280 Chapter 10. Solutions to Exercises and Problems p−1   is an integer number, since it is equal to p1 kp and p divides Observe that k1 k−1 p *p−1 $ −1 p−1%2 3 2 . We k=1 k k . Then the last sum is congruent, modulo p , to p k−1 prove that the sum is divisible by p. Observe that, modulo p, we have p−1 # k −1 k=1 p−1 k−1 2 ≡ p−1 # k=1 (k −1 )2 {(p − 1)(p − 2) · · · (p − k + 1) × [(k − 1)(k − 2) · · · (p − k + 1)]−1 }2 ≡ ≡ p−1 # k=1 p−1 # ! "2 (k −1 )2 (−1)(1)−1 (−2)(2)−1 · · · (−(k − 1))(k − 1)−1 (k −1 )2 (−1)2k−2 . k=1 But the inverse numbers of 1, 2, . . ., p − 1, modulo*p, are the same numbers p−1 in some other order. Then the sum is congruent to k=1 k 2 , which is equal to  (p − 1)p(2p − 1)/6. This is divisible by p, since p = 2, 3. Then 2p − 2 is divisible p 3 by p . Since 2p − 1 1 2p −1= −2 , p−1 2 p   3 it follows that 2p−1 p−1 − 1 is divisible by p , as we wanted. a b c d−c Solution 9.61. If ∈ S, then by (ii), 1 a b +1 = b a+b ∈ S and a b a b +1 = a a+b ∈ c d−c+c c ∈ S, then d−c+c = dc ∈ S and if d−c In particular, if c ∈ S, then a0 Consider a rational number q0 = b0 , with (a0 , b0 ) = 1 and 0 < q0 < 0 0 enough to see that either b0a−a or b0a−a belongs to S. 0 0 1 If q0 = 2 there is nothing to do. If q0 < 21 , then a0 < b0 − a0 . If b 0 − a0 < a0 . Let q1 =  a0 b0 −a0 , b0 −a0 a0 , if q0 < S. = dc ∈ S. 1. Then it is q0 > 12 , then 1 2 if q0 > 12 , then 0 < q1 < 1 and if q1 ∈ S, then q0 ∈ S. Now, if q1 = ab11 , with (a1 , b1 ) = 1, and considering  a1 1 b1 −a1 , if q1 < 2 , q2 = b1 −a1 1 a1 , if q1 > 2 , it follows that 0 < q2 < 1 and if q2 ∈ S, then q1 ∈ S. This process of going from qk to qk+1 is possible if no qk is equal to 12 ; otherwise, the proof is complete (qk ∈ 21 ∈ S implies that qk−1 , . . . , q0 ∈ S). 281 10.9 Solutions of Chapter 9 o , and b1 | b0 − a0 , The process cannot be infinite. If q0 < 21 , then ab11 = q1 = b0a−a 0 1 hence b1 ≤ b0 − a0 < b0 . If q0 > 2 , then b1 | a0 and b1 ≤ a0 < b0 . Hence, in any case b1 < b0 . Similarly, it follows that bk+1 < bk , for all k ≥ 0. Thus, {bk } is a decreasing infinite sequence of positive integers, which is impossible. 2 2 +b Solution 9.62. Suppose aab+1 = k and k is not a perfect square. Then a2 − kab + 2 b = k. Suppose (a0 , b0 ) is a solution of the equation. By symmetry, we can assume that a0 ≥ b0 > 0. We know that a0 is a root of the quadratic equation a2 − kab0 + b20 − k = 0. Let c1 be the other root of the previous equation; the two roots satisfy a0 + c1 = kb0 and a0 c1 = b20 − k, then c1 = kb0 − a0 is an integer. Now, since k is not a square, a0 c1 = 0, hence c1 = 0. If c1 < 0, then c21 − kc1 b0 + b20 ≥ c21 + k + b20 > k, which is a contradiction, therefore b2 −1 a2 −1 b2 −k c1 > 0. Moreover, c1 = 0a0 ≤ 0a0 ≤ 0a0 < a0 . Thus (c1 , b0 ) is a positive solution of a2 − kab + b2 = k, with c1 < a0 . Proceed in the same way to construct a decreasing sequence of positive integers a0 > c1 > c2 > · · · > 0, but this cannot happen. Solution 9.63. Since a, b, c are the roots of P (x), by Vieta’s formulas it follows that P (x) = x3 − (a + b + c)x2 + (ab + bc + ca)x − abc = x3 − 2007x + 2002, and then a + b + c = 0, ab + bc + ca = −2007, abc = −2002. Hence, a−1 a+1 b−1 b+1 c−1 c+1 abc − (ab + bc + ca) + a + b + c − 1 abc + ab + bc + ca + a + b + c + 1 −2002 − (−2007) − 1 4 1 = = =− . −2002 + (−2007) + 1 −4008 1002 = Solution 9.64. If x = a + b + c, y = abc, z = a1 + 1b + 1c are integers, also ab + bc + ca = abc( a1 + 1b + 1c ) = yz is an integer. Moreover, a, b, c are the roots of the polynomial w3 − xw2 + yzw − y = 0. Since the coefficients of the polynomial are integers and the coefficient of w3 is 1, by Gauss’ lemma or by the rational root theorem, a, b, c are integers. Suppose that 1 ≤ a ≤ b ≤ c, since 1 ≤ a1 + 1b + 1c ≤ a3 , it follows that a = 1, 2 or 3. If a = 1, then 1b + 1c ≤ 2, and the only possibilities are (b, c) = (1, 1) or (2, 2). If a = 2, then 21 ≤ 1b + 1c ≤ 1, hence (b, c) = (3, 6) or (4, 4). If a = 3, 1b + 1c = 23 , and the only solution is (b, c) = (3, 3). Thus, the solutions are (a, b, c) = (1, 1, 1), (1, 2, 2), (2, 3, 6), (2, 4, 4), (3, 3, 3). Solution 9.65. If one of a, b or c is zero, then one of the equations is linear and this one has a real solution. The discriminant of the equations are 4b2 − 4ca, 4c2 − 4ab, 4a2 − 4bc. Then, given that " ! (4b2 − 4ca) + (4c2 − 4ab) + (4a2 − 4bc) = 2 (a − b)2 + (b − c)2 + (c − a)2 ≥ 0, one of the discriminants is non-negative and therefore the equation with that discriminant has a real solution. 282 Chapter 10. Solutions to Exercises and Problems Solution 9.66. Let u, v be the roots of the quadratic polynomial and let α, β, γ be the roots of the cubic polynomial, with all of these roots being non-negative. By Vieta’s formulas, 2 uv = − a, 3 α + β + γ = −a, αβ + βγ + γα = b, αβγ = 8. u + v = 4, Now, using the inequality between the geometric mean and the arithmetic mean, 4= 4 2 2 = u+v 2 2 ≥ uv 2 = − a, 3 2 2 − a = (α + β + γ) ≥ 2 3 αβγ = 4. 3 3 Hence, on both inequalities the equality holds, and then u = v, α = β = γ and − 32 a = 4. Hence a = −6, α = β = γ = 2, therefore b = 12. Thus, the only pair is (a, b) = (−6, 12). Solution 9.67. We can choose appropriate signs in ±P (±x) in order to assume that a, b ≥ 0. (i) There are two cases: (1) c ≥ 0, |a| + |b| + |c| = a + b + c = P (1) ≤ 1. (2) c < 0, |a| + |b| + |c| = a + b − c = P (1) − 2P (0) ≤ 3. Then, 3 is the maximum that is attained with the polynomial P (x) = 2x2 − 1. (ii) There are four cases: (1) c ≥ 0, d ≥ 0, |a| + |b| + |c| + |d| = a + b + c + d = P (1) ≤ 1. (2) c ≥ 0, d < 0, |a| + |b| + |c| + |d| = a + b + c − d = P (1) − 2P (0) ≤ 3. (3) c < 0, d ≥ 0, |a| + |b| + |c| + |d| = a + b − c + d = 34 P (1) − 31 P (−1) + 8 1 8 1 3 P ( 2 ) + 3 P (− 2 ) ≤ 7. (4) c, d < 0, |a|+|b|+|c|+|d| = a+b−c−d = 53 P (1)−4P ( 12 )+ 34 P (− 21 ) ≤ 7. With the polynomial P (x) = 4x3 − 3x, the maximum 7 in this case, is attained. Solution 9.68. Notice that dx3 +cx2 +bx+a = x3 (d+c x1 +b x12 +a x13 ); then its roots are the inverse of the roots of ax3 + bx2 + cx + d. If {α, β, γ} are the roots of the polynomial ax3 + bx2 + cx + d, it follows, by Vieta’s formulas, that α + β + γ = − ab and α1 + β1 + γ1 = − dc . Using the inequality between the geometric mean and the bc = (α + β + γ)( α1 + β1 + γ1 ) ≥ 9. arithmetic mean, we can conclude that ad 283 10.9 Solutions of Chapter 9 √ Solution 9.69. Given that 2 x2 − 1 = x − get that x2 − p, and squaring both sides, we 4(x2 − 1) = x2 − 2x x2 − p + x2 − p 4x2 − 4 = 2x2 − 2x x2 − p − p 2x2 + (p − 4) = −2x x2 − p, then 2x2 + (p − 4) < 0, since x > 0. Squaring the last equation and simplifying, it follows that 4x4 + 4x2 (p − 4) + (p − 4)2 = 4x2 (x2 − p) 8x2 (p − 2) + (p − 4)2 = 0 x=± 4−p 8(2 − p) Since x is a positive real number, then p < 2 and x = √ 4−p . 8(2−p) . Solution 9.70. Let α, β, γ be the roots of P (x) that are positive. From Vieta’s d formulas, it follows that α + β + γ = − ab , αβ + βγ + γα = ac , αβγ = − . Since a d = P (0) < 0 and αβγ > 0, it follows that a > 0. Dividing by a3 , the inequality to be proved, it is enough to see that 2 b a 3 +9 d a −7 bc a2 ≤ 0, that in terms of α, β, γ is −2 (α + β + γ)3 − 9αβγ + 7 (α + β + γ) (αβ + βγ + γα) ≤ 0. After simplifying, the left-hand side of the previous inequality is   α2 β + αβ 2 + β 2 γ + βγ 2 + γ 2 α + γα2 ≤ 2 α3 + β 3 + γ 3 . This inequality follows from the rearrangement inequality applied in the following way, α2 β + β 2 γ + γ 2 α ≤ α3 + β 3 + γ 3 , β 2 α + γ 2 β + α2 γ ≤ α3 + β 3 + γ 3 . Solution 9.71. By Vieta’s formulas, it follows that −a = α+β+γ, b = αβ+βγ+γα and −c = αβγ. Since α2 = β +γ, then α2 = −α−a. Observe that α = 0, otherwise α = 0, and then c = 0, contradicting the fact that c is odd. Hence βγ = − αc and b = α(β + γ) + βγ = α3 − αc . 284 Chapter 10. Solutions to Exercises and Problems Therefore, 0 = α4 − bα − c = (α + a)2 − bα − c = α2 + 2αa + a2 − bα − c = −α − a + 2αa + a2 − bα − c = α(2a − b − 1) − (c − a2 + a). Observe that 2a − b − 1 = 0, otherwise 2a = b + 1, and since b is even, 2a would be odd, a contradiction. Hence, from the previous equation, it follows that α = c−a(a−1) 2a−b−1 is rational, and then it is an integer since the polynomial is monic and it has integer coefficients. On the other hand, −a = α(α + 1) is even. If β = γ, then 2β = β + γ = −a − α. If β is rational, it must be an integer; then from equation 2β = −a − α, it follows that α is even, but then c = −αβγ is even, a contradiction since c is odd. Solution 9.72. Let x0 be a real solution of equation x2 + px + q = 0. Then √ √ 1+ 1+4·1 1+ 5 −p ± p2 − 4q ≤ = = s. x0 = 2 2 2 Notice that x0 could be equal to s if p = q = −1. If y is a real root of an equation of the form t2 + pt + q = 0, with p, q ∈ [−1, 1], then any number z with absolute value less than or equal to the absolute value of y, would be a root too. To see this, let y 2 + py + q = 0, for p, q ∈ [−1, 1], and let z = αy, where |α| ≤ 1. The equation t2 + αpt + α2 q = 0 has coefficients αp and α2 q in the interval [−1, 1], since |α| ≤ 1. Moreover, z is a root of this equation, since z 2 + αpz + α2 q = (αy)2 + αp(αy) + α2 q = α2 (y 2 + py + q) = 0. Therefore, all solutions of the quadratic equations belong to the interval & √ √ ' − 1+2 5 , 1+2 5 . Solution 9.73. Define f : R → N by     1 n−1 f (x) = ⌊x⌋ + x + + ···+ x + − ⌊nx⌋. n n We have to show that f (x) = 0. Observe that         1 n−1 1 1 1 1 1 f x+ = x+ + x+ + + ···+ x + + − n x+ n n n n n n n     1 n−1 = x+ + ··· + x + + ⌊x + 1⌋ − ⌊nx + 1⌋, n n   and since ⌊x + k⌋ = ⌊x⌋ + k, for every integer k, it follows that f x + n1 = f (x), for every real number x. Hence, f is a periodic function with period n1 . In this 285 10.9 Solutions of Chapter 9 way it is enough to study f (x), for 0 ≤ x < then f (x) = 0 for every real number x. 1 n. But f (x) = 0 for all these values, Solution 9.74. The sought for identity can be rewritten as       1 1 n n n 1 + + 2+ + · · · + k+1 + + · · · = n. 2 2 2 2 2 2 We now use a special case the Hermite identity (see Problem 9.73 or Example  of  1 1.3.2), then for n = 2, x + 2 = ⌊2x⌋ − ⌊x⌋. This implies that           n n n n n + − 2 + · · · + k − k+1 + · · · = n. 2 2 2 2 2   n = 0, for k large enough. This last sum is telescopic and moreover 2k+1 ⌊n⌋ − Solution 9.75. This inequality can be proved using Hermite’s identity, but here we present an idea for a shorter proof. Let Sn be the right-hand side of the inequality. Then, if we set S0 = 0, we get Sn − Sn−1 = ⌊nx⌋ , for all n = 1, 2, . . . . n Then k(Sk − Sk−1 ) = ⌊kx⌋, for k = 1, 2, . . . , n + 1. Adding these n + 1 equations, it follows that −S1 − S2 − · · · − Sn + (n + 1)Sn+1 = ⌊(n + 1)x⌋ + ⌊nx⌋ + · · · + ⌊x⌋. Now proceed by induction. The basis n = 1 is clear. Suppose that Sk ≤ ⌊kx⌋, for 1 ≤ k ≤ n, then use the last identity for (n + 1)Sn , hence (n + 1)Sn+1 ≤ ⌊(n + 1)x⌋ + (⌊nx⌋ + ⌊x⌋) + · · · + (⌊x⌋ + ⌊nx⌋) . Using n times the fact that ⌊u⌋ + ⌊v⌋ ≤ ⌊u + v⌋ for any real numbers u and v, it follows that (n + 1)Sn+1 ≤ (n + 1)⌊(n + 1)x⌋, which ends the proof. Solution 9.76. First observe that since f (10) = 0 and f (10) = f (5) + f (2), then f (5) + f (2) = 0, but f (n) is non-negative, then f (5) = 0 and f (2) = 0. On the other hand, f (9) = f (3) + f (3) = 0. Then, given that 1985 = 5 · 397, it follows that f (1985) = 0, since f (1985) = f (5) + f (397) = 0 + f (397) = f (9) + f (397) = f (9 · 397) = f (3573) = 0. 286 Chapter 10. Solutions to Exercises and Problems Solution 9.77. The function must satisfy that f (y) > 0, for y > 0. Then f (xf (y)) = f (x) − 1 . xyf (y) (10.30) Let a = f (1) > 0. Taking x = 1 and then y = 1 in (10.30), it follows respectively that 1 1 =a− , for y ∈ R+ f (f (y)) = f (1) − yf (y) yf (y) (10.31) 1 + f (xa) = f (x) − , for x ∈ R . ax Taking x = 1 in the last equality, f (a) = f (1) − a1 = a − a1 . Taking x = a in the equation (10.30), it follows that f (af (y)) = f (a) − 1 1 1 =a− − . ayf (y) a ayf (y) (10.32) On the other hand, using equation (10.31), f (af (y)) = f (f (y)) − 1 1 1 =a− − . af (y) yf (y) af (y) (10.33) Combining equations (10.32) and (10.33), it follows that 1 1 1 1 + = + . a ayf (y) yf (y) af (y) + Hence f (y) = 1 + a−1 y , for y ∈ R . This is the only possible solution of the equation. Now substituting in the last equation, it follows that (a − 1)2 = 1, but since a > 0, the only choice is a = 2, and then f (x) = 1 + x1 is the only solution. Solution 9.78. For x ∈ [0, 1], |f (x)| = |f (x) − f (0)| < |x − 0| = x and |f (x)| = |f (x) − f (1)| < |x − 1| = 1 − x. Then |f (x)| < min {x, 1 − x}, for x ∈ [0, 1]. If |x − y| ≤ 12 , then |f (x) − f (y)| < |x − y| ≤ 12 . If |x − y| > 21 , without loss of generality, we can assume that 21 ≤ x ≤ 1 and that y < 12 . Since |f (x)| < min {x, 1 − x} = 1 − x and also |f (y)| < min {y, 1 − y} = y, it follows that |f (x) − f (y)| ≤ |f (x)| + |f (y)| < 1 − x + y = 1 − (x − y) < 12 . Solution 9.79. Let F = {f (n)} and G = {g(n)}, for n = 1, 2, . . . . Since g(1) = f (f (1)) + 1 > 1, then f (1) = 1 and g(1) = 2. Now we prove that if f (n) = k, then f (k) = k + n − 1 g(n) = k + n f (k + 1) = k + n + 1. (10.34) (10.35) (10.36) If we assume for the moment that these statements are true, they can be applied to f (1) = 1 to obtain g(1) = 2 and f (2) = 3. If we apply equation (10.34) to 287 10.9 Solutions of Chapter 9 f (2) = 3 and to the next numbers, we obtain a chain of results: f (3) = 4, f (4) = 6, f (6) = 9, f (9) = 14, f (14) = 22, f (22) = 35, f (35) = 56, f (56) = 90, f (90) = 145, f (145) = 234, f (234) = 378, .... But f (240) is not in this chain. Observe that equation (10.36) generates larger numbers; for instance, if we apply it to f (145) = 234, we get f (235) = 380. Looking at the previous values of the chain, we can see that applying equation (10.36), f (56) = 90 and then f (91) = 147, f (148) = 239. Finally, f (240) = 388. It only remains to prove equations (10.34), (10.35) and (10.36). Assuming that f (n) = k, it follows that the elements in the two disjoint subsets {f (1), f (2), . . . , f (k)} and {g(1), g(2), . . . , g(n)} cover all the natural numbers from 1 to g(n), since g(n) = f (f (n)) + 1 = f (k) + 1. Counting the elements in the sets, it follows that g(n) = k + n or g(n) = f (n) + n, which is equation (10.35). Equation (10.34) follows from k + n = g(n) = f (k) + 1. From equation g(n) − 1 = f (f (n)), notice that g(n) − 1 is an element of F , that is, two consecutive integers cannot be elements of G. Since k + n is an element of G, it follows that both k + n − 1 and k + n + 1 are elements of F , moreover, they are two consecutive elements of F . Therefore, equation (10.34) implies that k + n + 1 = f (k + 1). Solution 9.80. Take x = 0 and y = 1 in equations (9.2) and (9.1) to get f (1, 0) = f (0, 1) = 2. Moreover, if we take x = 0 and y = 0, by equations (9.3) and (9.1), we get f (1, 1) = f (0, f (1, 0)) = f (1, 0) + 1 = 3. Now, if we take x = 0 and y = 1 in (9.3) and using the previous results, it follows that f (1, 2) = f (0, f (1, 1)) = f (1, 1) + 1 = 4. We claim that f (1, y) = y + 2. (10.37) We already have verified this equation for y = 0, 1, 2. The inductive step follows from (9.3): f (1, k) = f (0, f (1, k − 1)) = f (1, k − 1) + 1 = (k − 1) + 2 + 1 = k + 2. Now, using induction, show that f (2, y) = 2y + 3. Observe that f (2, 0) = f (1, 1) = 3; now, from f (2, y + 1) = f (1, f (2, y)) = f (2, y) + 2, we obtain the inductive step. Also, it follows that f (3, 0) = f (2, 1) = 5 and f (3, y + 1) = f (2, f (3, y)) = 2f (3, y) + 3. Then, we get f (3, y) = 2f (3, y − 1) + 3 = 2(2f (3, y − 2) + 3) + 3 = · · · = 2y f (3, 0) + (2y−1 + · · · + 2 + 1)3 2y − 1 = 2y+3 − 3. = 2y (23 − 3) + 3 2−1 288 Chapter 10. Solutions to Exercises and Problems ·2 ·· Finally, once again by induction, we show that f (4, y) = 22 − 3, where the tower of numbers 2 has y + 3 floors. For this, notice that f (4, 0) = f (3, 1) = 24 − 3 = 13 and f (4, y + 1) = f (3, f (4, y)) = 2f (4,y)+3 − 3, and the inductive step is completed. Solution 9.81. Since f (n + m) − f (m) − f (n) = 0 or 1, it follows that f (m + n) ≥ f (m) + f (n). If m = n = 1, then f (2) ≥ 2f (1), but f (2) = 0, that is, 0 ≥ 2f (1), and since f (n) ≥ 0, then f (1) = 0. If m = 2 and n = 1, then f (3) = f (2) + f (1) + {0 or 1} = 0 or 1. Since f (3) > 0, then f (3) = 1. If m = n = 3, then f (3 + 3) = f (2 · 3) = f (3) + f (3) + {0 or 1}, that is, f (3 + 3) ≥ 2 · f (3) = 2. Notice that f (3 + 6) = f (3 + 2 · 3) ≥ f (3) + f (2 · 3) ≥ f (3) + 2 · f (3) = 3f (3). Hence, f (3n) = f (3 + (n − 1)3) ≥ f (3) + (n − 1)f (3) = n · f (3) = n. Therefore f (3n) ≥ n, for all n. If for some n0 the inequality is strict, then for all n ≥ n0 the inequality is also strict. Since f (9999) = f (3 · 3333) = 3333, it follows that f (3n) = n, for 3 ≤ n ≤ 3333, in particular f (3 · 1982) = 1982. Now, 1982 = f (3 · 1982) = f (2 · 1982 + 1982) ≥ f (2 · 1982) + f (1982) ≥ 3 · f (1982), then 1982 ≥ 3 · f (1982). Thus, f (1982) ≤ 1982 3 < 661. On the other hand, f (1982) = f (1980 + 2) ≥ f (1980) + f (2) = f (3 · 660) = 660. Therefore, 660 ≤ f (1982) < 661, that is, f (1982) = 660. Solution 9.82. First, we show that 1 is in the image of f . For x0 > 0, let y0 = 1 . f (x0 ) Then condition (i) states that f (x0 f (y0 )) = y0 f (x0 ) = 1, that is, 1 is in the image of f . Then there is a value of y such that f (y) = 1. This, together with x = 1 in (i), imply that f (1 · 1) = f (1) = yf (1). Since f (1) > 0 by hypothesis, then y = 1 and hence f (1) = 1. Taking y = x in (i), we obtain that f (xf (x)) = xf (x), for all x > 0. Then xf (x) is a fixed point of f . Now, if a and b are fixed points of f , then using (i) with x = a, y = b, we guarantee that f (ab) = ba, hence ab is also a fixed point of f . Thus, the set of fixed points of f is closed under multiplication. In particular, if a is a fixed point, all non-negative integer powers of a are fixed points, that is, 289 10.9 Solutions of Chapter 9 an is a fixed point for all non-negative integers n. Then, by (ii), there are no fixed points greater than 1. Since xf (x) is a fixed point, it follows that xf (x) ≤ 1 or f (x) ≤ Let a = xf (x), then f (a) = a. Now, use x = 1 f (a) a f then f 1 a = 1 a 1 a 1 , for all x. x and y = a in (i), to obtain = f (1) = 1 = af or f (10.38) 1 xf (x) = 1 a , 1 . xf (x) This shows that xf1(x) is also a fixed point of f for all x > 0. Then f (x) ≥ x1 . This and (10.38) imply that f (x) = x1 . This function clearly satisfies the conditions of the problem. Solution 9.83. Suppose that ⌊f (y)⌋ = 0 for some y, then the substitution x = 1 implies that f (y) = f (1)⌊f (y)⌋ = 0. Then if ⌊f (y)⌋ = 0 for all y, it follows that f (y) = 0 for all y. This function obviously satisfies the conditions of the problem. Now, we have to consider the case when ⌊f (a)⌋ =  0, for some a. Then it follows from f (⌊x⌋a) = f (x)⌊f (a)⌋, that f (x) = f (⌊x⌋a) . ⌊f (a)⌋ (10.39) This means that f (x1 ) = f (x2 ) if ⌊x1 ⌋ = ⌊x2 ⌋, then f (x) = f (⌊x⌋). Hence we can assume that a is an integer. Now, we have   1 1 f (a) = f 2a · = f (2a) f = f (2a)⌊f (0)⌋, 2 2 this implies that ⌊f (0)⌋ =  0, then we can assume that a = 0. Therefore, equation (10.39) implies that f (x) = ⌊ff (0) (0)⌋ = C = 0, for every x. Now, the condition of the problem is equivalent to equation C = C⌊C⌋, which is true exactly when ⌊C⌋ = 1. Then, the only functions that satisfy the conditions of the problem are f (x) = 0 and f (x) = C, with ⌊C⌋ = 1. Solution 9.84. For x = y = 1, we have f (f (1)) = f (1). Now, if we take x = 1 and y = f (1), and since f (f (1)) = f (1), it follows that [f (1)]2 = f (1). Then there are two possibilities, either f (1) = 1 or f (1) = 0. If f (1) = 1, substituting y = 1 in the functional equation, we get f (xf (1)) + f (x) = xf (1) + f (x), then f (x) = x, for all x ∈ R. But this function is not bounded, hence f (1) = 1 is not true. 290 Chapter 10. Solutions to Exercises and Problems If f (1) = 0, taking x = 1, we get f (f (y)) + yf (1) = f (y) + f (y), then f (f (y)) = 2f (y). If f (y) ∈ Img f , then 2f (y) ∈ Img f , and by induction, f n (y) = 2n f (y) ∈ Img f . We conclude that f (y) ≤ 0, because if f (y) > 0, it will follow that 2n f (y) ∈ Img f , for all n, which is impossible since the function f is upper bounded. that f (f (y)) = 2f (y), Substituting x by x2 and  x y by f (y) and,   x noticing we obtain f (xf (y)) + f (y)f f (y) . Then xf (y) − f (xf (y)) = = xf (y) + f 2 2     f (y)f x2 − f x2 f (y) ≥ 0, since f (x) ≤ 0, for all x. All these results together and equation (9.4) imply that yf (x) ≥ f (xy). Considering that yf (x) ≥ f (xy), and taking x > 0, y = x1 , we obtain f (x) ≥ 0. Since f (x) ≤ 0, then f (x) = 0. Clearly f (x) = 0, for all x ∈ R, satisfies the functional equation and it is bounded. Suppose that f is not identically zero, then there exists x0 < 0 such that f (x0 ) < 0. Let y0 = f (x0 ), then f (y0 ) = f (f (x0 )) = 2f (x0 ) = 2y0 . For any x < 0, it follows that y0 x > 0, then f (y0 x) = f (2y0 x) = 0. Hence, after substituting y by y0 in (9.4), we get f (2y0 x) + y0 f (x) = 2y0 x + f (xy0 ), thus f (x) = 2x, for all x < 0. Hence, the other solution is f (x) = 0 for x ≥ 0, and f (x) = 2x, for x < 0. Therefore, the only solutions are  0, if x ≥ 0, f (x) = 0 and f (x) = 2x, if x < 0. It is easy to verify that these functions satisfy the conditions of the problem. Solution 9.85. First notice that a2n > 1 and a2n+1 < 1, for all n ≥ 1. The proof of the following statement will be by induction for k ≥ 2. Pk : For every pair of positive integers a, b with (a, b) = 1 and a + b ≤ k, there exists an integer n such that an = ab . If k = 2, the only positive integers a, b that satisfy (a, b) = 1 and a + b ≤ 2, are a = b = 1, and in such a case a1 satisfies ab = a1 = 1. Now, suppose that the statement is true for k ≥ 2; we will prove that it is valid for k + 1. Let a, b be positive integers that satisfy (a, b) = 1 and a + b ≤ k + 1. If a > b, then a − b and b satisfy that (a − b, b) = 1 and (a − b) + b = a ≤ k, then by the induction hypothesis, there is an integer n with an = a−b b . Then a2n = an + 1 = a−b a +1= . b b If a < b, then b − a and a satisfy that (b − a, a) = 1 and (b − a) + a = b ≤ k; hence by the induction hypothesis, there is an integer n with an = b−a a . Then a2n+1 = 1 1 = = a2n an + 1 1 a = . b +1 b−a a 291 10.9 Solutions of Chapter 9 Let us now see the uniqueness of the representation. If an = am > 1, then m and n are even. However, if an = am < 1, then m and n are odd. In the first case, n = 2n′ and m = 2m′ , and then an = am implies that an′ = am′ . In the second case, n = 2n′ + 1 and m = 2m′ + 1, then an = am implies that a 1 ′ = a 1 ′ , and then an′ = am′ . Thus, in any case the equality an = am leads 2n 2m to another equality an′ = am′ , where the subindices are smaller. This process can only be applied a finite number of steps to conclude that n′ = m′ = 1 or n′ = 1 or m′ = 1. In the first case, it follows that n = m, and in the other cases we conclude that an′ = a1 = 1 or am′ = a1 = 1, but this is not possible, since at the beginning we pointed out that the an ’s are not equal to 1, when n > 1. Solution 9.86. Since an+1 = 1 an + n. Then an 1+nan , for n ≥ 0, it follows that 1 an+1 = 1+nan an = 1 1 = + 999 a1000 a999 1 + 999 + 998 = a998 .. = . 1 + 999 + 998 + · · · + 1 = a1 999 · 1000 = 1+ = 499501. 2 Therefore a1000 = 1 499501 . Solution 9.87. From the definition of xi it follows that for every integer k, x4k−3 = x2k−1 = −x4k−2 and x4k−1 = x4k = −x2k = xk . * Then, if we set Sn = ni=1 xi , it follows that S4k = k # ((x4k−3 + x4k−2 ) + (x4k−1 + x4k )) = k # (0 + 2xk ) = 2Sk , (10.40) (10.41) i=1 i=1 S4k+2 = S4k + (x4k+1 + x4k+2 ) = S4k . (10.42) *n *n Also, observe that Sn = i=1 xi ≡ i=1 1 = n mod 2. We will show, by induction on k, that Si ≥ 0, for all i ≤ 4k. The basis is true since x1 = x3 = x4 = 1, x2 = −1. For the inductive step, suppose that Si ≥ 0, for all i ≤ 4k. Using relations (10.40), (10.41) and (10.42), we obtain that S4k+4 = 2Sk+1 ≥ 0, S4k+3 S4k+2 = S4k ≥ 0, S4k+2 + S4k+4 = S4k+2 + x4k+3 = ≥ 0. 2 292 Chapter 10. Solutions to Exercises and Problems Then, it will be enough to show that S4k+1 ≥ 0. If k is odd, then S4k = 2Sk ≥ 0; since k is odd, Sk is also odd, and then S4k ≥ 2. Therefore S4k+1 = S4k +x4k+1 ≥ 1. Reciprocally, if k is even, then x4k+1 = x2k+1 = xk+1 . Thus S4k+1 = S4k +x4k+1 = 2Sk + xk+1 = Sk + Sk+1 ≥ 0 and the induction is complete. Solution 9.88. First, from the problem conditions, it follows that each an (n > s) can be expressed as an = aj1 + aj2 with j1 , j2 < n, j1 + j2 = n. If, say, j1 > s, then we can proceed in the same way with aj1 , and so on. Finally, we represent an as an = ai1 + · · · + aik , 1 ≤ ij ≤ s, (10.43) i1 + · · · + ik = n. (10.44) Moreover, if ai1 and ai2 are the numbers in (10.43), obtained on the last step, then i1 + i2 > s. Hence we can adjust (10.44) as 1 ≤ ij ≤ s, i1 + · · · + ik = n, i1 + i2 > s. (10.45) On the other hand, suppose that the indices i1 , . . . , ik satisfy conditions (10.45). Then, writing sj = i1 + · · · + ij , from the problem’s condition we have an = ask ≥ ask−1 + aik ≥ ask−2 + aik−1 + aik ≥ · · · ≥ ai1 + · · · + aik . Summarizing these observations, we get the following lemma. Lemma 10.9.1. For every n > r, we have an = max {ai1 + · · · + aik }, where the collection (i1 , . . . , ik ) satisfies (10.45). ) ( Now we write r = max aii | 1 ≤ i ≤ s , and fix some index l ≤ s, such that s = all . Consider some integer n ≥ s2 l + 2s and choose an expansion of an in the form (10.43), (10.45). Then we have n = i1 + · · · + ik ≤ sk, so k ≥ n/s ≥ sl + 2. Suppose that none of the numbers i3 , . . . , ik equals l. Then by the pigeonhole principle there is an index 1 ≤ j ≤ s which appears among i3 , . . . , ik at least l times, and surely j = l. Let us delete these l occurrences of j from (i1 , . . . , ik ), and add j occurrences of l instead, obtaining a sequence (i1 , i2 , i′3 , . . . , i′k′ ) also satisfying (10.45). Using the lemma, we have ai1 + · · · + aik = an ≥ ai1 + ai2 + ai′3 + · · · + ai′k′ , or, after removing the common terms, laj ≥ jal , then leads to laj = jal , hence al l ≤ aj j . The definition of l an = ai1 + ai2 + ai′3 + · · · + ai′k′ . Thus, for every n ≥ s2 l + 2s, we have found a representation of the form (10.43), (10.45) with ij = l, for some j ≥ 3. Rearranging the indices we may assume that ik = l. 293 10.9 Solutions of Chapter 9 Finally observe that in this representation the indices (i1 , . . . , ik−1 ) satisfy the conditions (10.45), with n replaced by n − l. Thus, from the lemma, we get an−l + al ≥ (ai1 + · · · + aik−1 ) + al = an , which, by the problem’s condition, implies an = an−l + al for each n ≥ s2 l + 2s, as desired. Solution 9.89. For each i = 1, 2, . . . , k − 1, let Pi be the set of all prime numbers congruent to i modulo k. Each prime number (except possibly k) is contained in exactly one of the sets P1 , P2 , . . . , Pk−1 . Since there are an infinite number of prime numbers, at least one of these sets is infinite, say Pi . Let p = x1 < x2 < · · · < −p , xn < · · · be its elements arranged in increasing order, and define an = xn+1 k for n = 1, 2, . . . . Then the sequence p + kan contains all members of Pi , starting with x2 . The numbers an are positive integers, p is prime and the sequence {an } is increasing, hence it satisfies the conditions of the problem. Solution 9.90. The hypothesis implies a + b + c = 2, ab + bc + ca = −1 and abc = 0. 2 Then a, x3 − 2x√ − x = 0, which are √ √ b, c are the roots of the cubic polynomial 0, 1 ± 2. Then we can assume that a = 1 + 2, b = 1 − 2 and c = 0. Hence, a2 + b2 = 6 and ab = −1. Now, we have that sn−1 sn+1 = (an−1 + bn−1 )(an+1 + bn+1 ) = a2n + b2n + n−1 n−1 2 a b (a + b2 ) = s2n − 2an bn + (ab)n−1 (a2 + b2 ) = s2n − 2(ab)n + 6(ab)n−1 = 2 sn − 8(−1)n , then s2n − sn−1 sn+1  = |8(−1)n | = 8. Solution 9.91. (i) We show that the series x20 x2 x2 + 1 + 2 + ··· , x1 x2 x3 with 1 = x0 ≥ x1 ≥ · · · > 0, (10.46) has sum greater than or equal to 4. This clearly implies that some partial sum of the series is greater than or equal to 3.999. Let L be the infimum (the greatest lower bound) of the sum of all series of the form (10.46). Clearly L ≥ 1, since the first term x11 ≥ 1. For all ǫ > 0 we can find a sequence {xn } such that L+ǫ> x20 x2 x2 + 1 + 2 + ··· . x1 x2 x3 (10.47) Setting yn = xn+1 x1 , with n ≥ 0, it follows that 1 = y0 ≥ y1 ≥ y2 ≥ · · · > 0. The series on the right-hand side of the inequality (10.47) can be written as 1 + x1 x1 y02 y2 y2 + 1 + 2 + ··· y1 y2 y3 . By definition of L, the series inside the parenthesis has a sum greater than or equal to ≥ L. Hence, by (10.47), we have that L + ǫ > x11 + x1 L. 294 Chapter 10. Solutions to Exercises and Problems Applying the inequality between the arithmetic mean and √the geometric mean on the right-hand side of the inequality we get L + ǫ > 2 L. Since this √ is true for all ǫ > 0, it follows that L ≥ 2 L. Thus, L2 ≥ 4L, and since L > 0, this implies L ≥ 4. (ii) Let xn = 1 2n , then ∞ ∞ # # 1 x2n = = 4, n−1 x 2 n=0 n=0 n+1 and the partial sums of this series are less than 4. Solution 9.92. By (i) and (ii), it follows that all the elements in the sequence are rational numbers. Suppose that ak = pq , with p and q integers such that (p, q) = 1, 2 2 ) then ak+1 = (2p q−q . Since (p, q) = 1, it follows that (2p2 − q 2 , q 2 ) = (2p2 , q 2 ) = 2 (2, q 2 ), which is equal to either 1 or 2. If q > 2, then the denominator of ak+1 is greater than the denominator of ak . Hence, the sequence of denominators will be increasing and therefore it cannot be an equality among the terms of the sequence. Moreover, if |ak | > 1, then writing |ak | = 1+e, it follows that ak+1 = 1+4e+2e2 > |ak |, which gives an increasing sequence. Therefore, in order that the terms of the sequence repeat themselves, the first term must have denominator at most 2 and must be between −1 and 1, this gives us only 5 possible values: • • • • • a0 a0 a0 a0 a0 = −1, which gives the sequence −1, 1, 1, 1, . . . . = − 12 , which gives the sequence − 21 , − 21 , − 21 , . . . . = 0, which gives the sequence 0, −1, 1, 1, 1, . . . . = 12 , which gives the sequence 12 , − 21 , − 21 , − 21 , . . . . = 1, which gives the sequence 1, 1, 1, 1, . . . . Solution 9.93. The first terms of the sequence are a0 = 0, a1 = 1, a2 = 2, a3 = 5, a4 = 12, a5 = 29, a6 = 70, a7 = 169. Observe that an+1 = 2an + an−1 = a2 an + a1 an−1 and an+2 = 2an+1 + an = 2(2an + an−1 ) + an = 5an + 2an−1 = a3 an + a2 an−1 . Then we can conjecture that an+m = am+1 an + am an−1 . (10.48) To show it we use induction on m. For m = 1, 2 it was already proved. Now, suppose that the equality is true for m = k − 1 and for m = k, and show that the relation holds for k + 1. Observe that an+k+1 = 2an+k + an+k−1 = 2(ak+1 an + ak an−1 ) + ak an + ak−1 an−1 = (2ak+1 + ak )an + (2ak + ak−1 )an−1 = ak+2 an + ak+1 an−1 , which finishes the induction. 295 10.9 Solutions of Chapter 9 If we let n = m in (10.48), we obtain the equality a2n = an (an+1 + an−1 ). Also note that if n is even, then an is even, which is easy to deduce using the formula of the sequence an = 2an−1 + an−2 . Now, if n is odd, then an ≡ 1 mod 4. In order to see this, use induction again. For n = 1, the result follows since a1 = 1, then assume that a2n−1 ≡ 1 mod 4. Then, since a2n+1 = 2a2n + a2n−1 , we only need to observe that a2n is even, which finishes the induction. Summarizing, if n is odd, that is, if there is no power of 2 dividing n, then an ≡ 1 mod 4, hence there is no power of 2 dividing an . If n is even, then n − 1 and n + 1 are odd, hence an+1 + an−1 ≡ 2 mod 4, that is, in the equality a2n = an (an+1 + an−1 ), only one extra factor of 2 appears inside the parenthesis, and this ends the proof. Solution 9.94. Since |am+1 + an − am+n+1 | ≤ 1 , m+n+1 |am + an+1 − am+n+1 | ≤ 1 , m+n+1 from the two inequalities it follows that |(am+1 − am ) − (an+1 − an )| ≤ 2 2 < . m+n+1 n Now, using twice the previous inequality, we get |(am+1 − am ) − (an+1 − an )| ≤ |(am+1 − am ) − (ak+1 − ak ) + (ak+1 − ak ) − (an+1 − an )| 2 2 4 + = . k k k Since k is arbitrary, |(am+1 − am ) − (an+1 − an )| is equal to 0, then an+1 − an is constant. ≤ |(am+1 − am ) − (ak+1 − ak )| + |(ak+1 − ak ) − (an+1 − an )| ≤ Solution 9.95. The function f (x) = nx + nx is decreasing in (0, n] (it can be seen that f ′ (x) < 0, or that the graph is decreasing with the identity f (x) − f (y) = √ (x−y)(xy−n2 ) n , for n ≥ 3. ). First we see, using induction in n, that n ≤ an ≤ √n−1 xyn √ 3 √ For n = 3 is clear, since 3 ≤ a3 = 2 ≤ 2 . Suppose the result true for n, % $ √ √ . On the other hand, an+1 = f (an ) ≥ f √ n = an+1 = f (an ) ≤ f ( n) = n+1 n n−1 √ n √ > n + 1, then the result follows. n−1 √ n Now, let us see that an < n + 1. If an+1 = f (an ) ≥ √n−1 is true for n ≥ 3, n−1 then an ≥ √n−2 for n ≥ 4. Since f is decreasing, then an+1 = f (an ) ≤ f n−1 √ n−2 = (n − 1)2 + n2 (n − 2) √ √ < n + 2, (n − 1)n n − 2 for n ≥ 4. Hence ⌊a2n ⌋ = n, for n ≥ 5. The case n = 4 follows easily after noticing that a4 = 13 6 . 296 Chapter 10. Solutions to Exercises and Problems Solution 9.96. Denote by l(n) the last digit of a positive integer n, that is, the units digit. The sequence {l(n)} is periodic with period 10. Now, for a fixed positive integer a, the sequence {l(an )} is periodic with period equal to 1 if a ends in 0, 1, 5, 6; the period is equal to 2 if a ends in 4 or 9; and the period is 4 if a ends in 2, 3, 7, 8. Since the least common multiple of 10 and 4 is 20, and if m = (n + 1)n+1 + (n + 2)n+2 + · · · + (n + 20)n+20 , then l(m) does not depend on n. We now calculate the last digit of 11 + 22 + 33 + · · · + 2020 . By the periodicity of the sequence of the form {l(an )}, the last digit of this number is the same as the one in 1 + 22 + 33 + 44 + 5 + 6 + 73 + 84 + 9 + 1 + 24 + 3 + 42 + 5 + 6 + 7 + 82 + 9. The units digit of this last number is 4. Therefore, the last digit of a sum of the form (n + 1)n+1 + (n + 2)n+2 + · · · + (n + 100)n+100 is equal to l(4 · 5) = l(20) = 0. Thus, bn is periodic with period 100. √ √ Solution 9.97. Notice that (1 + 3)2n+1 + (1 − 3)2n+1 is an even integer. In order to see this, simplify the binomials to obtain √ √ (1 + 3)2n+1 + (1 − 3)2n+1 2n+1 # 2n + 1 $ √ %j # 2n + 1 $√ %j 2n+1 3 + = − 3 j j j=0 j=0 =2 n n # # 2n + 1 $√ %2j 2n + 1 j =2 3 3 . 2j 2j j=0 j=0 √ √ since −1< 1− 3 < 0, it follows that −1 < (1− 3)2n+1 < 0, therefore  Now, √ 2n+1 √ 2n+1 √ √ 2n+1  √  1+ 3 + 1− 3 = 1+ 3 = (1+ 3)2n+1 +(1− 3)2n+1 is an even integer.  √ 2n+1  . To see this, observe that It can be shown that 2n+1 divides 1 + 3 √ %2n+1 $ √ √ √ √ %2n+1 √ 3 + 1− 3 = (1 + 3)(4 + 2 3)n + (1 − 3)(4 − 2 3)n √ √ √ √ = 2n (1 + 3)(2 + 3)n + 2n (1 − 3)(2 − 3)n . √ √ √ √ It is only left to see that 2 divides (1 + 3)(2 + 3)n + (1 − 3)(2 − 3)n , and for that use again Newton’s binomial theorem. $ 1+ Solution 9.98. The characteristic polynomial of the recursion an+2 = 3an+1 − 2an is λ2 −3λ+2, which has as roots λ = 2, 1. Then an = Aλn +B for some real numbers A and B which are determined by 3 = a1 = A · 2 + B and 5 = a2 = A · 22 + B. The previous system has solutions A = B = 1, then an = 2n + 1, for n ≥ 1. 297 10.9 Solutions of Chapter 9 Solution 9.99. The relation an+6 = an follows from the relation an+2 = an+1 − an , since an+6 = an+5 − an+4 = an+4 − an+3 − an+4 = −(an+2 − an+1 ) = −(an+1 − an − an+1 ) = an . Solution 9.100. Since a2 = 22 , a3 = 52 , a4 = 132 , we can conjecture that an = 2 . Use strong induction in order to show the result. A relation between 3 F2n−1 Fibonacci numbers that helps is F2n+1 = 3F2n−1 − F2n−3 . Solution 9.101. Calculate the first terms, u1 = 1 5 =2+ 2 2 u2 = u1 (u20 − 2) − u1 = u4 = = u5 = 2+ 1 2 1 23 1 23 + 3 2 1 25 + 5 2 23 + (22 − 2) − 2 + $ 1 2 =2+ 1 2 1 %2 1 −2 − 2+ 2 2 1 1 1 1 22 + 2 − 2 + = 2+ 22 − 1 + 2 2 2 2 2 $ 1 %2 1 2+ −2 − 2+ 2 2 1 1 1 22 + 2 − 2 + = 25 + 5 2 2 2 $ 1 1 1 %2 23 + 3 − 2 − 2 + = 211 + 11 . 2 2 2 u3 = u2 (u21 − 2) − u1 = = 1 2 1 2+ 2 2+ 2+ = 23 + 1 23 This allows us to conjecture that un = 2rn + 2r1n for some numbers rn which must be determined. If the numbers rn are integers, then ⌊un ⌋ = 2rn . If un = 2rn + 2r1n , then from the original equation it follows that   2 1 1 1 rn−1 rn un+1 = 2 + rn + rn−1 2 −2 − 2+ 2 2 2    1 = 2rn + 2−rn 22rn−1 + 2−2rn−1 − 2 + 2 = 2rn +2rn−1 + 2−rn −2rn−1 + 2rn −2rn−1 + 2−rn+2rn−1 − 2 + 1 2 . Therefore, if we can find a sequence {rn } that satisfies rn+1 = rn + 2rn−1 and rn − 2rn−1 = (−1)n , the proof is complete. The characteristic equation of the recursion rn+1 = rn + 2rn−1 is λ2 − λ− 2 = 0, and its roots are λ = 2, −1, and since r0 = 0 and r1 = 1, it follows that n n is the solution, and it is also solution of the other recursion. rn = 2 −(−1) 3 Hence, ⌊un ⌋ = 2 2n −(−1)n 3 , for all n ∈ N. 298 Chapter 10. Solutions to Exercises and Problems √ Solution 9.102. First √ notice that the integers n that satisfy ⌊ 2n⌋ = m are the ones that satisfy m ≤ 2n < m + 1. Then m2 ≤ 2n < (m + 1)2 , but even numbers between m2 (inclusive) and (m + 1)2 are m, if m is odd, and m + 1 if m is even. Hence the sequence an is 1, 2, 2, 2, 3, 3, 3, 4, 4, 4, 4, 4, 5, 5, 5, 5, 5, . . ., where the integer k appears k times in the sequence if k is odd or k + 1 times if k is even. When we write all the numbers 63, we arrive to the term an with n = 1 + 3 + 3 + 5 + 5 + · · · + 63 + 63 = 2(1 + 3 + · · · + 63) − 1 = 2(32)2 − 1 = 2047. This last term and the 62 previous ones are equal to 63, in particular a2020 = 63. Solution 9.103. Since P (0) = 0, it follows that P (x) = xQ(x), for some polynomial 1 Q(x) of degree n − 1 that satisfies Q(j) = j+1 , for j = 1, 2, . . . , n. If R(x) = (x + 1)Q(x) − 1, then the degree of R(x) is n and R(j) = 0, for j = 1, 2, . . . , n. Hence, R(x) = a0 (x − 1)(x − 2) . . . (x − n). If m > n, Q(m) = a0 (m − 1)(m − 2) . . . (m − n) + 1 . m+1 Evaluating R(x) in x = −1, it follows that −1 = a0 (−2)(−3) . . . (−n − 1), then n+1 a0 = (−1) (n+1)! . Hence, P (m) = mQ(m) = (−1)n+1 m(m − 1)(m − 2) . . . (m − n) m + . (n + 1)! m+1 m+1 Solution 9.104. Consider Q(x) = xP (x) − 1 = c(x − 1)(x − 2) . . . (x − 2n ). For x = 1, 2, . . . , 2n , it follows that 1 1 1 Q′ (x) = + + ··· + . Q(x) x−1 x−2 x − 2n Since Q(0) = −1 and Q′ (x) = P (x) + xP ′ (x), then 1 1 1 P (0) = Q′ (0) = − − − − · · · − n 1 2 2 = 1 1 − 2n+1 1 = 2 − n. 1 2 1− 2 Solution 9.105. Lemma. If P (x) is a polynomial of degree less than or equal to n, then n+1 # (−1)i i=0 n+1 P (i) = 0. i Proof of the lemma. Proceed by induction on n. For n = 0, P0 − P0 = 0. 1 1 0 P (0) − 1 P (1) = 299 10.9 Solutions of Chapter 9 Suppose valid the result for all n ≤ k and consider the polynomial P (x) of degree k + 1. The polynomial P (x) − P (x + 1) has degree less than or equal to k, then 0= k+1 # (−1)i i=0 k+1 # k+1 (P (i) − P (i + 1)) i k+2 # k+1 k+1 (−1)i P (i) P (i) + i−1 i i=1 i=0 , + k+1 # k+1 k+1 = P (0) + (−1)i + P (i) + (−1)k+2 P (k + 2) i i − 1 i=1 = = k+2 # (−1)i (−1)i i=0 k+2 P (i). i Hence, for the polynomial P (x) the result holds and therefore the proof of the lemma is complete.  Now, apply the lemma to the polynomial P (x) of the problem, 0= n+1 # (−1)i i=0 = n # n # n+1 n+1 (−1)i P (i) = i i i=0 n+1 i −1 + (−1)n+1 P (n + 1) (−1)i + (−1)n+1 P (n + 1), i=0 hence P (n + 1) is 1 if n is even and it is 0 if n is odd. Solution 9.106. Suppose that there are polynomials Q(x) and R(x) with integer coefficients such that P (x) = Q(x)R(x). Since P (0) = 3, we can assume, without loss of generality, that |Q(0)| = 3. If Q(x) = xk + ak−1 xk−1 + · · · + a0 , with a0 = ±3, R(x) = xl + bl−1 xl−1 + · · · + b0 and P (x) = xn + cn−1 xn−1 + · · · + c0 , it follows that cj = aj b0 + aj−1 b1 + · · · . Let j be the smallest index such that 3 does not divide aj , then 3 neither divides cj , since 3 ∤ b0 . Hence, j ≥ n − 1, and then k ≥ n − 1 and l ≤ 1. Thus the polynomial R(x) has the form ±x ± 1, but neither 1 or −1 are roots of P (x). Therefore, P (x) is irreducible over Z[x]. Solution 9.107. Suppose that the degree of Q(x) is n ≤ p − 2. Using the lemma in Problem 9.105, 0= p−1 p−1 # # p−1 Q(i) mod p, Q(i) ≡ (−1)i i i=0 i=0   since p−1 ≡ (−1)i mod p. But this is not possible if Q(0) = 0, Q(1) = 1 and i Q(i) ≡ 0, 1 mod p. 300 Chapter 10. Solutions to Exercises and Problems Solution 9.108. Since P (x) > 0, for x ≥ 0, the polynomial can be decomposed in the following way P (x) = a0 (x + a1 ) . . . (x + an )(x2 − b1 x + c1 ) . . . (x2 − bm x + cm ), with ai > 0, for 0 ≤ i ≤ n, and each quadratic polynomial x2 − bj x + cj has no real roots. Since the product of polynomials with positive coefficients is a polynomial with positive coefficients, and since the factors (x+ai ) already have positive coefficients, it will be enough to analyze the quadratic factors. Let x2 − bx + c be a quadratic polynomial, with b2 − 4c < 0. Then n # n i 2 x (x − bx + c) i i=0 , n+2 #+ n n n = −b +c xi i − 2 i − 1 i i=0 (1 + x)n (x2 − bx + c) = = n+2 # Ci xi , i=0 where + , n n n Ci = −b +c i−2 i−1 i ! " n! (b + c + 1)i2 − ((b + 2c)n + (2b + 3c + 1))i + c(n2 + 3n + 2) . = i!(n − i + 2)! Now, Ci will be negative if its discriminant is negative (depends of i). The discriminant is D = ((b + 2c)n + (2b + 3c + 1))2 − 4(b + c + 1)(c(n2 + 3n + 2)) = (b2 − 4c)n2 − 2U n + V, where U = 2b2 + bc + b − 4c and V = (2b + c + 1)2 − 4c. But since b2 − 4c < 0, it will be sufficient to take n large enough, and then we will do this with every quadratic factor. Solution 9.109. Notice that xP (x) = yP (y) ⇔ ⇔ a(x4 − y 4 ) + b(x3 − y 3 ) + c(x2 − y 2 ) + d(x − y) = 0 a(x3 + x2 y + xy 2 + y 3 ) + b(x2 + xy + y 2 ) + c(x + y) + d = 0. 301 10.9 Solutions of Chapter 9 2 If u = x + y and v = x2 + y 2 , then x2 + xy + y 2 = u 2+v and the previous equation becomes auv+ 2b (u2 +v)+cu+d = 0, or equivalently, (2au+b)v = −(bu2 +2cu+2d).   2 Since v ≥ u2 , it is clear that u2 |2au + b| ≤ 2 bu2 + 2cu + 2d, which is true only for a finite number of values of u. Since there are an infinite number of pairs of integers (x, y) with xP (x) = yP (y), there is an integer u such that xP (x) = (u − x)P (u − x) for an infinite number of integers x, but since P (x) is a polynomial, the latter is true for every real number x. If u = 0, then u is a root of P (x). If u = 0, then P (x) = −P (−x); this implies b = d = 0. Then P (x) = ax3 + cx = x(ax2 + c), hence u = 0 is a root of P (x). √ Solution 9.110. It is clear that (xyz)2 = abc, then xyz = ± abc, hence x = y= √ ± abc , c z= √ ± abc a √ ± abc , b solve the system. + b + c), xy = 21 (a + b −  , y = c), yz = 12 (b + c − a), zx = 12 (c + a − b). Hence x = ± (c+a−b)(a+b−c) 2(b+c−a)   (b+c−a)(a+b−c) (b+c−a)(c+a−b) ± ,z=± solve the system. 2(c+a−b) 2(a+b−c) Solution 9.111. It is clear that xy + yz + zx = 1 2 (a Solution 9.112. The system is equivalent to the following system: 1 1 1 + = , xy xz a 1 1 1 + = , yz yx b 1 1 1 + = . zx zy c By the previous problem, 1 =± x ( 1c + 1 a − 1b )( 1a + 1b − 1c ) =± 2( 1b + 1c − a1 ) (ab + bc − ca)(bc + ca − ab) 2abc(ca + ab − bc) and similarly for the other variables. Solution 9.113. If A, B, C are the expressions on the left-hand sides of the equations, it follows that −A + B + C = (−a + b + c)3 , A − B + C = (a − b + c)3 , A + B − C = (a + b − c)3 and −A + B + C = A − B + C = A + B − C = 1. The system is equivalent to −a + b + c = 1, a − b + c = 1, a + b − c = 1, which has the unique solution (a, b, c) = (1, 1, 1). Solution 9.114. Proceed by induction on n. The case n = 1 is trivial. Suppose that it is true for n > 1. The polynomial Q(x) = P (x + 1) − P (x) has degree n − 1 and takes integer values in the integers, then by the induction hypothesis, there are integers a0 , . . . , an−1 such that Q(x) = an−1 x x . + · · · + a0 0 n−1 302 Chapter 10. Solutions to Exercises and Problems For any integer x >0, it follows that P (x) = P(0) +    Q(0) + Q(1) + · · · + Q(x − 1). x = k+1 , for any integer k, we obtain Using the identity k0 + k1 + · · · + x−1 k the desired representation of P (x), P (x) = an−1 x x + P (0). + · · · + a0 1 n 1 Solution 9.115. Let r be a zero of P (x). Then |r|p −|r| ≤ |rp −r| = p. If |r| ≥ p p−1 , then 1 |r|p − |r| = |r|(|r|p−1 − 1) ≥ p p−1 (p − 1) > p, 1 which is a contradiction. Here we have used p p−1 > p−1 p−1 p p−1 , which follows from 1 p−1 p . = ((p − 1) + 1) . Therefore, |r| < p Suppose that P (x) is the product of two non-constant polynomials Q(x) and R(x) with integer coefficients. One of these polynomials, say Q(x), has constant term equal to ±p. On the other hand, the zeros r1 , r2 , . . . , rk of Q(x) satisfy |r1 |, . . . , 1 |rk | < p p−1 , and moreover r1 · · · rk = ±p, hence k ≥ p, which is impossible. Solution 9.116. Let P (x) = an xn + an−1 xn−1 + · · · + a1 x + a0 . For every x, the triplet (a, b, c) = (6x, 3x, −2x) satisfies the condition ab + bc + ca = 0. The condition in P (x) implies that P (3x) + P (5x) + P (−8x) = 2P (7x), for all x. Now comparing coefficients on both sides of the equality, it follows that the number K(i) = (3i + 5i + (−8)i − 2 · 7i ) = 0, if ai = 0. Since K(i) is negative for i odd and positive for i = 0 or for i ≥ 6 even, then ai = 0, for i = 2, 4. Therefore, P (x) = a2 x2 + a4 x4 , for any real numbers a2 and a4 . It is easy to see that all polynomials of the previous form satisfy the conditions set for the problem. Solution 9.117. We have shown that if for some integer t, Q(t) = t, then P (P (t)) = t (see Exercise 8.16). If such t also satisfies P (t) = t, the number of solutions is clearly at most the degree of P (x), which is equal to n. Let P (t1 ) = t2 , P (t2 ) = t1 , P (t3 ) = t4 and P (t4 ) = t3 , where t1 = ti , for i = 2, 3, 4. Then we have that t3 − t1 divides t4 − t2 and vice versa; therefore t3 − t1 = ±(t4 − t2 ). Similarly, we have that t3 − t2 = ±(t4 − t1 ). Suppose that we have positive signs in both equalities: t3 − t2 = t4 − t1 and t3 − t1 = t4 − t2 . Substracting these equalities, we find t1 − t2 = t2 − t1 , which is a contradiction. Then, at least one of the equalities has negative sign. For each one of those cases, this means that t3 + t4 = t1 + t2 , or equivalently, t1 + t2 − t3 − P (t3 ) = 0. Let C = t1 + t2 , then it has been proved that each integer number that is a fixed point of Q(x), different from t1 and t2 , is a root of the polynomial F (x) = C − x − P (x). This is also valid for t1 and t2 , and since P (x) has degree n > 1, the polynomial F (x) has the same degree, therefore it has no more than n roots. Hence, we have reached the result. 10.9 Solutions of Chapter 9 303 Solution 9.118. For the first positive integers, it follows that P (1) = P (02 + 1) = P (0)2 + 1 = 1 P (2) = P (12 + 1) = P (1)2 + 1 = 2 P (5) = P (22 + 1) = P (2)2 + 1 = 5 P (26) = P (52 + 1) = P (5)2 + 1 = 26. Then, for x0 = 0 and for n ≥ 1, it follows that xn = x2n−1 + 1. Then P (xn ) = P (x2n−1 + 1) = P (x2n−1 ) + 1 = x2n−1 + 1 = xn . Hence, P (x) has an infinite number of fixed points, therefore P (x) = x. Solution 9.119. Let P (1) = a, then it follows that a2 − 2a − 2 = 0. Since P (x) = (x − 1)P1 (x) + a, for some polynomial P1 (x), substituting in the original equation and simplifying leads to (x − 1)P1 (x)2 + 2aP1 (x) = 4(x + 1)P1 (2x2 − 1). For x = 1, it follows that 2aP1 (1) = 8P1 (1), and together with a = 4, implies that P1 (1) = 0. Hence, P1 (x) = (x − 1)P2 (x), for some polynomial P2 (x). Then P (x) = (x − 1)2 P2 (x) + a. Suppose that P (x) = (x − 1)n Q(x) + a, where Q(x) is a polynomial with Q(1) = 0. Again, substituting in the original equation and simplifying, we get (x−1)n Q(x)2 + 2aQ(x) = 2(2x + 2)n Q(2x2 − 1), which implies that Q(1) = 0, which is a contradiction. Therefore P (x) = a. Solution 9.120. It is clear that P (x) and Q(x) have the same degree, say n. The cases n = 0 and n = 1 are clear. Suppose that R(x) = P (x) − Q(x) = 0 and that 0 < k ≤ n − 1 is the degree of R(x), then P (P (x)) − Q(Q(x)) = [Q(P (x)) − Q(Q(x))] + R(P (x)). Writing Q(x) = xn + · · · + a1 x + a0 , we obtain Q(P (x)) − Q(Q(x)) = [P (x)n − Q(x)n ] + · · · + a1 [P (x) − Q(x)]. The main coefficient of the polynomial Q(P (x)) − Q(Q(x)) is n and it is equal 2 to the coefficient of the term xn −n+k . On the other hand, the degree of the polynomial R(P (x)) is equal to kn < n2 − n + k. Therefore the main coefficient of P (P (x))−Q(Q(x)) is n, which is a contradiction with the fact that the polynomial is zero. It is left to prove the case R(x) equal to some constant c. Then the condition P (P (x)) = Q(Q(x)) implies that Q(Q(x) + c) = Q(Q(x)) − c, hence the equality Q(y + c) = Q(y) − c follows for an infinite number of values of y. Thus Q(y + c) ≡ Q(y) − c, which is only possible for c = 0, which can be proved comparing the coefficients and using that Q(x) is monic. Notation The following notation is standard: N Z Q Q+ R R+ I C Zp ⇔ ⇒ a∈A A⊂B |x| |z| {x} ⌊x⌋ [a, b] (a, b) P (x) deg(P ) f : [a, b] → R f ′ (x) f ′′ (x) f (n) (x) f (x)n the positive integers or the natural numbers the integers the rational numbers the positive rational numbers the real numbers the positive real numbers the irrational numbers the complex numbers is {0, 1, . . . , p − 1} with the sum and product modulo p. if and only if imply the element a belongs to the set A A is subset of B the absolute value of the number x the module of the complex number z the fractional part of the number x the integer part of the number x the set of real numbers x such that a ≤ x ≤ b the set of real numbers x such that a < x < b the polynomial P in the variable x the degree of the polynomial P (x) the function f defined in [a, b] with values in R the derivative of the function f (x) the second derivative of the function f (x) the nth derivative of the function f (x) the nth power of a function f (x) © Springer Internationl Publishing Switzerland 2015 R.B. Manfrino et al, Topics in Algebra and Analysis, DOI 10.1007/978-3-319-11946-5 305 306 Notation f n (x) ∆f (x) the nth iteration of a function f (x) the diference operator of f (x) det A *n the determinant of a matrix A the sum a1 + a2 + · · · + an ai 3ni=1 ai 3i=1 a i=j i the product a1 · a2 · . . . · an the product for all a1 , a2 , . . . , an except aj max{a, b, . . . } min{a, b, . . . } √ x √ n x the maximum value among a, b, . . . the minimum value among a, b, . . . exp x = ex # f (a, b, . . . ) the exponential function cyclic the square root of x the nth root of the real number x represents the sum of the function f evaluated in all the cyclic permutations of the variables a, b, . . . We use the following notation for the source of the problems: AMC APMO IMO MEMO OMCC OIM OMM (country, year) American Mathematical Competition Asian Pacific Mathematical Olympiad International Mathematical Olympiad Middle European Mathematical Olympiad Mathematical Olympiad of Central America and the Caribean Iberoamerican Mathematical Olympiad Mexican Mathematical Olympiad problem corresponding to the mathematical olympiad celebrated in that country, in that year, in some stage Bibliography [1] Andreescu T., Andrica D., Complex numbers from A to . . . Z, Birkhäuser, 2005. [2] Andreescu T., Gelca R., Mathematical Olympiad Challenges, Birkhäuser, 2000. [3] Andreescu T., Enescu B., Mathematical Olympiad Treasures, Birkhäuser, 2006. [4] Barbeau E.J., Polynomials, Springer-Verlag, 1989. [5] Bulajich Manfrino R., Gómez Ortega J.A., Geometrı́a, Cuadernos de Olimpiadas, Instituto de Matemáticas de la Universidad Nacional Autónoma de México, Sociedad Matemática Mexicana, 2012. [6] Bulajich Manfrino R., Gómez Ortega J.A., Valdez Delgado R., Desigualdades, Cuadernos de Olimpiadas, Instituto de Matemáticas de la Universidad Nacional Autónoma de México, Sociedad Matemática Mexicana, 2010. [7] Bulajich Manfrino R., Gómez Ortega J.A., Valdez Delgado R., Inequalities: A Mathematical Olympiad Approach, Birkhäuser, 2009. [8] Cárdenas H., Lluis E., Raggi F., Tomás F. Álgebra Superior, Editorial Trillas, 1973. [9] Djukić D., Janković V., Matić I., Petrović N., The IMO Compendium, Springer, 2006. [10] Engel A., Problem-Solving Strategies, Springer, 1998. [11] Fine B., Resenberger G., The Fundamental Theorem of Algebra, Springer, 1997. [12] Goldberg S., Introduction to Difference Equations, Dover Publications, 1958. [13] Gómez Ortega J.A., Valdez Delgado R., Vázquez Padilla, Principio de las Casillas, Cuadernos de Olimpiadas, Instituto de Matemáticas de la Universidad Nacional Autónoma de México, Sociedad Matemática Mexicana, 2011. [14] Honsberger R., Ingenuity in Mathematics, vol. 23 in New Mathematical Library series, 1962. [15] Niven I., Montgomery H., Zuckerman H., An Introduction to the Theory of Numbers, Wiley, 5th edition, 1991. © Springer Internationl Publishing Switzerland 2015 R.B. Manfrino et al, Topics in Algebra and Analysis, DOI 10.1007/978-3-319-11946-5 307 308 Bibliography [16] Remmert R., Theory of Complex Functions, Springer, 1999. [17] Rudin W., Principles of Mathematical Analysis, McGraw-Hill, 1976. [18] Savchev S., Andreescu T., Mathematical Miniatures, The Mathematical Association of America, 2003. [19] Small C.G., Functional Equations and How to Solve Them, Springer, 2007. [20] Soberón P., Problem-Solving Methods in Combinatorics: An Approach to Olympiad Problems, Birkhäuser, 2013. [21] Spivak M., Calculus, Editorial Reverté, 1993. [22] Tabachnikov, S. (editor), Kvant Selecta: Algebra and Analysis II, American Mathematical Society, 1999. [23] Venkatachala B.J., Functional Equations. A Problem Solving Approach, Prism Books Pvt Ltd, 2002. Index Absolute value, 10 properties of, 11 Algorithm division, 66, 140 Euclid, 142 Arithmetic progression, 33 difference of the, 33 of order 2, 34 Bertrand’s postulate, 243 Binomial Newton, 54 square, 15 Binomial coefficient, 53 Cartesian Plane, 10 Complex number, 75 argument, 75 conjugate, 75 imaginary part, 75 module, 75 real part, 75 Decimal system, 8 Dense set, 101 Derangement, 128 Descartes’ rule of signs, 155 Determinant 2 × 2 matrix, 18 3 × 3 matrix, 18 properties of, 19 Difference operator, 111 Endpoints, 10 Equation characteristic, 122 difference, 111 © Springer Internationl Publishing Switzerland 2015 R.B. Manfrino et al, Topics in Algebra and Analysis, DOI 10.1007/978-3-319-11946-5 Factorial, 53 Factorization, 25 Formula Abel’s summation, 133 de Moivre, 79 interpolation, 151 Pascal, 54 Function, 89 non-increasing, 98 additive, 103 bijective, 94 bounded, 99 bounded above, 99 bounded below, 99 codomain, 89 constant, 89 continuous, 100 correspondence rule, 89 decreasing, 98 domain, 89 even, 96 graph, 89 identity, 89 image, 89 increasing, 98 injective, 94 iteration, 111 limit, 99 non-decreasing, 98 odd, 96 periodic, 97 range, 89 surjective, 94 Functional equations, 111 Cauchy, 102 Functions composition, 93 309 310 difference, 91 equality, 89 product, 91 quotient, 91 sum, 91 Geometric progession ratio of the, 36 Geometric progression, 36 Greater than, 6 Hanoi Towers, 44 Harmonic progression, 34 Identity Sophie Germain, 27 Imaginary axis, 75 Induction bases, 43 step, 43 Induction principle Cauchy’s, 49 simple, 43 strong, 48 Inequalities, 21 Inequality Cauchy–Schwarz, 69 helpful, 26 Nesbitt, 24 rearrangement, 24 Infinite descent, 57 Integers, 1 Interval close, 10 open, 10 Irrational number, 4 Koch’s snowflake, 130 Lemma Gauss, 147 growth, 86 Index Matrix 2 × 2, 17 3 × 3, 18 Mean arithmetic, 22, 49 geometric, 22, 49 harmonic, 22 quadratic, 22 Monomial, 139 Natural numbers, 1 Notable product three variables, 16 two variables, 15 Number line, 3 Numbers even, 32 fractional part, 14 greater than, 6 integer part, 12 Lucas, 128 odd, 32 square, 32 triangular, 31 Parameters, 153 Period, 97 Polynomial, 139 characteristic, 122 coefficients, 63, 139 commute, 157 conjugate, 154 constant, 63 cubic, 63 cyclotomic, 148 degree, 67, 139 derivative, 150 discriminant, 67 equality, 139 greatest common divisor, 64, 142 homogeneous, 161 integer coefficients, 145 irreducible, 146 linear, 63 main term, 139 monic, 63, 139 311 Index over the integers, 63 over the rationals, 63 primitive, 147 quadratic, 63 quotient, 64 reciprocal, 143, 144 remainder, 64 root, 63, 139, 141, 145 several variables, 161 solution, 63, 139 symmetric, 161 Tchebyshev, 158 zero, 63 Polynomials division, 64, 141 equality of, 63 product, 64 product by a constant, 64 subtraction of, 64 sum of, 64 Quadratic polynomial complex coefficients, 79 Rational numbers, 2 Real axis, 75 Real numbers, 4 Root primitive, 83 Roots multiple, 150 multiplicity, 67, 143 second-order equation, 67 unity, 82 Second-order equation, 67 discriminant, 67 Sequence, 115 bounded, 118 complete, 125 convergent, 126, 135 decreasing, 125 divergent, 126, 135 Fibonacci, 51 finite differences, 113 increasing, 124 limit, 126, 135 monotone decreasing, 125 monotone increasing, 124 periodic, 119 properties, 118 recursive, 118, 120 totally complete, 125 Series, 129 convergent, 129 derivative, 132 divergent, 129 geometric, 131 harmonic, 131 power, 131 formal, 131 Smaller than, 6 Smaller than or equal to, 7 Straight line oriented, 3 Subsequence, 127 Sum of Gauss, 32 of the cubes, 39 of the squares, 38 partial, 129 telescopic, 40 Theorem binomial, 54 Eisenstein, 147 factor, 66, 82 fundamental of Algebra, 81, 87 proof, 85 rational root, 146 Vieta formulas, 65, 145 jumping, 163